Coordenadas Sistemas (001-272)

Download as pdf or txt
Download as pdf or txt
You are on page 1of 272

CLASSICAL

MECHANICS

Peter Dourmashkin
Massachusetts Institute of Technology
Massachusetts Institute of Technology
Classical Mechanics

Peter Dourmashkin
This text is disseminated via the Open Education Resource (OER) LibreTexts Project (https://LibreTexts.org) and like the hundreds
of other texts available within this powerful platform, it is freely available for reading, printing and "consuming." Most, but not all,
pages in the library have licenses that may allow individuals to make changes, save, and print this book. Carefully
consult the applicable license(s) before pursuing such effects.
Instructors can adopt existing LibreTexts texts or Remix them to quickly build course-specific resources to meet the needs of their
students. Unlike traditional textbooks, LibreTexts’ web based origins allow powerful integration of advanced features and new
technologies to support learning.

The LibreTexts mission is to unite students, faculty and scholars in a cooperative effort to develop an easy-to-use online platform
for the construction, customization, and dissemination of OER content to reduce the burdens of unreasonable textbook costs to our
students and society. The LibreTexts project is a multi-institutional collaborative venture to develop the next generation of open-
access texts to improve postsecondary education at all levels of higher learning by developing an Open Access Resource
environment. The project currently consists of 14 independently operating and interconnected libraries that are constantly being
optimized by students, faculty, and outside experts to supplant conventional paper-based books. These free textbook alternatives are
organized within a central environment that is both vertically (from advance to basic level) and horizontally (across different fields)
integrated.
The LibreTexts libraries are Powered by MindTouch® and are supported by the Department of Education Open Textbook Pilot
Project, the UC Davis Office of the Provost, the UC Davis Library, the California State University Affordable Learning Solutions
Program, and Merlot. This material is based upon work supported by the National Science Foundation under Grant No. 1246120,
1525057, and 1413739. Unless otherwise noted, LibreTexts content is licensed by CC BY-NC-SA 3.0.
Any opinions, findings, and conclusions or recommendations expressed in this material are those of the author(s) and do not
necessarily reflect the views of the National Science Foundation nor the US Department of Education.
Have questions or comments? For information about adoptions or adaptions contact [email protected]. More information on our
activities can be found via Facebook (https://facebook.com/Libretexts), Twitter (https://twitter.com/libretexts), or our blog
(http://Blog.Libretexts.org).

This text was compiled on 04/09/2023


TABLE OF CONTENTS
Licensing

1: Introduction to Classical Mechanics


1.1: Introduction

2: Units, Dimensional Analysis, Problem Solving, and Estimation


2.1: The Speed of light
2.2: International System of Units
2.3: Dimensions of Commonly Encountered Quantities
2.4: Order of Magnitude Estimates - Fermi Problems

3: Vectors
3.1: Vector Analysis
3.2: Coordinate Systems
3.3: Vectors
3.4: Vector Product (Cross Product)

4: One Dimensional Kinematics


4.1: Introduction to One Dimensional Kinematics
4.2: Position, Time Interval, and Displacement
4.3: Velocity
4.4: Acceleration
4.5: Constant Acceleration
4.6: One Dimensional Kinematics and Integration

5: Two Dimensional Kinematics


5.1: Introduction to the Vector Description of Motion in Two Dimensions
5.2: Projectile Motion

6: Circular Motion
6.1: Introduction to Circular Motion
6.2: Circular Motion- Velocity and Angular Velocity
6.3: Circular Motion- Tangential and Radial Acceleration
6.4: Period and Frequency for Uniform Circular Motion
6.5: Angular Velocity and Angular Acceleration
6.6: Non-circular Central Motion

7: Newton’s Laws of Motion


7.1: Force and Quantity of Matter
7.2: Newton’s First Law
7.3: Momentum, Newton’s Second Law and Third Law
7.4: Newton’s Third Law- Action-Reaction Pairs

1 https://phys.libretexts.org/@go/page/28239
8: Applications of Newton’s Second Law
8.1: Force Laws
8.2: Fundamental Laws of Nature
8.3: Constraint Forces
8.4: Free-body Force Diagram
8.5: Tension in a Rope
8.6: Drag Forces in Fluids
8.7: Worked Examples

9: Circular Motion Dynamics


9.1: Introduction Newton’s Second Law and Circular Motion
9.2: Universal Law of Gravitation and the Circular Orbit of the Moon
9.3: Worked Examples Circular Motion
9.4: Appendix 9A The Gravitational Field of a Spherical Shell of Matter .

10: Momentum, System of Particles, and Conservation of Momentum


10.1: Introduction
10.2: Momentum (Quantity of Motion) and Average Impulse
10.3: External and Internal Forces and the Change in Momentum of a System
10.4: System of Particles
10.5: Center of Mass
10.6: Translational Motion of the Center of Mass
10.7: Constancy of Momentum and Isolated Systems
10.8: Momentum Changes and Non-isolated Systems
10.9: Worked Examples

11: Reference Frames


11.1: Introduction to Reference Frames
11.2: Galilean Coordinate Transformations
11.3: Law of Addition of Velocities - Newtonian Mechanics
11.4: Worked Examples

12: Momentum and the Flow of Mass


12.1: Introduction to Momentum and the Flow of Mass
12.2: Worked Examples
12.3: Rocket Propulsion

13: Energy, Kinetic Energy, and Work


13.1: The Concept of Energy and Conservation of Energy
13.2: Kinetic Energy
13.3: Kinematics and Kinetic Energy in One Dimension
13.4: Work done by Constant Forces
13.5: Work done by Non-Constant Forces
13.6: Work-Kinetic Energy Theorem
13.7: Power Applied by a Constant Force
13.8: Work and the Scalar Product
13.9: Work done by a Non-Constant Force Along an Arbitrary Path
13.10: Worked Examples
13.11: Work-Kinetic Energy Theorem in Three Dimensions

2 https://phys.libretexts.org/@go/page/28239
13.12: Appendix 13A Work Done on a System of Two Particles

14: Potential Energy and Conservation of Energy


14.1: Conservation of Energy
14.2: Conservative and Non-Conservative Forces
14.3: Changes in Potential Energies of a System
14.4: Change in Potential Energy and Zero Point for Potential Energy
14.5: Mechanical Energy and Conservation of Mechanical Energy
14.6: Spring Force Energy Diagram
14.7: Change of Mechanical Energy for Closed System with Internal Nonconservative Forces
14.8: Dissipative Forces- Friction
14.9: Worked Examples

15: Collision Theory


15.1: Introduction to Collision Theory
15.2: Reference Frames and Relative Velocities
15.3: Characterizing Collisions
15.4: One-Dimensional Collisions Between Two Objects
15.5: Worked Examples
15.6: Two Dimensional Elastic Collisions
15.7: Two-Dimensional Collisions in Center-of-Mass Reference Frame

16: Two Dimensional Rotational Kinematics


16.1: Introduction
16.2: Fixed Axis Rotation- Rotational Kinematics
16.3: Rotational Kinetic Energy and Moment of Inertia
16.4: Conservation of Energy for Fixed Axis Rotation
16.5: Appendix 16A- Proof of the Parallel Axis Theorem

17: Two-Dimensional Rotational Dynamics


17.1: Introduction to Two-Dimensional Rotational Dynamics
17.2: Vector Product (Cross Product)
17.3: Torque
17.4: Torque, Angular Acceleration, and Moment of Inertia
17.5: Torque and Rotational Work

18: Static Equilibrium


18.1: Introduction Static Equilibrium
18.2: Lever Law
18.3: Generalized Lever Law
18.4: Worked Examples
18.5: Appendix 18A The Torques About any Two Points are Equal for a Body in Static Equilibrium

19: Angular Momentum


19.1: Introduction
19.2: Angular Momentum about a Point for a Particle
19.3: Torque and the Time Derivative of Angular Momentum about a Point for a Particle
19.4: Conservation of Angular Momentum about a Point
19.5: Angular Impulse and Change in Angular Momentum

3 https://phys.libretexts.org/@go/page/28239
19.6: Angular Momentum of a System of Particles
19.7: Angular Momentum and Torque for Fixed Axis Rotation
19.8: Principle of Conservation of Angular Momentum
19.9: External Angular Impulse and Change in Angular Momentum

20: Rigid Body Kinematics About a Fixed Axis


20.1: Introduction
20.2: Constrained Motion - Translation and Rotation
20.3: Angular Momentum for a System of Particles Undergoing Translational and Rotational
20.4: Kinetic Energy of a System of Particles
20.5: Rotational Kinetic Energy for a Rigid Body Undergoing Fixed Axis Rotation
20.6: Appendix 20A Chasles’s Theorem- Rotation and Translation of a Rigid Body

21: Rigid Body Dynamics About a Fixed Axis


21.1: Introduction to Rigid Body Dynamics
21.2: Translational Equation of Motion
21.3: Translational and Rotational Equations of Motion
21.4: Translation and Rotation of a Rigid Body Undergoing Fixed Axis Rotation
21.5: Work-Energy Theorem
21.6: Worked Examples

22: Three Dimensional Rotations and Gyroscopes


22.1: Introduction to Three Dimensional Rotations
22.2: Gyroscope
22.3: Why Does a Gyroscope Precess?
22.4: Worked Examples

23: Simple Harmonic Motion


23.1: Introduction to Periodic Motion
23.2: Simple Harmonic Motion- Analytic
23.3: Energy and the Simple Harmonic Oscillator
23.4: Worked Examples
23.5: Damped Oscillatory Motion
23.6: Forced Damped Oscillator
23.7: Small Oscillations
23.8: Appendix 23A- Solution to Simple Harmonic Oscillator Equation
23.9: Appendix 23B - Complex Numbers
23.10: Solution to the Underdamped Simple Harmonic Oscillator
23.11: Solution to the Forced Damped Oscillator Equation

24: Physical Pendulums


24.1: Introduction to Physical Pendulums
24.2: Physical Pendulum
24.3: Worked Examples
24.4: Appendix 24A Higher-Order Corrections to the Period for Larger Amplitudes of a Simple Pendulum

25: Celestial Mechanics


25.1: Introduction- The Kepler Problem
25.2: Planetary Orbits

4 https://phys.libretexts.org/@go/page/28239
25.3: Energy and Angular Momentum, Constants of the Motion
25.4: Energy Diagram, Effective Potential Energy, and Orbits
25.5: Orbits of the Two Bodies
25.6: Kepler’s Laws
25.7: Worked Examples
25.8: Appendix 25A Derivation of the Orbit Equation
25.9: Appendix 25B Properties of an Elliptical Orbit
25.10: Appendix 25C Analytic Geometric Properties of Ellipses

26: Elastic Properties of Materials


26.1: Introduction
26.2: Stress and Strain in Tension and Compression
26.3: Shear Stress and Strain
26.4: Elastic and Plastic Deformation

27: Static Fluids


27.1: Introduction to Static Fluids
27.2: Density
27.3: Pressure in a Fluid
27.4: Pascal’s Law - Pressure as a Function of Depth in a Fluid of Uniform Density in a Uniform Gravitational Field
27.5: Compressibility of a Fluid
27.6: Archimedes’ Principle - Buoyant Force

28: Fluid Dynamics


28.1: Ideal Fluids
28.2: Velocity Vector Field
28.3: Mass Continuity Equation
28.4: Bernoulli’s Principle
28.5: Worked Examples- Bernoulli’s Equation
28.6: Laminar and Turbulent Flow

29: Kinetic Theory of Gases


29.1: Introduction- Gases
29.2: Temperature and Thermal Equilibrium
29.3: Internal Energy of a Gas
29.4: Ideal Gas
29.5: Atmosphere

Index
Glossary
Detailed Licensing

5 https://phys.libretexts.org/@go/page/28239
Licensing
A detailed breakdown of this resource's licensing can be found in Back Matter/Detailed Licensing.

1 https://phys.libretexts.org/@go/page/65339
CHAPTER OVERVIEW
1: Introduction to Classical Mechanics
1.1: Introduction

This page titled 1: Introduction to Classical Mechanics is shared under a CC BY-NC-SA 4.0 license and was authored, remixed, and/or curated by
Peter Dourmashkin (MIT OpenCourseWare) via source content that was edited to the style and standards of the LibreTexts platform; a detailed
edit history is available upon request.

1.1 https://phys.libretexts.org/@go/page/24423
1.1: Introduction
Classical mechanics is the mathematical science that studies the displacement of bodies under the action of forces. Gailieo Galilee
initiated the modern era of mechanics by using mathematics to describe the motion of bodies. His Mechanics, published in 1623,
introduced the concepts of force and described the constant accelerated motion of objects near the surface of the Earth. Sixty years
later Isaac Newton formulated his Laws of Motion, which he published in 1687 under the title, Philosophiae Naturalis Principia
Mathematica (Mathematical Principles of Natural Philosophy). In the third book, subtitled De mundi systemate (On the system of
the world), Newton solved the greatest scientific problem of his time by applying his Universal Law of Gravitation to determine the
motion of planets. Newton established a mathematical approach to the analysis of physical phenomena in which he stated that it
was unnecessary to introduce final causes (hypothesis) that have no experimental basis, “Hypotheses non fingo (I frame no
hypotheses), but that physical models are built from experimental observations and then made general by induction. This led to a
great century of applications of the principles of Newtonian mechanics to many new problems culminating in the work of Leonhard
Euler. Euler began a systematic study of the three dimensional motion of rigid bodies, leading to a set of dynamical equations now
known as Euler’s equations of motion.
Alongside this development and refinement of the concept of force and its application to the description of motion, the concept of
energy slowly emerged, culminating in the middle of the nineteenth century in the discovery of the principle of conservation of
energy and its immediate applications to the laws of thermodynamics. Conservation principles are now central to our study of
mechanics; the conservation of momentum, energy, and angular momentum enabled a new reformulation of classical mechanics.
During this period, the experimental methodology and mathematical tools of Newtonian mechanics were applied to other non-rigid
systems of particles leading to the development of continuum mechanics. The theories of fluid mechanics, wave mechanics, and
electromagnetism emerged leading to the development of the wave theory of light. However there were many perplexing aspects of
the wave theory of light, for example, does light propagate through a medium, the “ether”? A series of optics experiments,
culminating in the Michelson-Morley experiment in 1887 ruled out the hypothesis of a stationary medium. Many attempts were
made to reconcile the experimental evidence with classical mechanics but the challenges were more fundamental. The basics
concepts of absolute time and absolute space, which Newton had defined in the Principia, were themselves inadequate to explain a
host of experimental observations. Albert Einstein, by insisting on a fundamental rethinking of the concepts of space and time, and
the relativity of motion, in his special theory of relativity (1905) was able to resolve the apparent conflicts between optics and
Newtonian mechanics. In particular, special relativity provides the necessary framework for describing the motion of rapidly
moving objects (speed greater than v > 0.1 c ).
A second limitation on the validity of Newtonian mechanics appeared at the microscopic length scale. A new theory, statistical
mechanics, was developed relating the microscopic properties of individual atoms and molecules to the macroscopic or bulk
thermodynamic properties of materials. Started in the middle of the nineteenth century, new observations at very small scales
revealed anomalies in the predicted behavior of gases (heat capacity). It became increasingly clear that classical mechanics did not
adequately explain a wide range of newly discovered phenomena at the atomic and sub-atomic length scales. An essential
realization was that the language of classical mechanics was not even adequate to qualitatively describe certain microscopic
phenomena. By the early part of the twentieth century, quantum mechanics provided a mathematical description of microscopic
phenomena in complete agreement with our empirical knowledge of all non-relativistic phenomena.
In the twentieth century, as experimental observations led to a more detailed knowledge of the large-scale properties of the
universe, Newton’s Universal Law of Gravitation no longer accurately modeled the observed universe and needed to be replaced by
general relativity. By the end of the twentieth century and beginning of the twenty-first century, many new observations, for
example the accelerated expansion of the Universe, have required introduction of new concepts like dark energy that may lead once
again to a fundamental rethinking of the basic concepts of physics in order to explain observed phenomena.

This page titled 1.1: Introduction is shared under a not declared license and was authored, remixed, and/or curated by Peter Dourmashkin (MIT
OpenCourseWare) via source content that was edited to the style and standards of the LibreTexts platform; a detailed edit history is available upon
request.

1.1.1 https://phys.libretexts.org/@go/page/24417
CHAPTER OVERVIEW
2: Units, Dimensional Analysis, Problem Solving, and Estimation
But we must not forget that all things in the world are connected with one another and depend on one another, and that we
ourselves and all our thoughts are also a part of nature. It is utterly beyond our power to measure the changes of things by time.
Quite the contrary, time is an abstraction, at which we arrive by means of the change of things; made because we are not restricted
to any one definite measure, all being interconnected. A motion is termed uniform in which equal increments of space described
correspond to equal increments of space described by some motion with which we form a comparison, as the rotation of the earth.
A motion may, with respect to another motion, be uniform. But the question whether a motion is in itself uniform, is senseless.
With just as little justice, also, may we speak of an “absolute time” --- of a time independent of change. This absolute time can be
measured by comparison with no motion; it has therefore neither a practical nor a scientific value; and no one is justified in saying
that he knows aught about it. It is an idle metaphysical conception [1]- Ernst Mach
2.1: The Speed of light
2.2: International System of Units
2.3: Dimensions of Commonly Encountered Quantities
2.4: Order of Magnitude Estimates - Fermi Problems

[1] E. Mach, The Science of Mechanics, translated by Thomas J. McCormack, Open Court Publishing Company, La Salle, Illinois,
1960, p. 273.

This page titled 2: Units, Dimensional Analysis, Problem Solving, and Estimation is shared under a CC BY-NC-SA 4.0 license and was authored,
remixed, and/or curated by Peter Dourmashkin (MIT OpenCourseWare) via source content that was edited to the style and standards of the
LibreTexts platform; a detailed edit history is available upon request.

1
2.1: The Speed of light
When we observe and measure phenomena in the world, we try to assign numbers to the physical quantities with as much accuracy
as we can possibly obtain from our measuring equipment. For example, we may want to determine the speed of light, which we can
calculate by dividing the distance a known ray of light propagates over its travel time,
 distance 
 speed of light  =
 time. 

In 1983 the General Conference on Weights and Measures defined the speed of light to be

c  = 299, 792, 458 meters/second.

This number was chosen to correspond to the most accurately measured value of the speed of light and is well within the
experimental uncertainty.

This page titled 2.1: The Speed of light is shared under a CC BY-NC-SA 4.0 license and was authored, remixed, and/or curated by Peter
Dourmashkin (MIT OpenCourseWare) via source content that was edited to the style and standards of the LibreTexts platform; a detailed edit
history is available upon request.

2.1.1 https://phys.libretexts.org/@go/page/24424
2.2: International System of Units
The system of units most commonly used throughout science and technology today is the Système International (SI). It consists of
seven base quantities and their corresponding base units, shown in Table 2.2.1.
Table 2.2.1 : International System of Units
Base Quantity Base Unit

Length meter (m)

Mass kilogram (kg)

Time second (s)

Electric Current ampere (A)

Temperature kelvin (K)

Amount of Substance mole (mol)

Luminous Intensity candela (cd)

We shall refer to the dimension of the base quantity by the quantity itself, for example

 dim length  ≡  length  ≡ L,  dim mass  ≡  mass  ≡ M,  dim time  ≡  time  ≡ T.

Mechanics is based on just the first three of these quantities, the MKS or meter- kilogram-second system. An alternative metric
system, still widely used, is the CGS system (centimeter-gram-second).

Standard Mass
The unit of mass, the kilogram (kg), remains the only base unit in the International System of Units (SI) that is still defined in terms
of a physical artifact, known as the “International Prototype of the Standard Kilogram.” George Matthey (of Johnson Matthey)
made the prototype in 1879 in the form of a cylinder, 39 mm high and 39 mm in diameter, consisting of an alloy of 90 % platinum
and 10 % iridium. The international prototype is kept in the Bureau International des Poids et Mèsures (BIPM) at Sevres, France,
under conditions specified by the 1st Conférence Générale des Poids et Mèsures (CGPM) in 1889 when it sanctioned the prototype
and declared “This prototype shall henceforth be considered to be the unit of mass.” It is stored at atmospheric pressure in a
specially designed triple bell-jar. The prototype is kept in a vault with six official copies.
The 3rd Conférence Générale des Poids et Mèsures CGPM (1901), in a declaration intended to end the ambiguity in popular usage
concerning the word “weight” confirmed that:
The kilogram is the unit of mass; it is equal to the mass of the international prototype of the kilogram.
There is a stainless steel one-kilogram standard that is used for comparisons with standard masses in other laboratories. In practice
it is more common to quote a conventional mass value (or weight-in-air, as measured with the effect of buoyancy), than the
standard mass. Standard mass is normally only used in specialized measurements wherever suitable copies of the prototype are
stored.

Example 2.2.1: The International Prototype Kilogram

In order to minimize the effects of corrosion, the platinum-iridium prototype kilogram is a right cylinder with dimensions
chosen to minimize the surface area for a given fixed volume. The standard kilogram is an alloy of 90 % platinum and 10 %
iridium. The density of the alloy is ρ = 21.56g ⋅ cm . Based on this information,
−3

i. determine the radius of the prototype kilogram, and


ii. the ratio of the radius to the height.
Solution
The volume for a cylinder of radius r and height h is given by
2
V = πr h

2.2.1 https://phys.libretexts.org/@go/page/24425
The surface area can be expressed as a function of the radius r and the constant volume V according to

2 2
2V
A = 2π r + 2πrh = 2π r +
r

To find the smallest surface area for a fixed volume, minimize the surface area with respect to the radius by setting
dA 2V
0 = = 4πr −
2
dr r

which we can solve for the radius


1/3
V
r =( )

Because we also know that V = πr h


2
, we can rewrite Equation (2.2.5) as
2
πr h
3
r =

which implies that ratio of the radius to the height is


r 1
=
h 2

The standard kilogram is an alloy of 90% platinum and 10% iridium. The density of platinum is 21.45g ⋅ cm −3
and the density
of iridium is 22.55g ⋅ cm . Thus the density of the standard kilogram is
−3

−3 −3 −3
ρ = (0.90) (21.45g ⋅ cm ) + (0.10) (22.55g ⋅ cm ) = 21.56g ⋅ cm

and its volume is


−3 3
V = m/ρ = (1000g)/ (21.56g ⋅ cm ) = 46.38 cm

For the standard mass, the radius is


1/3 3 1/3
V 46.38cm
r =( ) =( ) ≅1.95cm
2π 2π

Because the prototype kilogram is an artifact, there are some intrinsic problems associated with its use as a standard. It may be
damaged, or destroyed. The prototype gains atoms due to environment wear and cleaning, at a rate of change of mass
corresponding to approximately 1μg/ year , (1μg ≡ 1 microgram  ≡ 1 × 10 g) . −6

Several new approaches to defining the SI unit of mass [kg] are currently being explored. One possibility is to define the
kilogram as a fixed number of atoms of a particular substance, thus relating the kilogram to an atomic mass. Silicon is a good
candidate for this approach because it can be grown as a large single crystal, in a very pure form.

Example 2.2.2: Mass of a Silicon Crystal

A given standard unit cell of silicon has a volume V and contains N atoms. The number of molecules in a given mole of
0 0

substance is given by Avogadro’s constant N = 6.02214129(27) × 10 mol . The molar mass of silicon is given by M .
A
23 −1
mol

Find the mass m of a volume V in terms of V , N , V , M , and N .


0 0 mol A

Solution
The mass m of the unit cell is the density rho of the silicon cell multiplied by the volume of the cell V ,
0 0

m0 = ρV0

The number of moles in the unit cell is the total mass, m , of the cell, divided by the molar mass M
0 mol

n0 = m0 / Mmol = ρV0 / Mmol

The number of atoms in the unit cell is the number of moles times the Avogadro constant, N , A

2.2.2 https://phys.libretexts.org/@go/page/24425
ρV0 NA
N0 = n0 NA =
Mmol

The density of the crystal is related to the mass m of the crystal divided by the volume V of the crystal,
m
ρ =
V

The number of atoms in the unit cell can be expressed as


mV0 NA
N0 =
V Mmol

The mass of the crystal is


Mmol  V
m = N0
NA V0

The molar mass, unit cell volume and volume of the crystal can all be measured directly. Notice that M /N is the mass of a
mol A

single atom, and (V /V ) N is the number of atoms in the volume. This accuracy of the approach depends on how accurate
0 0

the Avogadro constant can be measured. Currently, the measurement of the Avogadro constant has a relative uncertainty of 1
part in 10 , which is equivalent to the uncertainty in the present definition of the kilogram.
8

Atomic Clock and the Definition of the Second


Isaac Newton, in the Philosophiae Naturalis Principia Mathematica (“Mathematical Principles of Natural Philosophy”),
distinguished between time as duration and an absolute concept of time,
“Absolute true and mathematical time, of itself and from its own nature, flows equably without relation to anything external,
and by another name is called duration: relative, apparent, and common time, is some sensible and external (whether
accurate or unequable) measure of duration by means of motion, which is commonly used instead of true time; such as an
hour, a day, a month, a year. ”
The development of clocks based on atomic oscillations allowed measures of timing with accuracy on the order of 1 part in 10 , 14

corresponding to errors of less than one microsecond (one millionth of a second) per year. Given the incredible accuracy of this
measurement, and clear evidence that the best available timekeepers were atomic in nature, the second [s] was redefined in 1967 by
the International Committee on Weights and Measures as a certain number of cycles of electromagnetic radiation emitted by
cesium atoms as they make transitions between two designated quantum states:
The second is the duration of 9,192,631,770 periods of the radiation corresponding to the transition between the two hyperfine
levels of the ground state of the cesium 133 atom.

Meter
The meter [m] was originally defined as 1/10, 000, 000of the arc from the Equator to the North Pole along the meridian passing
through Paris. To aid in calibration and ease of comparison, the meter was redefined in terms of a length scale etched into a
platinum bar preserved near Paris. Once laser light was engineered, the meter was redefined by the 17th Conférence Générale des
Poids et Mèsures (CGPM) in 1983 to be a certain number of wavelengths of a particular monochromatic laser beam.
The meter is the length of the path traveled by light in vacuum during a time interval of 1/299 792 458 of a second

Example 2.2.1: Light-Year

Astronomical distances are sometimes described in terms of light-years [ly]. A light-year is the distance that light will travel in
one year [yr]. How far in meters does light travel in one year?
Solution
Using the relationship (distance) = ( speed of light ) ⋅ ( time ) , one light year corresponds to a distance. Because the speed
of light is given in terms of meters per second, we need to know how many seconds are in a year. We can accomplish this by
converting units. We know that

1 year = 365.25 days, 1 day = 24 hours, 1 hour = 60 minutes, 1 minute = 60 seconds

2.2.3 https://phys.libretexts.org/@go/page/24425
Putting this together we find that the number of seconds in a year is
24 hours  60 min  60s
 1 year  = (365.25 day ) ( )( )( ) = 31, 557, 600 s 
1 day  1 hour  1 min 

The distance that light travels in a one year is


299, 792, 458m 31, 557, 600s
15
1ly = ( )( ) (1yr) = 9.461 × 10 m
1s 1yr

The distance to the nearest star, a faint red dwarf star, Proxima Centauri, is 4.24 ly.

Radians
Consider the triangle drawn in Figure 2.2.1. The basic trigonometric functions of an angle θ in a right-angled triangle ONB are
sin(θ) = y/r , cos(θ) = x/r , and tan(θ) = y/x .

Figure 2.2.1 : Trigonometric relations. (CC BY-NC; Ümit Kaya)


It is very important to become familiar with using the measure of the angle θ itself as expressed in radians [rad]. Let θ be the angle
between two straight lines OX and OP . Draw a circle of radius r centered at O. The lines OP and OX cut the circle at the points
A and B where OA = OB = r . Denote the length of the arc AB by s , then the radian measure of θ is given by

θ = s/r

and the ratio is the same for circles of any radii centered at O -- just as the ratios y/r and y/x are the same for all right triangles
with the angle θ at O. As θ approaches 360 , s approaches the complete circumference 2πr of the circle, so that 360 = 2π rad.
∘ ∘

Figure 2.2: Radians compared to trigonometric functions. (CC BY-NC; Ümit Kaya)
Let’s compare the behavior of sin(θ) ,tan(θ) and θ itself for small angles. One can see from Figure 2.1 that s/r > y/r. It is less
obvious that y/x > θ . It is very instructive to plot sin(θ) , tan(θ) , and theta as functions of θ [rad] between 0 and π/2 on the
same graph (see Figure 2.2). For small θ , the values of all three functions are almost equal. But how small is “small”? An
acceptable condition is for θ << 1 in radians.
We can show this with a few examples. Recall that 360 = 2π rad, 57.3 = 1 rad , so an angle 6 ≅(6 ) (2π rad /360 ) ≅0.1
∘ ∘ ∘ ∘ ∘

rad when expressed in radians. In Table 2.2.2 we compare the value of θ (measured in radians) with sin(θ) ,tan(θ) , (θ − sin θ)/θ ,

2.2.4 https://phys.libretexts.org/@go/page/24425
and (θ − tan θ)/θ , for θ = 0.1 rad, 0.2 rad, 0.5 rad , and 1.0 rad.
Table 2.2.2 : Small Angle Approximation
θ [rad] θ [deg] sin(θ ) tan(θ ) (θ − sin θ)/θ (θ − tan θ)/θ

0.1 5.72958 0.09983 0.10033 0.00167 -0.00335

0.2 11.45916 0.19867 0.20271 0.00665 -0.01355

0.5 28.64789 0.47943 0.54630 0.54630 -0.09260

1.0 57.29578 0.84147 1.55741 0.15853 -0.55741

The values for (θ − sin θ)/θ , and (θ − tan θ)/θ , for θ = 0.2 rad are less than ±1.4 . Provided that θ is not too large, the
approximation that

sin(θ) ≃ tan(θ) ≃ θ

called the small angle approximation, can be used almost interchangeably, within some small percentage error. This is the basis of
many useful approximations in physics calculations.

Example 2.4: Parsec


A standard astronomical unit is the parsec. Consider two objects that are separated by a distance of one astronomical unit,
1AU = 1.50 × 10
11
m, which is the mean distance between the earth and sun. (One astronomical unit is roughly equivalent to
eight light minutes, 1AU = 8.3light-minutes). One parsec is the distance at which one astronomical unit subtends an angle
θ = 1 arcsecond  = (1/3600) degree. Suppose is a spacecraft is located in a space a distance 1 parsec from the Sun as shown

in Figure 2.2.3. How far is the spacecraft in terms of light years and meters?

Figure 2.2.3 : Definition of the parsec (CC BY-NC; Ümit Kaya)


Solution
Because one arc second corresponds to a very small angle, one parsec is therefore equal to distance divided by angle, hence
11
(1AU) 1.50 × 10 m
5 16
1pc = = (2.06 × 10 AU) ( ) = 3.09 × 10 m
(1/3600) 1AU

16
11y
= (3.09 × 10 m) ( ) = 3.26 ly
15
9.46 × 10 m

Steradians
The steradian [sr] is the unit of solid angle that, having its vertex in the center of a sphere, cuts off an area of the surface of the
sphere equal to that of a square with sides of length equal to the radius of the sphere. The conventional symbol for steradian
measure is Ω, the uppercase Greek letter “Omega.” The total solid angle Ω of a sphere is then found by dividing the surface area
sph

of the sphere by the square of the radius,


2 2
Ωsph = 4π r / r = 4π

2.2.5 https://phys.libretexts.org/@go/page/24425
This result is independent of the radius of the sphere.

Radiant Intensity
“The SI unit, candela, is the luminous intensity of a source that emits monochromatic radiation of frequency 540 × 10 12 −1
s , in a
given direction, and that has a radiant intensity in that direction of 1/683 watts per steradian.”
Note that "in a given direction" cannot be taken too literally. The intensity is measured per steradian of spread, so if the radiation
has no spread of directions, the luminous intensity would be infinite.

This page titled 2.2: International System of Units is shared under a CC BY-NC-SA 4.0 license and was authored, remixed, and/or curated by
Peter Dourmashkin (MIT OpenCourseWare) via source content that was edited to the style and standards of the LibreTexts platform; a detailed
edit history is available upon request.

2.2.6 https://phys.libretexts.org/@go/page/24425
2.3: Dimensions of Commonly Encountered Quantities
Many physical quantities are derived from the base quantities by a set of algebraic relations defining the physical relation between
these quantities. The dimension of the derived quantity is written as a power of the dimensions of the base quantities. For example
velocity is a derived quantity and the dimension is given by the relationship
−1
 dim velocity  = ( length )/( time ) = L ⋅ T

where L ≡  length , T ≡  time  . Force is also a derived quantity and has dimension
( mass )( dim velocity )
dim force =
( time )

where M ≡ mass . We can also express force in terms of mass, length, and time by the relationship
( mass )( length )
−2
dim force  = = M ⋅L⋅T
2
( time )

The derived dimension of kinetic energy is


2
dim kinetic energy = ( mass )(dim velocity ) ,

which in terms of mass, length, and time is


2
( mass )( length )
2 −2
 dim kinetic energy  = =M ⋅L ⋅T
2
( time )

The derived dimension of work is

dim work = ( dim force )( length ),

which in terms of our fundamental dimensions is


2
( mass )( length )
2 −2
 dim work = =M ⋅L ⋅T
2
( time )

So work and kinetic energy have the same dimensions. Power is defined to be the rate of change in time of work so the dimensions
are
2
 dim work   (dim force) (length)  ( mass )( length )
2 −3
dim power  = = = =M ⋅L ⋅T
3
 time   time  ( time )

In Table 2.3 we include the derived dimensions of some common mechanical quantities in terms of mass, length, and time.

Dimensional Analysis
There are many phenomena in nature that can be explained by simple relationships between the observed phenomena.

Table 2.3 Dimensions of Some Common Mechanical Quantities


M ≡  mass , L ≡  length, T ≡  time 

Quantity Dimension MKS unit

Angle dimensionless Dimensionless = radian


Solid Angle dimensionless Dimensionless = sterradian
Area L
2
m
2

Volume L
3
m
3

2.3.1 https://phys.libretexts.org/@go/page/24426
Quantity Dimension MKS unit

Frequency T
−1
s
−1
= hertz = Hz

Velocity L ⋅T
−1
m⋅s
−1

Acceleration L ⋅T
−2
m⋅s
−2

Angular Velocity T
−1
rad ⋅ s
−1

Angular Acceleration T
−2
rad ⋅ s
−2

Density M ⋅L
−3
kg ⋅ m
−3

Momentum M ⋅L ⋅T
−1
kg ⋅ m ⋅ s
−1

Angular Momentum M ⋅L
2
⋅T
−1
kg ⋅ m
2
⋅s
−1

Force M ⋅L ⋅T
−2
kg ⋅ m ⋅ s
−2
= newton  = N

Work, Energy M ⋅L
2
⋅T
−2
kg ⋅ m
2
⋅s
−2
= joule = J

Torque M ⋅L
2
⋅T
−2
kg ⋅ m
2
⋅s
−2

Power M ⋅L
2
⋅T
−3
kg ⋅ m
2
⋅s
−3
= watt = W

Pressure M ⋅L
−1
⋅T
−2
kg ⋅ m
−1
⋅s
−2
= pascal = Pa

Example 2.5 Period of a Pendulum

Consider a simple pendulum consisting of a massive bob suspended from a fixed point by a string. Let T denote the time
interval (period of the pendulum) that it takes the bob to complete one cycle of oscillation. How does the period of the simple
pendulum depend on the quantities that define the pendulum and the quantities that determine the motion?
Solution
What possible quantities are involved? The length of the pendulum l, the mass of the pendulum bob \{\text{m}\), the
gravitational acceleration g, and the angular amplitude of the bob θ are all possible quantities that may enter into a
0

relationship for the period of the swing. Have we included every possible quantity? We can never be sure but let’s first work
with this set and if we need more than we will have to think harder! Our problem is then to find a function f such that

T = f (l, m, g, θ0 )

We first make a list of the dimensions of our quantities as shown in Table 2.4.

Name of Quantity Symbol Dimensional Formula

Time of swing t T

Length of pendulum l L

Mass of pendulum m M

Gravitational acceleration g L⋅T


−2

Angular amplitude of swing θ0 No dimension

Our first observation is that the mass of the bob cannot enter into our relationship, as our final quantity has no dimensions of
mass and no other quantity has dimensions of mass. Let’s focus on the length of the string and the gravitational acceleration. In

2.3.2 https://phys.libretexts.org/@go/page/24426
order to eliminate length, these quantities must divide each other when appearing in some functional relation for the period T
If we choose the combination l/g , the dimensions are
 length 
2
dim[l/g] = = ( time )
2
 length / (time) 

It appears that the time of swing may proportional to the square root of this ratio. Thus we have a candidate formula
1/2
l
T ∼( )
g

in the above expression, the symbol “∼” represents a proportionality, not an approximation). Because the angular amplitude θ 0

is dimensionless, it may or may not appear. We can account for this by introducing some function y (θ ) into our relationship,
0

which is beyond the limits of this type of analysis. The period is then
1/2
l
T = y (θ0 ) ( )
g

We shall discover later on that y (θ0) is nearly independent of the angular amplitude θ for very small amplitudes and is equal
0

to y (θ ) = 2π ,
0

1/2
l
T = 2π( )
g

This page titled 2.3: Dimensions of Commonly Encountered Quantities is shared under a CC BY-NC-SA 4.0 license and was authored, remixed,
and/or curated by Peter Dourmashkin (MIT OpenCourseWare) via source content that was edited to the style and standards of the LibreTexts
platform; a detailed edit history is available upon request.

2.3.3 https://phys.libretexts.org/@go/page/24426
2.4: Order of Magnitude Estimates - Fermi Problems
Counting is the first mathematical skill we learn. We came to use this skill by distinguishing elements into groups of similar
objects, but counting becomes problematic when our desired objects are not easily identified, or there are too many to count. Rather
than spending a huge amount of effort to attempt an exact count, we can try to estimate the number of objects. For example, we can
try to estimate the total number of grains of sand contained in a bucket of sand. Because we can see individual grains of sand, we
expect the number to be very large but finite. Sometimes we can try to estimate a number, which we are fairly sure but not certain
is finite, such as the number of particles in the universe.
We can also assign numbers to quantities that carry dimensions, such as mass, length, time, or charge, which may be difficult to
measure exactly. We may be interested in estimating the mass of the air inside a room, or the length of telephone wire in the United
States, or the amount of time that we have slept in our lives. We choose some set of units, such as kilograms, miles, hours, and
coulombs, and then we can attempt to estimate the number with respect to our standard quantity.
Often we are interested in estimating quantities such as speed, force, energy, or power. We may want to estimate our natural
walking speed, or the force of wind acting against a bicycle rider, or the total energy consumption of a country, or the electrical
power necessary to operate a university. All of these quantities have no exact, well- defined value; they instead lie within some
range of values.
When we make these types of estimates, we should be satisfied if our estimate is reasonably close to the middle of the range of
possible values. But what does “reasonably close” mean? Once again, this depends on what quantities we are estimating. If we are
describing a quantity that has a very large number associated with it, then an estimate within an order of magnitude should be
satisfactory. The number of molecules in a breath
of air is close to 10 ; an estimate anywhere between 10 and 10 molecules is close enough. If we are trying to win a contest by
22 21 23

estimating the number of marbles in a glass container, we cannot be so imprecise; we must hope that our estimate is within 1% of
the real quantity. These types of estimations are called Fermi problems. The technique is named after the physicist Enrico Fermi,
who was famous for making these sorts of “back of the envelope” calculations.

Methodology for Estimation Problems


Estimating is a skill that improves with practice. Here are two guiding principles that may help you get started.
1. You must identify a set of quantities that can be estimated or calculated.
2. You must establish an approximate or exact relationship between these quantities and the quantity to be estimated in the
problem.
Estimations may be characterized by a precise relationship between an estimated quantity and the quantity of interest in the
problem. When we estimate, we are drawing upon what we know. But different people are more familiar with certain things than
others. If you are basing your estimate on a fact that you already know, the accuracy of your estimate will depend on the accuracy
of your previous knowledge. When there is no precise relationship between estimated quantities and the quantity to be estimated in
the problem, then the accuracy of the result will depend on the type of relationships you decide upon. There are often many
approaches to an estimation problem leading to a reasonably accurate estimate. So use your creativity and imagination!

Example 2.6: Lining Up Pennies


Suppose you want to line pennies up, diameter to diameter until the total length is 1 kilometer. How many pennies will you
need? How accurate is this estimation?
Solution
The first step is to consider what type of quantity is being estimated. In this example, we are estimating a dimensionless scalar
quantity, the number of pennies. We can now give a precise relationship for the number of pennies needed to mark off 1
kilometer
 total distance 
# of pennies  =
 diameter of penny 

We can estimate a penny to be approximately 2 centimeters wide. Therefore the number of pennies is

2.4.1 https://phys.libretexts.org/@go/page/24427
 total distance 
# of pennies  =
 length of a penny 

(1km)
=
5
(2cm) (1km/ 10 cm)

= 50, 000 pennies 

4
= 5 × 10  pennies. 

When applying numbers to relationships we must be careful to convert units whenever necessary. How accurate is this
estimation? If you measure the size of a penny, you will find out that the width is 1.9 cm, so our estimate was accurate to
within 5%. This accuracy was fortuitous. Suppose we estimated the length of a penny to be 1 cm. Then our estimate for the
total number of pennies would be within a factor of 2, a margin of error we can live with for this type of problem.

Example 2.7: Estimation of Mass of Water on Earth

Estimate the mass of the water on the Earth.


Solution
In this example we are estimating mass, a quantity that is a fundamental in SI units, and is measured in kg. We start by
approximating that the amount of water on Earth is approximately equal to the amount of water in all the oceans. Initially we
will try to estimate two quantities: the density of water and the volume of water contained in the oceans. Then the relationship
we want is

 mass  = ( density )( volume )

One of the hardest aspects of estimation problems is to decide which relationship applies. One way to check your work is to
check dimensions. Density has dimensions of mass/volume, so our relationship is correct dimensionally.
The density of fresh water is ρ = 1.0g ⋅ cm ; the density of seawater is slightly higher, but the difference won’t matter for
−3

this estimate. You could estimate this density by estimating how much mass is contained in a one-liter bottle of water. (The
density of water is a point of reference for all density problems. Suppose we need to estimate the density of iron. If we
compare iron to water, we might estimate that iron is 5 to 10 times denser than water. The actual density of iron is
ρ iron= 7.8g ⋅ cm
−3
).
Because there is no precise relationship, estimating the volume of water in the oceans is much harder. Let’s model the volume
occupied by the oceans as if the water completely covers the earth, forming a spherical shell of radius R and thickness d
E

(Figure 2.4.1, which is decidedly not to scale), where R is the radius of the earth and d is the average depth of the ocean. The
E

volume of that spherical shell is


2
 volume  ≅4π R d
earth 

Figure 2.4.1 : Volume of a spherical shell with a fixed radius. (CC BY-NC; Ümit Kaya)
We also estimate that the oceans cover about 75% of the surface of the earth. So we can refine our estimate that the volume of
the oceans is
2
 volume  ≅(0.75) (4π R d)
E

2.4.2 https://phys.libretexts.org/@go/page/24427
We therefore have two more quantities to estimate, the average depth of the ocean, which we can estimate as d ≅1 km , and
the radius of the earth, which is approximately R ≅6 × 10 km . (The quantity that you may remember is the circumference
3

of the earth, E about 25,000 miles. Historically the circumference of the earth was defined to be 4 × 10 m ). The radius R 7
E

and the circumference s are exactly related by


s = 2πR

Thus
s
RE =

4 −1
(2.5 × 10 mi) (1.6km ⋅ mi )
=

3
= 6.4 × 10 km

We will use R ≅6 × 10 km ; additional accuracy is not necessary for this problem, since the ocean depth estimate is clearly
E
3

less accurate. In fact, the factor of 75% is not needed, but included more or less from habit. Altogether, our estimate for the
mass of the oceans is
 mass  = ( density )( volume )

2
≅ρ(0.75) (4π R d)
E

5 3
1g 1kg (10 cm) 2
3
≅( )( )( ) (0.75)(4π) (6 × 10 km) (1km)
3 3 3
cm 10 g (1km)

20 20
≅3 × 10 kg ≅10 kg

This page titled 2.4: Order of Magnitude Estimates - Fermi Problems is shared under a CC BY-NC-SA 4.0 license and was authored, remixed,
and/or curated by Peter Dourmashkin (MIT OpenCourseWare) via source content that was edited to the style and standards of the LibreTexts
platform; a detailed edit history is available upon request.

2.4.3 https://phys.libretexts.org/@go/page/24427
CHAPTER OVERVIEW
3: Vectors
"Philosophy is written in this grand book, the universe which stands continually open to our gaze. But the book cannot be
understood unless one first learns to comprehend the language and read the letters in which it is composed. It is written in the
language of mathematics, and its characters are triangles, circles and other geometric figures without which it is humanly
impossible to understand a single word of it; without these, one wanders about in a dark labyrinth." - Galileo Galilei, The Assayer,
tr. Stillman Drake (1957), Discoveries and Opinions of Galileo pp. 237-8.
3.1: Vector Analysis
3.2: Coordinate Systems
3.3: Vectors
3.4: Vector Product (Cross Product)

This page titled 3: Vectors is shared under a CC BY-NC-SA 4.0 license and was authored, remixed, and/or curated by Peter Dourmashkin (MIT
OpenCourseWare) via source content that was edited to the style and standards of the LibreTexts platform; a detailed edit history is available upon
request.

1
3.1: Vector Analysis
Introduction to Vectors
Certain physical quantities such as mass or the absolute temperature at some point in space only have magnitude. A single number
can represent each of these quantities, with appropriate units, which are called scalar quantities. There are, however, other physical
quantities that have both magnitude and direction. Force is an example of a quantity that has both direction and magnitude
(strength). Three numbers are needed to represent the magnitude and direction of a vector quantity in a three dimensional space.
These quantities are called vector quantities. Vector quantities also satisfy two distinct operations, vector addition and
multiplication of a vector by a scalar. We can add two forces together and the sum of the forces must satisfy the rule for vector
addition. We can multiply a force by a scalar thus increasing or decreasing its strength. Position, displacement, velocity,
acceleration, force, and momentum are all physical quantities that can be represented mathematically by vectors. The set of vectors
and the two operations form what is called a vector space. There are many types of vector spaces but we shall restrict our attention
to the very familiar type of vector space in three dimensions that most students have encountered in their mathematical courses. We
shall begin our discussion by defining what we mean by a vector in three dimensional space, and the rules for the operations of
vector addition and multiplication of a vector by a scalar.

Properties of Vectors
→ →
A vector is a quantity that has both direction and magnitude. Let a vector be denoted by the symbol A . The magnitude of A is

| A| ≡ A . We can represent vectors as geometric objects using arrows. The length of the arrow corresponds to the magnitude of the
vector. The arrow points in the direction of the vector (Figure 3.1).

Figure 3.1: Vectors as arrows. (CC BY-NC; Ümit Kaya)


There are two defining operations for vectors:

(1) Vector Addition


→ → → → → → →
Vectors can be added. Let A and B be two vectors. We define a new vector, C = A + B , the “vector addition” of A and B , by
→ →
a geometric construction. Draw the arrow that represents A . Place the tail of the arrow that represents B at the tip of the arrow for
→ → →
A as shown in Figure 3.2a. The arrow that starts at the tail of A and goes to the tip of B is defined to be the “vector addition”
→ → → → →
C = A + B . There is an equivalent construction for the law of vector addition. The vectors A and B can be drawn with their
tails at the same point. The two vectors form the sides of a parallelogram. The diagonal of the parallelogram corresponds to the
→ → →
vector C = A + B , as shown in Figure 3.2b.

3.1.1 https://phys.libretexts.org/@go/page/24431
Figure 3.2b (CC BY-NC;
Figure 3.2a (CC BY-NC; Ümit Ümit Kaya)
Kaya)

Vector addition satisfies the following four properties:


(i) Commutativity
The order of adding vectors does not matter;
→ → → →
A + B = B +A

Our geometric definition for vector addition satisfies the commutative property (3.1.1). We can understand this geometrically
because in the head to tail representation for the addition of vectors, it doesn’t matter which vector you begin with, the sum is the
same vector, as seen in Figure 3.3.

Figure 3.3: Commutative property of vector addition. (CC BY-NC; Ümit Kaya)
(ii) Associativity
When adding three vectors, it doesn’t matter which two you start with
→ → → → → →
(A + B ) + C = A + ( B + C )

→ → → → → →
In Figure 3.4a, we add ( B + C ) + A , and use commutativity to get A + B) + C to arrive at the same vector as in Figure 3.4a.

3.1.2 https://phys.libretexts.org/@go/page/24431
Figure 3.4a Associative law. (CC BY-NC; Ümit Kaya)
(iii) Identity Element for Vector Addition
→ →
There is a unique vector, 0 , that acts as an identity element for vector addition. For all vectors A ,
→ → → → →
A + 0 = 0 +A = A

(iv) Inverse Element for Vector Addition


→ →
For every vector A there is a unique inverse vector −A such that
→ → →
A + (− A ) = 0

→ → → →
The vector −A has the same magnitude as A , | A | = | − A | = A but they point in opposite directions (Figure 3.5).

Figure 3.5 Additive inverse. (CC BY-NC; Ümit Kaya)

(2) Scalar Multiplication of Vectors


→ →
Vectors can be multiplied by real numbers. Let A be a vector. Let c be a real positive number. Then the multiplication of A by c is
→ → →
a new vector, which we denote by the symbol c A . The magnitude of c A is c times the magnitude of A (Figure 3.6a),
→ →
|c A | = c| A |

→ → → →
Let c > 0 , then the direction of cA is the same as the direction of A . However, the direction of −c A is opposite of A (Figure
3.6).

3.1.3 https://phys.libretexts.org/@go/page/24431

Figure 3.6 Multiplication of vector A by c > 0 , and −c < 0 . (CC BY-NC; Ümit Kaya)
Scalar multiplication of vectors satisfies the following properties:
(i) Associative Law for Scalar Multiplication
The order of multiplying numbers is doesn’t matter. Let b and c be real numbers. Then
→ → → →
b(c A ) = (bc) A = (cb A ) = c(b A )

(ii) Distributive Law for Vector Addition


Vectors satisfy a distributive law for vector addition. Let c be a real number. Then
→ → → →
c( A + B ) = c A + c B

Figure 3.7 illustrates this property.

Figure 3.7 Distributive Law for vector addition. (CC BY-NC; Ümit Kaya)
(iii) Distributive Law for Scalar Addition
Vectors also satisfy a distributive law for scalar addition. Let b and c be real numbers.Then
→ → →
(b + c) A = b A + c A

Our geometric definition of vector addition and scalar multiplication satisfies this condition as seen in Figure 3.8.

Figure 3.8 Distributive law for scalar multiplication. (CC BY-NC; Ümit Kaya)
(iv) Identity Element for Scalar Multiplication
The number 1 acts as an identity element for multiplication,
→ →
1A = A

3.1.4 https://phys.libretexts.org/@go/page/24431
Definition: Unit Vector
Dividing a vector by its magnitude results in a vector of unit length which we denote with a caret symbol

A
^
A =

| A|

→ →
Note that |A
^
| = | A |/| A | = 1

This page titled 3.1: Vector Analysis is shared under a CC BY-NC-SA 4.0 license and was authored, remixed, and/or curated by Peter
Dourmashkin (MIT OpenCourseWare) via source content that was edited to the style and standards of the LibreTexts platform; a detailed edit
history is available upon request.

3.1.5 https://phys.libretexts.org/@go/page/24431
3.2: Coordinate Systems
Physics involve the study of phenomena that we observe in the world. In order to connect the phenomena to mathematics we begin
by introducing the concept of a coordinate system. A coordinate system consists of four basic elements:
1. Choice of origin
2. Choice of axes
3. Choice of positive direction for each axis
4. Choice of unit vectors at every point in space
There are three commonly used coordinate systems: Cartesian, cylindrical and spherical. In this chapter, we will describe a
Cartesian coordinate system and a cylindrical coordinate system.

Cartesian Coordinate System


Cartesian coordinates consist of a set of mutually perpendicular axes, which intersect at a common point, the origin O. We live in a
three-dimensional spatial world; for that reason, the most common system we will use has three axes.

Choice of Origin
Choose an origin O at any point that is most convenient.

Choice of Axes
The simplest set of axes is known as the Cartesian axes, x-axis, y -axis, and the z -axis, that are at right angles with respect to each
other. Then each point P in space can be assigned a triplet of values (x , y , z ). The ranges of these values are:
P P P

−∞ < x < +∞ , −∞ < y


P < +∞ , −∞ < z
P < +∞ . P

Choice of Positive Direction


Our third choice is an assignment of positive direction for each coordinate axis. We shall denote this choice by the symbol + along
the positive axis. In physics problems we are free to choose our axes and positive directions any way that we decide best fits a
given problem. Problems that are very difficult using the 6 conventional choices may turn out to be much easier to solve by making
a thoughtful choice of axes.

Choice of Unit Vectors


We now associate to each point P in space, a set of three unit vectors ^ ^ ^
( i P , j P , kP ) . A unit vector has magnitude one:
∣^ ∣ ∣^ ∣ ∣^ ∣
∣ i P ∣ = 1, ∣ j P ∣ = 1,  and  ∣kP ∣ = 1 . We assign the direction of ^
iP to point in the direction of the increasing x-coordinate at the
point P . We define the directions for ^
jP and ^
kP P in the direction of the increasing y -coordinate and z -coordinate respectively,
(Figure 3.10). If we choose a different point S , and define a similar set of unit vectors ^ ^ ^
( i S , j S , kS ) , the unit vectors at S and P
satisfy the equalities
^ ^ ^ ^ ^ ^
iS = iP , j = j ,  and kS = kP ,
S P

because vectors are equal if they have the same direction and magnitude regardless of where they are located in space.

3.2.1 https://phys.libretexts.org/@go/page/24432
Figure 3.10 Choice of unit vectors at points P and S . (CC BY-NC; Ümit Kaya)
A Cartesian coordinate system is the only coordinate system in which Equation (3.2.1) holds for all pair of points. We therefore
drop the reference to the point P and use (^iP
^ ^
, j P , kP ) to represent the unit vectors in a Cartesian coordinate system (Figure 3.11).

Figure 3.11 Unit vectors in a Cartesian coordinate system. (CC BY-NC; Ümit Kaya)

Cylindrical Coordinate System


Many physical objects demonstrate some type of symmetry. For example, if you rotate a uniform cylinder about the longitudinal
axis (symmetry axis), the cylinder appears unchanged. The operation of rotating the cylinder is called a symmetry operation, and
the object undergoing the operation, the cylinder, is exactly the same as before the operation was performed. This symmetry
property of cylinders suggests a coordinate system, called a cylindrical coordinate system, that makes the symmetrical property
under rotations transparent.
First choose an origin O and axis through O, which we call the z -axis. The cylindrical coordinates for a point P are the three
numbers (r, θ, z) (Figure 3.12). The number z represents the familiar coordinate of the point P along the z -axis. The nonnegative
number r represents the distance from the z -axis to the point P . The points in space corresponding to a constant positive value of r
lie on a circular cylinder. The locus of points corresponding to r = 0 is the z -axis. In the plane z = 0 , define a reference ray
through O, which we shall refer to as the positive x-axis. Draw a line through the point P that is parallel to the z -axis. Let D
denote the point of intersection between that line P D and the plane z = 0 . Draw a ray OD from the origin to the point D. Let θ
denote the directed angle from the reference ray to the ray OD. The angle θ is positive when measured counterclockwise and
negative when measured clockwise.

3.2.2 https://phys.libretexts.org/@go/page/24432
Figure 3.12 Cylindrical Coordinates. (CC BY-NC; Ümit Kaya)
The coordinates (r, θ) are called polar coordinates. The coordinate transformations between (r, θ) and the Cartesian coordinates
(x, y) are given by

x = r cos θ,

y = r sin θ.

Conversely, if we are given the Cartesian coordinates (x, y) , the coordinates (r, θ) can be determined from the coordinate
transformations
2 2 1/2
r = +(x +y )

−1
θ = tan (y/x)

We choose a set of unit vectors (^


r P
^ ^
, θ P , kP ) at the point P as follows. We choose k
^
to point in the direction of increasing z . We
P

choose ^r to point in the direction of increasing r, directed radially away from the z -axis. We choose θ^ to point in the direction
P P

of increasing θ . This unit vector points in the counterclockwise direction, tangent to the circle (Figure 3.13a). One crucial
difference between cylindrical coordinates and Cartesian coordinates involves the choice of unit vectors. Suppose we consider a
different point S in the plane. The unit vectors (^
^ ^
rs , θ s , ks ) at the point S are also shown in Figure 3.13. Note that
^
rP ≠^
^ ^
rS  and θ p ≠ θ S because their direction differ. We shall drop the subscripts denoting the points at which the unit vectors are
defined at and simple refer to the set of unit vectors at a point as (^
r, θ , k) , with the understanding that the directions of the set
^ ^

(^
^
r, θ ) depend on the location of the point in question.

3.2.3 https://phys.libretexts.org/@go/page/24432
Figure 3.13: (a) Unit vectors at two different points in cylindrical coordinates. (b) Unit vectors in polar coordinates and Cartesian
coordinates. (CC BY-NC; Ümit Kaya)
The unit vectors (^
r, θ ) at the point P also are related to the Cartesian unit vectors ( i , j ) by the transformations
^ ^ ^

^ ^ ^
r = cos θ i + sin θ j

^ ^ ^
θ = − sin θ i + cos θ j

Similarly the inverse transformations are given by


^ ^
i = cos θ ^
r − sin θ θ

^ ^
j = sin θ ^
r + cos θ θ

A cylindrical coordinate system is also a useful choice to describe the motion of an object moving in a circle about a central point.
Consider a vertical axis passing perpendicular to the plane of motion passing through that central point. Then any rotation about
this vertical axis leaves circles unchanged.

This page titled 3.2: Coordinate Systems is shared under a CC BY-NC-SA 4.0 license and was authored, remixed, and/or curated by Peter
Dourmashkin (MIT OpenCourseWare) via source content that was edited to the style and standards of the LibreTexts platform; a detailed edit
history is available upon request.

3.2.4 https://phys.libretexts.org/@go/page/24432
3.3: Vectors
The Use of Vectors in Physics
From the last section we have three important ideas about vectors:
1. vectors can exist at any point P in space,
2. vectors have direction and magnitude, and
3. any two vectors that have the same direction and magnitude are equal no matter where in space they are located.
When we apply vectors to physical quantities it’s nice to keep in the back of our minds all these formal properties. However, from
the physicist’s point of view, we are interested in representing physical quantities such as displacement, velocity, acceleration,
force, impulse, and momentum as vectors. We can’t add force to velocity or subtract momentum from force. We must always
understand the physical context for the vector quantity. Thus, instead of approaching vectors as formal mathematical objects we
shall instead consider the following essential properties that enable us to represent physical quantities as vectors.

Vectors in Cartesian Coordinates


Vector Decomposition
Choose a coordinate system with an origin, axes, and unit vectors. We can decompose a vector into component vectors along each
coordinate axis (Figure 3.14).

Figure 3.14 Component vectors in Cartesian coordinates. (CC BY-NC; Ümit Kaya)

A vector A at P can be decomposed into the vector sum,
→ → → →
A = Ax + Ay + Az

→ →
where Ax is the x-component vector pointing in the positive or negative x-direction, Ay is the y -component vector pointing in the

positive or negative y -direction, and Az is the z -component vector pointing in the positive or negative z -direction.

Vector Components
Once we have defined unit vectors ^ ^ ^
, we then define the components of a vector. Recall our vector decomposition,
( i , j , k)

→ → → → →
A = Ax + Ay + Az. We define the x-component vector, Ax, as

^
A x = Ax i .


In this expression the term A (without the arrow above) is called the x-component of the vector
x Ax. The x-component A can be
x


1/2
positive, zero, or negative. It is not the magnitude of Ax which is given by (A 2
x) . The x-component A is a scalar quantity and
x


the x-component vector, Ax is a vector. In a similar fashion we define the y -component, Ay , and the z -component, Az , of the

vector A according to

3.3.1 https://phys.libretexts.org/@go/page/24433
→ →
^ ^
A y = Ay j , A z = Az k.


A vector A is represented by its three components (Ax , Ay , Az ) . Thus we need three numbers to describe a vector in three-

dimensional space. We write the vector A as

^ ^ ^
A = Ax i + Ay j + Az k

Magnitude

Using the Pythagorean theorem, the magnitude of A is,
−−−−−−−−−−−
2 2 2
A = √ Ax + Ay + Az

Direction
→ →
Let’s consider a vector A = (Ax , Ay , 0) . Because the z -component is zero, the vector A lies in the x − y plane. Let θ denote the

angle that the vector A makes in the counterclockwise direction with the positive x-axis (Figure 3.15).

Figure 3.15 Components of a vector in the xy -plane. (CC BY-NC; Ümit Kaya)
Then the x -component and y -component are

Ar = A cos(θ), Ay = A sin(θ)

We now write a vector in the xy -plane as



^ ^
A = A cos(θ) i + A sin(θ) j

Once the components of a vector are known, the tangent of the angle θ can be determined by
Ay A sin(θ)
= = tan(θ)
Ax A cos(θ)

and hence the angle θ is given by


Ay
−1
θ = tan ( )
Ax

Clearly, the direction of the vector depends on the sign of A and A . For example, if both A > 0 and A > 0 , then
x y x y

0 < θ < π/2 . If A < 0 and A > 0 then π/2 < θ < π . If A < 0 and A < 0 then π < θ < 3π/2 . If A > 0 and Ay < 0 , then
x y x y x

3π/2 < θ < 2 . Note that tanθ is a double valued function because

−Ay Ay Ay −Ay
= ,  and  =
−Ax Ax −Ax Ax

Unit Vectors
→ → → →
Unit vector in the direction of A : Let ^ ^ ^
A = Ax i + Av j + A7 k . Let A denote a unit vector in the direction of A . Then,

3.3.2 https://phys.libretexts.org/@go/page/24433

^ ^ ^
A Ax i + Ay j + Az k
^
A = =
→ 1/2
2 2 2
(Ax + Ay + Az )
| A|

Vector Addition
→ → → →
Let A and B be two vectors in the x − y plane. Let θ A
and θB denote the angles that the vectors A and B make (in the
counterclockwise direction) with the positive x -axis. Then

^ ^
A = A cos(θA ) i + A sin(θA ) j


^ ^
B = B cos(θB ) i + B sin(θB ) j

→ → → →
In Figure 3.16, the vector addition C = A + B is shown. Let θ denote the angle that the vector
C C makes with the positive x -
axis.

Figure 3.16 Vector addition using components. (CC BY-NC; Ümit Kaya)

From Figure 3.16, the components of C are

Cx = Ax + Bx , Cy = Ay + By

In terms of magnitudes and angles, we have


Cx = C cos(θC ) = A cos(θA ) + B cos(θB )

Cy = C sin(θC ) = A sin(θA ) + B sin(θB )


We can write the vector C as

^ ^ ^ ^
C = (Ax + Bx ) i + (Ay + By ) j = C cos(θC ) i + C sin(θC ) j

Example 3.1: Vector Addition


→ → → → → → → →
Given two vectors, ^ ^ ^ ^ ^ ^
A = 2 i + −3 j + 7 k and B = 5 i + j + 2 k , find: (a) | A|; (b) | B |; (c) A + B ; (d) A − B ; (e) a unit
→ →
vector A
^
pointing in the direction of A ; (f) a unit vector B
^
pointing in the direction of B ;
Solution
(a)
→ 1/2 −−
| A | = (2
2
+ (−3 )
2
+7 )
2
= √62 = 7.87 .
(b)
→ 1/2 −−
| B | = (5
2
+1
2
+2 )
2
= √30 = 5.48 .

3.3.3 https://phys.libretexts.org/@go/page/24433
(c)
→ →
^ ^ ^
A + B = (Ax + Bx ) i + (Ay + By ) j + (Az + Bz ) k

^ ^ ^
= (2 + 5) i + (−3 + 1) j + (7 + 2)k

^ ^ ^
= 7 i − 2 j + 9k

(d)
→ →
^ ^ ^
A − B = (Ax − Bx ) i + (Ay − By ) j + (Az − Bz ) k

^ ^ ^
= (2 − 5) i + (−3 − 1) j + (7 − 2)k

^ ^ ^
= −3 i − 4 j + 5 k

(e)
→ → →
A unit vector A
^
in the direction of A can be found by dividing the vector A by the magnitude of A . Therfore
→ →
^ ^ ^ ^ −−
A = A /| A | = (2 i + −3 j + 7 k)/ √62

(f)
→ →
−−
In a similar fashion, B
^
=
^ ^ ^
B /| B | = (5 i + j + 2 k)/ √30

Example 3.2 Sinking Sailboat


A Coast Guard ship is located 35 km away from a checkpoint in a direction 52 north of west. A distressed sailboat located in

still water 24 km from the same checkpoint in a direction 18 south of east is about to sink. Draw a diagram indicating the

position of both ships. In what direction and how far must the Coast Guard ship travel to reach the sailboat?
Solution
The diagram of the set-up is Figure 3.17.

3.3.4 https://phys.libretexts.org/@go/page/24433
Figure 3.17 Example 3.2. (CC BY-NC; Ümit Kaya)

Figure 3.18 Coordinate system for sailboat and ship. (CC BY-NC;
Ümit Kaya)

Choose the checkpoint as the origin of a Cartesian coordinate system with the positive x -axis in the East direction and the
positive y –axis in the North direction. Choose the corresponding unit vectors ^i and^j as shown in Figure 3.18. The Coast
Guard ship is then a distance r = 35 km at an angle θ = 180 − 52 = 128 from the positive x -axis, The position of the
1
∘ ∘ ∘

Coast Guard ship is then



^ ^
r 1 = r1 (cos θ1 i + sin θ1 j )


^ ^
r = −21.5km i + 27.6km j

and the position of the sailboat is



^ ^
r 2 = r2 (cos θ2 i + sin θ2 j )


^ ^
r 2 = 22.8km i − 7.4km j

3.3.5 https://phys.libretexts.org/@go/page/24433
Figure 3.19 Relative position vector from ship to sailboat. (CC BY-NC; Ümit Kaya)
The relative position vector from the Coast Guard ship to the sailboat is (Figure 3.19)
→ →
^ ^ ^ ^
r 2 − r 1 = (22.8km i − 7.4km j ) − (−21.5km i + 27.6km j )

→ →
^ ^
r 2 − r 1 = 44.4km i − 35.0km j

The distance between the ship and the sailboat is


∣→ → ∣ 2 2 1/2
r 2 − r 1 = ((44.4km) + (−35.0km) ) = 56.5km
∣ ∣

The rescue ship’s heading would be the inverse tangent of the ratio of the y - and x - components of the relative position vector,
−1 ∘
θ21 = tan (−35.0km/44.4km) = −38.3

or 38.3 South of East.


Example 3.3: Vector Addition


→ → → → → → → →
Two vectors A and B , such that | B | = 2| A | , have a resultant C = A + B of magnitude 26.5. The vector C makes an
→ → →
angle θc

= 41 with respect to vector A . Find the magnitude of each vector and the angle between vectors A and B .

Solution: We begin by making a sketch of the three vectors, choosing A to point in the positive x-direction (Figure 3.20).

Figure 3.20: Choice of coordinates system for Example 3.3. (CC BY-NC; Ümit Kaya)
→ → −−−−−−−−−−− → → →
2 2
Denote the magnitude of C by C ≡ | C | = √(Cx ) + (Cy ) = 26.5 . The components of C = A + B are given by

Cx = Ax + Bx = C cos θC = (26.5) cos(41 ) = 20

3.3.6 https://phys.libretexts.org/@go/page/24433

Cy = By = C sin θC = (26.5) sin(41 ) = 17.4.

→ →
From the condition that | B | = 2| A | , the square of their magnitudes satisfy
2 2 2
(Bx ) + (By ) = 4 (Ax ) .

Using Equations (3.3.17) and (3.3.18), Equation (3.3.19) becomes


2 2 2
(Cx − Ax ) + (Cy ) = 4 (Ax )

2 2 2 2
(Cx ) − 2 Cx Ax + (Ax ) + (Cy ) = 4 (Ax )

This is a quadratic equation


2 2
0 = 3 (Ax ) + 2 Cx Ax − C

which we solve for the component A : x

−−−−−−−−−−−−−−−−
2 − −−−−−−−−−−−−−−− −
−2 Cx ± √ (2 Cx ) + (4)(3) (C 2 ) 2
−2(20) ± √ (40)) + (4)(3)(26.5 )
2

Ax = = = 10.0
6 6


where we choose the positive square root because we originally chose Ax > 0 . The components of B are then given by
Equations (3.3.17) and (3.3.18):

Bx = Cx − Ax = 20.0 − 10.0 = 10.0

B. = 17.4

→ −−−−−−−−−−− →
The magnitude of | B | = √(Bx )
2
+ (By )
2
= 20.0 which is equal to two times the magnitude of | A | = 10.0 . The angle
→ →
between A and B is given by

−1 −1 ∘
θ = sin ( By /| B |) = sin (17.4/20.0N) = 60

Example 3.4 Vector Description of a Point on a Line


Consider two points, P with coordinates (x
1 1, y1 ) and P with coordinates (x
2 1, y1 ) are separated by distance d . Find a vector

A from the origin to the point on the line connecting P and P that is located a distance a from the point P (Figure 3.21).
1 2 2

Figure 3.3.21 : Example 3.4. (CC BY-NC; Ümit Kaya)


Solution
→ → → →
Let r = x ^i + y ^j be the position vector of P and r = x ^i + y ^j the position vector of
1 1 1 1 2 2 2 P2 . Let r 1 − r 2 be the
vector from P to P (Figure 3.22a). The unit vector pointing from P to P is given by
2 1 2 1

1/2
→ → ∣→ → ∣ → → 2 2
^
r21 = ( r 1 − r 2 ) / r 1 − r 2 = ( r 1 − r 2 ) /d
∣ ∣
, where d = ((x 2 − x1 ) + (y2 − y1 ) )

3.3.7 https://phys.libretexts.org/@go/page/24433
Figure 3.22a: Relative position vector Figure 3.22b: Relative position vector

→ → → −→
The vector s in Figure 3.22b connects A to the point at r1 , points in the direction of r12 and has length a . Therefore
→ → → → → →
s = a^
r21 = a ( r 1 − r 2 ) /d . The vector r 1 = A + s . Therefore

→ → → → → → → →
A = r 1 − s = r 1 − a ( r 1 − r 2 ) /d = (1 − a/d) r 1 + (a/d) r 2


^ ^ ^ ^
A = (1 − a/d) (x1 i + y1 j ) + (a/d) (x2 i + y2 j )

⎛ ⎞ ⎛ ⎞
→ a (x2 − x1 ) a (y2 − y1 )
A = ⎜x1 + ⎟^i + ⎜y1 + ⎟^j
⎜ 1/2
⎟ ⎜ 1/2

2 2 2 2
⎝ ( (x2 − x1 ) + (y2 − y1 ) ) ⎠ ⎝ ( (x2 − x1 ) + (y2 − y1 ) ) ⎠

Transformation of Vectors in Rotated Coordinate Systems


Consider two Cartesian coordinate systems S and S such that the
′ ′ ′
(x , y ) coordinate axes in S

are rotated by an angle θ with
respect to the (x, y) coordinate axes in S , (Figure 3.23).

Figure 3.23: Rotated coordinate systems. (CC BY-NC; Ümit Kaya)

The components of the unit vector i^ in the ^i and ^j direction are given by


′ ∣^ ∣
ix = i cos θ = cos θ
∣ ∣

and

′ ∣^ ∣
iy = i sin θ = sin θ.
∣ ∣

Therefore

^ ′ ^ ′ ^ ^ ^
i = ix i + iy j = i cos θ + j sin θ

A similar argument holds for the components of the unit vector j^ . The components of j^ in the ^i and ^j direction are given by
′ ′

3.3.8 https://phys.libretexts.org/@go/page/24433

′ ∣^ ∣
jx = − j sin θ = − sin θ
∣ ∣

and

′ ∣^ ∣
jy = j cos θ = cos θ.
∣ ∣

Therefore
^′ ′ ^ ′ ^ ^ ^
j = jx i + jy j = j cos θ − i sin θ

Conversely, from Figure 3.23 and similar vector decomposition arguments, the components of (\hat{\mathbf{i}}\) and
(\hat{\mathbf{j}}\) in S are given by

′ ′
^ ^ ^
i = i cos θ − j sin θ

′ ′
^ ^ ^
j = i sin θ + j cos θ


Consider a fixed vector ^ ^
r = xi +yj with components (x, y) in coordinate system S . In coordinate system S , the vector is given ′

→ ′ ′
^
by ′^ ′ ^ ′ ′
r = x i + y j ,  where  (x , y ) , where (x , y ) are the components in S , (Figure 3.24).
′ ′ ′

Figure 3.24: Transformation of vector components. (CC BY-NC; Ümit Kaya)


Using the Equations (3.3.20) and (3.3.21), we have that
→ ′ ′ ′ ′
^ ^ ^ ^ ^ ^
r = x i + y j = x ( i cos θ − j sin θ) + y ( j cos θ + i sin θ)

→ ′ ′
^ ^
r = (x cos θ + y sin θ) i + (x sin θ − y cos θ) j

Therefore the components of the vector transform according to



x = x cos θ + y sin θ


y = x sin θ − y cos θ

We now consider an alternate approach to understanding the transformation laws for the components of the position vector of a
→ →
fixed point in space. In coordinate system S , suppose the position vector r has length r = | r | and makes an angle ϕ with respect
to the positive x-axis (Figure 3.25).

Figure 3.25: Transformation of vector components of the position vector. (CC BY-NC; Ümit Kaya)

Then the components of r in S are given by

3.3.9 https://phys.libretexts.org/@go/page/24433
x = r cos ϕ

y = r sin ϕ


. In coordinate system S , the components of

r are given by

x = r cos(ϕ − θ)


y = r sin(ϕ − θ)

Apply the addition of angle trigonometric identities to Equations (3.3.29) and (3.3.30) yielding

x = r cos(ϕ − θ) = r cos ϕ cos θ + r sin ϕ sin θ = x cos θ + y sin θ


y = r sin(ϕ − θ) = r sin ϕ cos θ − r cos ϕ sin θ = y cos θ − x sin θ

in agreement with Equations (3.3.25) and (3.3.26).

Example 3.5 Vector Decomposition in Rotated Coordinate Systems



With respect to a given Cartesian coordinate system S , a vector A has components A = 5 , A = −3 , A = 0 . Consider ax y z

second coordinate system S such that the (x , y coordinate axes in S are rotated by an angle θ = 60 with respect to the
′ ′ ′ ′ ∘

(x, y) coordinate axes in S , (Figure 3.26).


a. What are the components A and A of vector A in coordinate system S ?
x y

b. Calculate the magnitude of the vector using the (A , A ) components and using the (A , A
x y x y) components. Does your
result agree with what you expect?

Figure 3.26 Example 3.4. (CC BY-NC; Ümit Kaya)


Solution:

We begin by considering the vector decomposition of A with respect to the coordinate system S ,

^ ^
A = Ax i + Ay j

→ → → →
Now we can use our results for the transformation of unit vectors and in terms of and , (Equations (3.3.22) and
′ ′

i j i j

(3.3.23)) in order decompose the vector A in coordinate system S ′

→ ′ ′ ′ ′
^ ^ ^ ^ ^ ^
A = A i + Ay j = Ax (cos θ i − sin θ j ) + Ay (sin θ i + cos θ j )

′ ′
^ ^
= (Ax cos θ + Ay sin θ) i + (−Ax sin θ + Ay cos θ) j

^ ^
= Ax i + Ay j

where

Ax′ = Ax cos θ + Ay sin θ

Ay ′ = −Ax sin θ + Ay cos θ

3.3.10 https://phys.libretexts.org/@go/page/24433

We now use the given information that Ax = 5 , Ay = −3 , and θ = 60

to solve for the components of A in coordinate
system S ′


Ax′ = Ax cos θ + Ay sin θ = (1/2)(5 − 3 √3)


Ay ′ = −Ax sin θ + Ay cos θ = (1/2)(−5 √3 − 3)

b) The magnitude can be calculated in either coordinate system


→ −−−−−−−−−−− −−−−−−−−−−
2 2 2 2 −−
| A | = √ (Ax ) + (Ay ) = √ (5 ) + (−3 ) = √34

→ −−−−−−−−−−−− − −−−−−−−−−−−−−−−−−−−−−−−−−−−−−− −
2 2 – 2 – 2 −−
| A | = √ (Ax′ ) + (Ay ′ ) = √ ((1/2)(5 − 3 √3)) + ((1/2)(−5 √3 − 3)) = √34


This result agrees with what I expect because the length of vector A independent of the choice of coordinate system.

This page titled 3.3: Vectors is shared under a CC BY-NC-SA 4.0 license and was authored, remixed, and/or curated by Peter Dourmashkin (MIT
OpenCourseWare) via source content that was edited to the style and standards of the LibreTexts platform; a detailed edit history is available upon
request.

3.3.11 https://phys.libretexts.org/@go/page/24433
3.4: Vector Product (Cross Product)
→ →
Let A and B be two vectors. Because any two non-parallel vectors form a plane, we denote the angle θ to be the angle between
→ → → → → →
the vectors A and B as shown in Figure 3.27. The magnitude of the vector product A × B of the vectors A and B is defined
→ →
to be product of the magnitude of the vectors A and B with the sine of the angle θ between the two vectors,
→ → → →
| A × B | = | A || B | sin(θ)

The angle θ between the vectors is limited to the values 0 ≤ θ ≤ π ensuring that sin(θ) ≥ 0.

Figure 3.27: Vector product geometry. (CC BY-NC; Ümit Kaya)


→ →
The direction of the vector product is defined as follows. The vectors A and B form a plane. Consider the direction perpendicular
to this plane. There are two possibilities: we shall choose one of these two (the one shown in Figure 3.27) for the direction of the
→ →
vector product A × B using a convention that is commonly called the “right-hand rule”.

Right-hand Rule for the Direction of Vector Product


→ → →
The first step is to redraw the vectors A and B so that the tails are touching. Then draw an arc starting from the vector A and

finishing on the vector B . Curl your right fingers the same way as the arc. Your right thumb points in the direction of the vector
→ →
product A × B (Figure 3.28).

Figure 3.28: Right-Hand Rule. (CC BY-NC; Ümit Kaya)


→ → → →
You should remember that the direction of the vector product A × B is perpendicular to the plane formed by A and B . We can
give a geometric interpretation to the magnitude of the vector product by writing the magnitude as
→ → → →
| A × B | = | A |(| B | sin(θ)).

→ → →
The term | A | sin θ is the projection of the vector A in the direction perpendicular to the vector B as shown in Figure 3.29(b). The
vector product of two vectors that are parallel (or anti-parallel) to each other is zero because the angle between the vectors is 0 (or
π) and sin(0) = 0 (or sin(π) = 0). Geometrically, two parallel vectors do not have a unique component perpendicular to their

common direction

3.4.1 https://phys.libretexts.org/@go/page/24434
→ → → →
Figure 3.29: Projection of (a) B perpendicular to A , (b) of A perpendicular to B . (CC BY-NC; Ümit Kaya)

Properties of the Vector Product


1. The vector product is anti-commutative because changing the order of the vectors changes the direction of the vector product by
the right hand rule:
→ → → →
A × B = −B × A

→ →
2. The vector product between a vector c A where c is a scalar and a vector B is
→ → → →
c A × B = c( A × B )

Similarly,
→ → → →
A × c B = c( A × B ).

→ → →
3. The vector product between the sum of two vectors A and B with a vector C is
→ → → → → → →
(A + B ) × C = A × C + B × C

Similarly,
→ → → → → → →
A × (B + C) = A × B + A × C.

Vector Decomposition and the Vector Product: Cartesian Coordinates


→ →
We first calculate that the magnitude of vector product of the unit vectors i and j :
^ ^ ^ ^
| i × j | = | i ∥ j | sin(π/2) = 1

because the unit vectors have magnitude |^i | = |^j | = 1 and sin(π/2) = 1. By the right hand rule, the direction of
→ →
i × j

is in the +k
^
as shown in Figure 3.30. Thus ^i × ^j = k
^

3.4.2 https://phys.libretexts.org/@go/page/24434
Figure 3.30: Vector product of ^i × ^j . (CC BY-NC; Ümit Kaya)
We note that the same rule applies for the unit vectors in the y and z directions,
^ ^ ^ ^ ^ ^
j × k = i, k× i = j

By the anti-commutatively property (1) of the vector product,


^ ^ ^ ^ ^ ^
j × i = −k, i × k = −j

The vector product of the unit vector ^i with itself is zero because the two unit vectors are parallel to each other, ( sin(0) = 0 ),
^ ^ ^ ^
| i × i | = | i ∥ i | sin(0) = 0.

The vector product of the unit vector ^j with itself and the unit vector k
^
with itself are also zero for the same reason,
^ ^ ^ ^
| j × j | = 0, | k × k| = 0.

With these properties in mind we can now develop an algebraic expression for the vector product in terms of components. Let’s

choose a Cartesian coordinate system with the vector B pointing along the positive \(x\)-axis with positive \(x\)-component Bx .
→ →
Then the vectors A and B can be written as

^ ^ ^
A = Ax i + Ay j + Az k

\\overrightarrow{\mathbf{B}}=B_{x} \hat{\mathbf{i}} \nonumber \]respectively. The vector product in vector components is


→ →
^ ^ ^ ^
A × B = (Ax i + Ay j + Az k) × Bx i

This becomes,
→ →
^ ^ ^ ^ ^ ^
A × B = (Ax i × Bx i ) + (Ay j × Bx i ) + (Az k × Bx i )

^ ^ ^ ^ ^ ^
= Ax Bx ( i × i ) + Ay Bx ( j × i ) + Az Bx (k × i ) 

^ ^
= −Ay Bx k + Az Bx j

The vector component expression for the vector product easily generalizes for arbitrary vectors

^ ^ ^
A = Ax i + Ay j + Az k


^ ^ ^
B = Bx i + By j + Bz k

to yield
→ →
^ ^ ^
A × B = (Ay Bz − Az By ) i + (Az Bx − Ax Bz ) j + (Ax By − Ay Bx ) k

Vector Decomposition and the Vector Product: Cylindrical Coordinates


Recall the cylindrical coordinate system, which we show in Figure 3.31. We have chosen two directions, radial and tangential in the
plane, and a perpendicular direction to the plane.

3.4.3 https://phys.libretexts.org/@go/page/24434
Figure 3.31: Cylindrical coordinates. (CC BY-NC; Ümit Kaya)

The unit vectors are at right angles to each other and so using the right hand rule, the vector product of the unit vectors are given by
the relations

^ ^ ^
r ×θ = k

^ ^
θ ×k = ^
r

^ ^
k×^
r =θ

→ → → →
Because the vector product satisfies A × B = −B × A, we also have that
^ ^
θ ×^
r = −k

^ ^
k × θ = −^
r

^ ^ ^
r × k = −θ

Finally

^ ^ ^ ^ ^
r ×^
r = θ ×θ = k×k = 0

Example 3.6: Vector Products


→ → → →
Given two vectors, ^ ^ ^ ^ ^ ^
A = 2 i + −3 j + 7 k and  B = 5 i + j + 2 k,  find  A × B

Solution
→ →
^ ^ ^
A × B = (Ay Bz − Az By ) i + (Az Bx − Ax Bz ) j + (Ax By − Ay Bx ) k

^ ^ ^
= ((−3)(2) − (7)(1)) i + ((7)(5) − (2)(2)) j + ((2)(1) − (−3)(5))k

^ ^ ^
= −13 i + 31 j + 17 k

Example 3.7: Law of Sines


→ → →
For the triangle shown in Figure 3.32(a), prove the law of wines, | A |/ sin α = | B |/ sin β = | C |/ sin γ, using the vector
product.

3.4.4 https://phys.libretexts.org/@go/page/24434
Figure 3.32(a): Example 3.6. (CC BY-NC; Ümit
Kaya) Figure 3.32(b): Vector analysis. (CC BY-NC; Ümit Kaya)

Solution
→ → → → → →
Consider the area of a triangle formed by three vectors A , B , and C , where A + B + C =0 (Figure 3.32(b)). Because
→ → → → → → → → → → → → → → →
A + B + C =0 , we have that 0 = A × ( A + B + C ) . Thus A × B = −A × C or | A × B | = | A × C | .
From Figure 17.7b, we see that
→ → → →
| A × B | = | A || B | sin γ

and
→ → → →
| A × C | = | A || C | sin β .
Therefore,
→ → → →
| A || B | sin γ = | A || C | sin β ,
and hence
→ →
| B |/ sin β = | C |/ sin γ .
→ →
A similar argument shows that | B |/ sin β = | A |/ sin α proving the law of sines.

Example 3.8: Unit Normal


→ →
Find a unit vector perpendicular to ^ ^ ^
A = i + j −k and ^ ^ ^
B = −2 i − j + 3 k.

Solution
→ → → → → → → →
The vector product A × B is perpendicular to both A and B. Therefore the unit vectors ^ = ± A × B /| A × B
n are
→ →
perpendicular to both A and B. We first calculate
→ →
^ ^ ^
A × B = (Ay Bz − Az By ) i + (Az Bx − Ax Bz ) j + (Ax By − Ay Bx ) k

^ ^ ^
= ((1)(3) − (−1)(−1)) i + ((−1)(2) − (1)(3)) j + ((1)(−1) − (1)(2))k

^ ^ ^
= 2 i − 5 j − 3k

We now calculate the magnitude


→ → 1/2
2 2 2 1/2
| A × B | = (2 +5 +3 ) = (38 ) .

Therefore the perpendicular unit vectors are

3.4.5 https://phys.libretexts.org/@go/page/24434
→ → → →
1/2
^ = ± A × B /| A × B | = ±(2 ^
n
^ ^
i − 5 j − 3 k)/(38 )

Example 3.9: Volume of Parallelepiped


→ → → → → →
Show that the volume of a parallelepiped with edges formed by the vectors A, B , and C is given by A ⋅ (B × C)

Solution
→ →
The volume of a parallelepiped is given by area of the base times height. If the base is formed by the vectors B , and C , then
→ → → → → →
the area of the base is given by the magnitude of B × C. The vector B × C = | B × C |n
^ where n
^ is a unit vector
perpendicular to the base (Figure 3.33).

Figure 3.33 Example 3.9. (CC BY-NC; Ümit Kaya)



The projection of the vector A along the direction n
^ gives the height of the parallelepiped. This projection is given by taking
→ →
the dot product of A with a unit vector and is equal to ^ = height.
A ⋅n Therefore
→ → → → → → → → →
A ⋅ ( B × C ) = A ⋅ (| B × C |)n
^ = (| B × C |) A ⋅ n
^ = (area)(height) = (volume)

Example 3.10: Vector Decomposition


→ → → →

Let A be an arbitrary vector and let n be a unit vector in some fixed direction. Show that ^ )n
A = (A ⋅ n ^ + (n
^ × A) × n
^ .
Solution
→ → →
Let A = A∥ n
^ + A⊥ e
^ where A∥ is the component A in the direction of n
^ , e
^ is the direction of projection of A in a plane
→ →
perpendicular to n
^ , and A is the component of
⊥ A in the direction of e
^ . Because e ^ = 0 we have that
^⋅n ^ = A∥
A ⋅n . Note
that

n
^ ×A =n
^ × (An
^ + A⊥ e
^) = n
^ × A⊥ e
^ = A⊥ (n
^ ×e
^)

The unit vector n ^ lies in the plane perpendicular to n


^ ×e ^ and is also perpendicular to e
^ . Therefore (n
^ ×e ^ is also a unit
^) × n

vector that is parallel to e
^ (by the right hand rule. So (n
^ × A) × n
^ =A ⊥e
^ . Thus
→ → →
^ + A⊥ e
A = A∥ n ^ = (A ⋅ n
^ )n
^ + (n
^ × A) × n
^

This page titled 3.4: Vector Product (Cross Product) is shared under a CC BY-NC-SA 4.0 license and was authored, remixed, and/or curated by
Peter Dourmashkin (MIT OpenCourseWare) via source content that was edited to the style and standards of the LibreTexts platform; a detailed
edit history is available upon request.

3.4.6 https://phys.libretexts.org/@go/page/24434
CHAPTER OVERVIEW
4: One Dimensional Kinematics
"In the first place, what do we mean by time and space? It turns out that these deep philosophical questions have to be analyzed
very carefully in physics, and this is not easy to do. The theory of relativity shows that our ideas of space and time are not as simple
as one might imagine at first sight. However, for our present purposes, for the accuracy that we need at first, we need not be very
careful about defining things precisely. Perhaps you say, “That’s a terrible thing—I learned that in science we have to define
everything precisely.” We cannot define anything precisely! If we attempt to, we get into that paralysis of thought that comes to
philosophers, who sit opposite each other, one saying to the other, “You don’t know what you are talking about!” The second one
says. “What do you mean by know? What do you mean by talking? What do you mean by you?”, and so on. In order to be able to
talk constructively, we just have to agree that we are talking roughly about the same thing. You know as much about time as you
need for the present, but remember that there are some subtleties that have to be discussed; we shall discuss this later [1]" - Richard
Feynman
4.1: Introduction to One Dimensional Kinematics
4.2: Position, Time Interval, and Displacement
4.3: Velocity
4.4: Acceleration
4.5: Constant Acceleration
4.6: One Dimensional Kinematics and Integration

References
1. Richard P. Feynman, Robert B. Leighton, Matthew Sands, The Feynman Lectures on Physics, Addison-Wesley, Reading,
Massachusetts, (1963), p. 12-2.

This page titled 4: One Dimensional Kinematics is shared under a CC BY-NC-SA 4.0 license and was authored, remixed, and/or curated by Peter
Dourmashkin (MIT OpenCourseWare) via source content that was edited to the style and standards of the LibreTexts platform; a detailed edit
history is available upon request.

1
4.1: Introduction to One Dimensional Kinematics
Kinematics is the mathematical description of motion. The term is derived from the Greek word kinema, meaning movement. In
order to quantify motion, a mathematical coordinate system, called a reference frame, is used to describe space and time. Once a
reference frame has been chosen, we shall introduce the physical concepts of position, velocity, and acceleration in a
mathematically precise manner. Figure 4.1 shows a Cartesian coordinate system in one dimension with unit vector ^i pointing in the
direction of increasing x -coordinate.

Figure 4.1.1 : A one-dimensional Cartesian coordinate system. (CC BY-NC; Ümit Kaya)

This page titled 4.1: Introduction to One Dimensional Kinematics is shared under a CC BY-NC-SA 4.0 license and was authored, remixed, and/or
curated by Peter Dourmashkin (MIT OpenCourseWare) via source content that was edited to the style and standards of the LibreTexts platform; a
detailed edit history is available upon request.

4.1.1 https://phys.libretexts.org/@go/page/24438
4.2: Position, Time Interval, and Displacement
Position
Consider a point-like object moving in one dimension. We denote the position coordinate of the object with respect to the choice of
origin by x(t). The position coordinate is a function of time and can be positive, zero, or negative, depending on the location of the
object. The position of the object with respect to the origin has both direction and magnitude, and hence is a vector (Figure 4.2.1),
which we shall denote as the position vector (or simply position) and write as

^
r (t) = x(t) i

We denote the position coordinate at t = 0 by the symbol x 0 ≡ x(t = 0) . The SI unit for position is the meter [m].

Figure 4.2.1 : The position vector, with reference to a chosen origin. (CC BY-NC; Ümit Kaya)

Time Interval
Consider a closed interval of time [t1,t2]. We characterize this time interval by the difference in endpoints of the interval,

Δt = t2 − t1 .

The SI units for time intervals are seconds [s].

Displacement
The displacement of a body during a time interval [t1, t2] (Figure 4.2.2) is defined to be the change in the position of the body
→ → →
^ ^
Δ r ≡ r (t2 ) − r (t1 ) = (x (t2 ) − x (t1 )) i ≡ Δx(t) i

Displacement is a vector quantity.

Figure 4.2.2 : The displacement vector of an object over a time interval is the vector difference between the two position vectors.
(CC BY-NC; Ümit Kaya)

This page titled 4.2: Position, Time Interval, and Displacement is shared under a CC BY-NC-SA 4.0 license and was authored, remixed, and/or
curated by Peter Dourmashkin (MIT OpenCourseWare) via source content that was edited to the style and standards of the LibreTexts platform; a
detailed edit history is available upon request.

4.2.1 https://phys.libretexts.org/@go/page/24439
4.3: Velocity
When describing the motion of objects, words like “speed” and “velocity” are used in natural language; however, when introducing
a mathematical description of motion, we need to define these terms precisely. Our procedure will be to define average quantities
for finite intervals of time and then examine what happens in the limit as the time interval becomes infinitesimally small. This will
lead us to the mathematical concept that velocity at an instant in time is the derivative of the position with respect to time.

Average Velocity
The x -component of the average velocity, Vx,ave , for a time interval Δt is defined to be the displacement Δx divided by the time
interval Δt,
Δx
vx,ave ≡
Δt

Because we are describing one-dimensional motion we shall drop the subscript x and denote

vave = vx,ave

When we introduce two-dimensional motion we will distinguish the components of the velocity by subscripts. The average velocity
is then

→ Δx
^ ^
v ave ≡ i = vave i
Δt

The SI units for average velocity are meters per second [m⋅ s-1]. The average velocity is not necessarily equal to the distance in the
time interval Δt traveled divided by the time interval Δt. For example, during a time interval, an object moves in the positive x-
direction and then returns to its starting position, the displacement of the object is zero, but the distance traveled is non-zero.

Instantaneous Velocity
Consider a body moving in one direction. During the time interval [t, t + Δt] , the average velocity corresponds to the slope of the
line connecting the points (t,x(t)) and (t +Δt, x(t + Δt)) . The slope, the rise over the run, is the change in position divided by the
change in time, and is given by

rise Δx x(t + Δt) − x(t)


vave ≡ = =
run Δt Δt

As Δt → 0 , the slope of the lines connecting the points (t, x(t)) and (t + Δt, x(t + Δt)) , approach slope of the tangent line to the
graph of the function x(t) at the time t (Figure 4.3.1).

Figure 4.3.1 : Plot of position vs. time showing the tangent line at time t. (CC BY-NC; Ümit Kaya)
The limiting value of this sequence is defined to be the x -component of the instantaneous velocity at the time t .
The x -component of instantaneous velocity at time t is given by the slope of the tangent line to the graph of the position function
at time t :

4.3.1 https://phys.libretexts.org/@go/page/24440
Δx x(t + Δt) − x(t) dx
v(t) ≡ lim vave = lim = lim ≡
Δt→0 Δt→0 Δt Δt→0 Δt dt

The instantaneous velocity vector is then



^
v (t) = v(t) i

The component of the velocity, v(t) , can be positive, zero, or negative, depending on whether the object is travelling in the positive
x -direction, instantaneously at rest, or the negative x-direction.

Example 4.3.1: Determining Velocity from Position

Consider an object that is moving along the x -coordinate axis with the position function given by
1
2
x(t) = x0 + bt
2

where x0 is the initial position of the object at t = 0 . We can explicitly calculate the x - component of instantaneous velocity
from Equation (4.3.5) by first calculating the displacement in the x -direction, Δx = x(t + Δt) − x(t) . We need to calculate the
position at time t + Δt ,
1 1
2 2 2
x(t + Δt) = x0 + b(t + Δt) = x0 + b (t + 2tΔt + Δt )
2 2

Then the x-component of instantaneous velocity is


1 2 2 1 2
x(t + Δt) − x(t) (x0 + b (t + 2tΔt + Δt )) − (x0 + bt )
2 2
v(t) = lim = lim
Δt→0 Δt Δt→0 Δt

This expression reduces to


1
v(t) = lim (bt + bΔt)
Δt→0 2

The first term is independent of the interval Δt and the second term vanishes because in the limit as Δt → 0 , the term (1/ 2)bΔt
→ 0 is zero. Therefore the x -component of instantaneous velocity at time t is

v(t) = bt

In Figure 4.3.2 we plot the instantaneous velocity, v(t) , as a function of time t .

Figure 4.3.2 : Plot of instantaneous velocity as a function of time. (CC BY-NC; Ümit Kaya)

Example 4.3.2: Mean Value Theorem

Consider an object that is moving along the x -coordinate axis with the position function given by
1
2
x(t) = x0 + v0 t + bt .
2

4.3.2 https://phys.libretexts.org/@go/page/24440
The graph of x(t) vs. t is shown in Figure 4.3.2.

Figure 4.3.2 : Intermediate Value Theorem. (CC BY-NC; Ümit Kaya)


The x -component of the instantaneous velocity is
dx(t)
v(t) = = v0 + bt
dt

For the time interval [ti, , the displacement of the object is


tf ]

1 1
2 2
x (tf ) − x (ti ) = Δx = v0 (tf − ti ) + b (t − t ) = v0 (tf − ti ) + b (tf − ti ) (tf + ti )
f i
2 2

Recall that the x -component of the average velocity is defined by the condition that

Δx = vave (tf − ti )

We can determine the average velocity by substituting Equation (4.3.15) into Equation (4.3.14) yielding
1
vave = v0 + b (tf + ti )
2

The Mean Value Theorem from calculus states that there exists an instant in time t1 , with ti< t1 < tf , such that the x -component
of the instantaneously velocity, v(t1) , satisfies

Δx = v (t1 ) (tf − ti )

Geometrically this means that the slope of the straight line (blue line in Figure 4.3.2) connecting the points (ti , x(ti)) to (tf ,
x(tf)) is equal to the slope of the tangent line (red line in Figure 4.6) to the graph of x(t) vs. t at the point (t1 ,x(t1)) (Figure 4.6),

v (t1 ) = vave

We know from Equation (4.3.13) that

v (t1 ) = v0 + b t1

We can solve for the time t1 by substituting Equations (4.3.19) and (4.3.16) into Equation (4.3.18) yielding

t1 = (tf + ti ) /2

This intermediate value v(t1) is also equal to one-half the sum of the initial velocity and final velocity
v (ti ) + v (tf ) (v0 + b ti ) + (v0 + b tf ) 1
v (t1 ) = = = v0 + b (tf + ti ) = v0 + b t1
2 2 2

4.3.3 https://phys.libretexts.org/@go/page/24440
For any time interval, the quantity (v (t ) + v (t )) /2 , is the arithmetic mean of the initial velocity and the final velocity (but
i f

unfortunately is also sometimes referred to as the average velocity). The average velocity, which we defined as
v ave= (x − x ) /Δt , and the arithmetic mean, (v (t ) + v (t )) /2 , are only equal in the special case when the velocity is a
f i i f

linear function in the variable t as in this example, (Equation (4.3.13)). We shall only use the term average velocity to mean
displacement divided by the time interval.

This page titled 4.3: Velocity is shared under a CC BY-NC-SA 4.0 license and was authored, remixed, and/or curated by Peter Dourmashkin (MIT
OpenCourseWare) via source content that was edited to the style and standards of the LibreTexts platform; a detailed edit history is available upon
request.

4.3.4 https://phys.libretexts.org/@go/page/24440
4.4: Acceleration
We shall apply the same physical and mathematical procedure for defining acceleration, as the rate of change of velocity with
respect to time. We first consider how the instantaneous velocity changes over a fixed time interval of time and then take the limit
as the time interval approaches zero.

Average Acceleration
Average acceleration is the quantity that measures a change in velocity over a particular time interval. Suppose during a time
interval Δt a body undergoes a change in velocity
→ → →
Δ v = v (t + Δt) − v (t)

The change in the x -component of the velocity, Δv, for the time interval [t, t + Δt] is then
Δv = v(t + Δt) − v(t)

Definition: x-Component of the Average Acceleration


The x-component of the average acceleration for the time interval Δt is defined to be

→ Δv (v(t + Δt) − v(t))


^ ^ ^
a ave = aave i ≡ i = i
Δt Δt

The SI units for average acceleration are meters per second squared, [m⋅s-2].

Instantaneous Acceleration
Consider the graph of the x -component of velocity, v(t) , (Figure 4.4.1).

Figure 4.4.1 : Graph of velocity vs. time showing the tangent line at time t . (CC BY-NC; Ümit Kaya)
The average acceleration for a fixed time interval Δt is the slope of the straight line connecting the two points (t, v(t)) and (t + Δt,
v(t + Δt)). In order to define the x - component of the instantaneous acceleration at time t, we employ the same limiting argument as
we did when we defined the instantaneous velocity in terms of the slope of the tangent line.

Definition: x - Component of the Instantaneous Acceleration


The x - component of the instantaneous acceleration at time t is the slope of the tangent line at time t of the graph of the x -
component of the velocity as a function of time,

Δv (v(t + Δt) − v(t)) dv


a(t) ≡ lim = lim ≡ (4.4.1)
Δt→0 Δt Δt→0 Δt dt

The instantaneous acceleration vector at time t is then

4.4.1 https://phys.libretexts.org/@go/page/24441

^
a (t) = a(t) i

Because the velocity is the derivative of position with respect to time, the x -component of the acceleration is the second
derivative of the position function,
2
dv d x
a = =
2
dt dt

Example 4.4.1: Determining Acceleration from Velocity

Let’s continue Example 4.1, in which the position function for the body is given by x = x + (1/2)bt , and the x-component
0
2

of the velocity is v = bt . The x-component of the instantaneous acceleration is the first derivative (with respect to time) of the
x - component of the velocity:
dv v(t + Δt) − v(t) bt + bΔt − bt
a = = lim = lim =b
dt Δt→0 Δt Δt→0 Δt

Note that in Equation 4.4.1, the ratio Δv/Δt is independent of t , consistent with the constant slope as shown in Figure 4.4.1.

This page titled 4.4: Acceleration is shared under a CC BY-NC-SA 4.0 license and was authored, remixed, and/or curated by Peter Dourmashkin
(MIT OpenCourseWare) via source content that was edited to the style and standards of the LibreTexts platform; a detailed edit history is
available upon request.

4.4.2 https://phys.libretexts.org/@go/page/24441
4.5: Constant Acceleration
When the x -component of the velocity is a linear function (Figure 4.5.1a ), the average acceleration, Δv / Δt, is a constant and
hence is equal to the instantaneous acceleration (Figure 4.5.1b).

Figure 4.5.1 : Constant acceleration: (a) velocity, (b) acceleration. (CC BY-NC; Ümit Kaya)
Let’s consider a body undergoing constant acceleration for a time interval [0, t], where Δt = t . Denote the x -component of the
velocity at time t =0 by (b) t. Therefore the x -component of the acceleration is given by
Δv v(t) − v0
a(t) = =
Δt t

Thus, the x -component of the velocity is a linear function of time given by

v(t) = v0 + at

Velocity: Area Under the Acceleration vs. Time Graph


In Figure 4.8(b), the area under the acceleration vs. time graph, for the time interval Δt = t − 0 = t , is

Area(a(t), t) = at

From Equation (4.5.2), the area is the change in the x -component of the velocity for the interval [0, t]:

Area(a(t), t) = at = v(t) − v0 = Δv

Displacement: Area Under the Velocity vs. Time Graph


In Figure 4.5.2 shows a graph of the x -component of the velocity vs. time for the case of constant acceleration (Equation (4.5.2)).

Figure 4.5.2 : Graph of velocity as a function of time for a constant. (CC BY-NC; Ümit Kaya)
The region under the velocity vs. time curve is a trapezoid, formed from a rectangle with area A 1 = v0 t , and a triangle with area
A = (1/2) (v(t) − v ) The total area of the trapezoid is given by
2 0

1
Area(v(t), t) = A1 + A2 = v0 t + (v(t) − v0 )
2

Substituting for the velocity (Equation (4.5.2)) yields


1
2
 Area(v(t), t) = v0 t + at
2

4.5.1 https://phys.libretexts.org/@go/page/24442
Recall that from Example 4.2 (setting b = a and Δt = t ),
1
vave = v0 + at = Δx/t
2

therefore Equation (4.5.6) can be rewritten as


1
Area(v(t), t) = ( v0 + at) t = vave t = Δx
2

The displacement is equal to the area under the graph of the x -component of the velocity vs. time. The position as a function of
time can now be found by rewriting Equation (4.5.8) as
1
2
x(t) = x0 + v0 t + at
2

Figure 4.5.3 shows a graph of this equation. Notice that at t =0 the slope is non-zero, corresponding to the initial velocity
component v 0

Figure 4.5.3 : Graph of position vs. time for constant acceleration. (CC BY-NC; Ümit Kaya)

Example 4.4 Accelerating Car


A car, starting at rest at t = 0 , accelerates in a straight line for 100 m with an unknown constant acceleration. It reaches a
speed of 20 m ⋅ s 1 and then continues at this speed for another 10 s.

a. Write down the equations for the position and velocity of the car as a function of time.
b. How long was the car accelerating?
c. What was the magnitude of the acceleration?
d. Plot speed vs. time, acceleration vs. time, and position vs. time for the entire motion.
e. What was the average velocity for the entire trip?
Solution
(a) For the acceleration a , the position x(t) and velocity v(t) as a function of time t for a car starting from rest are
2
x(t) = (1/2)at

vx (t) = at

b) Denote the time interval during which the car accelerated by t . We know that the position
1

x (t1 ) = 100m and v (t1 ) = 20m ⋅ s


−1
. Note that we can eliminate the acceleration a between the Equations (4.4.10) to
obtain

x(t) = (1/2)v(t)t

We can solve this equation for time as a function of the distance and the final speed giving
x(t)
t =2
v(t)

We can now substitute our known values for the position x (t1 ) = 100m and v (t1 ) = 20m ⋅ s
−1
and solve for the time
interval that the car has accelerated

4.5.2 https://phys.libretexts.org/@go/page/24442
x (t1 ) 100m
t1 = 2 =2 = 10s
−1
v (t1 ) 20m ⋅ s

c) We can substitute into either of the expressions in Equation (4.4.10); the second is slightly easier to use,
−1
v (t1 ) 20m ⋅ s −2
a = = = 2.0m ⋅ s
t1 10s

d) The x -component of acceleration vs. time, x -component of the velocity vs. time, and the position vs. time are piece-wise
functions given by
−2
2m ⋅ s ; 0 < t ≤ 10s
a(t) = {
0; 10s < t < 20s

−2
(2m ⋅ s ) t; 0 < t ≤ 10s
v(t) = {
−1
20m ⋅ s ; 10s ≤ t ≤ 20s

−2 2
(1/2) (2m ⋅ s )t ; 0 < t ≤ 10s
x(t) = {
−2
100m + (20m ⋅ s ) (t − 10s); 10s ≤ t ≤ 20s

The graphs of the x -component of acceleration vs. time, x -component of the velocity vs. time, and the position vs. time are
shown in Figure 4.11.
(e) After accelerating, the car travels for an additional ten seconds at constant speed and during this interval the car travels an
additional distance Δx = v (t ) × 10s = 200m (note that this is twice the distance traveled during the 10 s of acceleration), so
1

the total distance traveled is 300 m and the total time is 20 s, for an average velocity of
300m
−1
vave  = = 15m ⋅ s
20s

Figure 4.5.4 : Graphs of the x-components of acceleration, velocity and position as piecewise functions. (CC BY-NC; Ümit
Kaya)

Example 4.5.1: Catching a Bus

At the instant a traffic light turns green, a car starts from rest with a given constant acceleration, 3.0m ⋅ s . Just as the light
−2

turns green, a bus, traveling with a given constant velocity, 1.6 × 10 m ⋅ s , passes the car. The car speeds up and passes the
1 −2

bus sometime later. How far down the road has the car traveled, when the car passes the bus?
Solution
There are two moving objects, bus and the car. Each object undergoes one stage of one-dimensional motion. We are given the
acceleration of the car, the velocity of the bus, and infer that the position of the car and the bus are equal when the bus just
passes the car. Figure 4.5.5 shows a qualitative sketch of the position of the car and bus as a function of time.

4.5.3 https://phys.libretexts.org/@go/page/24442
Figure 4.5.5 : Position vs. time of the car and bus. (CC BY-NC; Ümit Kaya)
Choose a coordinate system with the origin at the traffic light and the positive x-direction such that car and bus are traveling in
the positive x-direction. Set time t = 0 as the instant the car and bus pass each other at the origin when the light turns green.
Figure 4.5.6 shows the position of the car and bus at time t .

Figure 4.5.6 : Coordinate system for car and bus. (CC BY-NC; Ümit Kaya)
Let x (t) denote the position function of the car, and x (t) the position function for the bus. The initial position and initial
1 2

velocity of the car are both zero, x = 0 and v = 0 , and the acceleration of the car is non-zero a ≠ 0 . Therefore the
1,0 1,0 1

position and velocity functions of the car are given by


1 2
x1 (t) = a1 t
2

v1 (t) = a1 t

The initial position of the bus is zero, (x (t) = 0 , the initial velocity of the bus is non-zero, v ≠ 0 , and the acceleration of
2,0 2,0

the bus is zero, (a_{2}\) = 0. Therefore the velocity is constant v (t) = v , and the position function for the bus is given by
2 2,0

x (t) = v
2 t.
2,0

Let t = t correspond to the time that the car passes the bus. Then at that instant, the position functions of the bus and car are
a

equal, x (t ) = x (t ) . We can use this condition to solve for t :


1 a 2 a a

1 −1
2v2,0 (2) (1.6 × 10 m ⋅ s )
2 1
(1/2)a1 ta = v2,0 ta ⇒ ta = = = 1.1 × 10 s
−2
a1 (3.0m ⋅ s )

Therefore the position of the car at t isa

2 2
1 −1
1 2v (2) (1.6 × 10 m ⋅ s )
2,0
2 2
x1 (ta ) = a1 ta = = = 1.7 × 10 m
−2
2 a1 (3.0m ⋅ s )

This page titled 4.5: Constant Acceleration is shared under a CC BY-NC-SA 4.0 license and was authored, remixed, and/or curated by Peter
Dourmashkin (MIT OpenCourseWare) via source content that was edited to the style and standards of the LibreTexts platform; a detailed edit
history is available upon request.

4.5.4 https://phys.libretexts.org/@go/page/24442
4.6: One Dimensional Kinematics and Integration
When the acceleration a(t) of an object is a non-constant function of time, we would like to determine the time dependence of the
position function x(t) and the x -component of the velocity v(t) . Because the acceleration is non-constant we no longer can use
Equations (4.4.2) and (4.4.9). Instead, we shall use integration techniques to determine these functions.

Change of Velocity as the Indefinite Integral of Acceleration


Consider a time interval t1 < t < t2 . Recall that by definition the derivative of the velocity v(t) is equal to the acceleration a(t) ,
dv(t)
= a(t)
dt

Integration is defined as the inverse operation of differentiation or the ‘anti-derivative’. For our example, the function v(t) is called
the indefinite integral of a(t) with respect to t , and is unique up to an additive constant C. We denote this by writing

v(t) + C = ∫ a(t)dt (4.6.1)

The symbol ∫ … dt means the ‘integral, with respect to t , of …”, and is thought of as the d inverse of the symbol d

dt
.... .
Equivalently we can write the differential dv(t) = a(t)dt , dt called the integrand, and then Equation 4.6.1 can be written as

v(t) + C = ∫ dv(t)

which we interpret by saying that the integral of the differential of function is equal to the function plus a constant.

Example 4.6.1: Non-constant Acceleration

Suppose an object at time t = 0 has initial non-zero velocity v and acceleration a(t) = bt , where b is a constant. Then
0
2

2 3
dv(t) = b t dt = d (b t /3) .

The velocity is then

3 3
v(t) + C = ∫ d (b t /3) = b t /3.

At t = 0 , we have that v 0 +C = 0 . Therefore C = −v0 and the velocity as a function of time is then
3
v(t) = v0 + (b t /3) .

Area as the Indefinite Integral of Acceleration


Consider the graph of a positive-valued acceleration function a(t) vs. t for the interval t1 ≤ t ≤ t2 , shown in Figure 4.6.1a .
Denote the area under the graph of a(t) over the interval t ≤ t ≤ t by A .
1 2
t2
t1

4.6.1 https://phys.libretexts.org/@go/page/24443
Figure 4.6.1 : (a; left) Area under the graph of acceleration over an interval t ≤ t ≤ t . (b; right) Intermediate value Theorem. The
1 2

shaded regions above and below the curve have equal areas . (CC BY-NC; Ümit Kaya)
The Intermediate Value Theorem states that there is at least one-time t such that the area A is equal to
c
t2

t1

t2
A = a (tc ) (t2 − t1 )
t1

In Figure 4.6.1b, the shaded regions above and below the curve have equal areas, and hence the area A under the curve is equal to t2
t1

the area of the rectangle given by a (t ) (t − t )


c 2 1

Figure 4.6.2 : Area function is additive. (CC BY-NC; Ümit Kaya)


We shall now show that the derivative of the area function is equal to the acceleration and therefore we can write the area function
as an indefinite integral. From Figure 4.6.2, the area function satisfies the condition that
t t+Δt t+Δt
A +A =A
t1 t t1

Let the small increment of area be denoted by ΔA t


t1
= At
t+Δt

1
− At
t

1
= At
t+Δt
. By the Intermediate Value Theorem
t
ΔA = a (tc ) Δt
t1

where t ≤ t c ≤ t + Δt . In the limit as Δt → 0


t t
dA ΔA
t1 t1
= lim = lim a (tc ) = a(t)
dt Δt→0 Δt tc →t

with the initial condition that when t = t , the area A


1
t1

t1
=0 is zero. Because v(t) is also an integral of a(t) , we have that

t
A =∫ a(t)dt = v(t) + C
t1

4.6.2 https://phys.libretexts.org/@go/page/24443
When t = t , the area A
1
t1
t1
=0 is zero, therefore v (t 1) +C = 0 , and so C = −v (t1 ) . Therefore Equation (4.6.8) becomes

t
A = v(t) − v (t1 ) = ∫ a(t)dt
t1

When we set t = t , Equation (4.6.9) becomes


2

t2
A = v (t2 ) − v (t1 ) = ∫ a(t)dt
t1

The area under the graph of the positive-valued acceleration function for the interval t 1 ≤ t ≤ t2 can be found by integrating a(t)

Change of Velocity as the Definite Integral of Acceleration


Let a(t) be the acceleration function over the interval t ≤ t ≤ t . Recall that the velocity v(t) is an integral of a(t) because
i f

dv(t)/dt = a(t) . Divide the time interval [ t , t ] into n equal time subintervals Δt = (t − t ) /n . For each subinterval [ t , t
i f ], f i j j+1

where the index j = 1, 2, … , n, t = t  and t


1 i =t , let t be a time such that t ≤ t ≤ t
n+1 f cj . Let j cj j+1

j=n

Sn = ∑ a (tcj ) Δt

Sn is the sum of the blue rectangle shown in Figure 4.16a for the case n = 4 . The Fundamental Theorem of Calculus states that in
the limit as n → ∞ , the sum is equal to the change in the velocity during the interval [t , t ] i f

j=n

lim Sn = lim ∑ a (tcj ) Δt = v (tf ) − v (ti )


n→∞ n→∞
j=1

Figure 4.6.3 : (a; left) Graph of a(t) vs. t . (b; right) Graph of a(t) vs. t . (CC BY-NC; Ümit Kaya)
The limit of the sum in Equation (4.5.12) is a number, which we denote by the symbol
tf j=n

∫ a(t)dt ≡ lim ∑ a (tcj ) Δt = v (tf ) − v (ti )


n→∞
ti j=1

and is called the definite integral of a(t) from t tot . The times t andt are called the limits of integration, t the lower limit and
i f i f i

t
f the upper limit. The definite integral is a linear map that takes a function a(t) defined over the interval [t , t ] and gives a i f

number. The map is linear because


tf tf tf

∫ (a1 (t) + a2 (t)) dt = ∫ a1 (t)dt + ∫ (t)dt


0 0 a2

Suppose the times t cj , j = 1, … , n are selected such that each t satisfies the Intermediate Value Theorem,
cj

4.6.3 https://phys.libretexts.org/@go/page/24443
dv (tcj )
Δvj ≡ v (tj+1 ) − v (tj ) = Δt = a (tc ) Δt
j
dt

where a(t ) is the instantaneous acceleration at a (tc )


j
, (Figure 4.16b). Then the sum of the cj cj changes in the velocity for the
interval [t , t ] is
i f

j=n
∑ Δvj = (v (t2 ) − v (t1 )) + (v (t3 ) − v (t2 )) + ⋯ + (v (tn+1 ) − v (tn )) = v (tn+1 ) − v (t1 ) = v (tf ) − v (ti )
j=1

where v (t ) = v (t )  and v (t ) = v (t ) Substituting Equation (4.5.15) into Equation (4.5.16) yields the exact result that the
f n+1 i 1

change in the x -component of the velocity is give by this finite sum.


j=n j=n

v (tf ) − v (ti ) = ∑ Δvj = ∑ a (tcj ) Δt

j=1 j=1

We do not specifically know the intermediate values a (t ) and so Equation (4.5.17) is not useful as a calculating tool. The
cj

statement of the Fundamental Theorem of Calculus is that the limit as n → ∞ of the sum in Equation (4.5.12) is independent of the
choice of the set of t . Therefore the exact result in Equation (4.5.17) is the limit of the sum.
cj

Thus we can evaluate the definite integral if we know any indefinite integral of the integrated a(t)dt = dv(t) , [t i, tf ] :

tf
tf
A =∫ a(t)dt
ti
ti

In Figure 4.6.3b, the red areas are an overestimate and the blue areas are an underestimate. As N → ∞ , the sum of the red areas
and the sum of the blue areas both approach zero. If there are intervals in which a(t) has negative values, then the summation is a
sum of signed areas, positive area above the t -axis and negative area below the t -axis.
We can determine both the change in velocity for the time interval [t , t ] and the area under the graph of a(t) vs. t for [t , t ] by
i f i f

integration techniques instead of limiting arguments. We can turn the linear map into a function of time, instead of just giving a
number, by setting t = t . In that case, Equation (4.5.13) becomes
f


t =t
′ ′
v(t) − v (ti ) = ∫ a (t ) dt

t =ti

Because the upper limit of the integral, tf = t , is now treated as a variable, we shall use the symbol t′ as the integration variable
instead of t

Displacement as the Definite Integral of Velocity


We can repeat the same argument for the definite integral of the x -component of the velocity v(t) vs. time t . Because x(t) is an
integral of v(t) the definite integral of v(t) for the time interval [t , t ] is the displacement
i f


t =tf
′ ′
x (tf ) − x (ti ) = ∫ v (t ) dt

t =ti

If we set t
f =t , then the definite integral gives us the position as a function of time

t =t
′ ′
x(t) = x (ti ) + ∫ v (t ) dt

t =ti

Summarizing the results of these last two sections, for a given acceleration a(t) , we can use integration techniques, to determine the
change in velocity and change in position for an interval [t , t], and given initial conditions (x , v ), we can determine the position
i i i

x(t) and the x-component of the velocity v(t) as functions of time.

Example 4.6.2: Non-constant Acceleration

Let’s consider a case in which the acceleration, a(t) , is not constant in time,
2
a(t) = b0 + b1 t + b2 t

The graph of the x -component of the acceleration vs. time is shown in Figure 4.6.4.

4.6.4 https://phys.libretexts.org/@go/page/24443
Figure 4.6.4 : Non-constant acceleration vs. time graph. (CC BY-NC; Ümit Kaya)

Denote the initial velocity at t = 0 by v0 . Then, the change in the x -component of the velocity as a function of time can be
found by integration:
′ ′
t =t t =t 2 3
′ ′ ′ ′2 ′
b1 t b2 t
v(t) − v0 = ∫ a (t ) dt = ∫ (b0 + b1 t + b2 t ) dt = b0 t + +
t =0
′ ′
t =0
2 3

The x -component of the velocity as a function in time is then


2 3
b1 t b2 t
v(t) = v0 + b0 t + +
2 3

Denote the initial position at t = 0 by x . The displacement as a function of time is


0


t =t
′ ′
x(t) − x0 = ∫ v (t ) dt

t =0

Use Equation (4.5.27) for the x-component of the velocity in Equation (4.5.24) and then integrate to determine the
displacement as a function of time:
′ ′
t =t t =t ′2 ′3 2 3 4
′ ′ ′
b1 t b2 t ′
b0 t b1 t b2 t
x(t) − x0 = ∫ v (t ) dt = ∫ ( v0 + b0 t + + ) dt = v0 t + + +

t =0

t =0
2 3 2 6 12

Finally the position as a function of time is then


2 3 4
b0 t b1 t b2 t
x(t) = x0 + vx,0 t + + +
2 6 12

Example 4.6.3: Bicycle and Car

A car is driving through a green light at t = 0 located at x = 0 with an initial speed v = 12m ⋅ s . At time t = 1s , the car c,0
−1
1

starts braking until it comes to rest at time t . The acceleration of the car as a function of time is given by the piecewise
2

function
0; 0 < t < t1 = 1s
ac (t) = {
b (t − t1 ) ; 1s < t < t2

where b = −(6 m ⋅s ).
a. Find the x -component of the velocity and the position of the car as a function of time.
b. A bicycle rider is riding at a constant speed of V and at t = 0 is 17 m behind the car. The bicyclist reaches the car when
b,0

the car just comes to rest. Find the speed of the bicycle.
Solution
a) In order to apply Equation (4.5.19), we shall treat each stage separately. For the time interval 0 < t < t , the acceleration is 1

zero so the x -component of the velocity is constant. For the second time interval t < t < t , the definite integral becomes 1 2


t =t
′ ′
vc (t) − vc (t1 ) = ∫ b (t − t1 ) dt

t =t1

4.6.5 https://phys.libretexts.org/@go/page/24443
Because v c (t1 ) = vc0 , the x -component of the velocity is then

vc0 ; 0 < t ≤ t1
vc (t) = { t =t

′ ′
vc0 + ∫ ′ b (t − t1 ) dt ; t1 ≤ t < t2
t =t1

Integrate and substitute the two endpoints of the definite integral, yields

vc0 ; 0 < t ≤ t1
vc (t) = {
1 2
vc0 + b (t − t1 ) ; t1 ≤ t < t2
2

In order to use Equation (4.5.25), we need to separate the definite integral into two integrals corresponding to the two stages of
motion, using the correct expression for the velocity for each integral. The position function is then

t t1
⎧ x ′
c0 + ∫ vc0 dt ; 0 < t ≤ t1
t=0
xc (t) = ⎨
′ 1 ′ 2
⎩ x (t ) + ∫ (vc0 + b (t − t1 ) ) dt; t1 ≤ t < t2
c 1 ′
t =t1 2

Upon integration we have



t =t
xc (0) + vc0 t; 0 < t ≤ t1 ∣

xc (t) = { ) ; t1 ≤ t < t2
′ 1 ′ 3 ∣
xc (t1 ) + (vc0 (t − t1 ) + b (t − t1 )
6 ∣
t=t1

We chose our coordinate system such that the initial position of the car was at the origin, x c0 = 0,  therefore xc (t1 ) = vc0 t1

So after substituting in the endpoints of the integration interval we have that

vc0 t; 0 < t ≤ t1
xc (t) = { 3
1
vc0 t1 + vc0 (t − t1 ) + b (t − t1 ) ; t1 ≤ t < t2
6

(b) We are looking for the instant t that the car has come to rest. So we use our expression for the x-component of the velocity
2

the interval t ≤ t < t , where we set t = t and v (t ) = 0 .


1 2 2 c 2

1 2
0 = vc (t2 ) = vc0 + b (t2 − t1 )
2

Solving for t yields


2

−−−−−−
2vc0
t2 = t1 + √−
b

where we have taken the positive square root. Substitute the given values then yields
−−−−−−−−−−−−
−1
2 (12m ⋅ s )
t2 = 1s + √− = 3s
−3
(−6m ⋅ s )

The position of the car at t is then given by


2

1 3
xc (t2 ) = vc0 t1 + vc0 (t2 − t1 ) + b (t2 − t1 )
6

−−−−− −− 1 3/2
xc (t2 ) = vc0 t1 + vc0 √−2 vc0 /b + b (−2 vc0 /b)
6

3/2
2 √2( v )
c0

xc (t2 ) = vc0 t1 +
1/2
3(−b)

−−−−− −−
where we used the condition that t 2 − t1 = √−2 vc0 /b . Substitute the given values then yields
– −1
3/2
– 3/2 4 √2 ( (12m ⋅ s )
4 √2(vc0 )
−1
xc (t2 ) = vc0 t1 + 2 = (12m ⋅ s ) (1s) + = 28m
1/2 −3 1/2
3(−b) 3 ((6m ⋅ s ))

b) Because the bicycle is traveling at a constant speed with an initial position = −17 m , the position of the bicycle is given by
x (t) = −17m + v t . The bicycle and b
b b car intersect at time t = 3 s , where x (t ) = x (t ) . Therefore
0 2 b 2 c 2

4.6.6 https://phys.libretexts.org/@go/page/24443
−17m + vb (3s) = 28m . So the speed of the bicycle is v b = 15m ⋅ s
−1
.

This page titled 4.6: One Dimensional Kinematics and Integration is shared under a CC BY-NC-SA 4.0 license and was authored, remixed, and/or
curated by Peter Dourmashkin (MIT OpenCourseWare) via source content that was edited to the style and standards of the LibreTexts platform; a
detailed edit history is available upon request.

4.6.7 https://phys.libretexts.org/@go/page/24443
CHAPTER OVERVIEW
5: Two Dimensional Kinematics
Where was the chap I saw in the picture somewhere? Ah yes, in the dead sea floating on his back, reading a book with a parasol
open. Couldn’t sink if you tried: so thick with salt. Because the weight of the water, no, the weight of the body in the water is equal
to the weight of the what? Or is it the volume equal to the weight? It’s a law something like that. Vance in High school cracking his
finger joints, teaching. The college curriculum. Cracking curriculum. What is weight really when you say weight? Thirty two feet
per second per second. Law of falling bodies: per second per second. They all fall to the ground. The earth. It’s the force of gravity
of the earth is the weight [1] - James Joyce
5.1: Introduction to the Vector Description of Motion in Two Dimensions
5.2: Projectile Motion

[1] James Joyce, Ulysses, The Corrected Text edited by Hans Walter Gabler with Wolfhard Steppe and Claus Melchior, Random
House, New York.

This page titled 5: Two Dimensional Kinematics is shared under a CC BY-NC-SA 4.0 license and was authored, remixed, and/or curated by Peter
Dourmashkin (MIT OpenCourseWare) via source content that was edited to the style and standards of the LibreTexts platform; a detailed edit
history is available upon request.

1
5.1: Introduction to the Vector Description of Motion in Two Dimensions
Where was the chap I saw in the picture somewhere? Ah yes, in the dead sea floating on his back, reading a book with a parasol
open. Couldn’t sink if you tried: so thick with salt. Because the weight of the water, no, the weight of the body in the water is equal
to the weight of the what? Or is it the volume equal to the weight? It’s a law something like that. Vance in High school cracking his
fingerjoints, teaching. The college curriculum. Cracking curriculum. What is weight really when you say weight? Thirtytwo feet
per second per second. Law of falling bodies: per second per second. They all fall to the ground. The earth. It’s the force of gravity
of the earth is the weight.
~ James Joyce
We have introduced the concepts of position, velocity and acceleration to describe motion in one dimension; however we live in a
multidimensional universe. In order to explore and describe motion in more than one dimension, we shall study the motion of a
projectile in two-dimension moving under the action of uniform gravitation.
We extend our definitions of position, velocity, and acceleration for an object that moves in two dimensions (in a plane) by treating
each direction independently, which we can do with vector quantities by resolving each of these quantities into components. For
example, our definition of velocity as the derivative of position holds for each component separately. In Cartesian coordinates, the

position vector r (t) with respect to some choice of origin for the object at time t is given by

^ ^
r (t) = x(t) i + y(t) j


The velocity vector v (t) at time t is the derivative of the position vector,

→ dx(t) dy(t)
^ ^ ^ ^
v (t) = i + j ≡ vx (t) i + vy (t) j
dt dt

where v x (t) ≡ dx(t)/dt and v y (t) ≡ dy(t)/dt denote the x - and y -components of the velocity respectively.

The acceleration vector a (t) is defined in a similar fashion as the derivative of the velocity vector,

→ dvx (t) dvy (t)


^ ^ ^ ^
a (t) = i + j ≡ ax (t) i + ay (t) j
dt dt

where a x (t) ≡ dvx (t)/dt and a y (t) ≡ dvy (t)/dt denote the x- and y-components of the acceleration

This page titled 5.1: Introduction to the Vector Description of Motion in Two Dimensions is shared under a CC BY-NC-SA 4.0 license and was
authored, remixed, and/or curated by Peter Dourmashkin (MIT OpenCourseWare) via source content that was edited to the style and standards of
the LibreTexts platform; a detailed edit history is available upon request.

5.1.1 https://phys.libretexts.org/@go/page/24445
5.2: Projectile Motion

Consider the motion of a body that is released at time t = 0 with an initial velocity v0 . Two paths are shown in Figure 5.1.

Figure 5.1 Actual orbit accounting for air resistance and parabolic orbit of a projectile
The dotted path represents a parabolic trajectory and the solid path represents the actual trajectory. The difference between the two
→air
paths is due to air resistance acting on the object, F
2
^
= −b v v , where v
^ is a unit vector in the direction of the velocity. (For the

orbits shown in Figure 5.1, b = 0.01N ⋅ s
2
⋅m
−2
, ∣


v 0 = 30.0m ⋅ s

, the initial launch angle with respect to the horizontal
θ0 = 21

and the actual horizontal distance traveled is 71.7% of the projectile orbit.). There are other factors that can influence the
path of motion; a rotating body or a special shape can alter the flow of air around the body, which may induce a curved motion or
lift like the flight of a baseball or golf ball. We shall begin our analysis by neglecting all interactions except the gravitational
interaction.

Figure 5.2 A coordinate sketch for parabolic motion.


Choose coordinates with the positive y-axis in the upward vertical direction and the positive x-axis in the horizontal direction in the
direction that the object is moving horizontally. Choose the origin at the ground immediately below the point the object is released.
→ →
Figure 5.2 shows our coordinate system with the position of the object r (t) at time t , the initial velocity v0 , and the initial angle
θ with respect to the horizontal, and the coordinate functions x(t) and y(t).
0

Initial Conditions:

Figure 5.3 A vector decomposition of the initial velocity


Decompose the initial velocity vector into its components:

^ ^
v 0 = vx,0 i + vy,0 j

5.2.1 https://phys.libretexts.org/@go/page/24446
The vector decomposition for the initial velocity is shown in Figure 5.3. Often the description of the flight of a projectile includes
the statement, “a body is projected with an initial speed v at an angle θ with respect to the horizontal.” The components of the
0 0

initial velocity can be expressed in terms of the initial speed and angle according to

vx,0 = v0 cos θ0

vy,0 = v0 sin θ0

Because the initial speed is the magnitude of the initial velocity, we have that
1/2
2 2
v0 = (v +v )
x,0 y,0

The angle θ is related to the components of the initial velocity by


0

−1
θ0 = tan (vy,0 / vx,0 )

Equation (5.1.8) will give two values for the angle θ0 , so care must be taken to choose the correct physical value. The initial
position vector generally is given by

^ ^
r 0 = x0 i + y0 j

Note that the trajectory in Figure 5.3 has x0 =0 , but this will not always be the case.

Force Diagram
We begin by neglecting all forces other than the gravitational interaction between the object and the earth. This force acts
downward with magnitude mg , where m is the mass of the object and g = 9.8m ⋅ s . Figure 5.4 shows the force diagram on the
−2

object.

Figure 5.4 Free-body force diagram on the object with the action of gravity
The vector decomposition of the force is
g

^
F = −mg j

Equations of Motions
The force diagram reminds us that the force is acting in the y -direction. Newton’s Second Law states that the sum of the force,
→total  →
F , acting on the object is equal to the product of the mass m and the acceleration vector a

→total  →
F =m a

→total  →g
Because we are modeling the motion with only one force, we have that F = F . This is a vector equation; the components
are equated separately:
−mg = may

0 = max

Therefore the y -component of the acceleration is

5.2.2 https://phys.libretexts.org/@go/page/24446
ay = −g

We see that the acceleration is a constant and is independent of the mass of the object. Notice that a < 0 . This is because we chosey

our positive y -direction to point upwards. The sign of the y -component of acceleration is determined by how we choose our
coordinate system. Because there are no horizontal forces acting on the object, we conclude that the acceleration in the horizontal
direction is also zero

ax = 0

Therefore the x -component of the velocity remains unchanged throughout the flight of the object.
The acceleration in the vertical direction is constant for all bodies near the surface of the Earth, independent of the mass of the
object, thus confirming Galileo’s Law of Free Falling Bodies. Notice that the equation of motion (Equation (5.1.14)) generalizes
the experimental observation that objects fall with constant acceleration. Our statement about the acceleration of objects near the
surface of Earth depends on our model force law Equation (5.1.10), and if subsequent observations show the acceleration is not
constant then we either must include additional forces (for example, air resistance), or modify the force law (for objects that are no
longer near the surface of Earth, or consider that Earth is a non-symmetric non-uniform body), or take into account the rotational
motion of the Earth.
We can now integrate the equation of motions (Equations (5.1.14) and (5.1.15)) separately for the x - and y - directions to find
expressions for the x - and y -components of velocity and position:

t =t
′ ′
vx (t) − vx,0 =∫ ax (t ) dt = 0 ⇒ vx (t) = vx,0 (5.2.1)

t =0

′ ′
t =t t =t
′ ′ ′
x(t) − x0 =∫ vx (t ) dt = ∫ vx,0 dt = vx,0 t ⇒ x(t) = x0 + vx,0 t (5.2.2)
′ ′
t =0 t =0

′ ′
t =t t =t
′ ′ ′
vy (t) − vy,0 =∫ ay (t ) dt = − ∫ gdt = −gt ⇒ vy (t) = vy,0 − gt (5.2.3)
′ ′
t =0 t =0

t=t t=t
′ ′ ′ 2 2
y(t) − y0 =∫ vy (t ) dt = ∫ (vy,0 − gt) dt = vy,0 t − (1/2)gt ⇒ y(t) = y0 + vy,0 t − (1/2)gt (5.2.4)

t=0 t =0

The complete set of vector equations for position and velocity for each independent direction of motion are given by
→ 2
^ ^ ^ ^
r (t) = x(t) i + y(t) j = (x0 + vx,0 t) i + (y0 + vy,0 t + (1/2)ay t ) j


^ ^ ^ ^
v (t) = vx (t) i + vy (t) j = vx,0 i + (vy,0 + ay t) j


^ ^ ^
a (t) = ax (t) i + ay (t) j = ay j

Example 5.2.1: Time of Flight and Maximum Height of a Projectile

A person throws a stone at an initial angle θ = 45 from the horizontal with an initial speed of v = 20m ⋅ s . The point of
0

0
−1

release of the stone is at a height d = 2 m above the ground. You may neglect air resistance. a) How long does it take the stone
to reach the highest point of its trajectory? b) What was the maximum vertical displacement of the stone? Ignore air resistance.
Solution: Choose the origin on the ground directly underneath the point where the stone is released. We choose the positive y-
axis in the upward vertical direction and the positive x-axis in the horizontal direction in the direction that the object is moving
horizontally. Set t = 0 the instant the stone is released. At t = 0 the initial conditions are then x = 0 and y = d . The initial x -
0 0

and y -components of the velocity are given by Equations (5.1.5) and (5.1.6).
At time t the stone has coordinates (x(t), y(t)) . These coordinate functions are shown in Figure 5.5.

5.2.3 https://phys.libretexts.org/@go/page/24446
Figure 5.5: Coordinate functions for stone

Figure 5.6 Plot of the y-component of the position as a function of time


The slope of this graph at any time t yields the instantaneous y-component of the velocity v (t) at that time t . Figure 5.5 is a
y

plot of y(t) vs. x(t) and Figure 5.6 is a plot of y(t) vs. t . There are several important things to notice about Figures 5.5 and 5.6.
The first point is that the abscissa axes are different in both figures. The second thing to notice is that at t = 0 , the slope of the
graph in Figure 5.5 is equal to
dy ∣ dy/dt ∣ vy,0
∣ =( )∣ = = tan θ0
dx ∣ t=0 dx/dt ∣t=0 vx,0

while at t = 0 the slope of the graph in Figure 5.6 is equal to


dy ∣
∣ = vy,0
dt ∣t=0

The slope of this graph in Figure 5.6 at any time t yields the instantaneous y-component of the velocity v (t) at that time t . Let
y

t =t 1correspond to the instant the stone is at its maximal vertical position, the highest point in the flight. The final thing to
notice about Figure 5.6 is that a t = t the slope is zero or v (t = t ) = 0 . Therefore
1 y 1

vy (t1 ) = v0 sin θ0 − gt1 = 0

Solving Equation (5.1.21) for t yields,


1

−1 ∘
v0 sin θ0 (20m ⋅ s ) sin(45 )
t1 = = = 1.44s
−2
g 9.8m ⋅ s

The graph in Figure 5.7 shows a plot of v (t) as a function of time. Notice that at t = 0 the intercept is positive indicting that
y

Vy,0 is positive which means that the stone was thrown upwards. The y -component of the velocity changes sign at t = t 1

indicating that the stone is reversing its direction and starting to move downwards.

5.2.4 https://phys.libretexts.org/@go/page/24446
Figure 5.7 y -component of the velocity as a function of time
We now substitute the expression for t = t (Equation (5.1.22)) into the y -component of the position in Equation (5.1.16) to
top

find the maximal height of the stone above the ground


2
v0 sin θ0 1 v0 sin θ0
y (t = ttop ) = d + v0 sin θ0 − g( )
g 2 g

2 ∘
2 2 −1 2
v0 sin θ0 (20m⋅ s ) sin ( 45 )
=d+ = 2m + −2
= 12.2m
2g 2(9.8m⋅ s )

Orbit Equation
So far our description of the motion has emphasized the independence of the spatial dimensions, treating all of the kinematic
quantities as functions of time. We shall now eliminate time from our equation and find the orbit equation of the body undergoing
projectile motion. We begin with the x -component of the position in Equation (5.1.16),

x(t) = x0 + vx,0 t

and solve Equation (5.1.24) for time t as a function of x(t),


x(t) − x0
t =
vx,0

The y -component of the position in Equation (5.1.16) is given by


1
2
y(t) = y0 + vy,0 t − gt
2

We then substitute Equation (5.1.25) into Equation (5.1.26) yielding


2
x(t) − x0 1 x(t) − x0
y(t) = y0 + vy,0 ( )− g( )
vx,0 2 vx,0

A little algebraic simplification yields the equation for a parabola:

1 g gx0 vy,0 vy,0 1 g


2 2
y(t) = − x(t) +( + ) x(t) − x0 − x + y0
2 2 2 0
2 v v vx,0 vx,0 2 v
x,0 x,0 x,0

The graph of y(t) as a function of x(t) is shown in Figure 5.8.

5.2.5 https://phys.libretexts.org/@go/page/24446
Figure 5.8 The parabolic orbit
The velocity vector is given by

→ dx(t) dy(t)
^ ^ ^ ^
v (t) = i + j ≡ vx (t) i + vy (t) j
dt dt

The direction of the velocity vector at a point (x(t), y(t)) can be determined from the components. Let θ be the angle that the
velocity vector forms with respect to the positive x -axis. Then
vy (t) dy/dt dy
−1 −1 −1
θ = tan ( ) = tan ( ) = tan ( )
vx (t) dx/dt dx

Differentiating Equation (5.1.28) with respect to x yields

dy g gx0 vy,0
=− x +( + )
2 2
dx v v vx,0
x,0 x,0

The direction of the velocity vector at a point (x(t), y(t)) is therefore

g gx0 vy,0
−1
θ = tan (− x +( + ))
2 2
v v vx,0
x,0 x,0

Although we can determine the angle of the velocity, we cannot determine how fast the body moves along the parabolic orbit from
our graph of y(x) ; the magnitude of the velocity cannot be determined from information about the tangent line.
If we choose our origin at the initial position of the body at t = 0 , then x 0 =0 and y0 =0 . Our orbit equation, Equation (5.1.28)
can now be simplified to
1 g vy,0
2
y(t) = − x(t) + x(t)
2
2 v vx,0
x,0

Example 5.2.2: Hitting the Bucket

A person is holding a pail while standing on a ladder. The person releases the pail from rest at a height h , above the ground. A 1

second person, standing a horizontal distance s from the pail, aims and throws a ball the instant the pail is released in order to
hit the pail. The person releases the ball at a height h above the ground, with an initial speed v , and at an angle θ with
2 0 0

respect to the horizontal. Assume that v is large enough so that the stone will at least travel a horizontal distance s before it
0

hits the ground. You may ignore air resistance.

5.2.6 https://phys.libretexts.org/@go/page/24446
Figure 5.9: Example 5.2
a. Find an expression for the angle θ that the person aims the ball in order to hit the pail. Does the answer depend on the
0

initial velocity?
b. Find an expression for the time of collision as a function of the initial speed of the ball \v , and the quantities \h , \h ,
0 1 2

and s .
c. Find an expression for the height above the ground where the collision occurred as a function of the initial speed of the ball
\v , and the quantities \h , \h , and s.
0 1 2

Solution
There are two objects involved in this problem. Each object is undergoing free fall, so there is only one stage of motion for
each object. The pail is undergoing one-dimensional motion. The ball is undergoing two-dimensional motion. The parameters
\h , \h , \v , and s are unspecified, so our answers will be functions of those quantities. Figure 5.9 shows a sketch of the
1 2 0

motion of all the bodies in this problem.


Choose an origin on the ground directly underneath the point where the ball is released, upwards for the positive y -direction
and towards the pail for the positive x -direction. Choose position coordinates for the pail as follows. The horizontal coordinate
is constant and given by x = s . The vertical coordinate represents the height above the ground and is denoted by y (t) . The
1 1

ball has coordinates (x (t), y (t)). We show these coordinates in the Figure 5.10.
2 2

Figure 5.10: Coordinate System


The pail undergoes constant acceleration a = −g in the vertical direction and the ball undergoes uniform motion in the
1,y

horizontal direction and constant acceleration in the vertical direction, with a = 0 and a = −g .
2,x 2,y

The initial conditions for the pail are (v1,0 )


y
= 0, x1,0 = s, y1,0 = h1 . The equations for position and velocity of the pail
simplify to
1
2
y1 (t) = h1 − gt
2

5.2.7 https://phys.libretexts.org/@go/page/24446
vy,1 (t) = −gt

The initial position is given by x = 0, y = h . The components of the initial velocity are given by (v ) = v sin(θ )
2,0 2,0 2 2,0 y 0 0

and (v ) = v cos(θ ) , where v is the magnitude of the initial velocity and θ is the initial angle with respect to the
2,0 x 0 0 0 0

horizontal. The equations for the position and velocity of the ball simplify to

x2 (t) = v0 cos(θ0 )t

v2x (t) = v0 cos(θ0 )

1
2
y2 (t) = h2 + v0 sin(θ0 )t − gt
2

v2,y (t) = v0 sin(θ0 ) − gt

Note that the quantities h , h , v , and s should be treated as known quantities although no numerical values were given. There
1 2 0

are six independent equations with 8 as yet unspecified quantities y (t), t, y (t), x (t), v (t), v (t), v (t) , and θ .
1 2 2 1,y 2,y 2,x 0

So we need two more conditions, in order to find expressions for the initial angle, θ , the time of collision, t , and the spatial
0 a

location of the collision point specified by y (t ) or y (t ) . At the collision time t = t the collision occurs when the two
1 a 2 a a

balls are located at the same position. Therefore

y1 (ta ) = y2 (ta )

x2 (ta ) = x1 = s

We shall now apply these conditions that must be satisfied in order for the ball to hit the pail.
1 1
2 2
h1 − gta = h2 + v0 sin(θ0 )ta − gta
2 2

s = v0 cos(θ0 )ta

Equation (5.1.42) simplifies to

v0 sin(θ0 )ta = h1 − h2

Dividing Equation (5.1.44) by Equation (5.1.43) yields


v0 sin(θ0 )ta h1 − h2
= tan(θ0 ) =
v0 cos(θ0 )ta s2

So the initial angle θ is independent of v , and is given by


0 0

−1
θ0 = tan ((h1 − h2 ) /s)

From the Figure 5.11 we can see that tan(θ0 ) = (h1 − h2 ) /s implies that the second person aims the ball at the initial
position of the pail.

Figure 5.11: Geometry of collision

5.2.8 https://phys.libretexts.org/@go/page/24446
In order to find the time that the ball collides with the pail, we begin by squaring both Equations (5.1.44) and (5.1.43), then
utilize the trigonometric identity sin (θ ) + cos (θ ) = 1 . Our squared equations become
2
0
2
0

2 2 2 2
v sin (θ0 )ta = (h1 − h2 )
0

2 2 2 2
v cos (θ0 )ta = s
0

Adding these equations together and using the identity sin 2


(θ0 ) + cos (θ0 ) = 1
2
and taking square roots yields
1/2
2 2
v0 ta = (s + (h1 − h2 ) )

We can solve Equation (5.1.49) for the time of collision


1 1/2
2 2
ta = (s + (h1 − h2 ) )
v0

We can now use the y -coordinate function of either the ball or the pail at t = t to find the height that the ball collides with the
a

pail. Because the pail had no initial y - component of the velocity, it’s easier to use the condition for the pail,

2 2
g (s + (h1 − h2 ) )

y1 (ta ) = h1 −
2
2v
0

Comments:
(1) Equations (5.1.49) and (5.1.50) can be arrived at in a very direct way. Suppose we analyze the motion in a reference frame

that is accelerating downward with A = −g^j . In that reference frame both the pail and the stone are not accelerating; the pail
is at rest and the stone is travelling with speed v , at an angle θ . Therefore in order to hit the stationary pail, the stone must be
0 0

thrown at the angle given by Equation (5.1.46) and the time that it takes to hit the stone is just given by distance traveled
divided by speed, Equation (5.1.50).

This page titled 5.2: Projectile Motion is shared under a CC BY-NC-SA 4.0 license and was authored, remixed, and/or curated by Peter
Dourmashkin (MIT OpenCourseWare) via source content that was edited to the style and standards of the LibreTexts platform; a detailed edit
history is available upon request.

5.2.9 https://phys.libretexts.org/@go/page/24446
CHAPTER OVERVIEW
6: Circular Motion
And the seasons they go round and round
And the painted ponies go up and down
We're captive on the carousel of time
We can't return we can only look
Behind from where we came
And go round and round and round
In the circle game [1] - Joni Mitchell

[1] Joni Mitchell, The Circle Game, Siquomb Publishing Company.


6.1: Introduction to Circular Motion
6.2: Circular Motion- Velocity and Angular Velocity
6.3: Circular Motion- Tangential and Radial Acceleration
6.4: Period and Frequency for Uniform Circular Motion
6.5: Angular Velocity and Angular Acceleration
6.6: Non-circular Central Motion

This page titled 6: Circular Motion is shared under a CC BY-NC-SA 4.0 license and was authored, remixed, and/or curated by Peter Dourmashkin
(MIT OpenCourseWare) via source content that was edited to the style and standards of the LibreTexts platform; a detailed edit history is
available upon request.

1
6.1: Introduction to Circular Motion
We shall now investigate a special class of motions, motion in a plane about a central point, a motion we shall refer to as central
motion, the most outstanding case of which is circular motion. Special cases often dominate our study of physics, and circular
motion about a central point is certainly no exception. There are many instances of central motion about a point; a bicycle rider on
a circular track, a ball spun around by a string, and the rotation of a spinning wheel are just a few examples. Various planetary
models described the motion of planets in circles before any understanding of gravitation. The motion of the moon around the earth
is nearly circular. The motions of the planets around the sun are nearly circular. Our sun moves in nearly a circular orbit about the
center of our galaxy, 50,000 light years from a massive black hole at the center of the galaxy. When Newton solved the two-body
under a gravitational central force, he discovered that the orbits can be circular, elliptical, parabolic or hyperbolic. All of these
orbits still display central force motion about the center of mass of the two-body system. Another example of central force motion
is the scattering of particles by a Coulombic central force, for example Rutherford scattering of an alpha particle (two protons and
two neutrons bound together into a particle identical to a helium nucleus) against an atomic nucleus such as a gold nucleus.
We shall begin by describing the kinematics of circular motion, the position, velocity, and acceleration, as a special case of two-
dimensional motion. We will see that unlike linear motion, where velocity and acceleration are directed along the line of motion, in
circular motion the direction of velocity is always tangent to the circle. This means that as the object moves in a circle, the direction
of the velocity is always changing. When we examine this motion, we shall see that the direction of the change of the velocity is
towards the center of the circle. This means that there is a non-zero component of the acceleration directed radially inward, which
is called the centripetal acceleration. If our object is increasing its speed or slowing down, there is also a non-zero tangential
acceleration in the direction of motion. But when the object is moving at a constant speed in a circle then only the centripetal
acceleration is non-zero.
In 1666, twenty years before Newton published his Principia, he realized that the moon is always “falling” towards the center of the
earth; otherwise, by the First Law, it would continue in some linear trajectory rather than follow a circular orbit. Therefore there
must be a centripetal force, a radial force pointing inward, producing this centripetal acceleration.

In all of these instances, when an object is constrained to move in a circle, there must exist a force F acting on the object directed
towards the center. Because Newton’s Second Law is a vector equality, the radial component of the Second Law is

Fr = m ar

This page titled 6.1: Introduction to Circular Motion is shared under a CC BY-NC-SA 4.0 license and was authored, remixed, and/or curated by
Peter Dourmashkin (MIT OpenCourseWare) via source content that was edited to the style and standards of the LibreTexts platform; a detailed
edit history is available upon request.

6.1.1 https://phys.libretexts.org/@go/page/24452
6.2: Circular Motion- Velocity and Angular Velocity

We begin our description of circular motion by choosing polar coordinates. In Figure 6.1 we sketch the position vector r (t) of the
object moving in a circular orbit of radius r.

Figure 6.1 A circular orbit with unit vectors.


At time t , the particle is located at the point P with coordinates (r, θ(t)) and position vector given by

r (t) = r^
r(t)

At the point P, consider two sets of unit vectors (^ r(t), θ (t)) and
^ ^ ^
(i, j) , as shown in Figure 6.1. The vector decomposition
expression for ^
r(t) and θ (t) in terms of i and j is given by
^ ^ ^

^ ^ ^
r(t) = cos θ(t) i + sin θ(t) j

^ ^ ^
θ (t) = − sin θ(t) i + cos θ(t) j

Before we calculate the velocity, we shall calculate the time derivatives of Equations (6.2.2) and (6.2.3). Let’s first begin with
r(t)/dt :
d^

d^
r(t) d dθ(t) dθ(t)
^ ^ ^ ^
= (cos θ(t) i + sin θ(t) j ) = (− sin θ(t) i + cos θ(t) j)
dt dt dt dt

dθ(t) dθ(t)
^ ^ ^
= (− sin θ(t) i + cos θ(t) j ) = θ (t)
dt dt

where we used the chain rule to calculate that


d dθ(t)
cos θ(t) = − sin θ(t)
dt dt

d dθ(t)
sin θ(t) = cos θ(t)
dt dt

The calculation for dθ^(t)/dt is similar:


^
dθ (t) d dθ(t) dθ(t)
^ ^ ^ ^
= (− sin θ(t) i + cos θ(t j ) = (− cos θ(t) i − sin(t) j)
dt dt dt dt

dθ(t) dθ(t)
^ ^ ^
= (− cos θ(t) i − sin θ(t) j ) = − r(t)
dt dt

The velocity vector is then



→ d r (t) ^
dr dθ
^ ^
v (t) = =r =r θ (t) = vθ θ (t)
dt dt dt

where the θ^-component of the velocity is given by

6.2.1 https://phys.libretexts.org/@go/page/24453

vθ = r
dt

a quantity we shall refer to as the tangential component of the velocity. Denote the magnitude of the velocity by v ≡ |v|⃗  The
angular speed is the magnitude of the rate of change of angle with respect to time, which we denote by the Greek letter ω,
∣ dθ ∣
ω ≡∣ ∣
∣ dt ∣

Geometric Derivation of the Velocity for Circular Motion



Consider a particle undergoing circular motion. At time t , the position of the particle is r (t) . During the time interval Δt the
→ →
particle moves to the position r (t + Δt) with a displacement Δ r .

Figure 6.2 Displacement vector for circular motion


→ →
The magnitude of the displacement, |Δ r | is represented by the length of the horizontal vector, Δ r joining the heads of the
displacement vectors in Figure 6.2 and is given by

|Δ r | = 2r sin(Δθ/2)

When the angle Δθ is small, we can approximate

sin(Δθ/2) ≅Δθ/2

This is called the small angle approximation, where the angle Δθ (and hence Δθ/2 ) is measured in radians. This fact follows from
an infinite power series expansion for the sine function given by
3 5
Δθ Δθ 1 Δθ 1 Δθ
sin( ) = − ( ) + ( ) −⋯
2 2 3! 2 5! 2

When the angle Δθ/2 is small, only the first term in the infinite series contributes, as successive terms in the expansion become
much smaller. For example, when Δθ/2 = π/30 ≅0.1 corresponding to 6 , \begin{equation}(\Delta \theta / 2)^{3} / 3 ! \cong 1.9

\times 10^{-4}\); this term in the power series is three orders of magnitude smaller than the first and can be safely ignored for small
angles.
Using the small angle approximation, the magnitude of the displacement is

|Δ r | ≅rΔθ

This result should not be too surprising since in the limit as Δθ approaches zero, the length of the chord approaches the arc length
rΔθ

The magnitude of the velocity, v , is proportional to the rate of change of the magnitude of the angle with respect to time,

→ |Δ r | r|Δθ| |Δθ| ∣ dθ ∣
v ≡ | v (t)| = lim = lim = r lim = r∣ ∣ = rω
Δ→0 Δt Δt→0 Δt Δt→0 Δt ∣ dt ∣

The direction of the velocity can be determined by considering that in the limit as Δt → 0 (note that Δθ → 0 ), the direction of the

displacement Δ r approaches the direction of the tangent to the circle at the position of the particle at time t (Figure 6.3).

6.2.2 https://phys.libretexts.org/@go/page/24453
Figure 6.3 Direction of the displacement approaches the direction of the tangent line
→ → →
Thus, in the limit Δt → 0, Δ r ⊥ r and so the direction of the velocity v (t) at time t is perpendicular to the position vector r(t)
and tangent to the circular orbit in the +θ^- direction for the case shown in Figure 6.3.

This page titled 6.2: Circular Motion- Velocity and Angular Velocity is shared under a CC BY-NC-SA 4.0 license and was authored, remixed,
and/or curated by Peter Dourmashkin (MIT OpenCourseWare) via source content that was edited to the style and standards of the LibreTexts
platform; a detailed edit history is available upon request.

6.2.3 https://phys.libretexts.org/@go/page/24453
6.3: Circular Motion- Tangential and Radial Acceleration
When the motion of an object is described in polar coordinates, the acceleration has two components, the tangential component a , θ

and the radial component, a . We can write the acceleration vector as


r


^
a = ar ^
r(t) + aθ θ (t)

Keep in mind that as the object moves in a circle, the unit vectors ^ ^
r(t) and θ (t) change direction and hence are not constant in
time.
We will begin by calculating the tangential component of the acceleration for circular motion. Suppose that the tangential velocity
v = rdθ/dt is changing in magnitude due to the presence of some tangential force; we shall now consider that dθ/dt is changing
θ

in time, (the magnitude of the velocity is changing in time). Recall that in polar coordinates the velocity vector Equation (6.2.8) can
be written as

→ dθ
^
v (t) = r θ (t)
dt

We now use the product rule to determine the acceleration.


→ 2 ^
→ d v (t) d θ(t) dθ(t) dθ (t)
^
a (t) = =r θ (t) + r
2
dt dt dt dt

Recall from Equation (6.2.3) that θ^(t) = − sin θ(t)^i + cos θ(t)^j . So we can rewrite Equation (6.3.3) as
2
→ d θ(t) dθ(t) d
^ ^ ^
a (t) = r θ (t) + r (− sin θ(t) i + cos θ(t) j )
dt2 dt dt

We again use the chain rule (Equations (6.2.5) and (6.2.6)) and find that
2
→ d θ(t) dθ(t) dθ(t) dθ(t)
^ ^ ^
a (t) = r θ (t) + r (− cos θ(t) i − sin θ(t) j)
2
dt dt dt dt

Recall that ω ≡ dθ/dt , and from Equation (6.2.2), ^ ^ ^


r(t) = cos θ(t) i + sin θ(t) j therefore the acceleration becomes
2 2
→ d θ(t) dθ(t)
^
a (t) = r θ (t) − r( ) ^
r(t)
2
dt dt

The tangential component of the acceleration is then


2
d θ(t)
aθ = r
2
dt

The radial component of the acceleration is given by


2
dθ(t)
2
ar = −r( ) = −rω <0
dt


Because a r <0 , that radial vector component 2
a r (t) = −rω ^r(t) is always directed towards the center of the circular orbit.

Example 6.1 Circular Motion Kinematics


A particle is moving in a circle of radius R. At t = 0 , it is located on the x -axis. The angle the particle makes with the positive x -
axis is given by θ(t) = At − Bt where A and B are positive constants. Determine (a) the velocity vector, and (b) the acceleration
3

vector. Express your answer in polar coordinates. At what time is the centripetal acceleration zero?
Solution:
The derivatives of the angle function θ(t) = At 3
− Bt are dθ/dt = 3At 2
−B and d 2 2
θ/dt = 6At . Therefore the velocity vector
is given by

6.3.1 https://phys.libretexts.org/@go/page/24454
→ dθ(t)
^ 2 ^
v (t) = R θ (t) = R (3At − Bt) θ (t)
dt

The acceleration is given by


2 2
→ d θ(t) dθ(t)
^
a (t) = R 2
θ (t) − R( ) ^
r(t)
dt dt

2
^ 2
^
= R(6At)θ (t) − R (3At − B) r(t)

The centripetal acceleration is zero at time t = t when


1

−−−−−
2
3At − B = 0 ⇒ t1 = √B/3A
1

This page titled 6.3: Circular Motion- Tangential and Radial Acceleration is shared under a CC BY-NC-SA 4.0 license and was authored,
remixed, and/or curated by Peter Dourmashkin (MIT OpenCourseWare) via source content that was edited to the style and standards of the
LibreTexts platform; a detailed edit history is available upon request.

6.3.2 https://phys.libretexts.org/@go/page/24454
6.4: Period and Frequency for Uniform Circular Motion
If the object is constrained to move in a circle and the total tangential force acting on the object is zero, F θ
total 
=0 then (Newton’s
Second Law), the tangential acceleration is zero,

aθ = 0

This means that the magnitude of the velocity (the speed) remains constant. This motion is known as uniform circular motion. The
acceleration is then given by only the acceleration radial component vector
→ 2
a r (t) = −rω (t)^
r(t) uniform circular motion .
Because the speed v = r|ω| is constant, the amount of time that the object takes to complete one circular orbit of radius r is also
constant. This time interval, T , is called the period. In one period the object travels a distance s = vT equal to the circumference,
s = 2πr ; thus

s = 2πr = vT

The period T is then given by


2πr 2πr 2π
T = = =
v rω ω

The frequency f is defined to be the reciprocal of the period,


1 ω
f = =
T 2π

The SI unit of frequency is the inverse second, which is defined as the hertz, [ s −1
] ≡ [Hz]

The magnitude of the radial component of the acceleration can be expressed in several equivalent forms since both the magnitudes
of the velocity and angular velocity are related by v = rω . Thus we have several alternative forms for the magnitude of the
centripetal acceleration. The first is that in Equation (6.5.3). The second is in terms of the radius and the angular velocity,
2
| ar | = rω

The third form expresses the magnitude of the centripetal acceleration in terms of the speed and radius,
2
v
| ar | =
r

Recall that the magnitude of the angular velocity is related to the frequency by ω = 2πf , so we have a fourth alternate expression
for the magnitude of the centripetal acceleration in terms of the radius and frequency,
2 2
| ar | = 4 π rf

A fifth form commonly encountered uses the fact that the frequency and period are related by f = 1/T = ω/2π . Thus we have the
fourth expression for the centripetal acceleration in terms of radius and period,
2
4π r
| ar | =
2
T

Other forms, such as 4π 2 2


r f /T or 2πrωf, while valid, are uncommon.
Often we decide which expression to use based on information that describes the orbit. A convenient measure might be the orbit’s
radius. We may also independently know the period, or the frequency, or the angular velocity, or the speed. If we know one, we can
calculate the other three but it is important to understand the meaning of each quantity.

Geometric Interpretation for Radial Acceleration for Uniform Circular Motion


An object traveling in a circular orbit is always accelerating towards the center. Any radial inward acceleration is called centripetal
acceleration. Recall that the direction of the velocity is always tangent to the circle. Therefore the direction of the velocity is
constantly changing because the object is moving in a circle, as can be seen in Figure 6.4. Because the velocity changes direction,
the object has a nonzero acceleration.

6.4.1 https://phys.libretexts.org/@go/page/24455
Figure 6.5 Change in velocity vector.

The calculation of the magnitude and direction of the acceleration is very similar to the calculation for the magnitude and direction

of the velocity for circular motion, but the change in velocity vector, Δ v is more complicated to visualize. The change in velocity
→ → →
Δ v = v (t + Δt) − v (t) is depicted in Figure 6.5. The velocity vectors have been given a common point for the tails, so that
→ →
the change in velocity, Δv can be visualized. The length |Δ v | of the vertical vector can be calculated in exactly the same way as

the displacement |Δ r |. The magnitude of the change in velocity is

|Δ v | = 2v sin(Δθ/2)

We can use the small angle approximation sin(Δθ/2) ≅Δθ/2 to approximate the magnitude of the change of velocity,

|Δ v | ≅v|Δθ|

The magnitude of the radial acceleration is given by



|Δ v | v|Δθ| |Δθ| ∣ dθ ∣
| ar | = lim = lim = v lim = v∣ ∣ = v|ω|
Δt→0 Δt Δt→0 Δt Δt→0 Δt ∣ dt ∣

The direction of the radial acceleration is determined by the same method as the direction of the velocity; in the limit
→ → →
Δθ → 0, Δ v ⊥ v and so the direction of the acceleration radial component vector a r (t) at time t is perpendicular to position

vector v (t) and directed inward, in the −r
^-direction.

This page titled 6.4: Period and Frequency for Uniform Circular Motion is shared under a CC BY-NC-SA 4.0 license and was authored, remixed,
and/or curated by Peter Dourmashkin (MIT OpenCourseWare) via source content that was edited to the style and standards of the LibreTexts
platform; a detailed edit history is available upon request.

6.4.2 https://phys.libretexts.org/@go/page/24455
6.5: Angular Velocity and Angular Acceleration
. Angular Velocity
We shall always choose a right-handed cylindrical coordinate system. If the positive z - axis points up, then we choose θ to be
increasing in the counterclockwise direction as shown in Figures 6.6.

Figure 6.6 Right handed coordinate system


For a point object undergoing circular motion about the z -axis, the angular velocity vector ω⃗  is directed along the z -axis with z -
component equal to the time derivative of the angle θ,

^ ^
ω⃗  = k = ωz k
dt

The SI units of angular velocity are [rad ⋅ s


−1
] Note that the angular speed is just the magnitude of the z -component of the
angular velocity,
∣ dθ ∣
ω ≡ | ωz | = ∣ ∣
∣ dt ∣

If the velocity of the object is in the +θ^-direction, (rotating in the counterclockwise direction in Figure 6.7(a)), then the z -
component of the angular velocity is positive, ω = dθ/dt > 0 The angular velocity vector then points in the +k
z
^
-direction as
shown in Figure 6.7(a). If the velocity of the object is in the −θ -direction, (rotating in the clockwise direction in Figure 6.7(b)),
^

then the z -component of the angular velocity angular velocity is negative, ω = dθ/dt < 0 . The angular velocity vector then
z

points in the −k
^
-direction as shown in Figure 6.7(b).

Figure 6.7(b) Angular velocity vector for motion with dθ / dt > 0 .

6.5.1 https://phys.libretexts.org/@go/page/24456
Figure 6.7(b) Angular velocity for motion with dθ / dt < 0 .
The velocity and angular velocity are related by
→ → → dθ dθ
^ ^
v = ω × r = k × r^
r =r θ
dt dt

Example 6.2 Angular Velocity


A particle is moving in a circle of radius R. At t = 0 , it is located on the x -axis. The angle the particle makes with the positive x -
axis is given by θ(t) = At − Bt where A and B are positive constants. Determine (a) the angular velocity vector, and (b) the
3

velocity vector. Express your answer in polar coordinates. (c) At what time, t = t is the angular velocity zero? (d) What is the
1

direction of the angular velocity for 1. t < t 2. t > t ? ?


1 1

Solution:
The derivative of θ(t) = At − Bt 3
is
dθ(t)
2
= A − 3Bt
dt

Therefore the angular velocity vector is given by

→ dθ(t)
^ 2 ^
ω (t) = k = (A − 3Bt ) k
dt

The velocity is given by

→ dθ(t)
^ 2 ^
v (t) = R θ (t) = R (A − 3Bt ) θ (t)
dt

The angular velocity is zero at time t = t when


1

−−−−−
2
A − 3Bt = 0 ⇒ t1 = √A/3B
1

dθ(t) →
For t < t1,
dt
2
= A − 3Bt
1
>0 hence ω (t) points in the positive k
^
-direction.
dθ(t) →
For t > t1,
dt
2
= A − 3Bt
1
<0 hence ω (t) points in the positive k
^
-direction.

Angular Acceleration
In a similar fashion, for a point object undergoing circular motion about the fixed z -axis, the angular acceleration is defined as
2
d θ
^ ^
α⃗  = k = αz k
2
dt

The SI units of angular acceleration are [rad ⋅ s


−2
] The magnitude of the angular acceleration is denoted by the Greek symbol
alpha,
2
∣d θ∣
α ≡ | α⃗ | = ∣ ∣
2
∣ dt ∣

6.5.2 https://phys.libretexts.org/@go/page/24456
There are four special cases to consider for the direction of the angular velocity. Let’s first consider the two types of motion with

α pointing in the k^
-direction: (i) if the object is rotating counterclockwise and speeding up then both dθ/dt > 0 and d θ/dt > 0 2 2

(Figure 6.8(a)) (ii) if the object is rotating clockwise and slowing down then dθ/dt < 0 but d θ/dt > 0 (Figure 6.8(b). There are
2 2

two corresponding cases in which α⃗  pointing in the −k ^


-direction (iii) if the object is rotating counterclockwise and slowing down
then dθ/dt > 0 but d θ/dt < 0 (Figure 6.9(a), (iv) if the object is rotating clockwise and speeding up then both dθ/dt < 0 and
2 2

d θ/dt < 0 (Figure 6.9(b).


2 2

Figure 6.8(a) Angular acceleration vector vector for motion with dθ/dt > 0 , and d 2
θ/dt
2
> 0

Figure 6.8(b) Angular velocity vector vector for motion with dθ/dt < 0 , and d 2
θ/dt
2
> 0

Figure 6.9(a) Angular acceleration vector vector for motion with dθ/dt > 0 , and d 2
θ/dt
2
< 0

6.5.3 https://phys.libretexts.org/@go/page/24456
Figure 6.9(b) Angular velocity vector vector for motion with dθ/dt < 0 , and d 2
θ/dt
2
< 0

Example 6.3 Integration and Circular Motion Kinematics


A point-like object is constrained to travel in a circle. The z -component of the angular acceleration of the object for the time
interval [0, t ] is given by the function
1

t
b (1 − ) ; 0 ≤ t ≤ t1
t1
αz (t) = {

0; t > t1

where b is a positive constant with units rad⋅S −2


.
a) Determine an expression for the angular velocity of the object at t = t . 1

b) Through what angle has the object rotated at time t = t ? 1

Solution:
a) The angular velocity at time t = t is given by
1

′ ′
t =t1 t =t1 ′ 2
t t bt1
′ ′ ′ 1
ωz (t1 ) − ωz (t = 0) = ∫ αz (t ) dt = ∫ b (1 − ) dt = b ( t1 − ) =

t =0

t =0
t1 2t1 2

b) In order to find the angle θ (t ) − θ(t = 0) that the object has rotated through at time
1 t = t1 , you first need to find ωz (t) by
integrating the z-component of the angular acceleration
′ ′
t =t t =t ′ 2
t t
′ ′ ′
ωz (t) − ωz (t = 0) = ∫ αz (t ) dt = ∫ b (1 − ) dt = b (t − )
t =0

t=0
t1 2t1

Because it started from rest, ω z (t = 0) = 0 , hence ω z (t) = b (t −


2t1
t
) ; 0 ≤ t ≤ t1

Then integrate ω z (t) between t = 0 and t = t to find that


1

′ ′
t =t1 t =t1 ′2 2 3 2
t t t bt
′ ′ ′ ′ 1 1 1
θ (t1 ) − θ(t = 0) = ∫ ωz (t ) dt = ∫ b (t − ) dt = b ( − ) =

t =0

t =0 2t1 2 6t1 3

This page titled 6.5: Angular Velocity and Angular Acceleration is shared under a CC BY-NC-SA 4.0 license and was authored, remixed, and/or
curated by Peter Dourmashkin (MIT OpenCourseWare) via source content that was edited to the style and standards of the LibreTexts platform; a
detailed edit history is available upon request.

6.5.4 https://phys.libretexts.org/@go/page/24456
6.6: Non-circular Central Motion
Let’s now consider central motion in a plane that is non-circular. In Figure 6.10, we show the spiral motion of a moving particle. In
polar coordinates, the key point is that the time derivative dr / dt of the position function r is no longer zero. The second derivative
d r/dt also may or may not be zero. In the following calculation we will drop all explicit references to the time dependence of
2 2

the various quantities. The position vector is still given by Equation (6.2.1), which we shall repeat below

r = r^
r

Because dr/dt ≠ 0 when we differentiate Equation (6.5.9), we need to use the product rule

→ d r dr dr
^
v = = ^
r +r
dt dt dt

Substituting Equation (6.2.4) into Equation (6.5.10)



→ d r dr dθ
^ ^ ^
v = = r +r θ = vr ^
r + vθ θ
dt dt dt

The velocity is no longer tangential but now has a radial component as well
dr
vr =
dt

In order to determine the acceleration, we now differentiate Equation (6.5.11), again using the product rule, which is now a little
more involved:
→ 2
^
2 ^
→ d v d r dr dr dr dθ d θ dθ dθ
^ ^ ^
a = = r+ + θ +r θ +r
2 2
dt dt dt dt dt dt dt dt dt

Now substitute Equations (6.2.4) and (6.2.7) for the time derivatives of the unit vectors in Equation (6.5.13), and after collecting
terms yields
→ d
2
r dθ
2
dr dθ d
2
θ ^
a =( 2
− r( ) )^
r + (2 +r 2

dt dt dt dt dt

^
= ar ^
r + aθ θ

The radial and tangential components of the acceleration are now more complicated than then in the case of circular motion due to
the non-zero derivatives of dr/dt and d r/dt . The radial component is
2 2

2 2
d r dθ
ar = − r( )
2
dt dt

and the tangential component is


2
dr dθ d θ
aθ = 2 +r
2
dt dt dt

The firs term in the tangential component of the acceleration, 2(dr/dt)(dθ/dt) has a special name, the coriolis acceleration,
dr dθ
acor = 2
dt dt

Example 6.4 Spiral Motion


A particle moves outward along a spiral starting from the origin at t = 0 . Its trajectory is given by r = bθ where b is a positive
constant with units [m ⋅ rad ] ⋅ θ increases in time according to θ = ct , where c > 0 is a positive constant (with units
−1 2

])
−2
[rad ⋅ s

a) Determine the acceleration as a function of time.


b) Determine the time at which the radial acceleration is zero.

6.6.1 https://phys.libretexts.org/@go/page/24734
c) What is the angle when the radial acceleration is zero?
d) Determine the time at which the radial and tangential accelerations have equal magnitude.
Solution:
a) The position coordinate as a function of time is given by r = bθ = bct . The acceleration is given by Equation (6.5.14). In order
2

to calculate the acceleration, we need to calculate the four derivatives dr/dt = 2bct, d r/dt = 2bc, dθ/dt = 2ct , and 2 2

d θ/dt = 2c . The acceleration is then


2 2

→ 3 4 2 2 2 2 3 4 2 2 ^
^
a = (2bc − 4b c t ) ^
r + (8b c t + 2b c t ) θ = (2bc − 4b c t ) ^
r + 10b c t θ

b) The radial acceleration is zero when


1/4
1
t1 = ( )
2
2c

c) The angle when the radial acceleration is zero is


2 –
θ1 = c t = √2/2
1

d) The radial and tangential accelerations have equal magnitude when after some algebra
3 4 2 2 4 2 2
(2bc − 4b c t ) = 10b c t ⇒ 0 =t + (5/2c)t − (1/2 c )

This equation has as only positive solution for t 2

1/2 −−
2 2
−(5/2c) ± ((5/2c ) + 2c ) √33 − 5
2
t = =
2
2 4c

Therefore the magnitudes of the two components are equal when


−−−−−− −
−−
√33 − 5
t2 =√
4c

This page titled 6.6: Non-circular Central Motion is shared under a CC BY-NC-SA 4.0 license and was authored, remixed, and/or curated by Peter
Dourmashkin (MIT OpenCourseWare) via source content that was edited to the style and standards of the LibreTexts platform; a detailed edit
history is available upon request.

6.6.2 https://phys.libretexts.org/@go/page/24734
CHAPTER OVERVIEW
7: Newton’s Laws of Motion
7.1: Force and Quantity of Matter
7.2: Newton’s First Law
7.3: Momentum, Newton’s Second Law and Third Law
7.4: Newton’s Third Law- Action-Reaction Pairs

This page titled 7: Newton’s Laws of Motion is shared under a CC BY-NC-SA 4.0 license and was authored, remixed, and/or curated by Peter
Dourmashkin (MIT OpenCourseWare) via source content that was edited to the style and standards of the LibreTexts platform; a detailed edit
history is available upon request.

1
7.1: Force and Quantity of Matter
I have not as yet been able to discover the reason for these properties of gravity from phenomena, and I do not feign hypotheses.
For whatever is not deduced from the phenomena must be called a hypothesis; and hypotheses, whether metaphysical or physical,
or based on occult qualities, or mechanical, have no place in experimental philosophy. In this philosophy particular propositions
are inferred from the phenomena, and afterwards rendered general by induction.
~Isaac Newton
In our daily experience, we can cause a body to move by either pushing or pulling that body. Ordinary language use describes this
action as the effect of a person’s strength or force. However, bodies placed on inclined planes, or when released at rest and undergo
free fall, will move without any push or pull. Galileo referred to a force acting on these bodies, a description of which he published
in Mechanics in 1623. In 1687, Isaac Newton published his three laws of motion in the Philosophiae Naturalis Principia
Mathematica (“Mathematical Principles of Natural Philosophy”), which extended Galileo’s observations. The First Law expresses
the idea that when no force acts on a body, it will remain at rest or maintain uniform motion; when a force is applied to a body, it
will change its state of motion.
Many scientists, especially Galileo, recognized the idea that force produces motion before Newton but Newton extended the
concept of force to any circumstance that produces acceleration. When a body is initially at rest, the direction of our push or pull
corresponds to the direction of motion of the body. If the body is moving, the direction of the applied force may change both the
direction of motion of the body and how fast it is moving. Newton defined the force acting on an object as proportional to the
acceleration of the object.
An impressed force is an action exerted upon a body, in order to change its state, either of rest, or of uniform motion in a right line.
In order to define the magnitude of the force, he introduced a constant of proportionality, the inertial mass, which Newton called
“quantity of matter”.
The quantity of matter is the measure of the same, arising from its density and bulk conjointly. Thus air of double density, in a
double space, is quadruple in quantity; in a triple space, sextuple in quantity. The same thing is to be understood of snow, and fine
dust or powders, that are condensed by compression or liquefaction, and of all bodies that are by any causes whatever differently
condensed. I have no regard in this place to a medium, if any such there is, that freely pervades the interstices between the parts of
bodies. It is this quantity that I mean hereafter everywhere under the name of body or mass. And the same is known by the weight of
each body, for it is proportional to the weight, as I have found by experiment on pendulums, very accurately made, which shall be
shown hereafter.
Suppose we apply a force to a body that is an identical copy of the standard mass, (we shall refer to this body as a standard body).

The force will induce the standard body to accelerate with magnitude | a | that can be measured by an accelerometer (any device

that measures acceleration). The magnitude of the force | F | acting on the standard body is defined to be the product of the standard

mass m with the magnitude of the acceleration | a |. Force is a vector quantity. The direction of the force on the standard body is
s

defined to be the direction of the acceleration of the body. Thus


→ →
F ≡ ms a

→ →
In order to justify the statement that force is a vector quantity, we need to apply two forces F1 and F2 simultaneously to our
→T
standard body and show that the resultant force F is the vector sum of the two forces when the forces are applied one at a time.

7.1.1 https://phys.libretexts.org/@go/page/24459
Figure 7.2 Force adds as vectors.
→ →
We apply each force separately and measure the accelerations a 1 and a 2 , noting that
→ →
F 1 = ms a 1

→ →
F 2 = ms a 2

When we apply the two forces simultaneously, we measure the acceleration a . The force by definition is now
→T →
F ≡ ms a

We then compare the accelerations. The results of these three measurements, and for that matter any similar experiment, confirms
that the accelerations add as vectors (Figure 7.1)
→ → →
a = a 1 + a 2

Therefore the forces add as vectors as well (Figure 7.2),


→T → →
F = F1 + F2

This last statement is not a definition but a consequence of the experimental result described by Equation (7.1.5) and our definition
of force.
Example 7.1 Vector Decomposition Solution
Two horizontal ropes are attached to a post that is stuck in the ground. The ropes pull the post producing the vector forces
→ →
^ ^
F 1 = 70N i + 20N j and F = −30N^i + 40N^j as shown in Figure 7.3. Find the direction and magnitude of the horizontal
2

component of a contact force of the ground on the post.

Figure 7.3 Example 7.1

7.1.2 https://phys.libretexts.org/@go/page/24459
Figure 7.4 Vector sum of the horizontal forces
Solution: Because the ropes are pulling the post horizontally, the contact force must have a horizontal component that is equal to
the negative of the sum of the two horizontal forces exerted by the rope on the post (Figure 7.4). There is an additional vertical
component of the contact force that balances the gravitational force exerted on the post by the earth. We restrict our attention to the
→ →
horizontal component of the contact force. Let F3 denote the sum of the forces due to the ropes. Then we can write the vector F3

as

^ ^ ^ ^
F 3 = (F1x + F2x ) i + (F1y + F2y ) j = (70N + −30N) i + (20N + 40N) j

^ ^
= (40N) i + (60N) j

Therefore the horizontal component of the contact force satisfies the condition that
→ → → →
^ ^
F hor = − F 3 = − ( F 1 + F 2 ) = (−40N) i + (−60N) j

∣→ ∣ −−−−−−−−−−−−−−−−
The magnitude is ∣ F hor ∣
2
= √(−40N) + (−60N)
2
= 72N . The horizontal component of the contact force makes an angle
∣ ∣

−1
60N ∘
θ = tan [ ] = 56.3
40N

as shown in the figure above.

Mass Calibration
So far, we have only used the standard body to measure force. Instead of performing experiments on the standard body, we can
calibrate the masses of all other bodies in terms of the standard mass by the following experimental procedure. We shall refer to the
mass measured in this way as the inertial mass and denote it by m . in

We apply a force of magnitude F to the standard body and measure the magnitude of the acceleration a . Then we apply the same
s

force to a second body of unknown mass m and measure the magnitude of the acceleration a . Because the same force is applied
in in

to both bodies,

F = min ain = ms as

the ratio of the inertial mass to the standard mass is equal to the inverse ratio of the magnitudes of the accelerations,
min as
=
ms ain

7.1.3 https://phys.libretexts.org/@go/page/24459
Therefore the second body has inertial mass equal to
as
min = ms
ain

This method is justified by the fact that we can repeat the experiment using a different force and still find that the ratios of the
acceleration are the same. For simplicity we shall denote the inertial mass by m .

This page titled 7.1: Force and Quantity of Matter is shared under a CC BY-NC-SA 4.0 license and was authored, remixed, and/or curated by
Peter Dourmashkin (MIT OpenCourseWare) via source content that was edited to the style and standards of the LibreTexts platform; a detailed
edit history is available upon request.

7.1.4 https://phys.libretexts.org/@go/page/24459
7.2: Newton’s First Law
The First Law of Motion, commonly called the “Principle of Inertia,” was first realized by Galileo. (Newton did not acknowledge
Galileo’s contribution.) Newton was particularly concerned with how to phrase the First Law in Latin, but after many rewrites
Newton choose the following expression for the First Law (in English translation):
Law 1: Every body continues in its state of rest, or of uniform motion in a right line, unless it is compelled to change that state by
forces impressed upon it.
Projectiles continue in their motions, so far as they are not retarded by the resistance of air, or impelled downwards by the force of
gravity. A top, whose parts by their cohesion are continually drawn aside from rectilinear motions, does not cease its rotation,
otherwise than as it is retarded by air. The greater bodies of planets and comets, meeting with less resistance in freer spaces,
preserve their motions both progressive and circular for a much longer time.
The first law is an experimental statement about the motions of bodies. When a body moves with constant velocity, there are either
no forces present or the sum of all the forces acting on the body is zero. If the body changes its velocity, it has non-zero
acceleration, and hence the sum of all the forces acting on the body must be non-zero as well. If the velocity of a body changes in
time, then either the direction or magnitude changes, or both can change.
After a bus or train starts, the acceleration is often so small we can barely perceive it. We are often startled because it seems as if
the station is moving in the opposite direction while we seem to be at rest. Newton’s First Law states that there is no physical way
to distinguish between whether we are moving or the station is moving, because there is nearly zero total force acting on the body.
Once we reach a constant velocity, our minds dismiss the idea that the ground is moving backwards because we think it is
impossible, but there is no actual way for us to distinguish whether the train is moving or the ground is moving.

This page titled 7.2: Newton’s First Law is shared under a CC BY-NC-SA 4.0 license and was authored, remixed, and/or curated by Peter
Dourmashkin (MIT OpenCourseWare) via source content that was edited to the style and standards of the LibreTexts platform; a detailed edit
history is available upon request.

7.2.1 https://phys.libretexts.org/@go/page/24460
7.3: Momentum, Newton’s Second Law and Third Law
Newton began his analysis of the cause of motion by introducing the quantity of motion:
Definition: Quantity of Motion
The quantity of motion is the measure of the same, arising from the velocity and quantity of matter conjointly.
The motion of the whole is the sum of the motion of all its parts; and therefore in a body double in quantity, with equal velocity, the
motion is double, with twice the velocity, it is quadruple.
Our modern term for quantity of motion is momentum and it is a vector quantity
→ →
p =m v


where m is the inertial mass and V is the velocity of the body. Newton’s Second Law states that
Law II: The change of motion is proportional to the motive force impressed, and is made in the direction of the right line in which
that force is impressed.
If any force generates a motion, a double force will generate double the motion, a triple force triple the motion, whether that force
is impressed altogether and at once or gradually and successively. And this motion (being always directed the same way with the
generating force), if the body moved before, is added or subtracted from the former motion, according as they directly conspire
with or are directly contrary to each other; or obliquely joined, when they are oblique, so as to produce a new motion compounded
from the determination of both.

Suppose that a force is applied to a body for a time interval Δt. The impressed force or impulse (a vector quantity I ) produces a
change in the momentum of the body,
→ →

I = F Δt = Δ p

From the commentary to the second law, Newton also considered forces that were applied continually to a body instead of
impulsively. The instantaneous action of the total force acting on a body at a time t is defined by taking the mathematical limit as
the time interval Δt becomes smaller and smaller,
→ →
→ Δp dp
F = lim ≡
Δt→0 Δt dt

When the mass remains constant in time, the Second Law can be recast in its more familiar form,

→ d v
F =m
dt

Because the derivative of velocity is the acceleration, the force is the product of mass and acceleration,
→ →
F =m a

Because we defined force in terms of change in motion, the Second Law appears to be a restatement of this definition, and devoid
of predictive power since force is only determined by measuring acceleration. What transforms the Second Law from just a
definition is the additional input that comes from force laws that are based on experimental observations on the interactions
between bodies. Throughout this book, we shall investigate these force laws and learn to use them in order to determine the forces
and accelerations acting on a body (left-hand-side of Newton’s Second Law). When a physical body is constrained to move along a
surface, or inside a container (for example gas molecules in a container), there are constraint forces that are not determined
beforehand by any force law but are only determined by their effect on the motion of the body. For any given constrained motion,
these constraint forces are unknown and must be determined by the particular motion of the body that we are studying, for example
the contact force of the surface on the body, or the force of the wall on the gas particles.
The right-hand-side of Newton’s Second Law is the product of mass with acceleration. Acceleration is a mathematical description
of how the velocity of a body changes. Knowledge of all the forces acting on the body enables us to predict the acceleration.
Equation (7.3.5) is known as the equation of motion. Once we know this equation we may be able to determine the velocity and

7.3.1 https://phys.libretexts.org/@go/page/24461
position of that body at all future times by integration techniques, or computational techniques. For constrained motion, if we know
the acceleration of the body, we can also determine the constraint forces acting on the body.

This page titled 7.3: Momentum, Newton’s Second Law and Third Law is shared under a CC BY-NC-SA 4.0 license and was authored, remixed,
and/or curated by Peter Dourmashkin (MIT OpenCourseWare) via source content that was edited to the style and standards of the LibreTexts
platform; a detailed edit history is available upon request.

7.3.2 https://phys.libretexts.org/@go/page/24461
7.4: Newton’s Third Law- Action-Reaction Pairs
Newton realized that when two bodies interact via a force, then the force on one body is equal in magnitude and opposite in
direction to the force acting on the other body.
Law III: To every action there is always opposed an equal reaction: or, the mutual action of two bodies upon each other are always
equal, and directed to contrary parts. Whatever draws or presses another is as much drawn or pressed by that other. If you press on
a stone with your finger, the finger is also pressed by the stone.
The Third Law, commonly known as the “action-reaction” law, is the most surprising of the three laws. Newton’s great discovery
was that when two objects interact, they each exert the same magnitude of force on each other but in opposite directions. We shall
refer to the pair of forces between two interacting bodies as an interaction pair of force, or more briefly as an interaction pair.

Consider two bodies engaged in a mutual interaction. Label the bodies 1 and 2 respectively. Let F 1,2 be the force on body 2 due to

the interaction with body 1, and F 2,1 be the force on body 1 due to the interaction with body 2. These forces are depicted in Figure
7.5.

Figure 7.5 Interaction pair of forces


These two vector forces are equal in magnitude and opposite in direction,
→ →
F 1,2 = − F 2,1

We shall employ these definitions, Newton’s three laws, and force laws to describe the motion of bodies, a subject known as
classical mechanics or Newtonian Mechanics, and hence explain a vast range of phenomena. Newtonian mechanics has important
limits. It does not satisfactorily explain systems of objects moving at speeds comparable to the speed of light ( v > 0.1 c ) where we
need the theory of special relativity, nor does it adequately explain the motion of electrons in atoms, where we need quantum
mechanics. We also need general relativity and cosmology to explain the largescale structure of the universe.

This page titled 7.4: Newton’s Third Law- Action-Reaction Pairs is shared under a CC BY-NC-SA 4.0 license and was authored, remixed, and/or
curated by Peter Dourmashkin (MIT OpenCourseWare) via source content that was edited to the style and standards of the LibreTexts platform; a
detailed edit history is available upon request.

7.4.1 https://phys.libretexts.org/@go/page/24462
CHAPTER OVERVIEW
8: Applications of Newton’s Second Law
8.1: Force Laws
8.2: Fundamental Laws of Nature
8.3: Constraint Forces
8.4: Free-body Force Diagram
8.5: Tension in a Rope
8.6: Drag Forces in Fluids
8.7: Worked Examples

This page titled 8: Applications of Newton’s Second Law is shared under a CC BY-NC-SA 4.0 license and was authored, remixed, and/or curated
by Peter Dourmashkin (MIT OpenCourseWare) via source content that was edited to the style and standards of the LibreTexts platform; a detailed
edit history is available upon request.

1
8.1: Force Laws
Those who are in love with practice without knowledge are like the sailor who gets into a ship without rudder or compass
and who never can be certain whether he is going. Practice must always be founded on sound theory
~Leonardo da Vinci
There are forces that don't change appreciably from one instant to another, which we refer to as constant in time, and forces that
don't change appreciably from one point to another, which we refer to as constant in space. The gravitational force on an object
near the surface of the earth is an example of a force that is constant in space. There are forces that depend on the configuration of a
system. When a mass is attached to one end of a spring, the spring force acting on the object increases in strength whether the
spring is extended or compressed. There are forces that spread out in space such that their influence becomes less with distance.
Common examples are the gravitational and electrical forces. The gravitational force between two objects falls off as the inverse
square of the distance separating the objects provided the objects are of a small dimension compared to the distance between them.
More complicated arrangements of attracting and repelling interactions give rise to forces that fall off with other powers of r :
constant, 1/r, 1/r , 1/r
2 3

A force may remain constant in magnitude but change direction; for example the gravitational force acting on a planet undergoing
circular motion about a star is directed towards the center of the circle. This type of attractive central force is called a centripetal
force.
A force law describes the relationship between the force and some measurable property of the objects involved. We shall see that
some interactions are describable by force laws and other interactions cannot be so simply described.

Hooke’s Law
In order to stretch or compress a spring from its equilibrium length, a force must be exerted on the spring. Consider an object of
mass m that is lying on a horizontal surface. Attach one end of a spring to the object and fix the other end of the spring to a wall.
Let l denote the equilibrium length of the spring (neither stretched or compressed). Assume that the contact surface is smooth and
0

hence frictionless in order to consider only the effect of the spring force. If the object is pulled to stretch the spring or pushed to
compress the spring, then by Newton’s Third Law the force of the spring on the object is equal and opposite to the force that the
object exerts on the spring. We shall refer to the force of the spring on the object as the spring force and experimentally determine a
relationship between that force and the amount of stretch or compress of the spring. Choose a coordinate system with the origin
located at the point of contact of the spring and the object when the spring-object system is in the equilibrium configuration.
Choose the ^i unit vector to point in the direction the object moves when the spring is being stretched. Choose the coordinate
function x to denote the position of the object with respect to the origin (Figure 8.1).

Figure 8.1 Spring attached to a wall and an object


Initially stretch the spring until the object is at position x . Then release the object and measure the acceleration of the object the
→ →
instant the object is released. The magnitude of the spring force acting on the object is | F | = m| a | Now repeat the experiment for

8.1.1 https://phys.libretexts.org/@go/page/24466
a range of stretches (or compressions). Experiments show that for each spring, there is a range of maximum values x > 0 for max

stretching and minimum values x min< 0 for compressing such that the magnitude of the measured force is proportional to the

stretched or compressed length and is given by the formula.



| F | = k|x|

where the spring constant k has units N ⋅ m −1


. The free-body force diagram is shown in Figure 8.2.

Figure 8.2 Spring force acting on object


The constant k is equal to the negative of the slope of the graph of the force vs. the compression or stretch (Figure 8.3).

Figure 8.3 Plot of x -component of the spring force F vs. x


x

The direction of the acceleration is always towards the equilibrium position whether the spring is stretched or compressed. This
type of force is called a restoring force. Let Fx denote the x -component of the spring force. Then

Fx = −kx

Now perform similar experiments on other springs. For a range of stretched lengths, each spring exhibits the same proportionality
between force and stretched length, although the spring constant may differ for each spring.
It would be extremely impractical to experimentally determine whether this proportionality holds for all springs, and because a
modest sampling of springs has confirmed the relation, we shall infer that all ideal springs will produce a restoring force, which is
linearly proportional to the stretched (or compressed) length. This experimental relation regarding force and stretched (or
compressed) lengths for a finite set of springs has now been inductively generalized into the above mathematical model for ideal
springs, a force law known as a Hooke’s Law.
This inductive step, referred to as Newtonian induction, is the critical step that makes physics a predictive science. Suppose a
spring, attached to an object of mass m, is stretched by an amount Δx . Use the force law to predict the magnitude of the force

between the rubber band and the object, | F | = k|Δx|, without having to experimentally measure the acceleration. Now use
Newton’s Second Law to predict the magnitude of the acceleration of the object.

→ | r | k|Δx|
| a | = =
m m

Carry out the experiment, and measure the acceleration within some error bounds. If the magnitude of the predicted acceleration
disagrees with the measured result, then the model for the force law needs modification. The ability to adjust, correct or even reject

8.1.2 https://phys.libretexts.org/@go/page/24466
models based on new experimental results enables a description of forces between objects to cover larger and larger experimental
domains.
Many real springs have been wound such that a force of magnitude F must be applied before the spring begins to stretch. The
0

value of F is referred to as the pre-tension of the spring. Under these circumstances, Hooke’s law must be modified to account for
0

this pretension,

Fx = −F0 − kx, x >0


{
Fx = +F1 − kx, x <0

Note the value of the pre-tension F and F may differ for compressing or stretching a spring.
0 1

This page titled 8.1: Force Laws is shared under a CC BY-NC-SA 4.0 license and was authored, remixed, and/or curated by Peter Dourmashkin
(MIT OpenCourseWare) via source content that was edited to the style and standards of the LibreTexts platform; a detailed edit history is
available upon request.

8.1.3 https://phys.libretexts.org/@go/page/24466
8.2: Fundamental Laws of Nature
Force laws are mathematical models of physical processes. They arise from observation and experimentation, and they have limited
ranges of applicability. Does the linear force law for the spring hold for all springs? Each spring will most likely have a different
range of linear behavior. So the model for stretching springs still lacks a universal character. As such, there should be some
hesitation to generalize this observation to all springs unless some property of the spring, universal to all springs, is responsible for
the force law.
Perhaps springs are made up of very small components, which when pulled apart tend to contract back together. This would suggest
that there is some type of force that contracts spring molecules when they are pulled apart. What holds molecules together? Can we
find some fundamental property of the interaction between atoms that will suffice to explain the macroscopic force law? This
search for fundamental forces is a central task of physics.
In the case of springs, this could lead into an investigation of the composition and structural properties of the atoms that compose
the steel in the spring. We would investigate the geometric properties of the lattice of atoms and determine whether there is some
fundamental property of the atoms that create this lattice. Then we ask how stable is this lattice under deformations. This may lead
to an investigation into the electron configurations associated with each atom and how they overlap to form bonds between atoms.
These particles carry charges, which obey Coulomb’s Law, but also the Laws of Quantum Mechanics. So in order to arrive at a
satisfactory explanation of the elastic restoring properties of the spring, we need models that describe the fundamental physics that
underline Hooke’s Law.

Universal Law of Gravitation


At points significantly far away from the surface of Earth, the gravitational force is no longer constant with respect to the distance
to the center of Earth. Newton’s Universal Law of Gravitation describes the gravitational force between two objects with masses,
m and m . This force points along the line connecting the objects, is attractive, and its magnitude is proportional to the inverse
1 2

square of the distance, r between the two point-like objects (Figure 8.4a). The force on object 2 due to the gravitational
1,2

interaction between the two objects is given by


→G m 1 m2
F 1,2 = −G ^
r1,2
2
r
1,2

→ → → ∣→ → ∣→
where r 1,2 = r 2 − r 1 is a vector directed from object 1 to object 2, r1,2 =

r 1,2


, and ^
r1,2 = r 1,2 / r 1,2



is a unit vector
directed from object 1 to object 2 (Figure 8.4b). The constant of proportionality in SI units is G = 6.67 × 10 −11
N⋅m
2
⋅ kg
−2
.

Figure 8.4 (b) Coordinate system for the two-body problem.

Principle of Equivalence:
The Principle of Equivalence states that the mass that appears in the Universal Law of Gravity is identical to the inertial mass that
is determined with respect to the standard kilogram. From this point on, the equivalence of inertial and gravitational mass will be
assumed and the mass will be denoted by the symbol m.

8.2.1 https://phys.libretexts.org/@go/page/24467
Gravitational Force near the Surface of the Earth
Near the surface of Earth, the gravitational interaction between an object and Earth is mutually attractive and has a magnitude of
∣→G ∣
∣ F earth,object ∣ = mg
∣ ∣

where g is a positive constant.


The International Committee on Weights and Measures has adopted as a standard value for the acceleration of an object freely
falling in a vacuum g = 9.80665m ⋅ s . The actual value of g varies as a function of elevation and latitude. If ϕ is the latitude and
−2

h the elevation in meters then the acceleration of gravity in SI units is


2 −4 −2
g = (9.80616 − 0.025928 cos(2ϕ) + 0.000069 cos (2ϕ) − 3.086 × 10 h) m ⋅ s

This is known as Helmert’s equation. The strength of the gravitational force on the standard kilogram at 42 latitude is ∘

9.80345N ⋅ kg and the acceleration due to gravity at sea level is therefore g = 9.80345m ⋅ s for all objects. At the equator,
−1 −2

g = 9.78m ⋅ s
−2
and at the poles g = 9.83m ⋅ s This difference is primarily due to the earth’s rotation, which introduces an
−2

apparent (fictitious) repulsive force that affects the determination of g as given in Equation (8.2.2) and also flattens the spherical
shape of Earth (the distance from the center of Earth is larger at the equator than it is at the poles by about 26.5 km ). Both the
magnitude and the direction of the gravitational force also show variations that depend on local features to an extent that's useful in
prospecting for oil, investigating the water table, navigating submerged submarines, and as well as many other practical uses. Such
variations in g can be measured with a sensitive spring balance. Local variations have been much studied over the past two decades
in attempts to discover a proposed “fifth force” which would fall off faster than the gravitational force that falls off as the inverse
square of the distance between the objects.

Electric Charge and Coulomb’s Law


Matter has properties other than mass. Matter can also carry one of two types of observed electric charge, positive and negative.
Like charges repel, and opposite charges attract each other. The unit of charge in the SI system of units is called the coulomb [C].
The smallest unit of “free” charge known in nature is the charge of an electron or proton, which has a magnitude of
−19
e = 1.602 × 10 C

It has been shown experimentally that charge carried by ordinary objects is quantized in integral multiples of the magnitude of this
free charge. The electron carries one unit of negative charge (q = −e) and the proton carries one unit of positive charge
e

(q = +e) . In an isolated system, the charge stays constant; in a closed system, an amount of unbalanced charge can neither be
p

created nor destroyed. Charge can only be transferred from one object to another.
Consider two point-like objects with charges q and q separated by a distance r in vacuum. By experimental observation, the
1 2 1,2

two objects repel each other if they are both positively or negatively charged (Figure 8.4a). They attract each other if they are
oppositely charged (Figure 8.5b). The force exerted on object 2 due to the interaction between objects 1 and 2 is given by
Coulomb's Law,
→E q1 q2
F 1,2 = ke ^
r1,2
2
r
1,2

→ ∣→
where ^
r 1,2 = r 1,2 / r 1,2



is a unit vector directed from object 1 to object 2, and in SI units, k e
9
= 8.9875 × 10 N ⋅ m
2
⋅C
−2
as
illustrated in the Figure 8.5a. This law was derived empirically by Charles Augustin de Coulomb in the late 18th century.

8.2.2 https://phys.libretexts.org/@go/page/24467
Figure 8.5 (a) and 8.5 (b) Coulomb interaction between two charges

Example 8.2.1: Coulomb’s Law and the Universal Law of Gravitation

Show that both Coulomb’s Law and the Universal Law of Gravitation satisfy Newton’s Third Law.
Solution
To see this, interchange 1 and 2 in the Universal Law of Gravitation to find the force on object 1 due to the interaction between
the objects. The only quantity to change sign is the unit vector

^
r2,1 = −^
r1,2

Then
→G m2 m 1 m1 m 2 →G
F 2,1 = −G ^
r2,1 = G ^
r1,2 = − F 1,2
2 2
r r
2,1 1,2

Coulomb’s Law also satisfies Newton’s Third Law since the only quantity to change sign is the unit vector, just as in the case
of the Universal Law of Gravitation.

This page titled 8.2: Fundamental Laws of Nature is shared under a CC BY-NC-SA 4.0 license and was authored, remixed, and/or curated by
Peter Dourmashkin (MIT OpenCourseWare) via source content that was edited to the style and standards of the LibreTexts platform; a detailed
edit history is available upon request.

8.2.3 https://phys.libretexts.org/@go/page/24467
8.3: Constraint Forces
Knowledge of all the external and internal forces acting on each of the objects in a system and applying Newton’s Second Law to
each of the objects determine a set of equations of motion. These equations of motion are not necessarily independent due to the
fact that the motion of the objects may be limited by equations of constraint. In addition there are forces of constraint that are
determined by their effect on the motion of the objects and are not known beforehand or describable by some force law. For
example: an object sliding down an inclined plane is constrained to move along the surface of the inclined plane (Figure 8.6a) and
the surface exerts a contact force on the object; an object that slides down the surface of a sphere until it falls off experiences a
contact force until it loses contact with the surface (Figure 8.6b); gas particles in a sealed vessel are constrained to remain inside
the vessel and therefore the wall must exert force on the gas molecules to keep them inside the vessel (8.6c); and a bead constrained
to slide outward along a rotating rod is acting on by time dependent forces of the rod on the bead (Figure 8.6d). We shall develop
methods to determine these constraint forces although there are many examples in which the constraint forces cannot be
determined.

Figure 8.6 Constrained motions: (a) particle sliding down inclined plane, (b) particles sliding down surface of sphere, (c) gas
molecules in a sealed vessel, and (d) bead sliding on a rotating rod

Contact Forces
Pushing, lifting and pulling are contact forces that we experience in the everyday world. Rest your hand on a table; the atoms that
form the molecules that make up the table and your hand are in contact with each other. If you press harder, the atoms are also
pressed closer together. The electrons in the atoms begin to repel each other and your hand is pushed in the opposite direction by
the table.
According to Newton’s Third Law, the force of your hand on the table is equal in magnitude and opposite in direction to the force
of the table on your hand. Clearly, if you push harder the force increases. Try it! If you push your hand straight down on the table,
the table pushes back in a direction perpendicular (normal) to the surface. Slide your hand gently forward along the surface of the
table. You barely feel the table pushing upward, but you do feel the friction acting as a resistive force to the motion of your hand.
This force acts tangential to the surface and opposite to the motion of your hand. Push downward and forward. Try to estimate the
magnitude of the force acting on your hand.
C
→ →
The force of the table acting on your hand, F ≡ C is called the contact force. This force has both a normal component to the
→ → → →
surface, C⊥ ≡ N , called the normal force, and a tangential component to the surface, C∥ ≡ f called the friction force (Figure
8.6).

8.3.1 https://phys.libretexts.org/@go/page/24468
Figure 8.6 Normal and tangential components of the contact force
The contact force, written in terms of its component forces, is therefore
→ → → → →
C = C⊥ + C∥ ≡ N + f

→ →
Any force can be decomposed into component vectors so the normal component, N , and the tangential component, f are not
independent forces but the vector components of the contact force, perpendicular and parallel to the surface of contact. The contact
force is a distributed force acting over all the points of contact between your hand and the surface.
For most applications we shall treat the contact force as acting at single point but precaution must be taken when the distributed
nature of the contact force plays a key role in constraining the motion of a rigid body.

In Figure 8.7, the forces acting on your hand are shown. These forces include the contact force, C of the table acting on your hand,

the force of your forearm, F forearm  acting on your hand (which is drawn at an angle indicating that you are pushing down on your
→g
hand as well as forward), and the gravitational interaction, F between the earth and your hand.

Figure 8.7 Forces on hand when moving towards the left


One point to keep in mind is that the magnitudes of the two components of the contact force depend on how hard you push or pull
your hand and in what direction, a characteristic of constraint forces, in which the components are not specified by a force law but
dependent on the particular motion of the hand.
Example 8.2 Normal Component of the Contact Force and Weight
Hold a block in your hand such that your hand is at rest (Figure 8.8). You can feel the “weight” of the block against your palm. But
what exactly do we mean by “weight”?

8.3.2 https://phys.libretexts.org/@go/page/24468
Figure 8.9 Forces on block
There are two forces acting on the block as shown in Figure 8.9. One force is the gravitational force between the earth and the
→g →
block, and is denoted by F = m g . The other force acting on the block is the contact force between your hand and the block.
Because our hand is at rest, this contact force on the block points perpendicular to the surface, and hence has only a normal

component, N Let N denote the magnitude of the normal force. Because the object is at rest in your hand, the vertical acceleration
is zero. Therefore Newton’s Second Law states that
→ →g →
N+ F = 0

Choose the positive direction to be upwards and then in terms of vertical components we have that

N − mg = 0

which can be solved for the magnitude of the normal force

N = mg

When we talk about the “weight” of the block, we often are referring to the effect the block has on a scale or on the feeling we have
when we hold the block. These effects are actually effects of the normal force. We say that a block “feels lighter” if there is an
additional force holding the block up. For example, you can rest the block in your hand, but use your other hand to apply a force
upwards on the block to make it feel lighter in your supporting hand.
The word “weight,” is often used to describe the gravitational force that Earth exerts on an object. We shall always refer to this
force as the gravitational force instead of “weight.” When you jump in the air, you feel “weightless” because there is no normal
force acting on you, even though Earth is still exerting a gravitational force on you; clearly, when you jump, you do not turn gravity
off!
This example may also give rise to a misconception that the normal force is always equal to the mass of the object times the
magnitude of the gravitational acceleration at the surface of the earth. The normal force and the gravitational force are two
completely different forces. In this particular example, the normal force is equal in magnitude to the gravitational force and directed
in the opposite direction because the object is at rest. The normal force and the gravitational force do not form a Third Law
interaction pair of forces. In this example, our system is just the block and the normal force and gravitational force are external
forces acting on the block.
Let’s redefine our system as the block, your hand, and Earth. Then the normal force and gravitational force are now internal forces
in the system and we can now identify the various interaction pairs of forces. We explicitly introduce our interaction pair notation
→g
to enable us to identify these interaction pairs: for example, let F E,B denote the gravitational force on the block due to the
→g
interaction with Earth. The gravitational force on Earth due to the interaction with the block is denoted by F B,E and these two
→g →g
forces form an interaction pair. By Newton’s Third Law, F = −F
E,B
. Note that these two forces are acting on different
B,E

objects, the block and Earth. The contact force on the block due to the interaction between the hand and the block is then denoted

8.3.3 https://phys.libretexts.org/@go/page/24468
→ → → →
by N . The force of the block on the hand, which we denote by N
H ,B , satisfies N
B,H = −N . Because we are including
B,H H ,B

your hand as part of the system, there are two additional forces acting on the hand. There is the gravitational force on your hand
→g →g →g →g
F E,H , satisfying F E,H = − F H ,E , where F H ,E is the gravitational force on Earth due to your hand. Finally there is the force of

your forearm holding your hand up, which we denote F . Because we are not including the forearm in our system, this force is
F ,H

an external force to the system. The forces acting on your hand are shown in diagram on your hand is shown in Figure 8.10, and the
just the interaction pairing of forces acting on Earth is shown in Figure 8.11 (we are not representing all other external forces acting
on the Earth).

Figure 8.11 Gravitational forces on earth due to object and hand

Kinetic and Static Friction


When a block is pulled along a horizontal surface or sliding down an inclined plane there is a lateral force resisting the motion. If
the block is at rest on the inclined plane, there is still a lateral force resisting the motion. This resistive force is known as dry
friction, and there are two distinguishing types when surfaces are in contact with each other. The first type is when the two objects
are moving relative to each other; the friction in that case is called kinetic friction or sliding friction. When the two surfaces are
non-moving but there is still a lateral force as in the example of the block at rest on an inclined plane, the force is called, static
friction.
Leonardo da Vinci was the first to record the results of measurements on kinetic friction over a twenty-year period between 1493–4
→k
and about 1515. Based on his measurements, the force of kinetic friction, f between two surfaces, he identified two key
properties of kinetic friction. The magnitude of kinetic friction is proportional to the normal force between the two surfaces,

fk = μk N

where μ is called the coefficient of kinetic friction. The second result is rather surprising in that the magnitude of the force is
k

independent of the contact surface. Consider two blocks of the same mass, but different surface areas. The force necessary to move
the blocks at a constant speed is the same. The block in Figure 8.12a has twice the contact area as the block shown in Figure 8.12b,
but when the same external force is applied to either block, the blocks move at constant speed. These results of da Vinci were
rediscovered by Guillaume Amontons and published in 1699. The third property that kinetic friction is independent of the speed of
moving objects (for ordinary sliding speeds) was discovered by Charles Augustin Coulomb.

Figure 8.12 (a) and (b): kinetic friction is independent of the contact area

8.3.4 https://phys.libretexts.org/@go/page/24468
→k
The kinetic friction on surface 2 moving relative to surface 1 is denoted by, f The direction of the force is always opposed to
1,2

the relative direction of motion of surface 2 relative to the surface 1. When one surface is at rest relative to our choice of reference
→k
frame we will denote the friction force on the moving object by f .
The second type of dry friction, static friction occurs when two surfaces are static relative to each other. Because the static friction
→s
force between two surfaces forms a third law interaction pair, will use the notation f to denote the static friction force on
1,2

surface 2 due to the interaction between surfaces 1 and 2. Push your hand forward along a surface; as you increase your pushing
force, the frictional force feels stronger and stronger. Try this! Your hand will at first stick until you push hard enough, then your
hand slides forward. The magnitude of the static frictional force, f , depends on how hard you push.
s

If you rest your hand on a table without pushing horizontally, the static friction is zero. As you increase your push, the static
friction increases until you push hard enough that your hand slips and starts to slide along the surface. Thus the magnitude of static
friction can vary from zero to some maximum value, (f ) when the pushed object begins to slip,
s max

0 ≤ fs ≤ (fs )
max

Is there a mathematical model for the magnitude of the maximum value of static friction between two surfaces? Through
experimentation, we find that this magnitude is, like kinetic friction, proportional to the magnitude of the normal force

(fs ) = μs N
max

Here the constant of proportionality is μ , the coefficient of static friction. This constant is slightly greater than the constant μ
s k

associated with kinetic friction, μ > μ . This small difference accounts for the slipping and catching of chalk on a blackboard,
s k

fingernails on glass, or a violin bow on a string.


The direction of static friction on an object is always opposed to the direction of the applied force (as long as the two surfaces are
→ →s
not accelerating). In Figure 8.13a, an external force, F is applied the left and the static friction, f is shown pointing to the right
→ →s
opposing the external force. In Figure 8.13b, the external force, F is directed to the right and the static friction, f , is now
pointing to the left.

Figure 8.13 (a) and (b): External forces and the direction of static friction.
Although the force law for the maximum magnitude of static friction resembles the force law for sliding friction, there are
important differences:
1. The direction and magnitude of static friction on an object always depends on the direction and magnitude of the applied forces
acting on the object, where the magnitude of kinetic friction for a sliding object is fixed.
2. The magnitude of static friction has a maximum possible value. If the magnitude of the applied force along the direction of the
contact surface exceeds the magnitude of the maximum value of static friction, then the object will start to slip (and be subject to
kinetic friction.) We call this the just slipping condition.

This page titled 8.3: Constraint Forces is shared under a CC BY-NC-SA 4.0 license and was authored, remixed, and/or curated by Peter
Dourmashkin (MIT OpenCourseWare) via source content that was edited to the style and standards of the LibreTexts platform; a detailed edit
history is available upon request.

8.3.5 https://phys.libretexts.org/@go/page/24468
8.4: Free-body Force Diagram
System
When we try to describe forces acting on a collection of objects we must first take care to specifically define the collection of
objects that we are interested in, which define our system. Often the system is a single isolated object but it can consist of multiple
objects.
Because force is a vector, the force acting on the system is a vector sum of the individual forces acting on the system
→ → →
F = F1 + F2 +⋯

A free-body force diagram is a representation of the sum of all the forces that act on a single system. We denote the system by a
large circular dot, a “point”. (Later on in the course we shall see that the “point” represents the center of mass of the system.) We
represent each force that acts on the system by an arrow (indicating the direction of that force). We draw the arrow at the “point”
representing the system. For example, the forces that regularly appear in free-body diagram are contact forces, tension, gravitation,
friction, pressure forces, spring forces, electric and magnetic forces, which we shall introduce below. Sometimes we will draw the
arrow representing the actual point in the system where the force is acting. When we do that, we will not represent the system by a
“point” in the free-body diagram.
Suppose we choose a Cartesian coordinate system, then we can resolve the force into its component vectors

^ ^ ^
F = Fx i + Fy j + Fz k

Each one of the component vectors is itself a vector sum of the individual component vectors from each contributing force. We can
use the free-body force diagram to make these vector decompositions of the individual forces. For example, the x - component of
the force is
Fx = F1,x + F2,x + ⋯

Modeling
One of the most central and yet most difficult tasks in analyzing a physical interaction is developing a physical model. A physical
model for the interaction consists of a description of the forces acting on all the objects. The difficulty arises in deciding which
forces to include. For example in describing almost all planetary motions, the Universal Law of Gravitation was the only force law
that was needed. There were anomalies, for example the small shift in Mercury’s orbit. These anomalies are interesting because
they may lead to new physics. Einstein corrected Newton’s Law of Gravitation by introducing General Relativity and one of the
first successful predictions of the new theory was the perihelion precession of Mercury’s orbit. On the other hand, the anomalies
may simply
be due to the complications introduced by forces that are well understood but complicated to model. When objects are in motion
there is always some type of friction present. Air friction is often neglected because the mathematical models for air resistance are
fairly complicated even though the force of air resistance substantially changes the motion. Static or kinetic friction between
surfaces is sometimes ignored but not always. The mathematical description of the friction between surfaces has a simple
expression so it can be included without making the description mathematically intractable. A good way to start thinking about the
problem is to make a simple model, excluding complications that are small order effects. Then we can check the predictions of the
model. Once we are satisfied that we are on the right track, we can include more complicated effects.

This page titled 8.4: Free-body Force Diagram is shared under a CC BY-NC-SA 4.0 license and was authored, remixed, and/or curated by Peter
Dourmashkin (MIT OpenCourseWare) via source content that was edited to the style and standards of the LibreTexts platform; a detailed edit
history is available upon request.

8.4.1 https://phys.libretexts.org/@go/page/24469
8.5: Tension in a Rope
Definition of Tension in a Rope
Let’s return to our example of the very light rope (object 2 with m 2 ≃0 ) that is attached to a block (object 1) at the point B , and

pulled by an applied force at point A F A,2 (Figure 8.18a).

Figure 8.18a Massless rope pulling a block

Choose a coordinate system with the -unit vector pointing upward in the normal direction to the surface, and the ^i -unit vector
^
j

pointing in the positive x -direction, (Figure 8.18b). The force diagrams for the system consisting of the rope and block is shown in

Figure 8.19, and for the rope and block separately in Figure 8.20, where F 2,1 the force on the block (object 1) due to the rope

(object 2), and F 1,2 is the force on the rope due to the block.

Figure 8.18b Forces acting on system consisting of block and rope


The forces on the rope and the block must each sum to zero. Because the rope is not accelerating, Newton’s Second Law applied to
the rope requires that F − F = m a (where we are using magnitudes for all the forces).
A,2 1,2 2

Figure 8.19 Separate force diagrams for rope and block


Because we are assuming the mass of the rope is negligible therefore

FA,2 − F1,2 = 0;  (massless rope) 

If we consider the case that the rope is very light, then the forces acting at the ends of the rope are nearly horizontal. Then if the
rope-block system is moving at constant speed or at rest, Newton’s Second Law is now

FA,2 − F1,2 = 0

Newton’s Second Law applied to the block in the +^i -direction requires that F2,1 − f = 0 Newton’s Third Law, applied to the
block-rope interaction pair requires that F = F . Therefore
1,2 2,1

FA,2 = F1,2 = F2,1 = f

8.5.1 https://phys.libretexts.org/@go/page/24470
Thus the applied pulling force is transmitted through the rope to the block since it has the same magnitude as the force of the rope
on the block. In addition, the applied pulling force is also equal to the friction force on the block.
How do we define “tension” at some point in a rope? Suppose make an imaginary slice of the rope at a point P , a distance x from P

point B , where the rope is attached to the block. The imaginary slice divides the rope into two sections, labeled L (left) and R
(right), as shown in Figure 8.20.

Figure 8.20 Imaginary slice through the rope


There is now a Third Law pair of forces acting between the left and right sections of the rope. Denote the force acting on the left
→ →
section by F R,L (x ) and the force acting on the right section by
P F L,R (xP ) Newton’s Third Law requires that the forces in this
interaction pair are equal in magnitude and opposite in direction.
→ →
F R,L (xp ) = − F L,R (xP )


The force diagram for the left and right sections are shown in Figure 8.21 where F 1,L is the force on the left section of the rope

due to the block-rope interaction. (We had previously denoted that force by F 1,2 ) Now denote the force on the right section of the
→ →
rope side due to the pulling force at the point A by F A,R (which we had previously denoted by F A,2 ).

Figure 8.21 Force diagram for the left and right sections of rope
The tension T (x ) at a point P in rope lying a distance x from one the left end of the rope, is the magnitude of the action -reaction
P

pair of forces acting at the point P ,


→ →
∣ ∣ ∣ ∣
T (xp ) = ∣ F R,L (xP )∣ = ∣ F L,R (xP )∣
∣ ∣ ∣ ∣

For a rope of negligible mass, under tension, as in the above case, (even if the rope is accelerating) the sum of the horizontal forces
applied to the left section and the right section of the rope are zero, and therefore the tension is uniform and is equal to the applied
pulling force,

T = FA,R

Example 8.3 Tension in a Massive Rope

Figure 8.22a Massive rope pulling a block

8.5.2 https://phys.libretexts.org/@go/page/24470
Consider a block of mass m that is lying on a horizontal surface. The coefficient of kinetic friction between the block and the
1

surface is μ . A uniform rope of mass m and length d is attached to the block. The rope is pulled from the side opposite the block
k 2

∣→ ∣
with an applied force of magnitude ∣ F A,2 ∣ = FA,2 . Because the rope is now massive, the pulling force makes an angle ϕ with
∣ ∣

respect to the horizontal in order to balance the gravitational force on the rope, (Figure 8.22a). Determine the tension in the rope as
a function of distance x from the block.
Solution: In the following analysis, we shall assume that the angle ϕ is very small and depict the pulling and tension forces as
essentially acting in the horizontal direction even though there must be some small vertical component to balance the gravitational
forces.
The key point to realize is that the rope is now massive and we must take in to account the inertia of the rope when applying
Newton’s Second Law. Consider an imaginary slice through the rope at a distance x from the block (Figure 8.22b), dividing the
rope into two sections. The right section has length d − x and mass m = (m /d) (d − x) .
R 2

mR = (m2 /d) (d − x) (8.5.1)

. The left section has length x and mass m L = (m2 /d) (x) .

Figure 8.22b Imaginary slice through the rope


The free body force diagrams for the two sections of the rope are shown in Figure 8.22c, where T (x) is the tension in the rope at a
∣→ ∣ ∣→ ∣
distance x from the block, and F 1,L = ∣ F 1,L ∣ ≡ ∣ F 1,2 ∣ is the magnitude of the force on the left-section of the rope due to the rope-
∣ ∣ ∣ ∣

block interaction.

Figure 8.22c Force diagram for the left and right sections of rope
Apply Newton’s Second Law to the right section of the rope yielding
m2
FA,R − T (x) = mR aR = (d − x)aR
d

where a is the x -component of the acceleration of the right section of the rope. Apply Newton’s Second Law to the left slice of
R

the rope yielding

T (x) − F1,L = mL aL = (m2 /d) x aL

where a is the x -component of the acceleration of the left piece of the rope.
L

8.5.3 https://phys.libretexts.org/@go/page/24470
Figure 8.23 Force diagram on sliding block

The force diagram on the block is shown in Figure 8.23. Newton’s Second Law on the block in the +^i - direction is
FL,1−f = m a
k 1 and in the +j^- direction is N − m g = 0 The kinetic friction force acting on the block is
1 1

f = μ N = μ m g Newton’s Second Law on the block in the + i -direction becomes


^
k k k 1

FL,1 − μk m1 g = m1 a1

Newton’s Third Law for the block-rope interaction is given by F L,1 = F1,L . Equation (8.5.8) then becomes

T (x) − (μk m1 g + m1 a1 ) = (m2 /d) x aL

Because the rope and block move together, the accelerations are equal which we denote by the symbol a ≡ a1 = aL . Then
Equation (8.5.10) becomes

T (x) = μk m1 g + (m1 + (m2 /d) x) a

This result is not unexpected because the tension is accelerating both the block and the left section and is opposed by the frictional
force.
Alternatively, the force diagram on the system consisting of the rope and block is shown in Figure 8.24.

Figure 8.24 Force diagram on block-rope system


Newton’s Second Law becomes

FA,R − μk m1 g = (m2 + m1 ) a

Solve Equation (8.5.12) for F A,R and substitute into Equation (8.5.7), and solve for the tension yielding Equation (8.5.11).
Example 8.4 Tension in a Suspended Rope
A uniform rope of mass M and length L is suspended from a ceiling (Figure 8.25). The magnitude of the acceleration due to gravity
is g . (a) Find the tension in the rope at the upper end where the rope is fixed to the ceiling. (b) Find the tension in the rope as a
function of the distance from the ceiling. (c) Find an equation for the rate of change of the tension with respect to distance from the
ceiling in terms of M , L , and g .

8.5.4 https://phys.libretexts.org/@go/page/24470
Figure 8.26 Coordinate system for suspended rope
Solution: (a) Begin by choosing a coordinate system with the origin at the ceiling and the positive y -direction pointing downward
(Figure 8.26). In order to find the tension at the upper end of the rope, choose as a system the entire rope. The forces acting on the
rope are the force at y = 0 holding the rope up, T ( y = 0) , and the gravitational force on the entire rope. The free-body force
diagram is shown in Figure 8.27.

Figure 8.27 Force diagram on rope


Because the acceleration is zero, Newton’s Second Law on the rope is M g − T (y = 0) = 0 . Therefore the tension at the upper end
is T (y = 0) = M g .
(b) Recall that the tension at a point is the magnitude of the action-reaction pair of forces acting at that point. Make an imaginary
slice in the rope a distance y from the ceiling separating the rope into an upper segment 1, and lower segment 2 (Figure 8.28a).
Choose the upper segment as a system with mass m = (M /L)y The forces acting on the upper segment are the gravitational
1

force, the force T ( y = 0) holding the rope up, and the tension T ( y) at the point y , that is pulling the upper segment down. The
free-body force diagram is shown in Figure 8.28b.

8.5.5 https://phys.libretexts.org/@go/page/24470
Figure 8.28 (a) Imaginary slice separates rope into two pieces. (b) Free-body force diagram on upper piece of rope
Apply Newton’s Second Law to the upper segment: m g + T (y) − T (y = 0) = 0 . Therefore the tension at a distance y from the
1

ceiling is T (y) = T (y = 0) − m g . Because m = (M /L)y is the mass of the segment piece and Mg is the tension at the upper
1 1

end, Newton’s Second Law becomes

T (y) = M g(1 − y/L)

As a check, we note that when y =L the tension T (y = L) = 0 which is what we expect because there is no force acting at the
lower end of the rope.
(c) Differentiate Equation (8.5.13) with respect to y yielding
dT
= −(M /L)g
dy

The rate that the tension is changing at a constant rate with respect to distance from the top of the rope.

Continuous Systems and Newton’s Second Law as a Differential Equations


We can determine the tension at a distance y from the ceiling in Example 8.4, by an alternative method, a technique that will
generalize to many types of “continuous systems”. Choose a coordinate system with the origin at the ceiling and the positive y -
direction pointing downward as in Figure 8.25. Consider as the system a small element of the rope between the points y and
y + Δy . This small element has length Δy The small element has mass Δm = (M /L)Δy and is shown in Figure 8.29.

Δm = (M /L)Δy

8.5.6 https://phys.libretexts.org/@go/page/24470
The forces acting on the small element are the tension, T (y) at y directed upward, the tension T (y + Δy) at y + Δy directed
downward, and the gravitational force Δmg directed downward. The tension T (y + Δy) is equal to the tension T ( y) plus a small
difference ΔT

T (y + Δy) = T (y) + ΔT

The small difference in general can be positive, zero, or negative. The free body force diagram is shown in Figure 8.30.

Figure 8.30 Free body force diagram on small mass element


Now apply Newton’s Second Law to the small element

Δmg + T (y) − (T (y) + ΔT ) = 0

The difference in the tension is then ΔT = −Δmg . We now substitute our result for the mass of the element Δm = (M /L)Δy ,
and find that that

ΔT = −(M /L)Δyg

Divide through by Δy yielding ΔT /Δy = −(M /L)g . Now take the limit in which the length of the small element goes to zero,
Δy → 0

ΔT
lim = −(M /L)g
Δy→0 Δy

Recall that the left hand side of Equation (8.5.18) is the definition of the derivative of the tension with respect to y , and so we
arrive at Equation (8.5.14),
dT
= −(M /L)g
dy

We can solve the differential equation, Equation (8.5.14), by a technique called separation of variables. We rewrite the equation as
dT = −(M /L)gdy and integrate both sides. Our integral will be a definite integral in which we integrate a ‘dummy’ integration

variable y and y = 0 to y = y and the corresponding T from T = T (y = 0) to T = T (y) :


′ ′ ′ ′ ′ ′

′ ′
T =T (y) y =y
′ ′
∫ dT = −(M /L)g ∫ dy

T =T (y=0) y ′ =0

After integration and substitution of the limits, we have that

T (y) − T (y = 0) = −(M /L)gy

Us the fact that tension at the top of the rope is T (y = 0) = M g and find that

T (y) = M g(1 − y/L)

in agreement with our earlier result, Equation (8.5.13).

This page titled 8.5: Tension in a Rope is shared under a CC BY-NC-SA 4.0 license and was authored, remixed, and/or curated by Peter
Dourmashkin (MIT OpenCourseWare) via source content that was edited to the style and standards of the LibreTexts platform; a detailed edit

8.5.7 https://phys.libretexts.org/@go/page/24470
history is available upon request.

8.5.8 https://phys.libretexts.org/@go/page/24470
8.6: Drag Forces in Fluids
When a solid object moves through a fluid it will experience a resistive force, called the drag force, opposing its motion. The fluid
may be a liquid or a gas. This force is a very complicated force that depends on both the properties of the object and the properties
of the fluid. The force depends on the speed, size, and shape of the object. It also depends on the density, viscosity and
compressibility of the fluid.
For objects moving in air, the air drag is still quite complicated but for rapidly Table 8.1 Drag Coefficients moving objects the
resistive force is roughly proportional to the square of the speed v , the cross-sectional area A of the object in a plane perpendicular
to the motion, the density ρ of the air, and independent of the viscosity of the air. Traditional the magnitude of the air drag for
rapidly moving objects is written as
1 2
Fdrag = CD Aρv
2

The coefficient C is called the drag coefficient, a dimensionless number that is a property of the object. Table 8.1 lists the drag
D

coefficient for some simple shapes, (each of these objects has a Reynolds number of order 104 ).

The above model for air drag does not extend to all fluids. An object dropped in oil, molasses, honey, or water will fall at different
rates due to the different viscosities of the fluid. For very low speeds, the drag force depends linearly on the speed and is also
proportional to the viscosity η of the fluid. For the special case of a sphere of radius R , the drag force law can be exactly deduced
from the principles of fluid mechanics and is given by
→ →
F drag = −6πηR v ( sphere )

This force law is known as Stokes’ Law. The coefficient of viscosity η has SI units of [N ⋅ m ⋅ s] = [Pa ⋅ s] = [kg ⋅ m
−2 −1 −1
⋅s ] ;
a cgs unit called the poise is often encountered . Some typical coefficients of viscosity are listed in Table 8.2.

8.6.1 https://phys.libretexts.org/@go/page/24471
This law can be applied to the motion of slow moving objects in a fluid, for example: very small water droplets falling in a
gravitational field, grains of sand settling in water, or the sedimentation rate of molecules in a fluid. In the later case, If we model a
molecule as a sphere of radius R , the mass of the molecule is proportional to R3 and the drag force is proportion to R , therefore
different sized molecules will have different rates of acceleration. This is the basis for the design of measuring devices that separate
molecules of different molecular weights.
In many physical situations the force on an object will be modeled as depending on the object’s velocity. We have already seen
static and kinetic friction between surfaces modeled as being independent of the surfaces’ relative velocity. Common experience
(swimming, throwing a Frisbee) tells us that the frictional force between an object and a fluid can be a complicated function of
velocity. Indeed, these complicated relations are an important part of such topics as aircraft design.
Example 8.5 Drag Force at Low Speeds

Figure 8.31 Example 8.5


A spherical marble of radius R and mass m is released from rest and falls under the influence of gravity through a jar of olive oil of
viscosity η . The marble is released from rest just below the surface of the olive oil, a height h from the bottom of the jar. The
gravitational acceleration is g (Figure 8.31). Neglect any force due to the buoyancy of the olive oil. (i) Determine the velocity of
→ →
the marble as a function of time, (ii) what is the maximum possible velocity v ∞ = v (t = ∞) (terminal velocity), that the marble
∣→ ∣
can obtain, (iii) determine an expression for the viscosity of olive oil η in terms of g , m, R , and v∞ =



(iv) determine an
expression for the position of the marble from just below the surface of the olive oil as a function of time.
Solution: Choose positive y -direction downwards with the origin at the initial position of the marble as shown in Figure 8.32(a).

8.6.2 https://phys.libretexts.org/@go/page/24471
Figure 8.32 (a) Coordinate system for marble; (b) free body force diagram on marble
There are two forces acting on the marble: the gravitational force, and the drag force which is given by Equation (8.6.2). The free
body diagram is shown in the Figure 8.32(b). Newton’s Second Law is then
dv
mg − 6πηRv = m
dt

where v is the y -component of the velocity of the marble. Let γ = 6πηR/m ; the SI units γ are −1
[S ] . Then Equation (8.6.3)
becomes
dv
g − γv =
dt

Suppose the object has an initial y -component of velocity v(t = 0) = 0 . We shall solve Equation (8.6.3) using the method of
separation of variables. The differential equation may be rewritten as
dv
= −γdt
(v − g/γ)

The integral version of Equation (8.6.5) is then


′ ′
v =v(t) ′ t =t
dv

∫ = −γ ∫ dt

v′ =0 v − g/γ t =0

Integrating both sides of Equation (8.6.6) yields


v(t) − g/γ
ln( ) = −γt
−g/γ

Recall that e ln x
=x , therefore upon exponentiation of Equation (8.6.7) yields
v(t) − g/γ
−γt
=e
−g/γ

Thus the y -component of the velocity as a function of time is given by


g mg
−γt −(6πηR/m)t
v(t) = (1 − e ) = (1 − e )
γ 6πηR

A plot of v(t) vs. t is shown in Figure 8.31 with parameters R = 5.00 × 10


−3
m , η = 8.10 × 10
−2 −1
kg ⋅ m
−1
⋅s ,
kg , and g/γ = 1.87m ⋅ s
−3 −1
m = 4.08 × 10

8.6.3 https://phys.libretexts.org/@go/page/24471
Figure 8.33 Plot of y -component of the velocity v(t) vs. t for marble falling through oil with g/γ = 1.87m ⋅ s −1
.
For large values of t , the term e −(6πηR/m)t
approaches zero, and the marble reaches a terminal velocity
mg
v∞ = v(t = ∞) =
6πηR

The coefficient of viscosity can then be determined from the terminal velocity by the condition that
mg
η =
6πRvter

Let ρ denote the density of the marble. The mass of the spherical marble is m = (4/3)ρ
m mR
3
. The terminal velocity is then
2
2 ρm R g
v∞ =

The terminal velocity depends on the square of the radius of the marble, indicating that larger marbles will reach faster terminal
speeds.
The position of the marble as a function of time is given by the integral expression

t =t
′ ′
y(t) − y(t = 0) = ∫ v (t ) dt

t =0

which after substitution of Equation (8.6.9) and integration using the initial condition that y(t = 0) = 0 , becomes
g g −γt
y(t) = t+ (e − 1)
γ γ2

Example 8.6 Drag Forces at High Speeds



An object of mass m at time t = 0 is moving rapidly with velocity V through a fluid of density ρ . Let A denote the cross-sectional
0

area of the object in a plane perpendicular to the motion. The object experiences a retarding drag force whose magnitude is given
by Equation (8.6.1). Determine an expression for the velocity of the object as a function of time.

Solution: Choose a coordinate system such that the object is moving in the positive x-direction ^
v = v i .  Set β = (1/2)CD Aρ .
Newton’s Second Law can then be written as

2
dv
−β v =
dt

An integral version of Equation (8.6.15) is then


′ ′
v =v(t) ′ t =t
dv ′
∫ = −β ∫ dt
′2

v =v0 v ′
t =0

Integration yields
1 1
−( − ) = −βt
v(t) v0

After some algebraic rearrangement the x -component of the velocity as a function of time is given by

8.6.4 https://phys.libretexts.org/@go/page/24471
v0 1
v(t) = = v0
1 + v0 βt 1 + t/τ

where τ . A plot of v(t) vs. t is shown in Figure 8.34 with initial conditions v
= 1/ v0 β 0 = 20m ⋅ s
−1
and β = 0.5s −1

Figure 8.34 Plot of v(t) vs. t for damping force F drag =


1

2
CD Aρv
2

This page titled 8.6: Drag Forces in Fluids is shared under a CC BY-NC-SA 4.0 license and was authored, remixed, and/or curated by Peter
Dourmashkin (MIT OpenCourseWare) via source content that was edited to the style and standards of the LibreTexts platform; a detailed edit
history is available upon request.

8.6.5 https://phys.libretexts.org/@go/page/24471
8.7: Worked Examples
Example 8.7 Staircase
An object of mass m at time t = 0 has speed v It slides a distance s along a horizontal floor and then off the top of a staircase
0

(Figure 8.35). The coefficient of kinetic friction between the object and the floor is μ The object strikes at the far end of the third
k

stair. Each stair has a rise of h and a run of d . Neglect air resistance and use g for the gravitational constant. (a) What is the
distance s that the object slides along the floor?

Figure 8.35 Object falling down a staircase


Solution: There are two distinct stages to the object’s motion, the initial horizontal motion and then free fall. The given final
position of the object, at the far end of the third stair, will determine the horizontal component of the velocity at the instant the
object left the top of the stairs. This in turn can be used to determine the time the object decelerated along the floor, and hence the
distance traveled on the floor. The given quantities are m , v0 , µk , g , h and d.
For the horizontal motion, choose coordinates with the origin at the initial position of the block. Choose the positive ^i direction to

be horizontal, directed to the left in Figure 8.35, ^j -direction to be vertical (up). The forces on the object are gravity m g ^
= −mg j
→ →
the normal force ^
N =Nj and the kinetic frictional force f k
^
= −fk i . The components of the vectors in Newton’s Second Law,
→ →
F =m a , are

−fk = max

N − mg = may

The object does not move in the y -direction; a = 0 and thus from the second expression in (8.6.19), N = mg The magnitude of
y

the frictional force is then f = μ N = μ mg , and the first expression in (8.6.19) gives the x -component of acceleration as
k k k

a = −μ g Becasue the acceleration is constant the x -component of the velocity is given by


x k

vx (t) = v0 + ax t

where v is the x -component of the velocity of the object when it just started sliding. The displacement is given by
0

1
2
x(t) − x0 = v0 t + ax t
2

Denote the time the block just leaves the landing by t , where x (t ) = s and the speed just when it reaches the landing
1 1

v (t ) = v
x 1 . The initial speed is v and x = 0 . Using the initial and final conditions, and the value of the acceleration, Equation
x,1 0 0

(8.6.21) becomes
1
2
s = v0 t1 − μk gt
1
2

Solve Equation (8.6.20) for the time the block reaches the edge of the landing,

8.7.1 https://phys.libretexts.org/@go/page/26795
vx,1 − v0 v0 − vx,1
t1 = =
−μk g μk g

Substituting Equation (8.6.23) into Equation (8.6.22) yields


2
v0 − vx,1 1 v0 − vx,1
s = v0 ( )− μk g( )
μk g 2 μk g

and after some algebra, we can rewrite Equation (8.6.24) as


2 2
v −v
0 x,1
s =
2 μk g

From the top of the stair to the far end of the third stair, the object is in free fall. Choose the positive ^i -direction to be horizontal,
directed to the left in Figure 8.35, and the positive ^j -direction to be vertical (up) and now choose the origin at the top of the stairs,
where the object first goes into free fall. The components of acceleration are a = 0 , a = −g , the initial x -component of velocity
x y

is v , the initial y -component of velocity is v = 0 , the initial x -position is x = 0 and the initial y -position is y = 0 . Reset
x,1 y,0 0 0

t = 0 when the object just leaves the landing. Let t denote the instant the object hits the stair, where y (t ) = −3h and
2 2

x (t ) = 3d . The equations describing the object’s position and speed at time t = t


2 are 2

x (t2 ) = 3d = vx,1 t2

1
2
y (t2 ) = −3h = − gt
2
2

Solve Equation (8.6.26) for t to yield


2

3d
t2 =
vx,1

Substitute Equation (8.6.28) into Equation (8.6.27) and eliminate the variable t 2

2
1 9d
3h = g
2
2 v
x,1

Equation (8.6.29) can now be solved for the square of the horizontal component of the velocity,
2
3gd
2
v =
x,1
2h

Now substitute Equation (8.6.30) into Equation (8.6.25) to determine the distance the object traveled on the landing,
2 2
v − (3gd /2h)
0
s =
2 μk g

Example 8.8 Cart Moving on a Track

Figure 8.36 A falling block will accelerate a cart on a track via the pulling force of the string. The force sensor measures the tension
in the string.
Consider a cart that is free to slide along a horizontal track (Figure 8.36). A force is applied to the cart via a string that is attached
to a force sensor mounted on the cart, wrapped around a pulley and attached to a block on the other end. When the block is released
the cart will begin to accelerate. The force sensor and cart together have a mass m , and the suspended block has mass m . C B

8.7.2 https://phys.libretexts.org/@go/page/26795
Neglect the small mass of the string and pulley, and assume the string is inextensible. The coefficient of kinetic friction between the
cart and the track is μ . Determine (i) the acceleration of the cart, and (ii) the tension in the string.
k

Solution: In general, we would like to draw free-body diagrams on all the individual objects (cart, sensor, pulley, rope, and block)
but we can also choose a system consisting of two (or more) objects knowing that the forces of interaction between any two objects
will cancel in pairs by Newton’s Third Law. In this example, we shall choose the sensor/cart as one free-body, and the block as the
other free-body. The free-body force diagram for the sensor/cart is shown in Figure 8.37.

Figure 8.37 Force diagram on sensor/cart with a vector decomposition of the contact force into horizontal and vertical components
→ →
There are three forces acting on the sensor/cart: the gravitational force m g , the pulling force T
C of the rope on the force
R,C

sensor, and the contact force between the track and the cart. In Figure 8.34, we decompose the contact force into its two
→ →
components, the kinetic frictional force f k
^
= −fk i and the normal force, ^
N =Nj .

The cart is only accelerating in the horizontal direction with a = a ^i so the component of the force in the vertical direction
C Cx

must be zero, a = 0 . We can now apply Newton’s Second Law in the horizontal and vertical directions and find that
Cy

^
i : TR,C − fk = mC aC,x

^
j : N − mC g = 0

From Equation (8.6.33), we conclude that the normal component is

N = mC g

We use Equation (8.6.34) for the normal force to find that the magnitude of the kinetic frictional force is

fk = μk N = μk mc g

Then Equation (8.6.32) becomes

TR,C − μk mC g = mC aC,x


The force diagram for the block is shown in Figure 8.38. The two forces acting on the block are the pulling force T R,B of the

string and the gravitational force mB g . We now apply Newton’s Second Law to the block and find that
^
JB : mB g − TR,B = mB aB,y

Figure 8.38 Forces acting on the block


In Equation (8.6.37), the symbol a B,yrepresents the component of the acceleration with sign determined by our choice of
downward direction for the unit vector j . Note that we made a different choice of direction for the unit vector in the vertical
^
B

direction in the freebody diagram for the block shown in Figure 8.37. Each free-body diagram has an independent set of unit
vectors that define a sign convention for vector decomposition of the forces acting on the free-body and the acceleration of the free-

8.7.3 https://phys.libretexts.org/@go/page/26795
body. In our example, with the unit vector pointing downwards in Figure 8.38, if we solve for the component of the acceleration
and it is positive, then we know that the direction of the acceleration is downwards.
There is a second subtle way that signs are introduced with respect to the forces acting on a free-body. In our example, the force
between the string and the block acting on the block points upwards, so in the vector decomposition of the forces acting on the

block that appears on the left-hand side of Equation (8.6.37), this force has a minus sign and the quantity ^
T R,B = −TR,B j B where
TR,B is assumed positive.
Our assumption that the mass of the rope and the mass of the pulley are negligible enables us to assert that the tension in the rope is
uniform and equal in magnitude to the forces at each end of the rope,

TR,B = TR,C ≡ T

We also assumed that the string is inextensible (does not stretch). This implies that the rope, block, and sensor/cart all have the
same magnitude of acceleration,

aC,x = aB,y ≡ a

Using Equations (8.6.38) and (8.6.39), we can now rewrite the equation of motion for the sensor/cart, Equation (8.6.36), as

T − μk mC g = mC a

and the equation of motion (8.6.37) for the block as

mB g − T = m B a

We have only two unknowns T and a , so we can now solve the two equations (8.6.40) and (8.6.41) simultaneously for the
acceleration of the sensor/cart and the tension in the rope. We first solve Equation (8.6.40) for the tension

T = μk mC g + mC a

and then substitute Equation (8.6.42) into Equation (8.6.41) and find that

mB g − (μk mC g + mC a) = mB a

We can now solve Equation (8.6.43) for the acceleration,


mB g − μk mC g
a =
mC + mB

Substitution of Equation (8.6.44) into Equation (8.6.42) gives the tension in the string,

T = μk mC g + mC a

mB g − μk mC g
= μk mC g + mC
mC + m B
mC mB
= (μk + 1) g
mC + m B

In this example, we applied Newton’s Second Law to two objects, one a composite object consisting of the sensor and the cart, and
the other the block. We analyzed the forces acting on each object and also any constraints imposed on the acceleration of each
object. We used the force laws for kinetic friction and gravitation on each free-body system. The three equations of motion enable
us to determine the forces that depend on the parameters in the example: the tension in the rope, the acceleration of the objects, and
normal force between the cart and the table.

Example 8.9 Pulleys and Ropes Constraint Conditions


Consider the arrangement of pulleys and blocks shown in Figure 8.39. The pulleys are assumed massless and frictionless and the
connecting strings are massless and inextensible. Denote the respective masses of the blocks as m , m , and m . The upper pulley
1 2 3

in the figure is free to rotate but its center of mass does not move. Both pulleys have the same radius R . (a) How are the
accelerations of the objects related? (b) Draw force diagrams on each moving object. (c) Solve for the accelerations of the objects
and the tensions in the ropes.

8.7.4 https://phys.libretexts.org/@go/page/26795
Figure 8.39 Constrained pulley system
Solution: (a) Choose an origin at the center of the upper pulley. Introduce coordinate functions for the three moving blocks, y , y , 1 2

and y . Introduce a coordinate function


3

yP (8.7.1)

for the moving pulley (the pulley on the lower right in Figure 8.40). Choose downward for positive direction; the coordinate system
is shown in the figure below then.

Figure 8.40 Coordinated system for pulley system


The length of string A is given by

lA = y1 + yP + πR

where πR is the arc length of the rope that is in contact with the pulley. This length is constant, and so the second derivative with
respect to time is zero,
2 2 2
d lA d y1 d yP
0 = = + = ay,1 + ay,P
2 2 2
dt dt dt

Thus block 1 and the moving pulley’s components of acceleration are equal in magnitude but opposite in sign,

ay,P = −ay,1

The length of string B is given by

lB = (y3 − yp ) + (y2 − yp ) + πR = y3 + y2 − 2 yP + πR

where πR is the arc length of the rope that is in contact with the pulley. This length is also constant so the second derivative with
respect to time is zero,
2 2 2 2
d lB d y2 d y3 d yp
0 = = + −2 = ay,2 + ay,3 − 2 ay,P
2 2 2 2
dt dt dt dt

We can substitute Equation (8.6.48) for the pulley acceleration into Equation (8.6.50) yielding the constraint relation between the
components of the acceleration of the three blocks,
0 = ay,2 + ay,3 + 2 ay,1

8.7.5 https://phys.libretexts.org/@go/page/26795
→ →
b) Free-body Force diagrams: the forces acting on block 1 are: the gravitational force m g and the pulling force T
1 of string A A,1

acting on the block 1. Denote the magnitude of this force by T Because the string is assumed to be massless and the pulley is
A

assumed to be massless and frictionless, the tension T in the string is uniform and equal in magnitude to the pulling force of the
A

string on the block. The free-body diagram on block 1 is shown in Figure 8.41(a).

Figure 8.41 Free-body force diagram on (a) block 1; (b) block 2; (c) block 3; (d) pulley
Newton’s Second Law applied to block 1 is then
^
j : m1 g − TA = m1 ay,1

→ →
The forces on the block 2 are the gravitational force m g and string B holding the block, T , with magnitude T . The free-
2 B,2 B

body diagram for the forces acting on block 2 is shown in Figure 8.41(b). Newton’s second Law applied to block 2 is
^
j : m2 g − TB = m2 ay,2

→ →
The forces on the block 3 are the gravitational force m g and string holding the block, T , with magnitude equal to T
3 B,3 B

because pulley P has been assumed to be both frictionless and massless. The free-body diagram for the forces acting on block 3 is
shown in Figure 8.41(c). Newton’s second Law applied to block 3 is
^
j : m3 g − TB = m3 ay,3

→ →
The forces on the moving pulley P are the gravitational force m P g = 0 (the pulley is assumed massless); string B pulls down on
→ →
the pulley on each side with a force, T which has magnitude T . String A holds the pulley up with a force T
B,P B with the A,P

magnitude T equal to the tension in string A . The free-body diagram for the forces acting on the moving pulley is shown in
A

Figure 8.41(d). Newton’s second Law applied to the pulley is


^
j : 2 TB − TA = mP ay,P = 0

Because the pulley is assumed to be massless, we can use this last equation to determine the condition that the tension in the two
strings must satisfy,

2 TB = TA

We are now in position to determine the accelerations of the blocks and the tension in the two strings. We record the relevant
equations as a summary.

0 = ay,2 + ay,3 + 2 ay,1

m1 g − TA = m1 ay,1

m2 g − TB = m2 ay,2

m3 g − TB = m3 ay,3

2 TB = TA

8.7.6 https://phys.libretexts.org/@go/page/26795
There are five equations with five unknowns, so we can solve this system. We shall first use Equation (8.6.61) to eliminate the
tension T in Equation (8.6.58), yielding
A

m1 g − 2 TB = m1 ay,1

We now solve Equations (8.6.59), (8.6.60) and (8.6.62) for the accelerations,
TB
ay,2 = g −
m2

TB
ay,3 = g −
m3

2TB
ay,1 = g −
m1

We now substitute these results for the accelerations into the constraint equation, Equation (8.6.57),
TB TB 4TB 1 1 4
0 = g− +g− + 2g − = 4g − TB ( + + )
m2 m3 m1 m2 m3 m1

We can now solve this last equation for the tension in string B ,
4g 4gm1 m2 m3
TB = =
1 1 4 m1 m 3 + m1 m2 + 4 m 2 m3
( + + )
m2 m3 m1

From Equation (8.6.61), the tension in string A is


8gm1 m2 m3
TA = 2 TB =
m1 m 3 + m 1 m2 + 4 m 2 m3

We find the acceleration of block 1 from Equation (8.6.65), using Equation (8.6.67) for the tension in string B,
2TB 8gm2 m3 m1 m3 + m 1 m2 − 4 m 2 m 3
ay,1 = g − = g− =g
m1 m1 m 3 + m1 m 2 + 4 m 2 m3 m1 m3 + m 1 m2 + 4 m 2 m 3

We find the acceleration of block 2 from Equation (8.6.63), using Equation (8.6.67) for the tension in string B,
TB 4gm1 m3 −3 m1 m3 + m1 m2 + 4 m2 m3
ay,2 = g − = g− =g
m2 m1 m 3 + m1 m 2 + 4 m 2 m3 m1 m3 + m 1 m2 + 4 m 2 m 3

Similarly, we find the acceleration of block 3 from Equation (8.6.64), using Equation (8.6.67) for the tension in string B,
TB 4gm1 m2 m1 m 3 − 3 m1 m2 + 4 m 2 m3
ay,3 = g − = g− =g
m3 m 1 m3 + m1 m 2 + 4 m2 m 3 m1 m 3 + m 1 m2 + 4 m 2 m3

As a check on our algebra we note that


2 a1,y + a2,y + a3,y =

m1 m3 +m1 m2 −4 m2 m3 −3 m1 m3 +m1 m2 +4 m2 m3 m1 m3 −3 m1 m2 +4 m2 m3
2g +g +g
m1 m3 +m1 m2 +4 m2 m3 m1 m3 +m1 m2 +4 m2 m3 m1 m3 +m1 m2 +4 m2 m3

=0

Example 8.10 Accelerating Wedge

8.7.7 https://phys.libretexts.org/@go/page/26795
Figure 8.42 Block on accelerating wedge

A 45o wedge is pushed along a table with constant acceleration A according to an observer at rest with respect to the table. A
block of mass m slides without friction down the wedge (Figure 8.42). Find its acceleration with respect to an observer at rest with
respect to the table. Write down a plan for finding the magnitude of the acceleration of the block. Make sure you clearly state which
concepts you plan to use to calculate any relevant physical quantities. Also clearly state any assumptions you make. Be sure you
include any free-body force diagrams or sketches that you plan to use.

Solution: Choose a coordinate system for the block and wedge as shown in Figure 8.43. Then ^
A = Ax,w i where Ax,w is the x-
component of the acceleration of the wedge.

Figure 8.43 Coordinate system for block on accelerating wedge


We shall apply Newton’s Second Law to the block sliding down the wedge. Because the wedge is accelerating, there is a constraint
relation between the x - and y - components of the acceleration of the block. In order to find that constraint we choose a coordinate
system for the wedge and block sliding down the wedge shown in the figure below. We shall find the constraint relationship
between the components of the accelerations of the block and wedge by a geometric argument. From the figure above, we see that
yb
tan ϕ =
l − (xb − xw )

Therefore

yb = (l − (xb − xw )) tan ϕ

If we differentiate Equation (8.6.73) twice with respect to time noting that


2
d l
=0
2
dt

8.7.8 https://phys.libretexts.org/@go/page/26795
we have that
2 2 2
d yb d xb d xw
= −( − ) tan ϕ
2 2 2
dt dt dt

Therefore

ab,y = − (ab,x − Ax,w ) tan ϕ

where
2
d xw
Ax,w =
2
dt

We now draw a free-body force diagram for the block (Figure 8.44). Newton’s Second Law in the ^i - direction becomes

N sin ϕ = mabx

and the ^j -direction becomes

N cos ϕ − mg = mab,y

Figure 8.44 Free-body force diagram on block


We can solve for the normal force from Equation (8.6.78):
mab,x
N =
sin ϕ

We now substitute Equation (8.6.76) and Equation (8.6.80) into Equation (8.6.79) yielding
mab,x
cos ϕ − mg = m (− (ab,x − Aw,x ) tan ϕ)
sin ϕ

We now clean this up yielding

m ab,x (cotan ϕ + tan ϕ) = m (g + Aw,x tan ϕ)

Thus the x-component of the acceleration is then


g + Aw,x tan ϕ
ab,x =
cotan ϕ + tan ϕ

From Equation (8.6.76), the y -component of the acceleration is then


g + Aw,x tan ϕ
ab,y = − (ab,x − Aw,x ) tan ϕ = − ( − Aw,x ) tan ϕ
cotan ϕ + tan ϕ

This simplifies to
Aw,x − g tan ϕ
ab,y =
cotan ϕ + tan ϕ

8.7.9 https://phys.libretexts.org/@go/page/26795
When ϕ = 45 , cotan 45
∘ ∘
= tan 45

=1 , and so Equation (8.6.83) becomes
g + Aw,x
ab,x =
2

and Equation (8.6.85) becomes


A−g
ab,y =
2

The magnitude of the acceleration is then


−−−−−−−−−−−−−−−−−−−−−−−
2 2
−−−−−−−− g + Awx Awx − g
2 2
a = √a +a = √( ) +( )
b,x b,y
2 2

−−−−−−−−−−
2 2
g + Aw
a = √( )
2

Example 8.11: Capstan


A device called a capstan is used aboard ships in order to control a rope that is under great tension. The rope is wrapped around a
fixed drum of radius R , usually for several turns (Figure 8.45 shows about three fourths turn as seen from overhead). The load on
the rope pulls it with a force T and the sailor holds the other end of the rope with a much smaller force T . The coefficient of
A B

static friction between the rope and the drum is μ . The sailor is holding the rope so that it is just about to slip. Show that
s

TB =T e
A
−μs θBA
where θ is the angle subtended by the rope on the drum.
BA

Figure 8.47. The right edge of the slice is at angle θ and the left edge of the slice is at θ + Δθ The angle edge end of the slice
makes with the horizontal is Δθ/2. There are four forces acting on this section of the rope. The forces are the normal force
between the capstan and the rope pointing outward, a static frictional force and the tensions at either end of the slice. The rope is
held at the just slipping point, so if the load exerts a greater force the rope will slip to the right. Therefore the direction of the static
frictional force between the capstan and the rope, acting on the rope, points to the left. The tension on the right side of the slice is
denoted by T (θ) ≡ T , while the tension on the left side of the slice is denoted by T (θ + Δθ) ≡ T + ΔT . Does the tension in this
slice from the right side to the left, increase, remain the same, or decrease? The tension decreases because the load on the left side
is less than the load on the right side. Note that ΔT < 0 .

8.7.10 https://phys.libretexts.org/@go/page/26795
Figure 8.47 Free-body force diagram on small slice of rope
The vector decomposition of the forces is given by
^
i : T cos(Δθ/2) − fs − (T + ΔT ) cos(Δθ/2)

^
j : −T sin(Δθ/2) + N − (T + ΔT ) sin(Δθ/2)

For small angles Δθ, cos(Δθ/2) ≅1 , and sin(Δθ/2) ≅Δθ/2 Using the small angle approximations, the vector decomposition of
the forces in the x -direction (the ^i direction) becomes

T cos(Δθ/2) − fs − (T + ΔT ) cos(Δθ/2) ≃ T − fs − (T + ΔT )

= −fs − ΔT

By the static equilibrium condition the sum of the x -components of the forces is zero,

−fs − ΔT = 0

The vector decomposition of the forces in the y -direction (the +j^-direction) is approximately
−T sin(Δθ/2) + N − (T + ΔT ) sin(Δθ/2) ≃ −T Δθ/2 + N − (T + ΔT )Δθ/2

= −T Δθ + N − ΔT Δθ/2

In the last equation above we can ignore the terms proportional to ΔT Δθ because these are the product of two small quantities and
hence are much smaller than the terms proportional to either ΔT or Δθ. The vector decomposition in the y -direction becomes

−T Δθ + N

Static equilibrium implies that this sum of the y -components of the forces is zero,

−T Δθ + N = 0

We can solve this equation for the magnitude of the normal force

N = T Δθ

The just slipping condition is that the magnitude of the static friction attains its maximum value

fs = (fs ) = μs N
max

We can now combine the Equations (8.6.92) and (8.6.97) to yield

ΔT = −μs N

Now substitute the magnitude of the normal force, Equation (8.6.96), into Equation (8.6.98), yielding

−μs T Δθ − ΔT = 0

Finally, solve this equation for the ratio of the change in tension to the change in angle,

8.7.11 https://phys.libretexts.org/@go/page/26795
ΔT
= −μs T
Δθ

The derivative of tension with respect to the angle θ is defined to be the limit
dT ΔT
≡ lim
dθ Δθ→0 Δθ

and Equation (8.6.100) becomes


dT
= −μs T

This is an example of a first order linear differential equation that shows that the rate of change of tension with respect to the angle
θ is proportional to the negative of the tension at that angle θ . This equation can be solved by integration using the technique of
separation of variables. We first rewrite Equation (8.6.102) as
dT
= −μs dθ
T

Integrate both sides, noting that when θ = 0 , the tension is equal to force of the load TA and when angle θ = θA,B the tension is
equal to the force T the sailor applies to the rope,
B

T =TB θ=θBA
dT
∫ = −∫ μs dθ
T =TA
T θ=0

The result of the integration is


TB
ln( ) = −μs θBA
TA

Note that the exponential of the natural logarithm


TB TB
exp(ln( )) =
TA TA

so exponentiating both sides of Equation (8.6.105) yields


TB
−μs θBA
=e
TA

the tension decreases exponentially,


−μs θBA
TB = TA e

Because the tension decreases exponentially, the sailor need only apply a small force to prevent the rope from slipping.

Example 8.12 Free Fall with Air Drag


Consider an object of mass m that is in free fall but experiencing air resistance. The magnitude of the drag force is given by
Equation (8.6.1), where ρ is the density of air, A is the cross-sectional area of the object in a plane perpendicular to the motion, and
CD the drag coefficient. Assume that the object is released from rest and very quickly attains speeds in which Equation (8.6.1)
applies. Determine (i) the terminal velocity, and (ii) the velocity of the object as a function of time.
Solution: Choose positive y -direction downwards with the origin at the initial position of the object as shown in Figure 8.48(a).

8.7.12 https://phys.libretexts.org/@go/page/26795
Figure 8.48 (a) Coordinate system for marble; (b) free body force diagram on marble
There are two forces acting on the object: the gravitational force, and the drag force which is given by Equation (8.6.1). The free
body diagram is shown in the Figure 8.48(b). Newton’s Second Law is then

2
dv
mg − (1/2)CD Aρv =m
dt

Set β = (1/2)C D Aρ . Newton’s Second Law can then be written as


dv
2
mg − β v =m
dt

Initially when the object is just released with v = 0 , the air drag is zero and the acceleration dv / dt is maximum. As the object
increases its velocity, the air drag becomes larger and dv / dt decreases until the object reaches terminal velocity and dv / dt = 0 .
Set dv / dt = 0 in Equation (8.6.15) and solve for the terminal velocity yielding.
−−−−−−
−−−
mg 2mg
v∞ = √ =√
β CD Aρ

Values for the magnitude of the terminal velocity is shown in Table 8.3 for a variety of objects with the same drag coefficient
CD = 0.5

Table 8.3 Terminal Velocities for Different Sized Objects with C D = 0.5

In order to integrate Equation (8.6.15), we shall apply the technique of separation of variables and integration by partial fractions.
First rewrite Equation (8.6.15) as
−β dv dv 1 1
dt = = = (− + ) dv
2 2
m 2
mg
(v − v∞ ) 2 v∞ (v + v∞ ) 2 v∞ (v − v∞ )
(v − )
β

An integral expression of Equation (8.6.112) is then


′ ′
v =v(t) ′ v=v(t) ′ t =t
dv dv β ′
−∫ +∫ =− ∫ dt

v =0
2 v∞ (v′ + v∞ ) v=0
2 v∞ (v′ − v∞ ) m t=0

Integration yields

8.7.13 https://phys.libretexts.org/@go/page/26795

′ ′ t =t
′ ′
v =v(t) dv v =v(t) dv β t ′
−∫ ′ ′
+∫ ′ ′
=− ∫ ′ dt
v =0 2 v∞ ( v +v∞ ) v =0 2 v∞ ( v −v∞ ) m t =0

v(t)+v∞ v∞ −v(t) β
1
(− ln( ) + ln( )) = − t
2v∞ v∞ v∞ m

After some algebraic manipulations, Equation (8.6.114) can be rewritten as


v∞ − v(t) 2 v∞ β
ln( ) =− t
v(t) + v∞ m

Exponentiate Equation (8.6.115) yields


v∞ − v(t) 2vω β
− t
( ) =e m

v(t) + v∞

After some algebraic rearrangement the y -component of the velocity as a function of time is given by
2vw β
t

1−e m v∞ β
v(t) = v∞ ( ) = v∞ tan h ( t)
2v∞ β
t m

1+e m


−− −− −−−−−−−−
v∞ β β mg βg (1/2) CD Aρg
 where  = √ =√ =√
m m β m m

This page titled 8.7: Worked Examples is shared under a not declared license and was authored, remixed, and/or curated by Peter Dourmashkin
(MIT OpenCourseWare) via source content that was edited to the style and standards of the LibreTexts platform; a detailed edit history is
available upon request.

8.7.14 https://phys.libretexts.org/@go/page/26795
CHAPTER OVERVIEW
9: Circular Motion Dynamics
9.1: Introduction Newton’s Second Law and Circular Motion
9.2: Universal Law of Gravitation and the Circular Orbit of the Moon
9.3: Worked Examples Circular Motion
9.4: Appendix 9A The Gravitational Field of a Spherical Shell of Matter .

Thumbnail: Our visible Universe contains billions of galaxies, whose very existence is due to the force of gravity. Gravity is
ultimately responsible for the energy output of all stars—initiating thermonuclear reactions in stars, allowing the Sun to heat
Earth, and making galaxies visible from unfathomable distances. Most of the dots you see in this image are not stars, but galaxies.
(credit: modification of work by NASA).

This page titled 9: Circular Motion Dynamics is shared under a CC BY-NC-SA 4.0 license and was authored, remixed, and/or curated by Peter
Dourmashkin (MIT OpenCourseWare) via source content that was edited to the style and standards of the LibreTexts platform; a detailed edit
history is available upon request.

1
9.1: Introduction Newton’s Second Law and Circular Motion
I shall now recall to mind that the motion of the heavenly bodies is circular, since the motion appropriate to a sphere is rotation in
a circle.
Nicholas Copernicus

We have already shown that when an object moves in a circular orbit of radius r with angular velocity ω it is most convenient to
choose polar coordinates to describe the position, velocity and acceleration vectors. In particular, the acceleration vector is given by
2 2
→ dθ d θ
^
a (t) = −r( ) ^
r(t) + r θ (t)
2
dt dt

→ →
Then Newton’s Second Law, F =m a can be decomposed into radial (^
r−) and tangential (θ −) components
^

2

Fr = −mr( ) ( circular motion )
dt

2
d θ
Fθ = mr ( circular motion )
2
dt

For the special case of uniform circular motion, d 2 2


θ/dt =0 , and so the sum of the tangential components of the force acting on
the object must therefore be zero,

Fθ = 0

This page titled 9.1: Introduction Newton’s Second Law and Circular Motion is shared under a CC BY-NC-SA 4.0 license and was authored,
remixed, and/or curated by Peter Dourmashkin (MIT OpenCourseWare) via source content that was edited to the style and standards of the
LibreTexts platform; a detailed edit history is available upon request.

9.1.1 https://phys.libretexts.org/@go/page/24473
9.2: Universal Law of Gravitation and the Circular Orbit of the Moon
An important example of (approximate) circular motion is the orbit of the Moon around the Earth. We can approximately calculate
the time T the Moon takes to complete one circle around the earth (a calculation of great importance to early lunar calendar
systems, which became the basis for our current model.) Denote the distance from the moon to the center of the earth by R . e,m

Because the Moon moves nearly in a circular orbit with angular speed ω = 2π/T it is accelerating towards the Earth. The radial
component of the acceleration (centripetal acceleration) is
2
4π Re,m
ar = −
2
T

According to Newton’s Second Law, there must be a centripetal force acting on the Moon directed towards the center of the Earth
that accounts for this inward acceleration.

Universal Law of Gravitation


Newton’s Universal Law of Gravitation describes the gravitational force between two bodies 1 and 2 with masses m and m 1 2

respectively. This force is a radial force (always pointing along the radial line connecting the masses) and the magnitude is
proportional to the inverse square of the distance that separates the bodies. Then the force on object 2 due to the gravitational
interaction between the bodies is given by,
→ m 1 m2
F 1,2 = −G ^
r1,2
2
r
1,2

where r is the distance between the two bodies and ^


1,2 r is the unit vector located at the position of object 2 and pointing from
1,2

object 1 towards object 2. The Universal Gravitation Constant is G = 6.67 × 10 N ⋅ m ⋅ kg . Figure 9.1 shows the direction
−11 2 −2

of the forces on bodies 1 and 2 along with the unit vector ^


r .
1,2

Figure 9.1 Gravitational force of interaction between two bodies


Newton realized that there were still some subtleties involved. First, why should the mass of the Earth act as if it were all placed at
the center? Newton showed that for a perfect sphere with uniform mass distribution, all the mass may be assumed to be located at
the center. (This calculation is difficult and can be found in Appendix 9A to this chapter.) We assume for the present calculation
that the Earth and the Moon are perfect spheres with uniform mass distribution.
Second, does this gravitational force between the Earth and the Moon form an action-reaction Third Law pair? When Newton first
explained the Moon’s motion in 1666, he had still not formulated the Third Law, which accounted for the long delay in the
publication of the Principia. The link between the concept of force and the concept of an action-reaction pair of forces was the last
piece needed to solve the puzzle of the effect of gravity on planetary orbits. Once Newton realized that the gravitational force
between any two bodies forms an action-reaction pair, and satisfies both the Universal Law of Gravitation and his newly
formulated Third Law, he was able to solve the oldest and most important physics problem of his time, the motion of the planets.
The test for the Universal Law of Gravitation was performed through experimental observation of the motion of planets, which
turned out to be resoundingly successful. For almost 200 years, Newton’s Universal Law was in excellent agreement with
observation. A sign of more complicated physics ahead, the first discrepancy only occurred when a slight deviation of the motion

9.2.1 https://phys.libretexts.org/@go/page/24474
of Mercury was experimentally confirmed in 1882. The prediction of this deviation was the first success of Einstein’s Theory of
General Relativity (formulated in 1915).
We can apply this Universal Law of Gravitation to calculate the period of the Moon’s orbit around the Earth. The mass of the Moon
is m = 7.36 × 10 kg and the mass of the Earth is m = 5.98 × 10 kg . The distance from the Earth to the Moon is
1
22
2
24

= 3.82 × 10 m . We show the force diagram in Figure 9.2.


8
R e,m

Figure 9.2 Gravitational force of moon


Newton’s Second Law of motion for the radial direction becomes
2
m1 m 2 4π Re,m
−G = −m1
2 2
Re,m T

We can solve this equation for the period of the orbit,


−−−−−−−
3
4π 2 Re,m
T =√
Gm2

Substitute the given values for the radius of the orbit, the mass of the earth, and the universal gravitational constant. The period of
the orbit is
−−−−−−−−−−−−−−−−−−−−−−−−−−−−−−−−−−
 3
2 8
 4π (3.82 × 10 m)
 6
T = = 2.35 × 10 s
⎷ (6.67 × 10−11 N ⋅ m2 ⋅ kg−2 ) (5.98 × 1024 kg)

This period is given in days by


1 day 
6
T = (2.35 × 10 s) ( ) = 27.2 days 
4
8.64 × 10 s

This period is called the sidereal month because it is the time that it takes for the Moon to return to a given position with respect to
the stars.
The actual time T between full moons, called the synodic month (the average period of the Moon’s revolution with respect to the
1

earth and is 29.53 days, it may range between 29.27 days and 29.83 days), is longer than the sidereal month because the Earth is
traveling around the Sun. So for the next full moon, the Moon must travel a little farther than one full circle around the Earth in
order to be on the other side of the Earth from the Sun (Figure 9.3).

Figure 9.3: Orbital motion between full moons


Therefore the time T between consecutive full moons is approximately T
1 1 ≃ T + ΔT where
ΔT ≃ T /12 = 2.3 (9.2.1)

9.2.2 https://phys.libretexts.org/@go/page/24474
days. So T 1 ≃ 29.5 days.

Kepler’s Third Law and Circular Motion


The first thing that we notice from the above solution is that the period does not depend on the mass of the Moon. We also notice
that the square of the period is proportional to the cube of the distance between the Earth and the Moon,
2 3
4π Re,m
2
T =
Gm2

This is an example of Kepler’s Third Law, of which Newton was aware. This confirmation was convincing evidence to Newton
that his Universal Law of Gravitation was the correct mathematical description of the gravitational force law, even though he still
could not explain what “caused” gravity.

This page titled 9.2: Universal Law of Gravitation and the Circular Orbit of the Moon is shared under a CC BY-NC-SA 4.0 license and was
authored, remixed, and/or curated by Peter Dourmashkin (MIT OpenCourseWare) via source content that was edited to the style and standards of
the LibreTexts platform; a detailed edit history is available upon request.

9.2.3 https://phys.libretexts.org/@go/page/24474
9.3: Worked Examples Circular Motion
Example 9.1 Geosynchronous Orbit
A geostationary satellite goes around the earth once every 23 hours 56 minutes and 4 seconds, (a sidereal day, shorter than the
noon-to-noon solar day of 24 hours) so that its position appears stationary with respect to a ground station. The mass of the earth is
kg . The mean radius of the earth is R = 6.37 × 10 m . The universal constant of gravitation is
24 6
m = 5.98 × 10
e e

G = 6.67 × 10
−11 2
N ⋅ m ⋅ kg . What is the radius of the orbit of a geostationary satellite? Approximately how many earth radii
−2

is this distance?
Solution: The satellite’s motion can be modeled as uniform circular motion. The gravitational force between the earth and the
satellite keeps the satellite moving in a circle (In Figure 9.4, the orbit is close to a scale drawing of the orbit). The acceleration of
the satellite is directed towards the center of the circle, that is, along the radially inward direction.

Figure 9.4 Geostationary satellite orbit (close to a scale drawing of orbit).


Choose the origin at the center of the earth, and the unit vector ^
r along the radial direction. This choice of coordinates makes sense

in this problem since the direction of acceleration is along the radial direction.
→ →
Let r be the position vector of the satellite. The magnitude of r (we denote it as r ) is the distance of the satellite from the center
s

of the earth, and hence the radius of its circular orbit. Let ω be the angular velocity of the satellite, and the period is T = 2π/ω The
acceleration is directed inward, with magnitude r ω ; in vector form,
s
2

→ 2
a = −rs ω ^r

Apply Newton’s Second Law to the satellite for the radial component. The only force in this direction is the gravitational force due
to the Earth,

2
F grav = −ms ω rs ^
r

The inward radial force on the satellite is the gravitational attraction of the earth,
m s me
2
−G ^
r = −ms ω rs ^
r
2
rs

Equating the ^
r components,

ms m e 2
G = ms ω rs
2
rs

Solving for the radius of orbit of the satellite r ,s

1/3
Gme
rs = ( )
ω2

The period T of the satellite’s orbit in seconds is 86164 s and so the angular speed is

9.3.1 https://phys.libretexts.org/@go/page/24475
2π 2π −5 −1
ω = = = 7.2921 × 10 s
T 86164s

Using the values of ω, G and m in Equation (9.3.5), we determine r ,


e s

7
rs = 4.22 × 10 m = 6.62 Re

Example 9.2 Double Star System


Consider a double star system under the influence of gravitational force between the stars. Star 1 has mass m and star 2 has mass
1

m . Assume that each star undergoes uniform circular motion such that the stars are always a fixed distance s apart (rotating
2

counterclockwise in Figure 9.5). What is the period of the orbit?

Figure 9.5 Two stars undergoing circular orbits about each other
Solution: Because the distance between the two stars doesn’t change as they orbit about each other, there is a central point where
the lines connecting the two objects intersect as the objects move, as can be seen in the figure above. (We will see later on in the
course that central point is the center of mass of the system.) Choose radial coordinates for each star with origin at that central

point. Let ^
r be a unit vector at Star 1 pointing radially away from the center of mass. The position of object 1 is then r
1 =r ^ r ,
1 1 1

where r is the distance from the central point. Let ^


1 r be a unit vector at Star 2 pointing radially away from the center of mass. The
2


position of object 2 is then r 2 = r2 ^
r2 , where r is the distance from the central point. Because the distance between the two stars
2

is fixed we have that

s = r1 + r2

The coordinate system is shown in Figure 9.6

9.3.2 https://phys.libretexts.org/@go/page/24475
Figure 9.6 Coordinate system for double star orbits
The gravitational force on object 1 is then
→ Gm1 m2
F 2,1 = − ^
r1
2
s

The gravitational force on object 2 is then


→ Gm1 m2
F 1,2 = − ^
r2
2
s

The force diagrams on the two stars are shown in Figure 9.7.

Figure 9.7 Force diagrams on objects 1 and 2


→ →
Let ω denote the magnitude of the angular velocity of each star about the central point. Then Newton’s Second Law, F 1 = m1 a 1

for Star 1 in the radial direction ^


1r is

m1 m2
2
−G = −m1 r1 ω
2
s

9.3.3 https://phys.libretexts.org/@go/page/24475
We can solve this for r ,1

m2
r1 = G
ω2 s2

→ →
Newton’s Second Law, F 2 = m2 a 2 for Star 2 in the radial direction ^
r is 2

m1 m2
2
−G = −m2 r2 ω
2
s

We can solve this for r 2

m1
r2 = G
2 2
ω s

Because s , the distance between the stars, is constant


m2 m1 (m2 + m1 )
s = r1 + r2 = G +G =G
2 2 2 2 2 2
ω s ω s ω s

Thus the magnitude of the angular velocity is


1/2
(m2 + m1 )
ω = (G )
3
s

and the period is then


1/2
2 3
2π 4π s
T = =( )
ω G (m2 + m1 )

Note that both masses appear in the above expression for the period unlike the expression for Kepler’s Law for circular orbits.
Equation (9.2.7). The reason is that in the argument leading up to Equation (9.2.7), we assumed that m << m , this was 1 2

equivalent to assuming that the central point was located at the center of the Earth. If we used Equation (9.3.8) instead we would
find that the orbital period for the circular motion of the Earth and moon about each other is
−−−−−−−−−−−−−−−−−−−−−−−−−−−−−−−−−−−−−−−−−−−−−−−
 3
2 8
 4π (3.82 × 10 m)
 6
T = = 2.33 × 10 s
⎷ −11 2 −2 24 22
(6.67 × 10 N⋅m ⋅ kg ) (5.98 × 10 kg + 7.36 × 10 kg)

which is 1.43 × 10 4
s = 0.17 d shorter than our previous calculation.
Example 9.3 Rotating Objects
Two objects 1 and 2 of mass m and m are whirling around a shaft with a constant angular velocity ω . The first object is a
1 2

distance d from the central axis, and the second object is a distance 2d from the axis (Figure 9.8). You may ignore the mass of the
strings and neglect the effect of gravity. (a) What is the tension in the string between the inner object and the outer object? (b) What
is the tension in the string between the shaft and the inner object?

Figure 9.8 Objects attached to a rotating shaft


Solution: We begin by drawing separate force diagrams, Figure 9.9a for object 1 and Figure 9.9b for object 2.

9.3.4 https://phys.libretexts.org/@go/page/24475
Figure 9.9 (a) and 9.9 (b) Free-body force diagrams for objects 1 and 2
→ →
Newton’s Second Law, F 1 = m1 a 1 , for the inner object in the radial direction is
2
^
r : T2 − T1 = −m1 dω

→ →
Newton’s Second Law, F 2 = m2 a 2 , for the outer object in the radial direction is
2
^
r : −T2 = −m2 2dω

The tension in the string between the inner object and the outer object is therefore
2
T2 = m2 2dω

Using this result for T in the force equation for the inner object yields
2

2 2
m2 2dω − T1 = −m1 dω

which can be solved for the tension in the string between the shaft and the inner object
2
T1 = dω (m1 + 2 m2 )

Example 9.4 Tension in a Rope


A uniform rope of mass mand length L is attached to shaft that is rotating at constant angular velocity ω . Find the tension in the
rope as a function of distance from the shaft. You may ignore the effect of gravitation.
Solution: Divide the rope into small pieces of length Δr , each of mass Δm = (m/L)Δr Consider the piece located a distance r
from the shaft (Figure 9.10).

Figure 9.10 Small slice of rotating rope


The radial component of the force on that piece is the difference between the tensions evaluated at the sides of the piece,
F = T (r + Δr) − T (r) , (Figure 9.11).
r

9.3.5 https://phys.libretexts.org/@go/page/24475
Figure 9.11 Free-body force diagram on small slice of rope
The piece is accelerating inward with a radial component a r = −rω
2
. Thus Newton’s Second Law becomes
2
Fr = −Δm ω r

2
T (r + Δr) − T (r) = −(m/L)Δrrω

Denote the difference in the tension by ΔT = T (r + Δr) − T (r) . After dividing through by Δr , Equation (9.3.9) becomes
ΔT
2
= −(m/L)rω
Δr

In the limit as Δr → 0 , Equation (9.3.10) becomes a differential equation,


dT
2
= −(m/L)ω r
dr

From this, we see immediately that the tension decreases with increasing radius. We shall solve this equation by integration

r =r r
dT ′ 2 ′ ′
T (r) − T (L) = ∫ dr = − (m ω /L) ∫ r dr


r =L
dr L

2 2 2
= − (m ω /2L) (r −L )

2 2 2
= (m ω /2L) (L −r )

We use the fact that the tension, in the ideal case, will vanish at the end of the rope, r = L . Thus,
2 2 2
T (r) = (m ω /2L) (L −r )

This last expression shows the expected functional form, in that the tension is largest closest to the shaft, and vanishes at the end of
the rope.
Example 9.5 Object Sliding in a Circular Orbit on the Inside of a Cone
Consider an object of mass m that slides without friction on the inside of a cone moving in a circular orbit with constant speed v . 0

The cone makes an angle θ with respect to a vertical axis. The axis of the cone is vertical and gravity is directed downwards. The
apex half-angle of the cone is θ as shown in Figure 9.12. Find the radius of the circular path and the time it takes to complete one
circular orbit in terms of the given quantities and g .

Figure 9.12 Object in a circular orbit on inside of a cone


Solution: Choose cylindrical coordinates as shown in the above figure. Choose unit vectors ^
r pointing in the radial outward
direction and k
^
pointing upwards. The force diagram on the object is shown in Figure 9.13.

9.3.6 https://phys.libretexts.org/@go/page/24475
Figure 9.13 Free-body force diagram on object
The two forces acting on the object are the normal force of the wall on the object and the gravitational force. Then Newton’s
Second Law in the ^
r -direction becomes

2
−mv
−N cos θ =
r

and in the k
^
-direction becomes

N sin θ − mg = 0

These equations can be re-expressed as


2
v
N cos θ = m
r

N sin θ = mg

We can divide these two equations,


N sin θ mg
=
N cos θ mv2 /r

yielding
rg
tan θ = 2
v

This can be solved for the radius,


2
v
r = tan θ
g

The centripetal force in this problem is the vector component of the contact force that is pointing radially inwards,

Fcent  = N cos θ = mg cot θ

where N sin θ = mg has been used to eliminate N in terms of m , g and θ . The radius is independent of the mass because the
component of the normal force in the vertical direction must balance the gravitational force, and so the normal force is proportional
to the mass.
Example 9.6 Coin on a Rotating Turntable
A coin of mass m (which you may treat as a point object) lies on a turntable, exactly at the rim, a distance R from the center. The
turntable turns at constant angular speed ω and the coin rides without slipping. Suppose the coefficient of static friction between the
turntable and the coin is given by µ . Let g be the gravitational constant. What is the maximum angular speed ω maxsuch that the
coin does not slip?

9.3.7 https://phys.libretexts.org/@go/page/24475
Figure 9.14 Coin on Rotating Turntable
Solution: The coin undergoes circular motion at constant speed so it is accelerating inward. The force inward is static friction and at
the just slipping point it has reached its maximum value. We can use Newton’s Second Law to find the maximum angular speed
ωmax . We choose a polar coordinate system and the free-body force diagram is shown in the figure below.

Figure 9.15 Free-body force diagram on coin


The contact force is given by
→ → →
^
C = N+ f s = N k − fs ^
r

The gravitational force is given by



^
F grav = −mgk

Newton’s Second Law in the radial direction is given by


2
−fs = −mRω

Newton’s Second Law, F z = m az in the z-direction, noting that the disc is static hence a z =0 , is given by

N − mg = 0

Thus the normal force is

N = mg

As ω increases, the static friction increases in magnitude until at ω = ωmax and static friction reaches its maximum value (noting
Equation (9.3.18)).

(fs ) = μN = μmg
max

At this value the disc slips. Thus substituting this value for the maximum static friction into Equation (9.3.16) yields
2
μmg = mRωmax

We can now solve Equation (9.3.20) for maximum angular speed ω max such that the coin does not slip
−−

μg
ωmax = √
R

This page titled 9.3: Worked Examples Circular Motion is shared under a CC BY-NC-SA 4.0 license and was authored, remixed, and/or curated
by Peter Dourmashkin (MIT OpenCourseWare) via source content that was edited to the style and standards of the LibreTexts platform; a detailed

9.3.8 https://phys.libretexts.org/@go/page/24475
edit history is available upon request.

9.3.9 https://phys.libretexts.org/@go/page/24475
9.4: Appendix 9A The Gravitational Field of a Spherical Shell of Matter .
When analyzing gravitational interactions between uniform spherical bodies we assumed we could treat each sphere as a point-like
mass located at the center of the sphere and then use the Universal Law of Gravitation to determine the force between the two point
like objects. We shall now justify that assumption. For simplicity we only need to consider the interaction between a spherical
object and a point-like mass. We would like to determine the gravitational force on the point-like object of mass m due to the
1

gravitational interaction with a solid uniform sphere of mass m and radius R . In order to determine the force law we shall first
2

consider the interaction between the point-like object and a uniform spherical shell of mass m and radius R . We will show that:
s

1) The gravitational force acting on a point-like object of mass m located a distance r > R from the center of a uniform spherical
1

shell of mass m and radius R is the same force that would arise if all the mass of the shell were placed at the center of the shell.
s

2) The gravitational force on an object of mass m placed inside a spherical shell of matter is zero.
1

The force law summarizes these results:


ms m1
→ −G ^
r, r >R
2
r
F s,1 (r) = {

0 , r <R

where ^
r is the unit vector located at the position of the object and pointing radially away from the center of the shell.

For a uniform spherical distribution of matter, we can divide the sphere into thin shells. Then the force between the point-like
object and each shell is the same as if all the mass of the shell were placed at the center of the shell. Then we add up all the
contributions of the shells (integration), the spherical distribution can be treated as point-like object located at the center of the
sphere justifying our assumption.
Thus it suffices to analyze the case of the spherical shell. We shall first divide the shell into small area elements and calculate the
gravitational force on the point-like object due to one element of the shell and then add the forces due to all these elements via
integration.
We begin by choosing a coordinate system. Choose our z -axis to be directed from the center of the sphere to the position of the

object, at position ^
r = zk , so that z ≥ 0 (Figure 9A.1 shows the object lying outside the shell with z > R )

Figure 9A.1 Object lying outside shell with z > R

Choose spherical coordinates as shown in Figure 9A.2.

9.4.1 https://phys.libretexts.org/@go/page/24476
Figure 9A.2 Spherical coordinates
For a point on the surface of a sphere of radius r = R , the Cartesian coordinates are related to the spherical coordinates by
x = R sin θ cos ϕ

y = R sin θ sin ϕ

z = R cos θ

where 0 ≤ θ ≤ π and 0 ≤ ϕ ≤ 2π .
Note that the angle θ in Figure 9A.2 and Equations (9.A.1) is not the same as that in plane polar coordinates or cylindrical
coordinates. The angle θ is known as the colatitude, the complement of the latitude. We now choose a small area element shown in
Figure 9A.3.

Figure 9A.3 Infinitesimal area element


The infinitesimal area element on the surface of the shell is given by
2
da = R sin θdθdϕ

Then the mass dm contained in that element is


2
dm = σda = σ R sin θdθdϕ

where σ is the surface mass density given by


2
σ = ms /4π R

9.4.2 https://phys.libretexts.org/@go/page/24476

The gravitational force F dm,m1on the object of mass m that lies outside the shell due to the infinitesimal piece of the shell (with
1

mass dm ) is shown in Figure 9A.4.

Figure 9A.4 Force on a point-like object due to piece of shell


The contribution from the piece with mass dm to the gravitational force on the object of mass m that lies outside the shell has a
1

component pointing in the negative k ^


-direction and a component pointing radially away from the z -axis. By symmetry there is
another mass element with the same differential mass dm = dm on the other side of the shell with same co-latitude θ but with ϕ

replaced by ϕ ± π ; this replacement changes the sign of x and y in Equations (9.A.1) but leaves z unchanged. This other mass
element produces a gravitational force that exactly cancels the radial component of the force pointing away from the z -axis.
Therefore the sum of the forces of these differential mass elements on the object has only a component in the negative k
^
-direction
(Figure 9A.5).

Figure 9A.5 Symmetric cancellation of components of force


Therefore we need only the z -component vector of the force due to the piece of the shell on the point-like object.

9.4.3 https://phys.libretexts.org/@go/page/24476
Figure 9A.6 Geometry for calculating the force due to piece of shell.
From the geometry of the set-up (Figure 9A.6) we see that
→ m1 dm
^ ^
(d F s,1 ) ≡ dF k = −G cos α k
2
z
r
s1

Thus
m1 dm Gms m1 cos α sin θdθdϕ
dFz = −G cos α = −
2 2
r 4π r
s1 s1

The integral of the force over the surface is then


θ=πϕ=2π
θ=πϕ=2π ∞
dm cos α Gms m cos α sin θdθdϕ
1
Fz = −Gm1 ∫ ∫ =− ∫ ∫
2 2
θ=0 ϕ=0 r 4π θ=0 ϕ=0 r
s1 s1

The ϕ -integral is straightforward yielding


θ=π cos α sin θdθ
Gms m
1
Fz = − ∫
2 θ=0

From Figure 9A.6 we can use the law of cosines in two different ways
2 2 2
r =R +z − 2Rz cos θ
s1

2 2 2
R =z +r − 2 rs,1 z cos α
s1

Differentiating the first expression in (9.A.5), with R and z constant yields,

2 rs,1 drs,1 = 2Rz sin θdθ

Hence
rs,1
sin θdθ = drs,1
Rz

and from the second expression in (9.A.5) we have that


1
2 2 2
cos α = [(z −R )+r ]
s1
2zrs,1

9.4.4 https://phys.libretexts.org/@go/page/24476
We now have everything we need in terms of r s,1 .
For the case when z > R, r s,1 varies from z − R to z + R. Substituting Equations (9.A.7) and (9.A.8) into Equation (9.A.3) and
using the limits for the definite integral yields
θ=π cos α sin θ
Gms m
1
Fz = − ∫ dθ
2 θ=0

z+R
Gms m1 1 1 1 rs,1 drs,1
2 2 2
=− ∫ [(z −R )+r ]
s,1 2
2 2z z−R
rs,1 r Rz
s,1

z+R z+R
Gms m1 1 drs,1
2 2
=− [(z −R )∫ +∫ drs,1 ]
2 2Rz 2 z−R
2
r z−R
s,1

No tables should be needed for these; the result is


z+R
2 2
Gms m1 1 (z −R )
Fz = − [− + rs,1 ]
2 2Rz 2 rs,1
z−R

Gms m1 1
=− [−(z − R) + (z + R) + 2R]
2
2 2Rz
Gms m1
=−
2
z

For the case when z < R, r s,1 varies from R − z to R + z . Then the integral is
R+z
2 2
Gms m1 1 (z −R )
Fz = − [− + rs,1 ]
2
2 2Rz rs,1
R−z

Gms m1 1
=− [−(z − R) − (z + R) + 2z]
2
2 2Rz

=0

So we have demonstrated the proposition that for a point-like object located on the z - axis a distance z from the center of a
spherical shell, the gravitational force on the point like object is given by
m s m1
⎧ −G
⎪ ^
→ k, z > R
z2
F s,1 (r) = ⎨
⎩ →

0 ,z < R

This proves the result that the gravitational force inside the shell is zero and the gravitational force outside the shell is equivalent to
putting all the mass at the center of the shell.

This page titled 9.4: Appendix 9A The Gravitational Field of a Spherical Shell of Matter . is shared under a CC BY-NC-SA 4.0 license and was
authored, remixed, and/or curated by Peter Dourmashkin (MIT OpenCourseWare) via source content that was edited to the style and standards of
the LibreTexts platform; a detailed edit history is available upon request.

9.4.5 https://phys.libretexts.org/@go/page/24476
CHAPTER OVERVIEW
10: Momentum, System of Particles, and Conservation of Momentum
10.1: Introduction
10.2: Momentum (Quantity of Motion) and Average Impulse
10.3: External and Internal Forces and the Change in Momentum of a System
10.4: System of Particles
10.5: Center of Mass
10.6: Translational Motion of the Center of Mass
10.7: Constancy of Momentum and Isolated Systems
10.8: Momentum Changes and Non-isolated Systems
10.9: Worked Examples

This page titled 10: Momentum, System of Particles, and Conservation of Momentum is shared under a CC BY-NC-SA 4.0 license and was
authored, remixed, and/or curated by Peter Dourmashkin (MIT OpenCourseWare) via source content that was edited to the style and standards of
the LibreTexts platform; a detailed edit history is available upon request.

1
10.1: Introduction
Law II: The change of motion is proportional to the motive force impressed, and is made in the direction of the right line in which
that force is impressed.
If any force generates a motion, a double force will generate double the motion, a triple force triple the motion, whether that force
is impressed altogether and at once or gradually and successively. And this motion (being always directed the same way with the
generating force), if the body moved before, is added or subtracted from the former motion, according as they directly conspire
with or are directly contrary to each other; or obliquely joined, when they are oblique, so as to produce a new motion compounded
from the determination of both.
Isaac Newton Principia
When we apply a force to an object, through pushing, pulling, hitting or otherwise, we are applying that force over a discrete
interval of time, Δt. During this time interval, the applied force may be constant, or it may vary in magnitude or direction. Forces
may also be applied continuously without interruption, such as the gravitational interaction between the earth and the moon. In this
chapter we will investigate the relationship between forces and the time during which they are applied, and in the process learn
about the quantity of momentum, the principle of conservation of momentum, and its use in solving a new set of problems
involving systems of particles.

This page titled 10.1: Introduction is shared under a CC BY-NC-SA 4.0 license and was authored, remixed, and/or curated by Peter Dourmashkin
(MIT OpenCourseWare) via source content that was edited to the style and standards of the LibreTexts platform; a detailed edit history is
available upon request.

10.1.1 https://phys.libretexts.org/@go/page/24480
10.2: Momentum (Quantity of Motion) and Average Impulse

Consider a point-like object (particle) of mass m that is moving with velocity v with respect to some fixed reference frame. The

quantity of motion or the momentum, p , of the object is defined to be the product of the mass and velocity
→ →
p =m v

Momentum is a reference frame dependent vector quantity, with direction and magnitude. The direction of momentum is the same
as the direction of the velocity. The magnitude of the momentum is the product of the mass and the instantaneous speed.
Units: In the SI system of units, momentum has units of [kg ⋅ m ⋅ s −1
] . There is no special name for this combination of units.

During a time interval Δt, a non-uniform force F is applied to the particle. Because we are assuming that the mass of the point-
like object does not change, Newton’s Second Law is then
→ →
→ → d v d(m v )
F =m a =m =
dt dt

Because we are assuming that the mass of the point-like object does not change, the Second Law can be written as

→ dp
F =
dt

The impulse of a force acting on a particle during a time interval [t, t + Δt] is defined as the definite integral of the force from t to
t + Δt


t =t+Δt
→ →
′ ′
I =∫ F (t ) dt

t =t

The SI units of impulse are [N ⋅ m] = [kg ⋅ m ⋅ s −1


] which are the same units as the units of momentum.
Apply Newton’s Second Law in Equation (10.2.4) yielding
→′ →

t =t+Δt

t =t+Δt
→ p = p (t+Δt)
→ → dp →′ → → →
′ ′ ′
I =∫ F (t ) dt = ∫ dt = ∫ d p = p (t + Δt) − p (t) = Δ p
′ →′ →

i =t

t =t
dt p = p (t)

Equation (10.2.5) represents the integral version of Newton’s Second Law: the impulse applied by a force during the time interval
[t, t + Δt] is equal to the change in momentum of the particle during that time interval.

The average value of that force during the time interval Δt is given by the integral expression

t =t+Δt
→ 1 →
′ ′
F ave = ∫ F (t ) dt
Δt ′
t =t

The product of the average force acting on an object and the time interval over which it is applied is called the average impulse,
→ →
I ave = F ave Δt

Multiply each side of Equation (10.2.6) by Δt resulting in the statement that the average impulse applied to a particle during the
time interval [t, t + Δt] is equal to the change in momentum of the particle during that time interval,


I ave =Δp

Example 10.1 Impulse for a Non-Constant Force


Suppose you push an object for a time Δt = 1.0s in the +x -direction. For the first half of the interval, you push with a force that
increases linearly with time according to

^ 1 −1
F (t) = bt i , 0 ≤ t ≤ 0.5s with b = 2.0 × 10 N ⋅ s

Then for the second half of the interval, you push with a linearly decreasing force,

10.2.1 https://phys.libretexts.org/@go/page/24481

^ 1
F (t) = (d − bt) i , 0.5s ≤ t ≤ 1.0s with d = 2.0 × 10 N

The force vs. time graph is shown in Figure 10.3. What is the impulse applied to the object?

Figure 10.3 Graph of force vs. time


Solution: We can find the impulse by calculating the area under the force vs. time curve. Since the force vs. time graph consists of
two triangles, the area under the curve is easy to calculate and is given by
→ 1 1
^
I =[ (bΔt/2)(Δt/2) + (bΔt/2)(Δt/2)] i
2 2

1 1
2^ 1 −1 2^ ^
= b(Δt) i = (2.0 × 10 N ⋅ s ) (1.0s) i = (5.0N ⋅ s) i .
4 4

This page titled 10.2: Momentum (Quantity of Motion) and Average Impulse is shared under a CC BY-NC-SA 4.0 license and was authored,
remixed, and/or curated by Peter Dourmashkin (MIT OpenCourseWare) via source content that was edited to the style and standards of the
LibreTexts platform; a detailed edit history is available upon request.

10.2.2 https://phys.libretexts.org/@go/page/24481
10.3: External and Internal Forces and the Change in Momentum of a System
So far we have restricted ourselves to considering how the momentum of an object changes under the action of a force. For
example, if we analyze in detail the forces acting on the cart rolling down the inclined plane (Figure 10.4), we determine that there
→ →
are three forces acting on the cart: the force F spring,cart the spring applies to the cart; the gravitational interaction F earth,cart

between the cart and the earth; and the contact force F between the inclined plane and the cart. If we define the cart as our
plane,cart

system, then everything else acts as the surroundings. We illustrate this division of system and surroundings in Figure 10.4.

Figure 10.4 A diagram of a cart as a system and its surroundings


The forces acting on the cart are external forces. We refer to the vector sum of these external forces that are applied to the system
(the cart) as the external force,
ext
→ → → →
F = F spring, cart  + F earth , cart  + F plane , cart 

Then Newton’s Second Law applied to the cart, in terms of impulse, is


tf ext
→ →

Δ p sys = ∫ F dt ≡ I sys
t0

Let’s extend our system to two interacting objects, for example the cart and the spring. The forces between the spring and cart are
now internal forces. Both objects, the cart and the spring, experience these internal forces, which by Newton’s Third Law are equal
in magnitude and applied in opposite directions. So when we sum up the internal forces for the whole system, they cancel. Thus the
sum of all the internal forces is always zero,
→int →
F = 0

External forces are still acting on our system; the gravitational force, the contact force between the inclined plane and the cart, and
also a new external force, the force between the spring and the force sensor. The force acting on the system is the sum of the
internal and the external forces. However, as we have shown, the internal forces cancel, so we have that
→ →ext →int →ext
F = F + F = F

This page titled 10.3: External and Internal Forces and the Change in Momentum of a System is shared under a CC BY-NC-SA 4.0 license and
was authored, remixed, and/or curated by Peter Dourmashkin (MIT OpenCourseWare) via source content that was edited to the style and
standards of the LibreTexts platform; a detailed edit history is available upon request.

10.3.1 https://phys.libretexts.org/@go/page/24482
10.4: System of Particles
Suppose we have a system of N particles labeled by the index i = 1, 2, 3, ⋯ , N. The force on the i th
particle is
j=N
→ →ext →
Fi = Fi + ∑ F i,j

j=1,j≠i


In this expression F j,i is the force on the th
i particle due to the interaction between the th
i and j
th
particles. We sum over all j
→ →
particles with j ≠ i since a particle cannot exert a force on itself (equivalently, we could define F i,i = 0 ), yielding the internal
force acting on the i particle,
th

j=N
→int →
Fi = ∑ F j,i

j=1,j≠i

The force acting on the system is the sum over all i particles of the force acting on each particle,
i=N i=N i=N j=N
→ → →ext → →ext
F = ∑ Fi = ∑ F +∑ ∑ F j,i = F
i

i=1 i=1 i=1 j=1,j≠i

Note that the double sum vanishes,


i=N j=N
→ →
∑ ∑ F j,i = 0

i=1 j=1,j≠i

because all internal forces cancel in pairs,


→ → →
F j,i + F i,j = 0

The force on the ith


particle is equal to the rate of change in momentum of the i th
particle,

→ dpi
Fi =
dt

When can now substitute Equation (10.4.6) into Equation (10.4.3) and determine that that the external force is equal to the sum
over all particles of the momentum change of each particle,
i=N →
→ext dpi
F =∑
dt
i=1

The momentum of the system is given by the sum


i=N
→ →
p sys = ∑ p i

i=1

momenta add as vectors. We conclude that the external force causes the momentum of the system to change, and we thus restate
and generalize Newton’s Second Law for a system of objects as

→ext d p sys
F =
dt

In terms of impulse, this becomes the statement


tf
→ →ext →
Δ p sys = ∫ F dt ≡ I
t0

This page titled 10.4: System of Particles is shared under a CC BY-NC-SA 4.0 license and was authored, remixed, and/or curated by Peter
Dourmashkin (MIT OpenCourseWare) via source content that was edited to the style and standards of the LibreTexts platform; a detailed edit

10.4.1 https://phys.libretexts.org/@go/page/24483
history is available upon request.

10.4.2 https://phys.libretexts.org/@go/page/24483
10.5: Center of Mass
Consider two point-like particles with masses m and m . Choose a coordinate system with a choice of origin such that body 1 has
1 2

→ →
position r 1 and body 2 has position r 2 (Figure 10.5).

Figure 10.5 Center of mass coordinate system.



The center of mass vector, R cm , of the two-body system is defined as
→ →
→ m1 r 1 + m2 r 2
R cm =
m1 + m 2

We shall now extend the concept of the center of mass to more general systems. Suppose we have a system of N particles labeled

by the index i = 1, 2, 3, ⋯ , N. Choose a coordinate system and denote the position of the i
th
particle as r i . The mass of the
system is given by the sum
i=N

msys = ∑ mi

i=1

and the position of the center of mass of the system of particles is given by
i=N
→ 1 →
R cm = ∑ mi r i
msys
i=1

→′
(For a continuous rigid body, each point-like particle has mass dm and is located at the position r The center of mass is then
defined as an integral over the body,
→′
→ ∫ dm r
bdy
R cm =
∫ dm
body

Example 10.2 Center of Mass of the Earth-Moon System


The mean distance from the center of the earth to the center of the moon is r = 3.84 × 10 m . The mass of the earth is
em
8

kg and the mass of the moon is m kg . The mean radius of the earth is r = 6.37 × 10 m . The
24 22 6
m = 5.98 × 10
e = 7.34 × 10 m e

mean radius of the moon is r = 1.74 × 10 m . Where is the location of the center of mass of the earthmoon system? Is it inside
m
6

the earth’s radius or outside?


Solution: The center of mass of the earth-moon system is defined to be
i=N
→ 1 → 1 → →
R cm = ∑ mi r i = ( me r e + mm r m )
msys me + mm
i=1

→ →
Choose an origin at the center of the earth and a unit vector ^i pointing towards the moon, then r e = 0 . The center of mass of the
earth-moon system is then

10.5.1 https://phys.libretexts.org/@go/page/24484

→ 1 mm r em mm rem
→ → ^
R cm = ( m e r e + mm r m ) = = i
me + mm me + mm me + mm

22 8
→ (7.34 × 10 kg) (3.84 × 10 m)
^ 6 ^
R cm = i = 4.66 × 10 m i
24 22
(5.98 × 10 kg + 7.34 × 10 kg)

The earth’s mean radius is r e = 6.37 × 10 m


6
so the center of mass of the earth-moon system lies within the earth.
Example 10.3 Center of Mass of a Rod
A thin rod has length L and mass M.

Figure 10.6 a) Uniform rod and b) non-uniform rod


(a) Suppose the rod is uniform (Figure 10.6a). Find the position of the center of mass with respect to the left end of the rod. (b)
Now suppose the rod is not uniform (Figure 10.6b) with a linear mass density that varies with the distance x from the left end
according to
λ0 2
λ(x) = x
2
L

where λ is a constant and has SI units [kg ⋅ m


0
−1
] . Find λ and the position of the center of mass with respect to the left end of
0

the rod.
Solution: (a) Choose a coordinate system with the rod aligned along the x -axis and the origin located at the left end of the rod. The
center of mass of the rod can be found using the definition given in Equation (10.5.4). In that expression dm is an infinitesimal

mass element and r is the vector from the origin to the mass element dm (Figure 10.6c).

Figure 10.6c Infinitesimal mass element for rod


Choose an infinitesimal mass element dm located a distance x from the origin. In this problem x will be the integration variable.
′ ′

Let the length of the mass element be dx . Then′


dm = λdx


The vector ′^
r =x i . The center of mass is found by integration

x x =L
→ 1 → 1 1 ∣ 1 L
′ ′^ ′2 ^ 2 ^ ^
R cm = ∫ r dm = ∫ x dx i = x ∣ i = (L − 0) i = i
M L ′ 2L ∣ ′ 2L 2
body  x =0 x =0

(b) For a non-uniform rod (Figure 10.6d),

10.5.2 https://phys.libretexts.org/@go/page/24484
Figure 10.6d Non-uniform rod
the mass element is found using Equation (10.5.8)

′ ′
λ0 ′2 ′
dm = λ (x ) dx = λ = x dx
2
L


The vector ′^
r =x i . The mass is found by integrating the mass element over the length of the rod

x=L x=L x =L
λ0 λ0 ∣ λ0 λ0
′ ′ ′2 ′ 3 3
M =∫ dm = ∫ λ (x ) dx = ∫ x dx = x ∣ = (L − 0) = L
2 2 ∣ 2
body 

x =0 L x =0
′ 3L x′ =0 3L 3

Therefore
3M
λ0 =
L

The center of mass is again found by integration


→ →
1 3 x ′ ′ ′^ 3 x ′3 ′^
R cm = ∫ r dm = ∫ λ (x ) x dx i = ∫ x dx i
M body λ0 L x′ =0 3 x′ =0
L


3 ′4 ∣ ^ 3 4 ^ 3 ^
R cm = x i = (L − 0) i = Li
4L
3 ∣ ′
4L
3
4
x =0

This page titled 10.5: Center of Mass is shared under a CC BY-NC-SA 4.0 license and was authored, remixed, and/or curated by Peter
Dourmashkin (MIT OpenCourseWare) via source content that was edited to the style and standards of the LibreTexts platform; a detailed edit
history is available upon request.

10.5.3 https://phys.libretexts.org/@go/page/24484
10.6: Translational Motion of the Center of Mass
The velocity of the center of mass is found by differentiation,

i=N
→ 1 → p sys
V cm = ∑ mi v i =
msys msys
i=1

The momentum is then expressed in terms of the velocity of the center of mass by


p sys = msys V cm

We have already determined that the external force is equal to the change of the momentum of the system (Equation (10.4.9)). If we
now substitute Equation (10.6.2) into Equation (10.4.9), and continue with our assumption of constant masses m , we have that
i

→ →
→ext d p sys d V cm →
F = = msys = msys A cm
dt dt

→ →
where A the derivative with respect to time of V is the acceleration of the center of mass. From Equation (10.6.3) we can
cm cm

conclude that in considering the linear motion of the center of mass, the sum of the external forces may be regarded as acting at the
center of mass.
Example 10.4 Forces on a Baseball Bat

Suppose you push a baseball bat lying on a nearly frictionless table at the center of mass, position 2, with a force F (Figure 10.7).
Will the acceleration of the center of mass be greater than, equal to, or less than if you push the bat with the same force at either
end, positions 1 and 3

Figure 10.7 Forces acting on a baseball bat


Solution: The acceleration of the center of mass will be equal in the three cases. From our previous discussion, (Equation (10.6.3)),
the acceleration of the center of mass is independent of where the force is applied. However, the bat undergoes a very different
motion if we apply the force at one end or at the center of mass. When we apply the force at the center of mass all the particles in
the baseball bat will undergo linear motion (Figure 10.7a).

Figure 10.7a Force applied at center of mass


When we push the bat at one end, the particles that make up the baseball bat will no longer undergo a linear motion even though
the center of mass undergoes linear motion. In fact, each particle will rotate about the center of mass of the bat while the center of
mass of the bat accelerates in the direction of the applied force (Figure 10.7b).

10.6.1 https://phys.libretexts.org/@go/page/24485
Figure 10.7b Force applied at end of bat

This page titled 10.6: Translational Motion of the Center of Mass is shared under a CC BY-NC-SA 4.0 license and was authored, remixed, and/or
curated by Peter Dourmashkin (MIT OpenCourseWare) via source content that was edited to the style and standards of the LibreTexts platform; a
detailed edit history is available upon request.

10.6.2 https://phys.libretexts.org/@go/page/24485
10.7: Constancy of Momentum and Isolated Systems
Suppose we now completely isolate our system from the surroundings. When the external force acting on the system is zero,
→ext →
F = 0

the system is called an isolated system. For an isolated system, the change in the momentum of the system is zero,
→ →
Δ p sys = 0  (isolated system) 

therefore the momentum of the isolated system is constant. The initial momentum of our system is the sum of the initial momentum
of the individual particles,
→ → →
p sys,i = m1 v 1,i + m2 v 2,i + ⋯

The final momentum is the sum of the final momentum of the individual particles,
→ → →
p sys,f = m1 v 1,f + m2 v 2,f + ⋯

Note that the right-hand-sides of Equations. (10.7.3) and (10.7.4) are vector sums.
When the external force on a system is zero, then the initial momentum of the system equals the final momentum of the system,
→ →
p sys,i = p sys,f

This page titled 10.7: Constancy of Momentum and Isolated Systems is shared under a CC BY-NC-SA 4.0 license and was authored, remixed,
and/or curated by Peter Dourmashkin (MIT OpenCourseWare) via source content that was edited to the style and standards of the LibreTexts
platform; a detailed edit history is available upon request.

10.7.1 https://phys.libretexts.org/@go/page/24755
10.8: Momentum Changes and Non-isolated Systems
Suppose the external force acting on the system is not zero,
→ext →
F ≠ 0

and hence the system is not isolated. By Newton’s Third Law, the sum of the force on the surroundings is equal in magnitude but
opposite in direction to the external force acting on the system,
→sur  →ext 
F = −F

It’s important to note that in Equation (10.8.2), all internal forces in the surroundings sum to zero. Thus the sum of the external
force acting on the system and the force acting on the surroundings is zero,
→sur →ext →
F + F = 0

→ext
We have already found (Equation (10.4.9)) that the external force F acting on a system is equal to the rate of change of the
momentum of the system. Similarly, the force on the surrounding is equal to the rate of change of the momentum of the
surroundings. Therefore the momentum of both the system and surroundings is always conserved.
For a system and all of the surroundings that undergo any change of state, the change in the momentum of the system and its
surroundings is zero,
→ → →
Δ p sys + Δ p sur = 0

Equation (10.8.4) is referred to as the Principle of Conservation of Momentum.

This page titled 10.8: Momentum Changes and Non-isolated Systems is shared under a CC BY-NC-SA 4.0 license and was authored, remixed,
and/or curated by Peter Dourmashkin (MIT OpenCourseWare) via source content that was edited to the style and standards of the LibreTexts
platform; a detailed edit history is available upon request.

10.8.1 https://phys.libretexts.org/@go/page/24756
10.9: Worked Examples
Problem Solving Strategies
When solving problems involving changing momentum in a system, we shall employ our general problem solving strategy
involving four basic steps:
1. Understand – get a conceptual grasp of the problem.
2. Devise a Plan - set up a procedure to obtain the desired solution.
3. Carry our your plan – solve the problem!
4. Look Back – check your solution and method of solution.
We shall develop a set of guiding ideas for the first two steps.
1. Understand – get a conceptual grasp of the problem
The first question you should ask is whether or not momentum is constant in some system that is changing its state after undergoing
an interaction. First you must identify the objects that compose the system and how they are changing their state due to the
interaction. As a guide, try to determine which objects change their momentum in the course of interaction. You must keep track of
the momentum of these objects before and after any interaction. Second, momentum is a vector quantity so the question of whether
momentum is constant or not must be answered in each relevant direction. In order to determine this, there are two important
considerations. You should identify any external forces acting on the system. Remember that a non-zero external force will cause
the momentum of the system to change, (Equation (10.4.9) above),

→ext d p sys
F =
dt

Equation (10.9.1) is a vector equation; if the external force in some direction is zero, then the change of momentum in that
direction is zero. In some cases, external forces may act but the time interval during which the interaction takes place is so small
that the impulse is small in magnitude compared to the momentum and might be negligible. Recall that the average external
impulse changes the momentum of the system
→ →ext →
I = F Δtint = Δ p sys

→ →
If the interaction time is small enough, the momentum of the system is constant, Δ p → 0 If the momentum is not constant then
you must apply either Equation (10.9.1) or Equation (10.9.2). If the momentum of the system is constant, then you can apply
Equation (10.7.5),
→ →
p sys,i = p sys,f

If there is no net external force in some direction, for example the x -direction, the component of momentum is constant in that
direction, and you must apply

psys,x,i = psys,x,f

2. Devise a Plan - set up a procedure to obtain the desired solution


Draw diagrams of all the elements of your system for the two states immediately before and after the system changes its state.
Choose symbols to identify each mass and velocity in the system. Identify a set of positive directions and unit vectors for each
state. Choose your symbols to correspond to the state and motion (this facilitates an easy interpretation, for example (v ) x,i 1

represents the x -component of the velocity of object 1 in the initial state and (v ) represents the x -component of the velocity of
x,f 1

object 1 in the final state). Decide whether you are using components or magnitudes for your velocity symbols. Since momentum is
a vector quantity, identify the initial and final vector components of the momentum. We shall refer to these diagrams as momentum
flow diagrams. Based on your model you can now write expressions for the initial and final momentum of your system. As an
example in which two objects are moving only in the x -direction, the initial x -component of the momentum is

psys ,x,i = m1 (vx,i ) + m2 (vx,i ) +⋯


1 2

10.9.1 https://phys.libretexts.org/@go/page/24757
The final x -component of the momentum is

psys, x,f = m1 (vx,f ) + m2 (vx,f ) +⋯


1 2

If the x -component of the momentum is constant then

psys,x,i = psys,x,f

We can now substitute Equations (10.9.5) and (10.9.6) into Equation (10.9.7), yielding

m1 (vx,i ) + m2 (vx,i ) + ⋯ = m1 (vx,f ) + m2 (vx,f ) +⋯


1 2 1 2

Equation (10.9.8) can now be used for any further analysis required by a particular problem. For example, you may have enough
information to calculate the final velocities of the objects after the interaction. If so then carry out your plan and check your
solution, especially dimensions or units and any relevant vector directions.
Example 10.5 Exploding Projectile
An instrument-carrying projectile of mass m accidentally explodes at the top of its trajectory. The horizontal distance between
1

launch point and the explosion is x . The projectile breaks into two pieces that fly apart horizontally. The larger piece, m has three
i 3

times the mass of the smaller piece, m . To the surprise of the scientist in charge, the smaller piece returns to earth at the launching
2

station. Neglect air resistance and effects due to the earth’s curvature. How far away, x , from the original launching point does
3,f

the larger piece land?

Figure 10.8 Exploding projectile trajectories


Solution: We can solve this problem two different ways. The easiest approach is utilizes the fact that the external force is the
gravitational force and therefore the center of mass of the system follows a parabolic trajectory. From the information given in the
problem m = m /4 and m = 3m /4 . Thus when the two objects return to the ground the center of mass of the system has
2 1 3 1

traveled a distance R = 2x . We now use the definition of center of mass to find where the object with the greater mass hits the
cm i

ground. Choose an origin at the starting point. The center of mass of the system is given by
→ →
→ m2 r 2 + m3 r 3
R cm =
m2 + m 3

→ → →
So when the objects hit the ground ^
R cm = 2 xi i , the object with the smaller mass returns to the origin, r 2 = 0 , and the position

vector of the other object is ^
r 3 = x3,f i . So using the definition of the center of mass,

10.9.2 https://phys.libretexts.org/@go/page/24757
^ ^
(3 m1 /4) x3,f i (3 m1 /4) x3,f i 3
^ ^
2 xi i = = = x3,f i
m1 /4 + 3 m1 /4 m1 4

Therefore
8
x3,f = xi
3

Note that the neither the vertical height above ground nor the gravitational acceleration g entered into our solution.
Alternatively, we can use conservation of momentum and kinematics to find the distance traveled. Because the smaller piece
returns to the starting point after the collision, the velocity of the smaller piece immediately after the explosion is equal to the
negative of the velocity of original object immediately before the explosion. Because the collision is instantaneous, the horizontal
component of the momentum is constant during the collision. We can use this to determine the speed of the larger piece after the
collision. The larger piece takes the same amount of time to return to the ground as the projectile originally takes to reach the top of
the flight. We can therefore determine how far the larger piece traveled horizontally.
We begin by identifying various states in the problem.
Initial state, time t 0 =0 : the projectile is launched.
State 1 time t : the projectile is at the top of its flight trajectory immediately before the explosion. The mass is m and the velocity
1 1


of the projectile is ^
v 1 = v1 i .
State 2 time t : immediately after the explosion, the projectile has broken into two pieces, one of mass m moving backwards (in
2 2

→ →
the negative x -direction) with velocity v2 =−v1 . The other piece of mass m3 is moving in the positive x -direction with

velocity ^
v 3 = v3 i , (Figure 10.8).
State 3: the two pieces strike the ground at time t = 2t one at the original launch site 1 and the other at a distance, x from the
f 1 3,f

launch site, as indicated in Figure 10.8. The pieces take the same amount of time to reach the ground Δt = t because both pieces 1

are falling from the same height as the original piece reached at time t and each has no component of velocity in the vertical
1

direction immediately after the explosion. The momentum flow diagram with state 1 as the initial state and state 2 as the final state
are shown in the upper two diagrams in Figure 10.8.
The initial momentum at time t immediately before the explosion is
1

ss
→ →
p (t1 ) = m1 v 1

The momentum at time t immediately after the explosion is


2

→gs → → 1 → 3 →
p (t2 ) = m2 v 2 + m3 v 3 = − m1 v 1 + m1 v 3
4 4

During the duration of the instantaneous explosion, impulse due to the external gravitational force may be neglected and therefore
the momentum of the system is constant. In the horizontal direction, we have that
→ 1 → 3 →
m1 v 1 = − m1 v 1 + m1 v 3
4 4

Equation (10.9.11) can now be solved for the velocity of the larger piece immediately after the collision,
→ 5→
v3 = v1
3

The larger piece travels a distance


5 5
x3,f = v3 t1 = v1 t1 = xi
3 3

Therefore the total distance the larger piece traveled from the launching station is
5 8
xf = xi + xi = xi
3 3

in agreement with our previous approach.

10.9.3 https://phys.libretexts.org/@go/page/24757
Example 10.6 Landing Plane and Sandbag

Figure 10.9 Plane and sandbag


A light plane of mass 1000 kg makes an emergency landing on a short runway. With its engine off, it lands on the runway at a
speed of 40m ⋅ s A hook on the plane snags a cable attached to a 120 kg sandbag and drags the sandbag along. If the coefficient
−1

of friction between the sandbag and the runway is μ = 0.4 , and if the plane’s brakes give an additional retarding force of
k

magnitude 1400 N , how far does the plane go before it comes to a stop?
Solution: We shall assume that when the plane snags the sandbag, the collision is instantaneous so the momentum in the horizontal
direction remains constant,

px,i = px,1

We then know the speed of the plane and the sandbag immediately after the collision. After the collision, there are two external
forces acting on the system of the plane and sandbag, the friction between the sandbag and the ground and the braking force of the
runway on the plane. So we can use the Newton’s Second Law to determine the acceleration and then one-dimensional kinematics
to find the distance the plane traveled since we can determine the change in kinetic energy.
The momentum of the plane immediately before the collision is

^
p i = mp vp,i i

The momentum of the plane and sandbag immediately after the collision is

^
p 1 = (mp + ms ) vp,1 i

Because the x - component of the momentum is constant, we can substitute Equations (10.9.16) and (10.9.17) into Equation
(10.9.15) yielding
mp vp,i = (mp + ms ) vp,1

The speed of the plane and sandbag immediately after the collision is
mp vp,i
vp,1 =
mp + ms

The forces acting on the system consisting of the plane and the sandbag are the normal force on the sandbag,

^
N g,s = Ng,s j

the frictional force between the sandbag and the ground



^ ^
f k = −fk i = −μk Ng,s i ,

the braking force on the plane



^
F g,p = −Fg,p i

and the gravitational force on the system,



^
(mp + ms ) g = − (mp + ms ) g j

Newton’s Second Law in the ^i -direction becomes

−Fg,p − fk = (mp + ms ) ax

If we just look at the vertical forces on the sandbag alone then Newton’s Second Law in the ^j -direction becomes

10.9.4 https://phys.libretexts.org/@go/page/24757
N − ms g = 0

The frictional force on the sandbag is then



^ ^
f k = −μk Ng,s i = −μk ms g i

Newton’s Second Law in the 1


^ -direction becomes

−Fg,p − μk ms g = (mp + ms ) ax

The x -component of the acceleration of the plane and the sand bag is then
−Fg,p − μk ms g
ax =
mp + m s

We choose our origin at the location of the plane immediately after the collision, x (0) = 0 . Set t = 0 immediately after the
p

collision. The x -component of the velocity of the plane immediately after the collision is v = v . Set t = t when the plane x,0 p,1 f

just comes to a stop. Because the acceleration is constant, the kinematic equations for the change in velocity is

vx,f (tf ) − vp,1 = ax tf

We can solve this equation for t = t , where v


f x,f (tf ) = 0

tf = −vp,1 / ax t

Then the position of the plane when it first comes to rest is


2
1 1 vp,1
2
xp (tf ) − xp (0) = vp,1 tf + ax t =−
f
2 2 ax

Then using x p (0) =0 and substituting Equation (10.9.26) into Equation (10.9.27) yields
2
1 (mp + ms ) v
p,1
xp (tf ) =
2 (Fg,p + μk ms g)

We now use the condition from conservation of the momentum law during the collision, Equation (10.9.19) in Equation (10.9.28)
yielding
2 2
mp v
p,i
xp (tf ) =
2 (mp + ms ) (Fg,p + μk ms g)

Substituting the given values into Equation (10.9.28) yields


2
2 −1
(1000kg) (40m ⋅ s )
2
xp (tf ) = = 3.8 × 10 m
−2
2(1000kg + 120kg) (1400N + (0.4)(120kg) (9.8m ⋅ s ))

This page titled 10.9: Worked Examples is shared under a CC BY-NC-SA 4.0 license and was authored, remixed, and/or curated by Peter
Dourmashkin (MIT OpenCourseWare) via source content that was edited to the style and standards of the LibreTexts platform; a detailed edit
history is available upon request.

10.9.5 https://phys.libretexts.org/@go/page/24757
CHAPTER OVERVIEW
11: Reference Frames
11.1: Introduction to Reference Frames
11.2: Galilean Coordinate Transformations
11.3: Law of Addition of Velocities - Newtonian Mechanics
11.4: Worked Examples

This page titled 11: Reference Frames is shared under a CC BY-NC-SA 4.0 license and was authored, remixed, and/or curated by Peter
Dourmashkin (MIT OpenCourseWare) via source content that was edited to the style and standards of the LibreTexts platform; a detailed edit
history is available upon request.

1
11.1: Introduction to Reference Frames
Examples of this sort, together with the unsuccessful attempts to discover any motion of the earth relatively to the “light
medium” suggest that the phenomena of electromagnetism as well as mechanics possess no properties corresponding to the
idea of absolute rest. They suggest rather that, …, the same laws of electrodynamics and optics will be valid for all frames of
reference for which the equations of mechanics hold good. We will raise this conjecture (the purport of which will hereafter
be called the “Principle of Relativity”) to the status of a postulate, and also introduce another postulate, …, namely that
light is always propagated in empty space with a definite velocity c, which is independent of the state of motion of the
emitting body.
~Albert Einstein
In order to describe physical events that occur in space and time such as the motion of bodies, we introduced a coordinate system.
Its spatial and temporal coordinates can now specify a space-time event. In particular, the position of a moving body can be
described by space-time events specified by its space-time coordinates. You can place an observer at the origin of coordinate
system. The coordinate system with your observer acts as a reference frame for describing the position, velocity, and acceleration
of bodies. The position vector of the body depends on the choice of origin (location of your observer) but the displacement,
velocity, and acceleration vectors are independent of the location of the observer.
You can always choose a second reference frame that is moving with respect to the first reference frame. Then the position,
velocity and acceleration of bodies as seen by the different observers do depend on the relative motion of the two reference frames.
The relative motion can be described in terms of the relative position, velocity, and acceleration of the observer at the origin, O , in
reference frame S with respect to a second observer located at the origin, O′ , in reference frame S′.

This page titled 11.1: Introduction to Reference Frames is shared under a CC BY-NC-SA 4.0 license and was authored, remixed, and/or curated
by Peter Dourmashkin (MIT OpenCourseWare) via source content that was edited to the style and standards of the LibreTexts platform; a detailed
edit history is available upon request.

11.1.1 https://phys.libretexts.org/@go/page/24487
11.2: Galilean Coordinate Transformations

Let the vector R point from the origin of frame S to the origin of reference frame S

. Suppose an object is located at a point 1.

Denote the position vector of the object with respect to origin of reference frame S by r . Denote the position vector of the object
→′
with respect to origin of reference frame S by

r .

Figure 11.1 Two reference frames S and S′.


The position vectors are related by
→′ → →
r = r −R

These coordinate transformations are called the Galilean Coordinate Transformations. They enable the observer in frame S to
predict the position vector in frame S′, based only on the position vector in frame S and the relative position of the origins of the
two frames.

The relative velocity between the two reference frames is given by the time derivative of the vector R , defined as the limit as of
the displacement of the two origins divided by an interval of time, as the interval of time becomes infinitesimally small,

→ dR
V =
dt

Relatively Inertial Reference Frames and the Principle of Relativity


If the relative velocity between the two reference frames is constant, then the relative acceleration between the two reference
frames is zero,

→ dV →
A = = 0
dt

When two reference frames are moving with a constant velocity relative to each other as above, the reference frames are called
relatively inertial reference frames.
We can reinterpret Newton’s First Law
Law 1: Every body continues in its state of rest, or of uniform motion in a right line, unless it is compelled to change that state by
forces impressed upon it.
as the Principle of Relativity:
In relatively inertial reference frames, if there is no net force impressed on an object at rest in frame S, then there is also no net
force impressed on the object in frame S′ .

This page titled 11.2: Galilean Coordinate Transformations is shared under a CC BY-NC-SA 4.0 license and was authored, remixed, and/or
curated by Peter Dourmashkin (MIT OpenCourseWare) via source content that was edited to the style and standards of the LibreTexts platform; a
detailed edit history is available upon request.

11.2.1 https://phys.libretexts.org/@go/page/24488
11.3: Law of Addition of Velocities - Newtonian Mechanics
Suppose the object in Figure 11.1 is moving; then observers in different reference frames will measure different velocities. Denote
→ → →′ →′
the velocity of the object in frame S by v = d r /dt , and the velocity of the object in frame S′ by v = d r /dt . Since the

derivative of the position is velocity, the velocities of the object in two different reference frames are related according to
→′ → →
d r d r dR
= −
dt′ dt dt

→ ′
→ →
v = v −V

This is called the Law of Addition of Velocities.

This page titled 11.3: Law of Addition of Velocities - Newtonian Mechanics is shared under a CC BY-NC-SA 4.0 license and was authored,
remixed, and/or curated by Peter Dourmashkin (MIT OpenCourseWare) via source content that was edited to the style and standards of the
LibreTexts platform; a detailed edit history is available upon request.

11.3.1 https://phys.libretexts.org/@go/page/24489
11.4: Worked Examples
Example 11.1 Relative Velocities of Two Moving Planes
An airplane A is traveling northeast with a speed of v A = 160m ⋅ s
−1
. A second airplane B is traveling southeast with a speed of

vB = 200m ⋅ s
−1
. (a) Choose a coordinate system and write down an expression for the velocity of each airplane as vectors, VA
→ → →
and V Carefully use unit vectors to express your answer. (b) Sketch the vectors V and V on your coordinate system. (c)
B A B

Find a vector expression that expresses the velocity of aircraft A as seen from an observer flying in aircraft B. Calculate this vector.
What is its magnitude and direction? Sketch it on your coordinate system.

Figure 11.2 (b): Coordinate System


An observer at rest with respect to the ground defines a reference frame S . Choose a coordinate system shown in Figure 11.2b.

According to this observer, airplane A is moving with velocity ^ ^
v A = vA cos θA i + vA sin θA j , and airplane B is moving with

velocity ^ ^
v B = vB cos θB i + vB sin θB j . According to the information given in the problem airplane A flies northeast so
→ – –
θA = π/4 and airplane B flies southeast east so θB = −π/4 . Thus v A = (80 √2m ⋅ s
−1 ^
) i + (80 √2m ⋅ s
−1 ^
) j and
→ – –
v B = (100 √2m ⋅ s
−1 ^
) i − (100 √2m ⋅ s
−1 ^
) j .
Consider a second observer moving along with airplane B, defining reference frame S′ . What is the velocity of airplane A
according to this observer moving in airplane B ? The velocity of the observer moving along in airplane B with respect to an
→ →
observer at rest on the ground is just the velocity of airplane B and is given by V = v = v cos θ ^i + v sin θ ^j . Using the
B B B B B

Law of Addition of Velocities, Equation (11.3.2), the velocity of airplane A with respect to an observer moving along with Airplane
B is given by
′ →
→ → ^ ^ ^ ^
v = v A − V = (vA cos θA i + vA sin θA j ) − (vB cos θB i + vB sin θB j )
A

^ ^
= (vA cos θA − vB cos θB ) i + (vA sin θA − vB sin θB ) j

– −1 – −1 ^ – −1 – −1 ^
= ((80 √2m ⋅ s ) − (100 √2m ⋅ s )) i + ((80 √2m ⋅ s ) + (100 √2m ⋅ s )) j .

– −1 ^ – −1 ^
= − (20 √2m ⋅ s ) i + (180 √2m ⋅ s ) j

′ ^ ′ ^
=v i +v j
Ax Ay

Figure 11.3 shows the velocity of airplane A with respect to airplane B in reference frame S′ .

11.4.1 https://phys.libretexts.org/@go/page/24490
Figure 11.3 Airplane A as seen from observer in airplane B
The magnitude of velocity of airplane A as seen by an observer moving with airplane B is given by
1/2 1/2
′ – 2 – 2
∣v ⃗  ∣ = (v′2 ′2
+v ) = ( (−20 √2m ⋅ s
−1
) + (180 √2m ⋅ s
−1
) ) = 256m ⋅ s
−1
∣ A∣ Ax Ay

The angle of velocity of airplane A as seen by an observer moving with airplane B is given by,

′ −1 ′ ′ −1 – −1 – −1
θ = tan (v /v ) = tan ((180 √2m ⋅ s ) / (−20 √2m ⋅ s ))
A Ay Ax

−1 ∘ ∘ ∘
= tan (−9) = 180 − 83.7 = 96.3

Example 11.2 Relative Motion and Polar Coordinates


By relative velocity we mean velocity with respect to a specified coordinate system. (The term velocity, alone, is understood to be

relative to the observer’s coordinate system.) (a) A point is observed to have velocity VA , relative to coordinate system A . What is
→ →
its velocity relative to coordinate system B , which is displaced from system A by distance R ? ( R can change in time.) (b)
Particles a and b move in opposite directions around a circle with the magnitude of the angular velocity ω , as shown in Figure

11.4. At t = 0 they are both at the point ^^
r = l j where l is the radius of the circle. Find the velocity of a relative to b .

Figure 11.5 Particles a and b moving relative to each other


Solution: (a) The position vectors are related by
→ → →
r B = r A −R

The velocities are related by the taking derivatives, (law of addition of velocities Equation (11.3.2))

→ →
vB = vA −V

11.4.2 https://phys.libretexts.org/@go/page/24490
(b) Let’s choose two reference frames; frame B is centered at particle b, and frame A is centered at the center of the circle in Figure
11.5. Then the relative position vector between the origins of the two frames is given by

R = l^
r

The position vector of particle a relative to frame A is given by


→ ′
r A = l^
r

The position vector of particle b in frame B can be found by substituting Equations (11.4.7) and (11.4.6) into Equation (11.4.4),

→ → ′
r B = r A − R = l^
r − l^
r

We can decompose each of the unit vectors ^


r and ^
r with respect to the Cartesian unit vectors 1

^ and ^
j (see Figure 11.5),

^ ^
^
r = − sin θ i + cos θ j

′ ^ ^
^
r = sin θ i + cos θ j

Then Equation (11.4.8) giving the position vector of particle b in frame B becomes
→ ′
^ ^ ^ ^ ^
r B = l^
r − l^
r = l(sin θ i + cos θ j ) − l(− sin θ i + cos θ j ) = 2l sin θ i

In order to find the velocity vector of particle a in frame B (i.e. with respect to particle b), differentiate Equation (11.4.11)
→ d dθ
^ ^ ^
vB = (2l sin θ) i = (2l cos θ) i = 2ωl cos θ i
dt dt

Example 11.3 Recoil in Different Frames


A person of mass m is standing on a cart of mass m . Assume that the cart is free to move on its wheels without friction. The
1 2

person throws a ball of mass m θ with respect to the horizontal as measured by the person in the cart. The ball is thrown with a
3

speed v with respect to the cart (Figure 11.6). (a) What is the final velocity of the ball as seen by an observer fixed to the ground?
0

(b) What is the final velocity of the cart as seen by an observer fixed to the ground? (c) With respect to the horizontal, what angle
the fixed observer see the ball leave the cart?

Figure 11.6 Recoil of a person on cart due to thrown ball


Solution: a), b) Our reference frame will be that fixed to the ground. We shall take as our initial state that before the ball is thrown
(cart, ball, throwing person stationary) and our final state that after the ball is thrown. We are assuming that there is no friction, and
so there are no external forces acting in the horizontal direction. The initial x -component of the total momentum is zero,
px,0 = 0 total
After the ball is thrown, the cart and person have a final momentum

^
p f , cart  = − (m2 + m1 ) vf , cart  i

as measured by the person on the ground, where v = 0 is the speed of the person and cart. (The person’s center of mass will
f ,cart

move with respect to the cart while the ball is being thrown, but since we’re interested in velocities, not positions, we need only
assume that the person is at rest with respect to the cart after the ball is thrown.)

11.4.3 https://phys.libretexts.org/@go/page/24490
The ball is thrown with a speed v and at an angle θ with respect to the horizontal as measured by the person in the cart. Therefore
0

the person in the cart throws the ball with velocity


→′
^ ^
v f , ball  = v0 cos θ i + v0 sin θ j

Because the cart is moving in the negative x -direction with speed v = 0 just as the ball leaves the person’s hand, the x -
f ,cart

component of the velocity of the ball as measured by an observer on the ground is given by

vxf , ball  = v0 cos θ − vf , cart 

The ball appears to have a smaller x -component of the velocity according to the observer on the ground. The velocity of the ball as
measured by an observer on the ground is

^ ^
v f , ball  = (v0 cos θ − vf , cart ) i + v0 sin θ j

The final momentum of the ball according to an observer on the ground is



^ ^
p f , ball  = m3 [(v0 cos θ − vf , can  ) i + v0 sin θ j ]

The momentum flow diagram is shown in (Figure 11.7).

Figure 11.7 Momentum flow diagram for recoil


Because the x -component of the momentum of the system is constant, we have that

0 = (px,f ) + (px,f )
cart  ball 

= − (m2 + m1 ) vf , cart  + m3 (v0 cos θ − vf , cart )

We can solve Equation (11.4.19) for the final speed and velocity of the cart as measured by an observer on the ground,
m3 v0 cos θ
vf , cart  =
m 2 + m 1 + m3

→ m3 v0 cos θ
^ ^
v f , cart  = vf , cart  i = i
m2 + m 1 + m3

Note that the y -component of the momentum is not constant because as the person is throwing the ball he or she is pushing off the
cart and the normal force with the ground exceeds the gravitational force so the net external force in the y -direction is non-zero.
Substituting Equation (11.4.20) into Equation (11.4.17) gives

^ ^
v f , ball  = (v0 cos θ − vf , cart ) i + v0 sin θ j

m1 + m2
^ ^
= (v0 cos θ) i + (v0 sin θ) j
m 1 + m2 + m 3


As a check, note that in the limit m << m + m , V f ,  ball  has speed v and is directed at an angle θ above the horizontal; the
3 1 2 0

fact that the much more massive person-cart combination is free to move doesn’t affect the flight of the ball as seen by the fixed
observer. Also note that in the unrealistic limit m >> m + m the ball is moving at a speed much smaller than v as it leaves the
1 2 0

cart.

11.4.4 https://phys.libretexts.org/@go/page/24490
c) The angle ϕ at which the ball is thrown as seen by the observer on the ground is given by
(vf , ball ) v0 sin θ
y
−1 −1
ϕ = tan = tan
(vf , ball ) [(m1 + m2 ) / (m1 + m2 + m3 )] v0 cos θ
x

−1
m1 + m 2 + m3
= tan [( ) tan θ]
m 1 + m2

For arbitrary values for the masses, the above expression will not reduce to a simplified form. However, we can see that
tan ϕ > tan θ for arbitrary masses, and that in the limit m << m + m , ϕ → θ and in the unrealistic limit
3 1 2

m ≫ m + m , ϕ → π/2 . Can you explain this last odd prediction?


3 1 2

This page titled 11.4: Worked Examples is shared under a CC BY-NC-SA 4.0 license and was authored, remixed, and/or curated by Peter
Dourmashkin (MIT OpenCourseWare) via source content that was edited to the style and standards of the LibreTexts platform; a detailed edit
history is available upon request.

11.4.5 https://phys.libretexts.org/@go/page/24490
CHAPTER OVERVIEW
12: Momentum and the Flow of Mass
12.1: Introduction to Momentum and the Flow of Mass
12.2: Worked Examples
12.3: Rocket Propulsion

This page titled 12: Momentum and the Flow of Mass is shared under a CC BY-NC-SA 4.0 license and was authored, remixed, and/or curated by
Peter Dourmashkin (MIT OpenCourseWare) via source content that was edited to the style and standards of the LibreTexts platform; a detailed
edit history is available upon request.

1
12.1: Introduction to Momentum and the Flow of Mass
Even though the release was pulled, the rocket did not rise at first, but the flame came out, and there was a steady roar. After a
number of seconds it rose, slowly until it cleared the flame, and then at express-train speed, curving over to the left, and striking the
ice and snow, still going at a rapid rate. It looked almost magical as it rose, without any appreciably greater noise or flame, as if it
said, “I've been here long enough; I think I'll be going somewhere else, if you don't mind.”
Robert Goddard

Preface: The Challenger Flight

When the Rogers Commission in 1986 investigated the Challenger disaster, a commission member, physicist Richard
Feynman, made an extraordinary demonstration during the hearings.
“He (Feynman) also learned that rubber used to seal the solid rocket booster joints using O-rings, failed to expand when the
temperature was at or below 32 degrees F (0 degrees C). The temperature at the time of the Challenger liftoff was 32 degrees F.
Feynman now believed that he had the solution, but to test it, he dropped a piece of the O-ring material, squeezed with a C-
clamp to simulate the actual conditions of the shuttle, into a glass of ice water. Ice, of course, is 32 degrees F. At this point one
needs to understand exactly what role the O-rings play in the solid rocket booster (SRB) joints. When the material in the SRB
start to heat up, it expands and pushes against the sides of the SRB. If there is an opening in a joint in the SRB, the gas tries to
escape through that opening (think of it like water in a tea kettle escaping through the spout.) This leak in the Challenger's SRB
was easily visible as a small flicker in a launch photo. This flicker turned into a flame and began heating the fuel tank, which
then ruptured. When this happened, the fuel tank released liquid hydrogen into the atmosphere where it exploded. As Feynman
explained, because the O-rings cannot expand in 32 degree weather, the gas finds gaps in the joints, which led to the explosion
of the booster and then the shuttle itself.”
In the Report of the Presidential Commission on the Space Shuttle Challenger Accident (1986), Appendix F - Personal
observations on the reliability of the Shuttle, Feynman wrote
The Challenger flight is an excellent example. … The O-rings of the Solid Rocket Boosters were not designed to erode.
Erosion was a clue that something was wrong. Erosion was not something from which safety can be inferred. There was no
way, without full understanding, that one could have confidence that conditions the next time might not produce erosion three
times more severe than the time before. Nevertheless, officials fooled themselves into thinking they had such understanding
and confidence, in spite of the peculiar variations from case to case. A mathematical model was made to calculate erosion. This
was a model based not on physical understanding but on empirical curve fitting. To be more detailed, it was supposed a stream
of hot gas impinged on the O-ring material, and the heat was determined at the point of stagnation (so far, with reasonable
physical, thermodynamic laws). But to determine how much rubber eroded it was assumed this depended only on this heat by a
formula suggested by data on a similar material. A logarithmic plot suggested a straight line, so it was supposed that the
erosion varied as the .58 power of the heat, the .58 being determined by a nearest fit. At any rate, adjusting some other
numbers, it was determined that the model agreed with the erosion (to depth of one-third the radius of the ring). There is
nothing much so wrong with this as believing the answer! Uncertainties appear everywhere. How strong the gas stream might
be was unpredictable, it depended on holes formed in the putty. Blow-by showed that the ring might fail even though not, or
only partially eroded through. The empirical formula was known to be uncertain, for it did not go directly through the very data
points by which it was determined. There were a cloud of points some twice above, and some twice below the fitted curve, so
erosions twice predicted were reasonable from that cause alone. Similar uncertainties surrounded the other constants in the
formula, etc., etc. When using a mathematical model careful attention must be given to uncertainties in the model. ...
In any event this has had very unfortunate consequences, the most serious of which is to encourage ordinary citizens to fly in
such a dangerous machine, as if it had attained the safety of an ordinary airliner. The astronauts, like test pilots, should know
their risks, and we honor them for their courage. Who can doubt that McAuliffe was equally a person of great courage, who
was closer to an awareness of the true risk than NASA management would have us believe? Let us make recommendations to
ensure that NASA officials deal in a world of reality in understanding technological weaknesses and imperfections well
enough to be actively trying to eliminate them. …. For a successful technology, reality must take precedence over public
relations, for nature cannot be fooled.

12.1.1 https://phys.libretexts.org/@go/page/24494
So far we have restricted ourselves to considering systems consisting of discrete objects or point-like objects that have fixed
amounts of mass. We shall now consider systems in which material flows between the objects in the system, for example we shall
consider coal falling from a hopper into a moving railroad car, sand leaking from railroad car fuel, grain moving forward into a
railroad car, and fuel ejected from the back of a rocket, In each of these examples material is continuously flows into or out of an
object. We have already shown that the total external force causes the momentum of a system to change,

→total  dp
system 
F ext =
dt

We shall analyze how the momentum of the constituent elements our system change over a time interval [t, t + Δt] , and then
consider the limit as Δt → 0 . We can then explicit calculate the derivative on the right hand side of Equation (12.2.1) and Equation
(12.2.1) becomes
→ → → →
→total  d p system  Δ p system  p system (t + Δt) − p system (t)
F = = lim = lim
ext
dt Δt→0 Δt Δt→0 Δt

We need to be very careful how we apply this generalized version of Newton’s Second Law to systems in which mass flows
between constituent objects. In particular, when we isolate elements as part of our system we must be careful to identify the mass
Δm of the material that continuous flows in or out of an object that is part of our system during the time interval Δt under
consideration.
We shall consider four categories of mass flow problems that are characterized by the momentum transfer of the material of mass
Δm .

Transfer of Material into an Object, but no Transfer of Momentum


Consider for example rain falling vertically downward with speed u into car of mass m moving forward with speed v . A small
amount of falling rain Δm has no component of momentum in the direction of motion of the car. There is a transfer of rain into
r

the car but no transfer of momentum in the direction of motion of the car (Figure 12.1).

Figure 12.1 Transfer of rain mass into the car but no transfer of momentum in direction of motion

Transfer of Material Out of an Object, but no Transfer of Momentum


The material continually leaves the object but it does not transport any momentum away from the object in the direction of motion
of the object (Figure 12.2). Consider an ice skater gliding on ice at speed v holding a bag of sand that is leaking straight down with
respect to the moving skater. The sand continually leaves the bag but it does not transport any momentum away from the bag in the
direction of motion of the object. In Figure 12.2, sand of mass Δm leaves the bag.
s

Figure 12.2 Transfer of mass out of object but no transfer of momentum in direction of motion

12.1.2 https://phys.libretexts.org/@go/page/24494
Transfer of Material Impulses Object Via Transfer of Momentum
Suppose a fire hose is used to put out a fire on a boat of mass m . Assume the column of water moves horizontally with speed u .
b

The incoming water continually hits the boat propelling it forward. During the time interval Δt a column of water of mass Δm s

will hit the boat that is moving forward with speed v increasing it’s speed (Figure 12.3).

Figure 12.3 Transfer of mass of water increases speed of boat

Material Continually Ejected From Object results in Recoil of Object


When fuel of mass Δm is ejected from the back of a rocket with speed u relative to the rocket, the rocket of mass m recoils
f r

forward. Figure 12.4a shows the recoil of the rocket in the reference frame of the rocket. The rocket recoils forward with speed
Δv . In a reference frame in which the rocket is moving forward with speed v , then the speed after recoil is v + Δv . The speed
r r r r

of the backwardly ejected fuel is u − v (Figure 12.4b).


r

Figure 12.4 Transfer of mass out of rocket provides impulse on rocket in (a) reference frame of rocket, (b) reference frame in which
rocket moves with speed v r


We must carefully identify the momentum of the object and the material transferred at time t in order to determine p system (t) . We

must also identify the momentum of the object and the material transferred at time t + Δt in order to determine p system (t + Δt)

as well. Recall that when we defined the momentum of a system, we assumed that the mass of the system remain constant.
Therefore we cannot ignore the momentum of the transferred material at time t + Δt even though it may have left the object; it is
still part of our system (or at time t even though it has not flowed into the object yet).

This page titled 12.1: Introduction to Momentum and the Flow of Mass is shared under a CC BY-NC-SA 4.0 license and was authored, remixed,
and/or curated by Peter Dourmashkin (MIT OpenCourseWare) via source content that was edited to the style and standards of the LibreTexts
platform; a detailed edit history is available upon request.

12.1.3 https://phys.libretexts.org/@go/page/24494
12.2: Worked Examples
Example 12.2.1: Filling a Coal Car

An empty coal car of mass m starts from rest under an applied force of magnitude F . At the same time coal begins to run into
0

the car at a steady rate b from a coal hopper at rest along the track (Figure 12.5). Find the speed when a mass m of coal has
c

been transferred.

Figure 12.5 Filling a coal car


Solution
We shall analyze the momentum changes in the horizontal direction, which we call the x -direction. Because the falling coal
does not have any horizontal velocity, the falling coal is not transferring any momentum in the x -direction to the coal car. So
we shall take as our system the empty coal car and a mass m of coal that has been transferred. Our initial state at t = 0 is when
c

the coal car is empty and at rest before any coal has been transferred. The x -component of the momentum of this initial state is
zero,

px (0) = 0

Our final state at t = t is when all the coal of mass m = bt has been transferred into the car that is now moving at speed
f c f

v . The x -component of the momentum of this final state is


f

px (tf ) = (m0 + mc ) vf = (m0 + b tf ) vf

There is an external constant force Fx =F applied through the transfer. The momentum principle applied to the x -direction is
tf

∫ Fx dt = Δpx = px (tf ) − px (0)


0

Because the force is constant, the integral is simple and the momentum principle becomes

F tf = (m0 + b tf ) vf

So the final speed is


F tf
vf =
(m0 + b tf )

Example 12.2.2: Emptying a Freight Car

A freight car of mass m contains sand of mass m . At t = 0 a constant horizontal force of magnitude F is applied in the
c s

direction of rolling and at the same time a port in the bottom is opened to let the sand flow out at the constant rate
b = dm /dt . Find the speed of the freight car when all the sand is gone (Figure 12.6). Assume that the freight car is at rest at
s

t = 0.

12.2.1 https://phys.libretexts.org/@go/page/24495
Figure 12.6 Emptying a freight car
Solution
Choose the positive x -direction to point in the direction that the car is moving. Choose for the system the amount of sand in

the fright car at time t, m (t). At time t , the car is moving with velocity
c
^
v c (t) = vc (t) i . The momentum diagram for the
system at time t is shown in the diagram on the left in Figure 12.7.

Figure 12.7 Momentum diagram at time t and at time t + Δt


The momentum of the system at time t is given by
→ →
p sys (t) = mc (t) v c (t)

During the time interval [t, t + Δt] , an amount of sand of mass Δm leaves the freight car and the mass of the freight car
s

changes by m (t + Δt) = m (t) + Δm , where Δm = −Δm . At the end of the interval the car is moving with velocity
c c c c s

→ → →
. The momentum diagram for the system at time
^
v c (t + Δt) = v c (t) + Δ v c = (vc (t) + Δvc ) i t + Δt is shown in the
diagram on the right in Figure 12.7. The momentum of the system at time t + Δt is given by
→ → → → →
p sys (t + Δt) = (Δms + mc (t) + Δmc ) ( v c (t) + Δ v c ) = mc (t) ( v c (t) + Δ v c )

→ →
Note that the sand that leaves the car is shown with velocity v (t) + Δ v . This implies that all the sand leaves the car with
c c

the velocity of the car at the end of the interval. This is an approximation. Because the sand leaves continuous, the velocity will
→ → →
vary from v (t) to v (t) + Δ v but so does the change in mass of the car and these two contributions to the system’s
c c c

moment exactly cancel. The change in momentum of the system is then


→ → → → → → →
Δ p sys = p sys (t + Δt) − p sys (t) = mc (t) ( v c (t) + Δ v c ) − mc (t) v c (t) = mc (t)Δ v c


Throughout the interval a constant force ^
F =F i is applied to the system so the momentum principle becomes
→ → → →
→ p sys (t + Δt) − p sys (t) Δvc d vc
F = lim = lim mc (t) = mc (t)
Δt→0 Δt Δt→0 Δt dt

Because the motion is one-dimensional, Equation (12.3.9) written in terms of x -components becomes

12.2.2 https://phys.libretexts.org/@go/page/24495
dvc
F = mc (t)
dt

Denote by initial mass of the car by m = m + m where m is the mass of the car and m is the mass of the sand in the
c,0 c s c s

car at t = 0 . The mass of the sand that has left the car at time t is given by
t t
dms
ms (t) = ∫ dt = ∫ bdt = bt
0
dt 0

Thus

mc (t) = mc,0 − bt = mc + ms − bt

Therefore Equation (12.3.10) becomes


dvc
F = (mc + ms − bt)
dt

This equation can be solved for the x -component of the velocity at time t , vc (t) (which in this case is the speed) by the
method of separation of variables. Rewrite Equation (12.3.13) as
F dt
dvc =
(mc + ms − bt)

Then integrate both sides of Equation (12.3.14) with the limits as shown
′ ′
v =vc (t) t =t ′
F dt

∫ dvc = ∫

v′ =0

t =0 mc + ms − b t

Integration yields the speed of the car as a function of time



′ =t
F ′
∣ F mc + ms − bt F mc + ms
vc (t) = − ln(mc + ms − b t )∣ =− ln( ) = ln( )
b ∣ t=0 b mc + ms b mc + ms − bt

In writing Equation (12.3.16), we used the property that ln(a) − ln(b) = ln(a/b) and therefore ln(a/b) = − ln(b/a) . Note
mc +ms
that m c + ms ≥ mc + ms − bt , so the term ln( mc +ms −bt
) ≥0 , and the speed of the car increases as we expect.

Example 12.2.3: Filling a Freight Car

Grain is blown into car A from car B at a rate of b kilograms per second. The grain leaves the chute vertically downward, so
that it has the same horizontal velocity, u as car B , (Figure 12.8). Car A is initially at rest before any grain is transferred in and
has mass m . At the moment of interest, car A has mass m and speed v . Determine an expression for the speed car A as a
A,0 A

function of time t.

Figure 12.8 Filling a freight car


Solution
Choose positive x -direction to the right in the direction the cars are moving. Define the system at time t to be the car and grain
that is already in it, which together has mass m (t) and the small amount of material of mass Δm that is blown into car A
A g

during the time interval [t, t + Δt] At time that is moving with x -component of the velocity V . At time t , car A is moving A

→ →
with velocity ^
v A (t) = vA (t) i and the material blown into car is moving with velocity ^
u = ui At time t + Δt car A is

12.2.3 https://phys.libretexts.org/@go/page/24495
→ →
moving with velocity v (t) + Δ v = (v (t) + Δv ) ^i , and the mass of car A is m (t + Δt) = m
A A A A A A
(t) + ΔmA where
Δm A
= Δm . The momentum diagram for times t and for t + Δt is shown in Figure 12.9.
g

Figure 12.9 Momentum diagram at times t and t + Δt


The momentum at time t is
→ → →
P sys (t) = mA (t) v A (t) + Δmg u

The momentum at time t + Δt is


→ → →
P sys (t + Δt) = (mA (t) + ΔmA ) ( v A (t) + Δ v A )

There are no external forces acting on the system in the x -direction and the external forces acting on the system perpendicular
to the motion sum to zero, so the momentum principle becomes
→ →
→ P sys (t + Δt) − P sys (t)
0 = lim
Δt→0 Δt

Using the results above (Equations (12.3.17) and (12.3.18), the momentum principle becomes
→ → → →
(mA (t) + ΔmA ) ( v A (t) + Δ v A ) − (mA (t) v A (t) + Δmg u )

0 = lim
Δt→0 Δt

which after using the condition that Δm A = Δmg and some rearrangement becomes
→ →
→ ΔmA ( v A (t) − u ) →
→ mA (t)Δ v A ΔmA Δ v A
0 = lim + lim + lim
Δt→0 Δt Δt→0 Δt Δt→0 Δt


In the limit as , the product Δm AΔVA is a second order differential (the product of two first order differentials) and the term

ΔmA Δ v A /Δt approaches zero, therefore the momentum principle yields the differential equation

→ d vA dmA → →
0 = mA (t) + ( v A (t) − u )
dt dt

The x -component of Equation (12.3.22) is then


dvA dmA
0 = mA (t) + (vA (t) − u)
dt dt

Rearranging terms and using the fact that the material is blown into car A at a constant rate b ≡ dm A /dt , we have that the rate
of change of the x -component of the velocity of car A is given by
dvA (t) b (u − vA (t))
=
dt mA (t)

We cannot directly integrate Equation (12.3.24) with respect to dt because the mass of the car A is a function of time. In order
to find the x -component of the velocity of car A we need to know the relationship between the mass of car A and the x -

12.2.4 https://phys.libretexts.org/@go/page/24495
component of the velocity of the car A . There are two approaches. In the first approach we separate variables in Equation
(12.3.24) where we have suppressed the dependence on t in the expressions for m and V yielding A A

dvA dmA
=
u − vA mA

which becomes the integral equation


′ ′
v =vA (t) ′ m =mA (t) ′
A dv A
′ dm
A A
∫ =∫
′ ′
v

=0
u −v m

=mA,0
m
A A A A

where m A,0 is the mass of the car before any material has been blown in. After integration we have that
u mA (t)
ln = ln
u − vA (t) mA,0

Exponentiate both side yields


u mA (t)
=
u − vA (t) mA,0

We can solve this equation for the x -component of the velocity of the car
mA (t) − mA,0
vA (t) = u
mA (t)

Because the material is blown into the car at a constant rate b ≡ dm A /dt , the mass of the car as a function of time is given by

mA (t) = mA,0 + bt

Therefore substituting Equation (12.3.30) into Equation (12.3.29) yields the x -component of the velocity of the car as a
function of time
bt
vA (t) = u
mA,0 + bt

In a second approach, we substitute Equation (12.3.30) into Equation (12.3.24) yielding

dvA b (u − vA )
=
dt mA,0 + bt

Separate variables in Equation (12.3.32):


dvA bdt
=
u − vA mA,0 + bt

which then becomes the integral equation


′ ′ ′
v =vA (t) ′ t =t ′
A dv dt
A
∫ =∫
′ ′
v

=0
u −v ′
t =0
mA,0 + b t
A
A

Integration yields

u mA,0 + bt
ln = ln
u − vA (t) mA,0

Again exponentiate both sides resulting in

u mA,0 + bt
=
u − vA (t) mA,0

After some algebraic manipulation we can find the speed of the car as a function of time

12.2.5 https://phys.libretexts.org/@go/page/24495
bt
vA (t) = u
mA,0 + bt

in agreement with Equation (12.3.31).


Check result:
We can rewrite Equation (12.3.37) as
(mA,0 + bt) vA (t) = btu

which illustrates the point that the momentum of the system at time t is equal to the momentum of the grain that has been
transferred to the system during the interval [0,t].

Example 12.2.4: Boat and Fire Hose

A burning boat of mass m is initially at rest. A fire fighter stands on a bridge and sprays water onto the boat. The water leaves
0

the fire hose with a speed u at a rate α (measured in kg ⋅ s ). Assume that the motion of the boat and the water jet are
−1

horizontal, that gravity does not play any role, and that the river can be treated as a frictionless surface. Also assume that the
change in the mass of the boat is only due to the water jet and that all the water from the jet is added to the boat, (Figure
12.10).

Figure 12.10 Example 12.4


a. In a time interval [t, t + Δt] , an amount of water Δm hits the boat. Choose a system. Is the total momentum constant in
your system? Write down a differential equation that results from the analysis of the momentum changes inside your
system.
b. Integrate the differential equation you found in part a), to find the velocity v(m) as a function of the increasing mass m of
the boat, m , and u.
0

Solution
Let’s take as our system the boat, the amount of water of mass Δm that enters the boat during the time interval [t, t + Δt]
w

and whatever water is in the boat at time t . The water from the fire hose has a speed u . Denote the mass of the boat (including
some water) at time t by m ≡ m (t) , and the speed of the boat by v ≡ v (t) . At time t + Δt the speed of the boat is v + Δv .
b b b

Choose the positive x - direction in the direction that the boat is moving. Then the x -components of the momentum of the
system at time t and t + Δt are shown in Figure 12.11.

12.2.6 https://phys.libretexts.org/@go/page/24495
Figure 12.11 Momentum diagrams for burning boat
Because we are assuming that the burning boat slides with negligible resistance and that gravity has a negligible effect on the
arc of the water jet, there are no external forces acting on the system in the x -direction. Therefore the x -component of the
momentum of the system is constant during the interval [t, t + Δt] and so
px (t + Δt) − px (t)
0 = lim
Δt→0 Δt

Using the information from the figure above, Equation (12.3.39) becomes
(mb + Δmw ) (v + Δv) − (Δmw u + mb v)
0 = lim
Δt→0 Δt

Equation (12.3.40) simplifies to


Δv Δmw Δmw Δv Δmw
0 = lim mb + lim v + lim − lim u
Δt→0 Δt Δt→0 Δt Δt→0 Δt Δt→0 Δt

The third term vanishes when we take the limit Δt → 0 because it is of second order in the infinitesimal quantities (in this case
Δm Δv ) and when so dividing by Δt the quantity is of first order and hence vanishes since both Δm
w → 0 and Δv → 0 w

Equation (12.3.41) becomes


Δv Δmw Δmw
0 = lim mb + lim v − lim u
Δt→0 Δt Δt→0 Δt Δt→0 Δt

We now use the definition of the derivatives:


Δv dv Δmw dmw
lim = ; lim =
Δt→0 Δt dt Δt→0 Δt dt

in Equation (12.3.42) to fund the differential equation describing the relation between the acceleration of the boat and the time
rate of change of the mass of water entering the boat
dv dmw
0 = mb + (v − u)
dt dt

The mass of the boat is increasing due to the addition of the water. Let m w (t) denote the mass of the water that is in the boat at
time t .Then the mass of the boat can be written as

mb (t) = m0 + mw (t)

where m is the mass of the boat before any water entered. Note we are neglecting the effect of the fire on the mass of the
0

boat. Differentiating Equation (12.3.45) with respect to time yields


dmb dmw
=
dt dt

12.2.7 https://phys.libretexts.org/@go/page/24495
Then Equation (12.3.44) becomes
dv dmb
0 = mb + (v − u)
dt dt

(b) We can integrate this equation through the separation of variable technique. Rewrite Equation (12.3.47) as (cancel the
common factor dt )
dv dmb
=−
v−u mb

We can then integrate both sides of Equation (12.3.48) with the limits as shown
v(t) mb (t)
dv dmb
∫ = −∫
v=0
v−u m0
mb

Integration yields
v(t) − u mb (t)
ln( ) = − ln( )
−u m0

Recall that ln(a/b) = − ln(b/a) so Equation (12.3.50) becomes


v(t) − u m0
ln( ) = ln( )
−u mb (t)

Also recall that exp(ln(a/b)) = a/b and so exponentiating both sides of Equation (12.3.51) yields
v(t) − u m0
=
−u mb (t)

So the speed of the boat at time t can be expressed as


m0
v(t) = u (1 − )
mb (t)

Check result:
We can rewrite Equation (12.3.52) as

mb (t)(v(t) − u) = −m0 u ⇒ mb (t)v(t) = (mb (t) − m0 ) u

Recall that the mass of the water that enters the car during the interval [0,t] is mw (t) = mb (t) − m0 . Therefore Equation
(12.3.54) becomes

mb (t)v(t) = mw (t)u

During the interaction between the jet of water and the boat, the water transfers an amount of momentum m (t)u to the boat w

and car producing a momentum m (t)v(t) . Because all the water that collides with the boat ends up in the boat, all the
b

interaction forces between the jet of water and the boat are internal forces. The boat recoils forward and the water recoils
backward and through collisions with the boat stays in the boat. Therefore if we choose as our system, all of the water that
eventually ends up in the boat and the boat then the momentum principle states

psys (t) = psys (0)

where p sys (0) = mw (t)u is the momentum of all of the water that eventually ends up in the boat.
Note that the problem didn’t ask to find the speed of the boat as a function t . We shall now show how to find that. We begin by
observing that
dmb dmw
= ≠α
dt dt

where the constant α is measured in kg ⋅ s and is specified as a given constant according to the information in the problem
−1

statement. The reason is that α is the rate that the water is ejected from the hose but not the rate that the water enters the boat.

12.2.8 https://phys.libretexts.org/@go/page/24495
Figure 12.12 Mass per unit length of water jet
Consider a small amount of water that is moving with speed u that, in a time interval Δt , flows through a cross sectional area
oriented perpendicular to the flow (see Figure 12.12). The area is larger than the cross sectional area of the jet of water. The
amount of water that floes through the area element Δm = λuΔt where λ is the mass per unit length of the jet and uΔt is the
length of the jet that flows through the area in the interval Δt . The mass rate of water that flows through the cross sectional
area element is then
Δm
α = = λu
Δt

In the Figure 12.13 we consider a small length uΔt of the water jet that is just behind the boat at time t . During the time
interval [t, t + Δt] , the boat moves a distance vΔt .

Figure 12.13 Amount of water that enter boat in time interval [t, t + Δt]
Only a fraction of the length uΔt of water enters the boat and is given by
α
Δmw = λ(u − v)Δt = (u − v)Δt
u

Dividing Equation (12.3.59) through by Δt and taking limits we have that


dmw Δmw α v
= lim = (u − v) = α (1 − )
dt Δt→0 Δt u u

Substituting Equation (12.3.53) and Equation (12.3.46) into Equation (12.3.60) yields
dmb v m0
= α (1 − ) =α
dt u mb (t)

We can integrate this equation by separating variables to find an integral expression for the mass of the boat as a function of
time
m0 (t) t

∫ mb d mb = α m0 ∫ dt
m0 t=0

12.2.9 https://phys.libretexts.org/@go/page/24495
We can easily integrate both sides of Equation (12.3.62) yielding
1 2 2
(mb (t) − m ) = α mb,0 t
0
2

The mass of the boat as a function of time is then


−−−−−−−
αt
mb (t) = m0 √ 1 + 2
m0

We now substitute Equation (12.3.64) into Equation (12.3.65)yielding the speed of the burning boat as a function of time

⎛ ⎞
1
v(t) = u ⎜1 − −−−−−−−⎟
αt
⎝ √1 + 2 ⎠
mb,0

This page titled 12.2: Worked Examples is shared under a CC BY-NC-SA 4.0 license and was authored, remixed, and/or curated by Peter
Dourmashkin (MIT OpenCourseWare) via source content that was edited to the style and standards of the LibreTexts platform; a detailed edit
history is available upon request.

12.2.10 https://phys.libretexts.org/@go/page/24495
12.3: Rocket Propulsion

A rocket at time t = ti is moving with velocity V r,i with respect to a fixed reference frame. During the time interval [ ti , tf ] the

rocket continuously burns fuel that is continuously ejected backwards with velocity u relative to the rocket. This exhaust velocity
is independent of the velocity of the rocket. The rocket must exert a force to accelerate the ejected fuel backwards and therefore by
Newton’s Third law, the fuel exerts a force that is equal in magnitude but opposite in direction accelerating the rocket forward. The

rocket velocity is a function of time, v r (t) . Because fuel is leaving the rocket, the mass of the rocket is also a function of time,

mr (t) , and is decreasing at a rate dmr /dt . Let F ext denote the total external force acting on the rocket. We shall use the
→ → → →
momentum principle, to determine a differential equation that relates d v r /dt , dmr /dt , u , v r (t),  and  F ext , an equation
known as the rocket equation.
We shall apply the momentum principle during the time interval [t, t + Δt] with Δt taken to be a small interval (we shall
eventually consider the limit that Δt → 0 ), and t < t < t During this interval, choose as our system the mass of the rocket at
i f

time t ,

msys = mr (t) = mr,d + mf (t)

where mr,d is the dry mass of the rocket and mf (t) is the mass of the fuel in the rocket at time t . During the time interval

[t, t + Δt], a small amount of fuel of mass Δm (in the limit that Δt → 0, Δm → 0 is ejected backwards with velocity u to the
f f

rocket. Before the fuel is ejected, it is traveling at the velocity of the rocket and so during the time interval [t, t + Δt] , the elected

fuel undergoes a change in momentum and the rocket recoils forward. At time t + Δt the rocket has velocity v (t + Δt) . r

Although the ejected fuel continually changes its velocity, we shall assume that the fuel is all ejected at the instant t + Δt and then
consider the limit as Δt → 0 . Therefore the velocity of the ejected fuel with respect to the fixed reference frame is the vector sum
→ →
of the relative velocity of the fuel with respect to the rocket and the velocity of the rocket, u + v (t + Δt) Figure 12.14 r

represents momentum diagrams for our system at time t and t + Δt relative to a fixed inertial reference frame in which velocity of

the rocket at time t is v r (t) .

Figure 12.14 Momentum diagrams for system at time t and t + Δt


The momentum of the system at time t is
→ →
p sys (t) = mr (t) v r (t)

Note that the mass of the system at time t is

msys = mr (t)

The momentum of the system at time t + Δt is


→ → → →
p sys (t + Δt) = mr (t + Δt) v r (t + Δt) + Δmf ( u + v r (t + Δt))

where m r (t + Δt) = mr (t) + Δmr . With this notation the mass of the system at time t + Δt is given by

12.3.1 https://phys.libretexts.org/@go/page/24496
msys = mr (t + Δt) + Δmf = mr (t) + Δmr + Δmf

Because the mass of the system is constant, setting Equation (12.3.69) equal to Equation (12.3.71) requires that

Δmr = −Δmf

The momentum of the system at time t + Δt (Equation (12.3.70)) can be rewritten as


→ → → →
p sys (t + Δt) = (mr (t) + Δmr v r (t + Δt) − Δmr ( u + v r (t + Δt))

→ → →
p sys (t + Δt) = mr (t) v r (t + Δt) − Δmr u

We can now apply Newton’s Second Law in the form of the momentum principle,
→ → →
→ ( mr (t) v r (t+Δt)−Δ mr u )−mr (t) v r (t)

F ext = limΔt→0
Δt

→ →
v r (t+Δt)− v r (t) Δmr →
= mr (t) limΔt→0 − limΔt→0 u
Δt Δt

We now take the limit as



→ d vr dmr →
F ext = mr (t) − u
dt dt

Equation (12.3.75) is known as the rocket equation.


→ →
Suppose the rocket is moving in the positive x -direction with an external force given by ^
F ext = Fext,x i Then ^
u = −u i where u

> 0 is the relative speed of the fuel and it is moving in the negative x -direction, ^
v r = vr,x i Then the rocket equation (Equation
(12.3.75)) becomes
dvr,x dmr
Fext,x = mr (t) + u
dt dt

Note that the rate of decrease of the mass of the rocket, dm r /dt , is equal to the negative of the rate of increase of the exhaust fuel

dmr dmf
=−
dt dt

We can rewrite Equation (12.3.76) as


dmr dvr,x
Fext,x − u = mr (t)
dt dt

The second term on the left-hand-side of Equation (12.3.78) is called the thrust
dmr dmf
Fthrust,x = − u = u
dt dt

Note that this is not an extra force but the result of the forward recoil due to the ejection of the fuel. Because we are burning fuel at
a positive rate dm /dt > 0 and the speed u > 0 , the direction of the thrust is in the positive x -direction.
f

Rocket Equation in Gravity-free Space


We shall first consider the case in which there are no external forces acting on the system, then Equation (12.3.78) becomes
dmr dvr,x
− u = mr (t)
dt dt

In order to solve this equation, we separate the variable quantities v r,x (t) and m r (t) and multiply both sides by dt yielding
dmr
dvr,x = −u
mr (t)

We now integrate both sides of Equation (12.3.81) with limits corresponding to the values of the x -component of the velocity and
mass of the rocket at times t when the ejection of the burned fuel began and the time t when the process stopped,
i f

12.3.2 https://phys.libretexts.org/@go/page/24496
′ ′
vr,x =vr,x,f mr =mr,f
u
′ ′
∫ dvr,x = − ∫ d mr


vr,x =vr,x,i

mr =mr,i
mr

Performing the integration and substituting in the values at the endpoints yields
mr,f
vr,x,f − vr,x,i = −u ln( )
mr,i

Because the rocket is losing fuel, m r,f < mr,i , we can rewrite Equation (12.3.83) as
mr,i
vr,x,f − vr,x,i = u ln( )
mr,f

We note ln(m /m ) > 1 . Therefore v


r,i r,f >v
r,x,f as we expect. After a slight rearrangement of Equation (12.3.84), we have an
r,x,i

expression for the x -component of the velocity of the rocket as a function of the mass m of the rocket r

mr,i
vr,x,f = vr,x,i + u ln( )
mr,f

Let’s examine our result. First, let’s suppose that all the fuel was burned and ejected. Then m r,f ≡ mr,d is the final dry mass of the
rocket (empty of fuel). The ratio
mr,i
R =
mr,d

is the ratio of the initial mass of the rocket (including the mass of the fuel) to the final dry mass of the rocket (empty of fuel). The
final velocity of the rocket is then

vr,x,f = vr,x,i + u ln R

This is why multistage rockets are used. You need a big container to store the fuel. Once all the fuel is burned in the first stage, the
stage is disconnected from the rocket. During the next stage the dry mass of the rocket is much less and so R is larger than the
single stage, so the next burn stage will produce a larger final speed then if the same amount of fuel were burned with just one stage
(more dry mass of the rocket). In general rockets do not burn fuel at a constant rate but if we assume that the burning rate is
constant where
dmf dmr
b = =−
dt dt

then we can integrate Equation (12.3.88)


′ ′
mr =mr (t) t =t

′ ′
∫ dmr = −b ∫ dt
′ ′
m =mr,i t =ti

and find an equation that describes how the mass of the rocket changes in time

mr (t) = mr,i − b (t − ti )

For this special case, if we set t


f =t in Equation (12.3.85), then the velocity of the rocket as a function of time is given by
mr,i
vr,x,f = vr,x,i + u ln( )
mr,i − bt

Example 12.3.1: Single-Stage Rocket

Before a rocket begins to burn fuel, the rocket has a mass of m = 2.81 × 10 kg , of which the mass of the fuel is
r,i
7

= 2.46 × 10 kg . The fuel is burned at a constant rate with total burn time is 510 s and ejected at a speed u = 3000 m/s
7
m f ,i

relative to the rocket. If the rocket starts from rest in empty space, what is the final speed of the rocket after all the fuel has
been burned?
Solution

12.3.3 https://phys.libretexts.org/@go/page/24496
The dry mass of the rocket is m r,d ≡ mr,i − mf ,i = 0.35 × 10 kg
7
, hence R = mr,i / mr,d = 8.03 . The final speed of the
rocket after all the fuel has burned is

vr,f = Δvr = u ln R = 6250m/s

Example 12.3.2: Two-Stage Rocket

Now suppose that the same rocket in Example 12.4 burns the fuel in two stages ejecting the fuel in each stage at the same
relative speed. In stage one, the available fuel to burn is m = 2.03 × 10 kg with burn time 150 s . Then the empty fuel
f ,1,i
7

tank and accessories from stage one are disconnected from the rest of the rocket. These disconnected parts have a mass
m = 1.4 × 10 kg All the remaining fuel with mass is burned during the second stage with burn time of 360 s . What is the
6

final speed of the rocket after all the fuel has been burned?
Solution
The mass of the rocket after all the fuel in the first stage is burned is m r,1,d = mr,1,i − mf ,1,i = 0.78 × 10 kg
7
and
R =m
1 /m
r,1,i = 3.60 . The change in speed after the first stage is complete is
r,1,d

Δvr,1 = u ln R1 = 3840m/s

After the empty fuel tank and accessories from stage one are disconnected from the rest of the rocket, the remaining mass of
the rocket is m = 2.1 × 10 kg . The remaining fuel has mass m
r,2,d
6
= 4.3 × 10 kg . The mass of the rocket plus the
f ,2,i
6

unburned fuel at the beginning of the second stage is m = 6.4 × 10 kg . Then R = m


r,2,i
6
/m = 3.05 Therefore the
2 r,2,i r,2,d

rocket increases its speed during the second stage by an amount

Δvr,2 = u ln R2 = 3340m/s

The final speed of the rocket is the sum of the change in speeds due to each stage,

vf = Δvr = u ln R1 + u ln R2 = u ln(R1 R2 ) = 7190m/s

which is greater than if the fuel were burned in one stage. Plots of the speed of the rocket as a function time for both one-stage
and two-stage burns are shown Figure 12.15.

Figure 12.15 Plots of speed of rocket for both one-stage burn and two-stage burn

Rocket in a Constant Gravitational Field:


Now suppose that the rocket takes off from rest at time t = 0 in a constant gravitational field then the external force is
total 
→ →
F ext = mr g

12.3.4 https://phys.libretexts.org/@go/page/24496
Choose the positive x -axis in the upward direction then Fex,x (t) = −mr (t)g . Then the rocket equation (Equation (12.3.75)
becomes
dmr dvr,x
−mr (t)g − u = mr (t)
dt dt

Multiply both sides of Equation (12.3.97) by dt , and divide both sides by m r (t) . Then Equation (12.3.97) can be written as
dmr
dvr,x = −gdt − u
mr (t)

We now integrate both sides


vr,x (t) mr (t) ′ t
dmr ′

∫ dvr,x = −u ∫ −g∫ dt

vr,x, =0 mr,j
mr 0

where m r,i is the initial mass of the rocket and the fuel. Integration yields
mr (t) mr,i
vr,x (t) = −u ln( ) − gt = u ln( ) − gt
mr,i mr (t)

After all the fuel is burned at t = t , the mass of the rocket is equal to the dry mass m
f r,f = mr,d and so

vr,x (tf ) = u ln R − gtf

The first term on the right hand side is independent of the burn time. However the second term depends on the burn time. The
shorter the burn time, the smaller the negative contribution from the third turn, and hence the rocket ends up with a larger final
speed. So the rocket engine should burn the fuel as fast as possible in order to obtain the maximum possible speed.

This page titled 12.3: Rocket Propulsion is shared under a CC BY-NC-SA 4.0 license and was authored, remixed, and/or curated by Peter
Dourmashkin (MIT OpenCourseWare) via source content that was edited to the style and standards of the LibreTexts platform; a detailed edit
history is available upon request.

12.3.5 https://phys.libretexts.org/@go/page/24496
CHAPTER OVERVIEW
13: Energy, Kinetic Energy, and Work
13.1: The Concept of Energy and Conservation of Energy
13.2: Kinetic Energy
13.3: Kinematics and Kinetic Energy in One Dimension
13.4: Work done by Constant Forces
13.5: Work done by Non-Constant Forces
13.6: Work-Kinetic Energy Theorem
13.7: Power Applied by a Constant Force
13.8: Work and the Scalar Product
13.9: Work done by a Non-Constant Force Along an Arbitrary Path
13.10: Worked Examples
13.11: Work-Kinetic Energy Theorem in Three Dimensions
13.12: Appendix 13A Work Done on a System of Two Particles

This page titled 13: Energy, Kinetic Energy, and Work is shared under a CC BY-NC-SA 4.0 license and was authored, remixed, and/or curated by
Peter Dourmashkin (MIT OpenCourseWare) via source content that was edited to the style and standards of the LibreTexts platform; a detailed
edit history is available upon request.

1
13.1: The Concept of Energy and Conservation of Energy
Acceleration of the expansion of the universe is one of the most exciting and significant discoveries in physics, with implications
that could revolutionize theories of quantum physics, gravitation, and cosmology. With its revelation that close to the three-quarters
of the energy density of the universe, given the name dark energy, is of a new, unknown origin and that its exotic gravitational
“repulsion” will govern the fate of the universe, dark energy and the accelerating universe becomes a topic not just of great
interest to research physicists but to science students at all levels.
Eric Linder
The transformation of energy is a powerful concept that enables us to describe a vast number of processes:
Falling water releases stored gravitational potential energy, which can become the kinetic energy associated with a coherent motion
of matter. The harnessed mechanical energy can be used to spin turbines and alternators, doing work to generate electrical energy,
transmitted to consumers along power lines. When you use any electrical device, the electrical energy is transformed into other
forms of energy. In a refrigerator, electrical energy is used to compress a gas into a liquid. During the compression, some of the
internal energy of the gas is transferred to the random motion of molecules in the outside environment. The liquid flows from a
highpressure region into a low-pressure region where the liquid evaporates. During the evaporation, the liquid absorbs energy from
the random motion of molecules inside of the refrigerator. The gas returns to the compressor.
“Human beings transform the stored chemical energy of food into various forms necessary for the maintenance of the functions of
the various organ system, tissues and cells in the body.” A person can do work on their surroundings – for example, by pedaling a
bicycle – and transfer energy to the surroundings in the form of increasing random motion of air molecules, by using this catabolic
energy.
Burning gasoline in car engines converts chemical energy, stored in the molecular bonds of the constituent molecules of gasoline,
into coherent (ordered) motion of the molecules that constitute a piston. With the use of gearing and tire/road friction, this motion is
converted into kinetic energy of the car; the automobile moves.
Stretching or compressing a spring stores elastic potential energy that can be released as kinetic energy.
The process of vision begins with stored atomic energy released as electromagnetic radiation (light), which is detected by exciting
photoreceptors in the eye, releasing chemical energy.
When a proton fuses with deuterium (a hydrogen atom with a neutron and proton for a nucleus), helium-three is formed (with a
nucleus of two protons and one neutron) along with radiant energy in the form of photons. The combined internal energy of the
proton and deuterium are greater than the internal energy of the helium-three. This difference in internal energy is carried away by
the photons as light energy.
There are many such processes involving different forms of energy: kinetic energy, gravitational energy, thermal energy, elastic
energy, electrical energy, chemical energy, electromagnetic energy, nuclear energy and more. The total energy is always conserved
in these processes, although different forms of energy are converted into others.
Any physical process can be characterized by two states, initial and final, between which energy transformations can occur. Each
form of energy E , where “ j ” is an arbitrary label identifying one of the N forms of energy, may undergo a change during this
j

transformation,
ΔEj ≡  Ef inal,j − Einitial,j

Conservation of energy means that the sum of these changes is zero,


N

ΔE1 + ΔE2 + ⋯ + ΔEN = ∑ ΔEj = 0

j=1

Two important points emerge from this idea. First, we are interested primarily in changes in energy and so we search for relations
that describe how each form of energy changes. Second, we must account for all the ways energy can change. If we observe a
process, and the sum of the changes in energy is not zero, either our expressions for energy are incorrect, or there is a new type of
change of energy that we had not previously discovered. This is our first example of the importance of conservation laws in
describing physical processes, as energy is a key quantity conserved in all physical processes. If we can quantify the changes of
different forms of energy, we have a very powerful tool to understand nature.

13.1.1 https://phys.libretexts.org/@go/page/24501
We will begin our analysis of conservation of energy by considering processes involving only a few forms of changing energy. We
will make assumptions that greatly simplify our description of these processes. At first we shall only consider processes acting on
bodies in which the atoms move in a coherent fashion, ignoring processes in which energy is transferred into the random motion of
atoms. Thus we will initially ignore the effects of friction. We shall then treat processes involving friction between consider rigid
bodies. We will later return to processes in which there is an energy transfer resulting in an increase or decrease in random motion
when we study the First Law of Thermodynamics.
Energy is always conserved but we often prefer to restrict our attention to a set of objects that we define to be our system. The rest
of the universe acts as the surroundings. We illustrate this division of system and surroundings in Figure 13.1.

Figure 13.1 A diagram of a system and its surroundings with boundary


Because energy is conserved, any energy that leaves the system must cross through the boundary and enter the surroundings.
Consider any physical process in which energy transformations occur between initial and final states. We assert that
when a system and its surroundings undergo a transition from an initial state to a final state, the change in energy is zero,
ΔE = ΔEsystem + ΔEsurrounding = 0

Equation (13.1.3) is called conservation of energy and is our operating definition for energy. We will sometime refer to Equation
(13.1.3) as the energy principle. In any physical application, we first identify our system and surroundings, and then attempt to
quantify changes in energy. In order to do this, we need to identify every type of change of energy in every possible physical
process. When there is no change in energy in the surroundings then the system is called a closed system, and consequently the
energy of a closed system is constant.
ΔEsystem  = 0 (closed system) .
If we add up all known changes in energy in the system and surroundings and do not arrive at a zero sum, we have an open
scientific problem. By searching for the missing changes in energy, we may uncover some new physical phenomenon. Recently,
one of the most exciting open problems in cosmology is the apparent acceleration of the expansion of the universe, which has been
attributed to dark energy that resides in space itself, an energy type without a clearly known source.

This page titled 13.1: The Concept of Energy and Conservation of Energy is shared under a CC BY-NC-SA 4.0 license and was authored,
remixed, and/or curated by Peter Dourmashkin (MIT OpenCourseWare) via source content that was edited to the style and standards of the
LibreTexts platform; a detailed edit history is available upon request.

13.1.2 https://phys.libretexts.org/@go/page/24501
13.2: Kinetic Energy
The first form of energy that we will study is an energy associated with the coherent motion of molecules that constitute a body of
mass m; this energy is called the kinetic energy (from the Greek word kinetikos which translates as moving). Let us consider a car
moving along a straight road (along which we will place the x -axis). For an observer at rest with respect to the ground, the car has

velocity ^
v = vx i The speed of the car is the magnitude of the velocity, v ≡ |v x|

The kinetic energy K of a non-rotating body of mass m moving with speed v is defined to be the positive scalar quantity.
1 2
K ≡ mv
2

The kinetic energy is proportional to the square of the speed. The SI units for kinetic energy are [kg ⋅ m ⋅ s ] . This combination
2 −2

of units is defined to be a joule and is denoted by [J], thus IJ ≡ 1kg ⋅ m ⋅ s (The SI unit of energy is named for James Prescott
2 −2

Joule.) The above definition of kinetic energy does not refer to any direction of motion, just the speed of the body.

Let’s consider a case in which our car changes velocity. For our initial state, the car moves with an initial velocity v = v ^i i x,i

along the x -axis. For the final state (at some later time), the car has changed its velocity and now moves with a final velocity

^
v f = vx,f i . Therefore the change in the kinetic energy is
Example 13.1 Change in Kinetic Energy of a Car
Suppose car A increases its speed from 10 to 20 mph and car B increases its speed from 50 to 60 mph. Both cars have the same
mass m. (a) What is the ratio of the change of kinetic energy of car B to the change of kinetic energy of car A? In particular, which
car has a greater change in kinetic energy? (b) What is the ratio of the change in kinetic energy of car B to car A as seen by an
observer moving with the initial velocity of car A?
Solution: (a) The ratio of the change in kinetic energy of car B to car A is
1 2 1 2 2 2
ΔKB m (vB,f ) − m (vB,i ) (vB,f ) − (vB,i )
2 2
= =
1 2 1 2 2 2
ΔKA m (vA,f ) − m (vA,i ) (vA,f ) − (vA,i )
2 2

2 2
(60mph) − (50mph)
= = 11/3
2 2
(20mph) − (10mph)

Thus car B has a much greater increase in its kinetic energy than car A.
(b) In a reference moving with the speed of car A , car A increases its speed from rest to 10 mph and car B increases its speed from
40 to 50 mph. The ratio is now
1 2 1 2 2 2
ΔKB m (vB,f ) − m (vB,0 ) (vB,f ) − (vB,0 )
2 2
= =
1 2 1 2 2 2
ΔKA m (vA,f ) − m (vA,0 ) (vA,f ) − (vA,0 )
2 2

2 2
(50mph) − (40mph)
= =9
2
(10mph)

The ratio is greater than that found in part a). Note that from the new reference frame both car A and car B have smaller increases
in kinetic energy.

This page titled 13.2: Kinetic Energy is shared under a CC BY-NC-SA 4.0 license and was authored, remixed, and/or curated by Peter
Dourmashkin (MIT OpenCourseWare) via source content that was edited to the style and standards of the LibreTexts platform; a detailed edit
history is available upon request.

13.2.1 https://phys.libretexts.org/@go/page/24502
13.3: Kinematics and Kinetic Energy in One Dimension
Constant Accelerated Motion
Let’s consider a constant accelerated motion of a rigid body in one dimension in which we treat the rigid body as a point mass.
Suppose at t = 0 the body has an initial x - component of the velocity given by V . If the acceleration is in the direction of the
x,i

displacement of the body then the body will increase its speed. If the acceleration is opposite the direction of the displacement then
the acceleration will decrease the body’s speed. The displacement of the body is given by
1
2
Δx = vx,i t + ax t
2

The product of acceleration and the displacement is


1
2
ax Δx = ax ( vx,i t + ax t )
2

The acceleration is given by

Δvx (vx,f − vx,i )


ax = =
Δt t

Therefore
(vx,f − vx,i ) 1 (vx,f − vx,i ) 2
ax Δx = ( vx,i t + t )
t 2 t

Equation (13.3.4) becomes


1 1 1
2 2
ax Δx = (vx,f − vx,i ) (vx,i ) + (vx,f − vx,i ) (vx,f − vx,i ) = v − v
x,f x,i
2 2 2

If we multiply each side of Equation (13.3.5) by the mass m of the object this kinematical result takes on an interesting
interpretation for the motion of the object. We have
1 1
2 2
m ax Δx = mv −m v = Kf − Ki
x,f x,i
2 2

Recall that for one-dimensional motion, Newton’s Second Law is Fx = m ax , for the motion considered here, Equation (13.3.6)
becomes

Fx Δx = Kf − Ki

Non-constant Accelerated Motion


If the acceleration is not constant, then we can divide the displacement into N intervals indexed by j = 1 to N . It will be convenient
to denote the displacement intervals by Δx the corresponding time intervals by Δt and the x -components of the velocities at the
j j

beginning and end of each interval as V x,j−1 and V . Note that the x -component of the velocity at the beginning and end of the
x,j

first interval j =1is then v = v and the velocity at the end of the last interval, j = N is v
x,1 x,i =v . Consider the sum of the , x,N x,j

products of the average acceleration (a ) and displacement Δx in each interval,


x,j
ave
j

j=N

∑ (ax,j ) Δxj
ave 

j=1

The average acceleration over each interval is equal to


Δvx,j (vx,j+1 − vx,j )
(ax,j ) = =
ave
Δtj Δtj

and so the contribution in each integral can be calculated as above and we have that
1 1
2 2
(ax,j ) Δxj = v − v
ave  x,j x,j−1
2 2

13.3.1 https://phys.libretexts.org/@go/page/24503
When we sum over all the terms only the last and first terms survive, all the other terms cancel in pairs, and we have that
j=N
1 1
2 2
∑ (ax,j ) Δxj = v − v
ave  x,f x,i
2 2
j=1

In the limit as N → ∞ and Δx → 0 for all j (both conditions must be met!), the limit of the sum is the definition of the definite
j

integral of the acceleration with respect to the position,


j=N x=xf

lim ∑ (ax,j ) Δxj ≡ ∫ ax (x)dx


N→ ∞
ave 
x=xi
Δ x →0
j
j=1

Therefore In the limit as N → ∞ and Δx j → 0 for all j , with v x,N → vx,f Equation (13.3.11) becomes
x=xf
1
2 2
∫ ax (x)dx = (v −v )
x,f x,i
x=xi
2

This integral result is consequence of the definition that a x ≡ dvx /dt . The integral in Equation (13.3.13) is an integral with respect
to space, while our previous integral
t=tf

∫ ax (t)dt = vx,f − vx,i


t=ti

requires integrating acceleration with respect to time. Multiplying both sides of Equation (13.3.13) by the mass m yields
x=xf
1
2 2
∫ m ax (x)dx = m (v −v ) = Kf − Ki
x,f x,i
x=xi
2

When we introduce Newton’s Second Law in the form F x = m ax , then Equation (13.3.15) becomes
x=xf

∫ Fx (x)dx = Kf − Ki
x=xi

The integral of the x -component of the force with respect to displacement in Equation (13.3.16) applies to the motion of a point-
like object. For extended bodies, Equation (13.3.16) applies to the center of mass motion because the external force on a rigid body
causes the center of mass to accelerate.

This page titled 13.3: Kinematics and Kinetic Energy in One Dimension is shared under a CC BY-NC-SA 4.0 license and was authored, remixed,
and/or curated by Peter Dourmashkin (MIT OpenCourseWare) via source content that was edited to the style and standards of the LibreTexts
platform; a detailed edit history is available upon request.

13.3.2 https://phys.libretexts.org/@go/page/24503
13.4: Work done by Constant Forces
We will begin our discussion of the concept of work by analyzing the motion of an object in one dimension acted on by constant
forces. Let’s consider the following example: push a cup forward with a constant force along a desktop. When the cup changes
velocity (and hence kinetic energy), the sum of the forces acting on the cup must be non-zero according to Newton’s Second Law.
a
→ → → →
There are three forces involved in this motion: the applied pushing force F ; the contact force C ≡ N+ f k ; and gravity
→g →
F =m g . The force diagram on the cup is shown in Figure 13.2.

Figure 13.2 Force diagram for cup.


Let’s choose our coordinate system so that the +x -direction is the direction of the forward motion of the cup. The pushing force
can then be described by
→a
a^
F = Fx i

Suppose a body moves from an initial point xi to a final point xf so that the displacement of the point the force acts on is
→a
Δx ≡ xf − xi . The work done by a constant force F
a^
= Fx i acting on the body is the product of the component of the force F a
x

and the displacement Δx ,


a a
W = Fx Δx

Work is a scalar quantity; it is not a vector quantity. The SI unit for work is
−2 2 −2
[1N ⋅ m] = [1kg ⋅ m ⋅ s ] [1m] = [1kg ⋅ m ⋅s ] = [1J]

Note that work has the same dimension and the same SI unit as kinetic energy. Because our applied force is along the direction of
motion, both F > 0 and Δx > 0 In this example, the work done is just the product of the magnitude of the applied force and the
a
x

distance through which that force acts and is positive. In the definition of work done by a force, the force can act at any point on the
body. The displacement that appears in Equation (13.4.2) is not the displacement of the body but the displacement of the point of
application of the force. For point-like objects, the displacement of the point of application of the force is equal to the displacement
of the body. However for an extended body, we need to focus on where the force acts and whether or not that point of application
undergoes any displacement in the direction of the force as the following example illustrates.
Example 13.2 Work Done by Static Fiction
Suppose you are initially standing and you start walking by pushing against the ground with your feet and your feet do not slip.
What is the work done by the static friction force acting on you?
Solution: When you apply a contact force against the ground, the ground applies an equal and opposite contact force on you. The
tangential component of this constant force is the force of static friction acting on you. Since your foot is at rest while you are
pushing against the ground, there is no displacement of the point of application of this static friction force. Therefore static friction
does zero work on you while you are accelerating. You may be surprised by this result but if you think about energy transformation,
chemical energy stored in your muscle cells is being transformed into kinetic energy of motion and thermal energy.
When forces are opposing the motion, as in our example of pushing the cup, the kinetic friction force is given by

13.4.1 https://phys.libretexts.org/@go/page/24504
f

^ ^ ^
F = fk,x i = −μk N i = −μk mg i

Here the component of the force is in the opposite direction as the displacement. The work done by the kinetic friction force is
negative,
f
W = −μk mgΔx

Since the gravitation force is perpendicular to the motion of the cup, the gravitational force has no component along the line of
motion. Therefore the gravitation force does zero work on the cup when the cup is slid forward in the horizontal direction. The
normal force is also perpendicular to the motion, and hence does no work.
We see that the pushing force does positive work, the kinetic friction force does negative work, and the gravitation and normal
force does zero work.
Example 13.3 Work Done by Force Applied in the Direction of Displacement
Push a cup of mass 0.2 kg along a horizontal table with a force of magnitude 2.0 N for a distance of 0.5 m. The coefficient of
friction between the table and the cup is μ = 0.10 Calculate the work done by the pushing force and the work done by the friction
k

force.
Solution: The work done by the pushing force is
a a
W = Fx Δx = (2.0N)(0.5m) = 1.0J

The work done by the friction force is


f −2
W = −μk mgΔx = −(0.1)(0.2kg) (9.8m ⋅ s ) (0.5m) = −0.10J

Example 13.4 Work Done by Force Applied at an Angle to the Direction of Displacement
Suppose we push the cup in the previous example with a force of the same magnitude but at an angle θ = 30 upwards with ∘

respect to the table. Calculate the work done by the pushing force. Calculate the work done by the kinetic friction force.
Solution: The force diagram on the cup and coordinate system is shown in Figure 13.3.

Figure 13.3 Force diagram on cup.


The x -component of the pushing force is now
a a ∘
Fx = F cos(θ) = (2.0N) (cos(30 )) = 1.7N

The work done by the pushing force is


a a −1
W = Fx Δx = (1.7N)(0.5m) = 8.7 × 10 J

The kinetic friction force is


→f
^
F = −μk N i

In this case, the magnitude of the normal force is not simply the same as the weight of the cup. We need to find the y -component of
the applied force,

13.4.2 https://phys.libretexts.org/@go/page/24504
a a ∘
Fy =F sin(θ) = (2.0N) (sin(30 ) = 1.0N

To find the normal force, we apply Newton’s Second Law in the y -direction,
a
Fy + N − mg = 0

Then the normal force is


a −2 −1
N = mg − Fy = (0.2kg) (9.8m ⋅ s ) − (1.0N) = 9.6 × 10 N

The work done by the kinetic friction force is


f −1 −2
W = −μk N Δx = −(0.1) (9.6 × 10 N) (0.5m) = 4.8 × 10 J

Example 13.5 Work done by Gravity Near the Surface of the Earth
Consider a point-like body of mass m near the surface of the earth falling directly towards the center of the earth. The gravitation
→ →
force between the body and the earth is nearly constant, F = m g . Let’s choose a coordinate system with the origin at the
grav

surface of the earth and the + y -direction pointing away from the center of the earth Suppose the body starts from an initial point
y and falls to a final point y closer to the earth. How much work does the gravitation force do on the body as it falls?
i f

Solution: The displacement of the body is negative, Δy ≡ y f − yi < 0 . The gravitation force is given by
→g → g^ ^
F = m g = Fy j = −mg j

The work done on the body is then


g g
W = Fy Δy = −mgΔy

For a falling body, the displacement of the body is negative, Δy ≡ y − y < 0 ; therefore the work done by gravity is positive,
f i

> 0 . The gravitation force is pointing in the same direction as the displacement of the falling object so the work should be
g
W

positive.
When an object is rising while under the influence of a gravitation force, Δy ≡ y − y > 0 . The work done by the gravitation
f i

force for a rising body is negative, W < 0 , because the gravitation force is pointing in the opposite direction from that in which
g

the object is displaced.


It’s important to note that the choice of the positive direction as being away from the center of the earth (“up”) does not make a
difference. If the downward direction were chosen positive, the falling body would have a positive displacement and the
g
gravitational force as given in Equation (13.4.15) would have a positive downward component; the product F Δy would still be y

positive.

This page titled 13.4: Work done by Constant Forces is shared under a CC BY-NC-SA 4.0 license and was authored, remixed, and/or curated by
Peter Dourmashkin (MIT OpenCourseWare) via source content that was edited to the style and standards of the LibreTexts platform; a detailed
edit history is available upon request.

13.4.3 https://phys.libretexts.org/@go/page/24504
13.5: Work done by Non-Constant Forces

Consider a body moving in the x -direction under the influence of a non-constant force in the x -direction, F = F ^i . The body x

moves from an initial position x to a final position x . In order to calculate the work done by a non-constant force, we will divide
i f

up the displacement of the point of application of the force into a large number N of small displacements Δx where the index j j

marks the j displacement and takes integer values from 1 to N . Let (F )


th denote the average value of the x -component of
x,j
ave 

the force in the displacement interval [x , x ]. For the j displacement interval we calculate the contribution to the work.
j−1 j th

Wj = (Fx,j ) Δxj
ave 

This contribution is a scalar so we add up these scalar quantities to get the total work
j=N j=N

WN = ∑ Wj = ∑ (Fx,j ) Δxj
ave 

j=1 j=1

The sum in Equation (13.5.2) depends on the number of divisions N and the width of the intervals Δx . In order to define a j

quantity that is independent of the divisions, we take the limit as N → ∞ and |Δx | → 0 for all j . The work is then
j

j=N x=xf

W = lim ∑ (Fx,j ) Δxj = ∫ Fx (x)dx


ave 
N→ ∞

∣ ∣ j=1 x=xi
Δx →0
∣ j∣

This last expression is the definite integral of the x -component of the force with respect to the parameter x . In Figure 13.5 we
graph the x -component of the force as a function of the parameter x . The work integral is the area under this curve between
x = x and x = x .
i f

Figure 13.5 Plot of x -component of a sample force F x (x) as a function of x .

Example 13.5.1: Work done by the Spring Force

Connect one end of an unstretched spring of length l with spring constant k to an object resting on a smooth frictionless table
0

and fix the other end of the spring to a wall. Choose an origin as shown in the figure. Stretch the spring by an amount x and i

release the object. How much work does the spring do on the object when the spring is stretched by an amount x ? f

Figure 13.6 Equilibrium, initial and final states for a spring


Solution
We first begin by choosing a coordinate system with our origin located at the position of the object when the spring is
unstretched (or uncompressed). We choose the ^i unit vector to point in the direction the object moves when the spring is being

13.5.1 https://phys.libretexts.org/@go/page/24505
stretched. We choose the coordinate function x to denote the position of the object with respect to the origin. We show the
coordinate function and free-body force diagram in the figure below.

Figure 13.6a Spring force


The spring force on the object is given by (Figure 13.6a)

^ ^
F = Fx i = −kx i

In Figure 13.7 we show the graph of the x -component of the spring force, F x (x) , as a function of x

Figure 13.7 Plot of spring force F x (x) vs. displacement x


The work done is just the area under the curve for the interval x to x , i f

′ ′
x =xf x =xf
′ ′ ′ ′
1
2 2
W =∫ Fx (x ) dx = ∫ −kx dx = − k (x −x )
f i

x =xi

x =xi
2

This result is independent of the sign of x and x because both quantities appear as squares. If the spring is less stretched or
i f

compressed in the final state than in the initial state, then the absolute value, |x | < |x | and the work done by the spring force
f i

is positive. The spring force does positive work on the body when the spring goes from a state of “greater tension” to a state of
“lesser tension.”

This page titled 13.5: Work done by Non-Constant Forces is shared under a CC BY-NC-SA 4.0 license and was authored, remixed, and/or curated
by Peter Dourmashkin (MIT OpenCourseWare) via source content that was edited to the style and standards of the LibreTexts platform; a detailed
edit history is available upon request.

13.5.2 https://phys.libretexts.org/@go/page/24505
13.6: Work-Kinetic Energy Theorem
There is a direct connection between the work done on a point-like object and the change in kinetic energy the point-like object
undergoes. If the work done on the object is nonzero, this implies that an unbalanced force has acted on the object, and the object
will have undergone acceleration. For an object undergoing one-dimensional motion the left hand side of Equation (13.3.16) is the
work done on the object by the component of the sum of the forces in the direction of displacement,
x=xf
1 1
2 2
W =∫ Fx dx = mv − mv = Kf − Ki = ΔK
f i
x=xi
2 2

When the work done on an object is positive, the object will increase its speed, and negative work done on an object causes a
decrease in speed. When the work done is zero, the object will maintain a constant speed. In fact, the work-energy relationship is
quite precise; the work done by the applied force on an object is identically equal to the change in kinetic energy of the object.

Example 13.6.1: Gravity and the Work-Energy Theorem

Suppose a ball of mass m = 0.2kg starts from rest at a height y = 15m above the surface of the earth and falls down to a
0

height y = 5.0m above the surface of the earth. What is the change in the kinetic energy? Find the final velocity using the
f

work-energy theorem.
Solution
As only one force acts on the ball, the change in kinetic energy is the work done by gravity,
g
W = −mg (yf − y0 )

−1 −2 1
= (−2.0 × 10 kg) (9.8m ⋅ s ) (5m − 15m) = 2.0 × 10 J

The ball started from rest, vy,0 =0 . So the change in kinetic energy is
1 1 1
2 2 2
ΔK = mv − mv = mv
y,f y,0 y,f
2 2 2

We can solve Equation (13.6.3) for the final velocity using Equation (13.6.2)
−−−−−−−−−−− −
−−−−− −−−−− 1
g 2 (2.0 × 10 J)
2ΔK 2W
1 −1
vy,f =√ =√ = √ = 1.4 × 10 m ⋅ s
m m 0.2kg

For the falling ball in a constant gravitation field, the positive work of the gravitation force on the body corresponds to an
increasing kinetic energy and speed. For a rising body in the same field, the kinetic energy and hence the speed decrease since
the work done is negative.

Example 13.6.2: Final Kinetic Energy of Moving Cup

A person pushes a cup of mass 0.2 kg along a horizontal table with a force of magnitude 2.0 N at an angle of 30 with respect ∘

to the horizontal for a distance of 0.5 m as in Example 13.4. The coefficient of friction between the table and the cup is
μ = 0.1 . If the cup was initially at rest, what is the final kinetic energy of the cup after being pushed 0.5 m? What is the final
k

speed of the cup?


Solution
The total work done on the cup is the sum of the work done by the pushing force and the work done by the friction force, as
given in Equations (13.4.9) and (13.4.14),
a f a
W = W +W = (Fx − μk N ) (xf − xi )

−2 −1
= (1.7N − 9.6 × 10 N) (0.5m) = 8.0 × 10 J

The initial velocity is zero so the change in kinetic energy is just


1 1 1
2 2 2
ΔK = mv − mv = mv
y,f y,0 y,f
2 2 2

13.6.1 https://phys.libretexts.org/@go/page/24506
Thus the work-kinetic energy theorem, Equation(13.6.1)), enables us to solve for the final kinetic energy,
1 −1
2
Kf = mv = ΔK = W = 8.0 × 10 J
f
2

We can solve for the final speed,


−−−−−−−−−−−−−
−−−− −−−− −1
2Kf 2W 2 (8.0 × 10 J)
−1
vy,f =√ =√ =√ = 2.9m ⋅ s
m m 0.2kg

This page titled 13.6: Work-Kinetic Energy Theorem is shared under a CC BY-NC-SA 4.0 license and was authored, remixed, and/or curated by
Peter Dourmashkin (MIT OpenCourseWare) via source content that was edited to the style and standards of the LibreTexts platform; a detailed
edit history is available upon request.

13.6.2 https://phys.libretexts.org/@go/page/24506
13.7: Power Applied by a Constant Force
a

Suppose that an applied force F acts on a body during a time interval Δt and the displacement of the point of application of the
force is in the x -direction by an amount Δx. The work done, ΔW , during this interval is a

a a
ΔW = Fx Δx

where F is the x -component of the applied force. (Equation (13.7.1) is the same as Equation (13.4.2).)
a
x

The average power of an applied force is defined to be the rate at which work is done,
a a
ΔW Fx Δx
a a
Pave = = = Fx vave ,x
Δt Δt

The average power delivered to the body is equal to the component of the force in the direction of motion times the component of
the average velocity of the body. Power is a scalar quantity and can be positive, zero, or negative depending on the sign of work.
The SI units of power are called watts [W] and [1W] = [1J ⋅ s ] . −1

The instantaneous power at time t is defined to be the limit of the average power as the time interval [t, t + Δt] approaches zero,
a a
a
ΔW Fx Δx Δx
a a
P = lim = lim = Fx ( lim ) = Fx vx
Δt→0 Δt Δt→0 Δt Δt→0 Δt

The instantaneous power of a constant applied force is the product of the component of the force in the direction of motion and the
instantaneous velocity of the moving object.

Example 13.7.1: Gravitational Power for a Falling Object

Suppose a ball of mass m = 0.2kg starts from rest at a height y = 15m above the surface of the earth and falls down to a
0

height y = 5.0m above the surface of the earth. What is the average power exerted by the gravitation force? What is the
f

instantaneous power when the ball is at a height y = 5.0m above the surface of the Earth? Make a graph of power vs. time.
f

You may ignore the effects of air resistance.


Solution
There are two ways to solve this problem. Both approaches require calculating the time interval Δt for the ball to fall. Set
t = 0 for the time the ball was released. We can solve for the time interval Δt = t
0 that it takes the ball to fall using the f

equation for a freely falling object that starts from rest,


1
2
yf = y0 − gt
f
2

Thus the time interval for falling is


−−−−−−−−−
−−−−−−−−−−−−−−−−− −
2 2
tf = √ (y0 − yf ) = √ (15m − 5m) = 1.4s
g 9.8m ⋅ s−2

First approach: we can calculate the work done by gravity,


g
W = −mg (yf − y0 )

−1 −2 1
= (−2.0 × 10 kg) (9.8m ⋅ s ) (5m − 15m) = 2.0 × 10 J

Then the average power is


1
g
ΔW 2.0 × 10 J
1
Pave  = = = 1.4 × 10 W
Δt 1.4s

Second Approach. We calculate the gravitation force and the average velocity. The gravitation force is
g −1 −2
Fy = −mg = − (2.0 × 10 kg) (9.8m ⋅ s ) = −2.0N

The average velocity is

13.7.1 https://phys.libretexts.org/@go/page/26935
Δy 5m − 15m
−1
vave y = = = −7.0m ⋅ s
Δt 1.4s

The average power is therefore


g g
Pave  = Fy vave y = (−mg)vave ,y

−1 1
= (−2.0N) (−7.0m ⋅ s ) = 1.4 × 10 W

In order to find the instantaneous power at any time, we need to find the instantaneous velocity at that time. The ball takes a
time t = 1.4s to reach the height y = 5.0m. The velocity at that height is given by
f f

−2 1 −1
vy = −gtf = − (9.8m ⋅ s ) (1.4s) = −1.4 × 10 m ⋅ s

So the instantaneous power at time t f = 1.4s is


g g 2
P = Fy vy = (−mg) (−gtf ) = m g tf

2
−2 1
= (0.2kg) (9.8m ⋅ s ) (1.4s) = 2.7 × 10 W

If this problem were done symbolically, the answers given in Equation (13.7.11) and Equation (13.7.12) would differ by a
factor of two; the answers have been rounded to two significant figures.
The instantaneous power grows linearly with time. The graph of power vs. time is shown in Figure 13.8. From the figure, it
should be seen that the instantaneous power at any time is twice the average power between t = 0 and that time.

Figure 13.8 Graph of power vs. time

Example 13.7.2: Power Pushing a Cup

A person pushes a cup of mass 0.2 kg along a horizontal table with a force of magnitude 2.0 N at an angle of 30 with respect ∘

to the horizontal for a distance of 0.5 m , as in Example 13.4. The coefficient of friction between the table and the cup is
μ = 0.1 . What is the average power of the pushing force? What is the average power of the kinetic friction force?
k

Solution
We will use the results from Examples 13.4 and 13.7 but keeping extra significant figures in the intermediate calculations. The
work done by the pushing force is
a a −1
W = Fx (xf − x0 ) = (1.732N)(0.50m) = 8.660 × 10 J

The final speed of the cup is v


x,f = 2.860m ⋅ s
−1
. Assuming constant acceleration, the time during which the cup was pushed
is
2 (xf − x0 )
tf = = 0.3496s
vx,f

The average power of the pushing force is then, with Δt = t , f

a −1
ΔW 8.660 × 10 J
a
Pave  = = = 2.340W
Δt 0.3496s

or 2.3W to two significant figures. The work done by the friction force is

13.7.2 https://phys.libretexts.org/@go/page/26935
f
W = fk (xf − x0 )

−2 −2
= −μk N (xf − x0 ) = − (9.6 × 10 N) (0.50m) = − (4.8 × 10 J)

The average power of kinetic friction is


f −2
f
ΔW −4.8 × 10 J
−1
Pave  = = = −1.4 × 10 W
Δt 0.3496s

The time rate of change of the kinetic energy for a body of mass m moving in the x-direction is
dK d 1 dvx
2
= ( m vx ) = m vx = m ax vx
dt dt 2 dt

By Newton’s Second Law, F x = m ax and so Equation (13.7.18) becomes


dK
= Fx vx = P
dt

The instantaneous power delivered to the body is equal to the time rate of change of the kinetic energy of the body.

This page titled 13.7: Power Applied by a Constant Force is shared under a not declared license and was authored, remixed, and/or curated by
Peter Dourmashkin (MIT OpenCourseWare) via source content that was edited to the style and standards of the LibreTexts platform; a detailed
edit history is available upon request.

13.7.3 https://phys.libretexts.org/@go/page/26935
13.8: Work and the Scalar Product
We shall introduce a vector operation, called the scalar product or “dot product” that takes any two vectors and generates a scalar
quantity (a number). We shall see that the physical concept of work can be mathematically described by the scalar product between
the force and the displacement vectors.

Scalar Product
→ →
Let A and B be two vectors. Because any two non-collinear vectors form a plane, we define the angle θ to be the angle between
→ →
the vectors A and B as shown in Figure 13.9. Note that θ can vary from 0 to π.

Figure 13.9 Scalar product geometry.


→ → → → → →
The scalar product A ⋅ B of the vectors A and B is defined to be product of the magnitude of the vectors A and B with the
cosine of the angle θ between the two vectors:
→ →
A ⋅ B = AB cos(θ)

→ → → →
where A = | A | and B =∣ B represent the magnitude of A and B respectively. The scalar product can be positive, zero, or
negative, depending on the value of cos θ. The scalar product is always a scalar quantity.
The angle formed by two vectors is therefore
→ →
⎛ A ⋅ B ⎞
−1
θ = cos ⎜ ⎟
→ →
⎝ ⎠
| A || B |

→ →
The magnitude of a vector A is given by the square root of the scalar product of the vector A with itself.
→ → →
1/2
| A| = (A ⋅ A)

We can give a geometric interpretation to the scalar product by writing the definition as
→ →
A ⋅ B = (A cos(θ))B

→ →
In this formulation, the term Acosθ is the projection of the vector B in the direction of the vector B . This projection is shown in
→ → →
Figure 13.10a. So the scalar product is the product of the projection of the length of A in the direction of B with the length of B .
Note that we could also write the scalar product as
→ →
A ⋅ B = A(B cos(θ))

→ →
Now the term B cos(θ) is the projection of the vector B in the direction of the vector A as shown in Figure 13.10b. From this
→ → →
perspective, the scalar product is the product of the projection of the length of B in the direction of A with the length of A .

13.8.1 https://phys.libretexts.org/@go/page/26936
Figure 13.10 (a) and (b) Projection of vectors and the scalar product
From our definition of the scalar product we see that the scalar product of two vectors that are perpendicular to each other is zero
since the angle between the vectors is π/2 and cos(π/2) = 0.
We can calculate the scalar product between two vectors in a Cartesian coordinates system as follows. Consider two vectors
→ →
^ ^ ^
A = Ax i + Ay j + Az k and ^ ^ ^
B = Bx i + By j + Bz k . Recall that
^ ^ ^ ^ ^ ^
i ⋅ i = j ⋅ j = k⋅k = 1

^ ^ ^ ^ ^ ^
i ⋅ j = j ⋅k = i ⋅k =0

→ →
The scalar product between A and B is then
→ →
A ⋅ B = Ax Bx + Ay By + Az Bz

The time derivative of the scalar product of two vectors is given by


→ →
d d
(A ⋅ B ) = (Ax Bx + Ay By + Az Bz )
dt dt

d d d d d d
= (Ax ) Bx + (Ay ) By + (Az ) Bz + Ax (Bx ) + Ay (By ) + Az (Bz )
dt dt dt dt dt dt

→ → → →
d d
=( A) ⋅ B + A ⋅ ( B)
dt dt

→ → →
In particular when A = B , then the time derivative of the square of the magnitude of the vector A is given by
d d → → d → → → d → d → →
2
A = (A ⋅ A) = ( A) ⋅ A + A ⋅ ( A) = 2 ( A) ⋅ A
dt dt dt dt dt

Kinetic Energy and the Scalar Product


For an object undergoing three-dimensional motion, the velocity of the object in Cartesian components is given by

^ ^ ^
v = vx i + vy j + vz k . Recall that the magnitude of a vector is given by the square root of the scalar product of the vector with
itself,
→ → →
1/2 1/2
2 2 2
A ≡ | A| ≡ (A ⋅ A) = (Ax + Ay + Az )

Therefore the square of the magnitude of the velocity is given by the expression
2 → → 2 2 2
v ≡ ( v ⋅ v ) = vx + vy + vz

Hence the kinetic energy of the object is given by


1 → → 1
2 2 2
K = m( v ⋅ v ) = m (vx + vy + vz )
2 2

13.8.2 https://phys.libretexts.org/@go/page/26936
Work and the Scalar Product
Work is an important physical example of the mathematical operation of taking the scalar product between two vectors. Recall that
when a constant force acts on a body and the point of application of the force undergoes a displacement along the x -axis, only the
component of the force along that direction contributes to the work,

W = Fx Δx


Suppose we are pulling a body along a horizontal surface with a force F . Choose coordinates such that horizontal direction is the x
→ →
-axis and the force F forms an angle β with the positive x -direction. In Figure 13.11 we show the force vector ^ ^
F = Fx i + Fy j
→ →
and the displacement vector of the point of application of the force . Note that
^
Δ x = Δx i is the component of the
^
Δ x = Δx i

displacement and hence can be greater, equal, or less than zero (but is shown as greater than zero in the figure for clarity). The
→ →
scalar product between the force vector F and the displacement vector Δ x is
→ →
^ ^ ^
F ⋅ Δ x = (Fx i + Fy j ) ⋅ (Δx i ) = Fx Δx

Figure 13.11 Force and displacement vectors


The work done by the force is then
→ →
ΔW = F ⋅ Δ x

In general, the angle β takes values within the range −π ≤ β ≤ π (in Figure 13.11, 0 ≤ β ≤ π/2 ). Because the x -component of
→ →
the force is F
x = F cos(β) where F = |F| denotes the magnitude of F , the work done by the force is
→ →
W = F ⋅ Δ x = (F cos(β))Δx

Example 13.10 Object Sliding Down an Inclined Plane


An object of mass m = 4.0 kg , starting from rest, slides down an inclined plane of length l = 3.0 m . The plane is inclined by an
angle of θ = 30 to the ground. The coefficient of kinetic friction is μ = 0.2 (a) What is the work done by each of the three forces

k

while the object is sliding down the inclined plane? (b) For each force, is the work done by the force positive or negative? (c) What
is the sum of the work done by the three forces? Is this positive or negative?
Solution: (a) and (b) Choose a coordinate system with the origin at the top of the inclined plane and the positive x -direction
pointing down the inclined plane, and the positive y - direction pointing towards the upper right as shown in Figure 13.12. While
the object is sliding down the inclined plane, three uniform forces act on the object, the gravitational force which points downward
and has magnitude F = mg , the normal force N which is perpendicular to the surface of the inclined plane, and the friction force
g

which opposes the motion and is equal in magnitude to f = μ N . A force diagram on the object is shown in Figure 13.13.
k k

13.8.3 https://phys.libretexts.org/@go/page/26936
Figure 13.12 Coordinate system for object sliding down the inclined plane

Figure 13.13 Free-body force diagram for object


In order to calculate the work we need to determine which forces have a component in the direction of the displacement. Only the
component of the gravitational force along the positive x -direction F = mg sin θ and the friction force are directed along the
gx

displacement and therefore contribute to the work. We need to use Newton’s Second Law to determine the magnitudes of the
normal force. Because the object is constrained to move along the positive x -direction, a = 0 , Newton’s Second Law in the ^j -
y

direction N − mg cos θ = 0 . Therefore N = mg cos θ and the magnitude of the friction force is f = μ mg cos θ .
k k

With our choice of coordinate system with the origin at the top of the inclined plane and the positive x -direction pointing down the

inclined plane, the displacement of the object is given by the vector Δ r ^
= Δx i (Figure 13.14).

Figure 13.14 Force vectors and displacement vector for object


→g →f →N
The vector decomposition of the three forces are F = mg sin θ^i − mg cos θ^j , F = −μ mg cos θ^i , andk F
^
= mg cos θ j .
The work done by the normal force is zero because the normal force is perpendicular the displacement
→N →
N ^ ^
W = F ⋅ Δ r = mg cos θ j ⋅ l i = 0

Then the work done by the friction force is negative and given by

13.8.4 https://phys.libretexts.org/@go/page/26936
f
→ →
f ^ ^
W = F ⋅ Δ r = −μk mg cos θ i ⋅ l i = −μk mg cos θl < 0

Substituting in the appropriate values yields


f −2 ∘
W = −μk mg cos θl = −(0.2)(4.0kg) (9.8m ⋅ s ) (3.0m) (cos(30 )(3.0m) = −20.4J

The work done by the gravitational force is positive and given by


g
→ →
g ^ ^ ^
W = F ⋅ Δ r = (mg sin θ i − mg cos θ j ) ⋅ l i = mgl sin θ > 0

Substituting in the appropriate values yields


g −2 ∘
W = mgl sin θ = (4.0kg) (9.8m ⋅ s ) (3.0m) (sin(30 ) = 58.8J

(c) The scalar sum of the work done by the three forces is then
g f
W =W +W = mgl (sin θ − μk cos θ)

−2 ∘ ∘
W = (4.0kg) (9.8m ⋅ s ) (3.0m) (sin(30 ) − (0.2) (cos(30 )) = 38.4J.

This page titled 13.8: Work and the Scalar Product is shared under a not declared license and was authored, remixed, and/or curated by Peter
Dourmashkin (MIT OpenCourseWare) via source content that was edited to the style and standards of the LibreTexts platform; a detailed edit
history is available upon request.

13.8.5 https://phys.libretexts.org/@go/page/26936
13.9: Work done by a Non-Constant Force Along an Arbitrary Path

Suppose that a non-constant force F acts on a point-like body of mass m while the body is moving on a three dimensional curved

path. The position vector of the body at time t with respect to a choice of origin is r (t) . In Figure 13.15 we show the orbit of the
→ →
body for a time interval [ ti , tf ] moving from an initial position r i ≡ r (t = ti ) at time t = ti to a final position
→ →
r f ≡ r (t = tf ) at time t = t . f

Figure 13.15 Path traced by the motion of a body.


We divide the time interval [ ti , tf ] into N smaller intervals with ,
[ tj−1 , tj ] j = 1, ⋯ , N with tN = tf . Consider two position
→ → → →
vectors r j ≡ r (t = tj ) and r j−1 ≡ r (t = tj−1 ) the displacement vector during the corresponding time interval as
→ → → →
Δ r j = r j − r j−1 . Let F denote the force acting on the body during the interval [t j−1 , . The average force in this interval is
tj ]


( F j) and the average work ΔW done by the force during the time interval [t
j j−1 , tj ] is the scalar product between the average
ave

force vector and the displacement vector,


→ →
ΔWj = ( F j ) ⋅Δ r j
ave

The force and the displacement vectors for the time interval [t j−1 , tj ] are shown in Figure 13.16 (note that the subscript “ave” on

( F j) has been suppressed).
ave

Figure 13.16 An infinitesimal work element.


We calculate the work by adding these scalar contributions to the work for each interval [t j−1 , , for j =1 to N ,
tj ]

j=N j=N
→ →
WN = ∑ ΔWj = ∑ ( F j ) ⋅Δ r j

j=1 j=1 ave

We would like to define work in a manner that is independent of the way we divide the interval, so we take the limit as N → ∞
→ ∣
and ∣

Δ r j → 0

for all j . In this limit, as the intervals become smaller and smaller, the distinction between the average force and
the actual force vanishes. Thus if this limit exists and is well defined, then the work done by the force is

13.9.1 https://phys.libretexts.org/@go/page/26937
j=N f
→ → → →
W = lim ∑ ( F j) ⋅Δ r j =∫ F ⋅d r
N→ ∞
→ ave  i
∣ ∣ j=1
Δ r →0
∣ j∣

→ →
Notice that this summation involves adding scalar quantities. This limit is called the line integral of the force F . The symbol d r

is called the infinitesimal vector line element. At time t, d r is tangent to the orbit of the body and is the limit of the displacement
→ → →
vector Δ r = r (t + Δt) − r (t) as Δt approaches zero. In this limit, the parameter t does not appear in the expression in
Equation (13.8.19).
→ →
In general this line integral depends on the particular path the body takes between the initial position r i and the final position r f ,

which matters when the force F is nonconstant in space, and when the contribution to the work can vary over different paths in
space. We can represent the integral in Equation (13.8.19) explicitly in a coordinate system by specifying the infinitesimal vector

line element d r and then explicitly computing the scalar product.

Work Integral in Cartesian Coordinates


In Cartesian coordinates the line element is

^ ^ ^
d r = dx i + dy j + dzk

where dx , dy , and dz represent arbitrary displacements in the ^i −, ^j −, and k


^
-direction respectively as seen in Figure 13.17.

Figure 13.17 A line element in Cartesian coordinates.


The force vector can be represented in vector notation by

^ ^ ^
F = Fx i + Fy j + Fz k

The infinitesimal work is the sum of the work done by the component of the force times the component of the displacement in each
direction,
dW = Fx dx + Fy dy + Fz dz

Equation (13.8.22) is just the scalar product


→ →
^ ^ ^ ^ ^ ^
dW = F ⋅ d r = (Fx i + Fy j + Fz k) ⋅ (dx i + dy j + dzk)

= Fx dx + Fy dy + Fz dz

The work is
→ → → → → → → → −−−−−−−→
r = r f r = r f r = r f r = r f
→ →
W =∫ F ⋅d r =∫ (Fx dx + Fy dy + Fz dz) = ∫ Fx dx + ∫ ⋅ dy + ∫ Fz dz
→ → → → → → → → →
r = r 0 r = r 0 r = r 0 r = r 0 z= r 0

Work Integral in Cylindrical Coordinates


In cylindrical coordinates the line element is

^ ^
d r = dr^
r + rdθθ + dzk

13.9.2 https://phys.libretexts.org/@go/page/26937
where dr , rdθ , and dz represent arbitrary displacements in the ^
r−, θ − and k− directions respectively as seen in Figure 13.18.
^ ^


Figure 13.18 Displacement vector d s between two points
The force vector can be represented in vector notation by

^ ^
F = Fr ^
r + Fθ θ + Fz k

The infinitesimal work is the scalar product


→ →
^ ^ ^ ^
dW = F ⋅ d r = (Fr ^
r + Fθ θ + Fz k) ⋅ (dr^
r + rdθθ + dzk)

= Fr dr + Fθ rdθ + Fz dz

The work is
→ → → → → → → → → →
r = r f r = r f r = r f r = r f r = r f

W =∫ ⋅ r =∫ (Fr dr + Fθ rdθ + Fz dz) = ∫ Fr dr + ∫ Fθ rdθ + ∫ Fz dz
→ → → → → → → → → →
r = r 0 r = r 0 r = r 0 r = r 0 r = r 0

This page titled 13.9: Work done by a Non-Constant Force Along an Arbitrary Path is shared under a not declared license and was authored,
remixed, and/or curated by Peter Dourmashkin (MIT OpenCourseWare) via source content that was edited to the style and standards of the
LibreTexts platform; a detailed edit history is available upon request.

13.9.3 https://phys.libretexts.org/@go/page/26937
13.10: Worked Examples
Example 13.11 Work Done in a Constant Gravitation Field
The work done in a uniform gravitation field is a fairly straightforward calculation when the body moves in the direction of the

field. Suppose the body is moving under the influence of gravity, F = −mg^j along a parabolic curve. The body begins at the
point (x , y ) and ends at the point (x , y ). What is the work done by the gravitation force on the body?
0 0 f f


Solution: The infinitesimal line element d r is therefore

^ ^
d r = dx i + dy j

The scalar product that appears in the line integral can now be calculated,
→ →
^ ^ ^
F ⋅ d r = −mg j ⋅ [dx i + dy j ] = −mgdy

This result is not surprising since the force is only in the y -direction. Therefore the only non-zero contribution to the work integral
is in the y -direction, with the result that
rf y=yf y=yf
→ →
W =∫ F ⋅d r =∫ Fy dy = ∫ −mgdy = −mg (yf − y0 )
r0 y=y0 y=y0

In this case of a constant force, the work integral is independent of path.


Example 13.12 Hooke’s Law Spring-Body System
Consider a spring-body system lying on a frictionless horizontal surface with one end of the spring fixed to a wall and the other end
attached to a body of mass m (Figure 13.19). Calculate the work done by the spring force on body as the body moves from some
initial position to some final position.

Figure 13.19 A spring-body system.


Solution: Choose the origin at the position of the center of the body when the spring is relaxed (the equilibrium position). Let x be
the displacement of the body from the origin. We choose the +^i unit vector to point in the direction the body moves when the
spring is being stretched (to the right of x = 0 in the figure). The spring force on the body is then given by

^ ^
F = Fx i = −kx i

The work done by the spring force on the mass is


x=xf
1
2 2
Wspring  = ∫ (−kx)dx = − k (x −x )
f 0
x=x0
2

Example 13.13 Work done by the Inverse Square Gravitation Force


Consider a body of mass m in moving in a fixed orbital plane about the sun. The mass of the sun is m . How much work does the
s

gravitation interaction between the sun and the body done on the body during this motion?

13.10.1 https://phys.libretexts.org/@go/page/26938
Solution: Let’s assume that the sun is fixed and choose a polar coordinate system with the origin at the center of the sun. Initially
the body is at a distance r from the center of the sun. In the final configuration the body has moved to a distance r < r from the
0 f 0


center of the sun. The infinitesimal displacement of the body is given by d r = dr^
^
r + rdθθ . The gravitation force between the
sun and the body is given by
→ Gms m
F grav = Fgrav ^
r =− ^
r
r2

The infinitesimal work done work done by this gravitation force on the body is given by
→ →
^
dW = F grav ⋅ d r = (Fgrav,r ^
r) ⋅ (dr^
r + rdθθ ) = Fgrav,r dr

Therefore the work done on the object as the object moves from r to r is given by the integrali f

rf rf rf
→ → Gmsun m
W =∫ F grav ⋅ d r = ∫ Fgrav,r dr = ∫ (− ) dr
ri ri ri
r2

Upon evaluation of this integral, we have for the work


rf rf
Gmsun m Gmsun m ∣ 1 1
W =∫ (− ) dr = ∣ = Gmsun m ( − )
2 ∣
ri
r r r
rf ri
i

Because the body has moved closer to the sun, r f < ri , hence 1/r f > 1/ ri . Thus the work done by gravitation force between the
sun and the body, on the body is positive,
1 1
W = Gmsun m ( − ) >0
rf ri

We expect this result because the gravitation force points along the inward radial direction, so the scalar product and hence work of
the force and the displacement is positive when the body moves closer to the sun. Also we expect that the sign of the work is the
same for a body moving closer to the sun as a body falling towards the earth in a constant gravitation field, as seen in Example
4.7.1 above.
Example 13.14 Work Done by the Inverse Square Electrical Force
Let’s consider two point-like bodies, body 1 and body 2, with charges q and q respectively interacting via the electric force alone.
1 2

Body 1 is fixed in place while body 2 is free to move in an orbital plane. How much work does the electric force do on the body 2
during this motion?
Solution: The calculation in nearly identical to the calculation of work done by the gravitational inverse square force in Example
13.13. The most significant difference is that the electric force can be either attractive or repulsive while the gravitation force is
always attractive. Once again we choose polar coordinates centered on body 2 in the plane of the orbit. Initially a distance r 0

separates the bodies and in the final state a distance r separates the bodies. The electric force between the bodies is given by
f

→ 1 q1 q2
F elec = Felec ^
r = Felec,r ^
r = ^
r
2
4πε0 r

The work done by this electric force on the body 2 is given by the integral
rf rf rf
→ → 1 q1 q2
W =∫ F elec ⋅ d r = ∫ Felec,r dr = ∫ dr
2
ri ri
4πε0 ri r

Evaluating this integral, we have for the work done by the electric force
rf rf
1 q1 q2 1 q1 q2 ∣ 1 1 1
W =∫ dr = − ∣ =− q1 q2 ( − )
2 2 ∣
ri
4πε0 r 4πε0 r ri
4πε0 rf ri

If the charges have opposite signs, q q < 0 , we expect that the body 2 will move closer to body 1 so
1 2 rf < ri and 1/ rf > 1/ ri .
From our result for the work, the work done by electrical force in moving body 2 is positive,
1 1 1
W =− q1 q2 ( − ) >0
4πε0 rf ri

13.10.2 https://phys.libretexts.org/@go/page/26938
Once again we see that bodies under the influence of electric forces only will naturally move in the directions in which the force
does positive work. If the charges have the same sign, then q q > 0 . They will repel with r > r and 1/r < 1/r . Thus the
1 2 f i f i

work is once again positive:


1 1 1
W =− q1 q2 ( − ) >0
4πε0 rf ri

This page titled 13.10: Worked Examples is shared under a not declared license and was authored, remixed, and/or curated by Peter Dourmashkin
(MIT OpenCourseWare) via source content that was edited to the style and standards of the LibreTexts platform; a detailed edit history is
available upon request.

13.10.3 https://phys.libretexts.org/@go/page/26938
13.11: Work-Kinetic Energy Theorem in Three Dimensions
Recall our mathematical result that for one-dimensional motion
f f f f
dvx dx 1 1
2 2
m∫ ax dx = m ∫ dx = m ∫ dvx =m∫ vx dvx = mv − mv
x,f x,i
i i dt i dt i 2 2

Using Newton’s Second Law in the form F x = m ax , we concluded that


f
1 1
2 2
∫ Fx dx = mv − mv
x,f x,i
i
2 2

Equation (13.11.2) generalizes to the y - and z -directions:


f
1 1
2 2
∫ Fy dy = mv − mv
y,f y,i
i 2 2

f
1 1
2 2
∫ Fz dz = mv − mv
z,f z,i
i
2 2

Adding Equations (13.11.2), (13.11.3), and (13.11.4) yields


f
1 1
2 2 2 2 2 2
∫ (Fx dx + Fy dy + Fz dz) = m (v +v +v )− m (v +v +v )
x,f y,f z,f x,i y,i z,i
i 2 2


Recall (Equation (13.8.24)) that the left hand side of Equation (13.11.5) is the work done by the force F on the object
f f f
→ →
W =∫ dW = ∫ (Fx dx + Fy dy + Fz dz) = ∫ F ⋅d r
i i i

The right hand side of Equation (13.11.5) is the change in kinetic energy of the object
1 1 1 1
2 2 2 2 2 2 2 2
ΔK ≡ Kf − Ki = mv − mv = m (v +v +v )− m (v +v +v )
f 0 x,f y,f z,f x,i y,i z,i
2 2 2 2

Therefore Equation (13.11.5) is the three dimensional generalization of the work-kinetic energy theorem
f
→ →
∫ F ⋅ d r = Kf − Ki
i

When the work done on an object is positive, the object will increase its speed, and negative work done on an object causes a
decrease in speed. When the work done is zero, the object will maintain a constant speed.

Instantaneous Power Applied by a Non-Constant Force for Three Dimensional Motion


Recall that for one-dimensional motion, the instantaneous power at time t is defined to be the limit of the average power as the time
interval [t, t + Δt] approaches zero,
a
P (t) = Fx (t)vx (t)

A more general result for the instantaneous power is found by using the expression for dW as given in Equation (13.8.23),
→ →
dW F ⋅d r → →
P = = = F ⋅ v
dt dt

The time rate of change of the kinetic energy for a body of mass m is equal to the power,

dK 1 d d v →
→ → → → → →
= m (v ⋅ v) =m ⋅ v =m a ⋅ v = F ⋅ v =P
dt 2 dt dt

where the we used Equation (13.8.9), Newton’s Second Law and Equation (13.11.10).

13.11.1 https://phys.libretexts.org/@go/page/26939
This page titled 13.11: Work-Kinetic Energy Theorem in Three Dimensions is shared under a not declared license and was authored, remixed,
and/or curated by Peter Dourmashkin (MIT OpenCourseWare) via source content that was edited to the style and standards of the LibreTexts
platform; a detailed edit history is available upon request.

13.11.2 https://phys.libretexts.org/@go/page/26939
13.12: Appendix 13A Work Done on a System of Two Particles
We shall show that the work done by an internal force in changing a system of two particles of masses m1 and m1 respectively
from an initial state A to a final state B is equal to
1
2 2
Wc = μ (v −v )
B A
2

where 2
v
B
is the square of the relative velocity in state B, 2
v
A
is the square of the relative velocity in state A, and
μ = m1 m2 / (m + m ) .
1 2

Consider two bodies 1 and 2 and an interaction pair of forces shown in Figure 13A.1.

Figure 13A.1 System of two bodies interacting


We choose a coordinate system shown in Figure 13A.2.

Figure 13A.2 Coordinate system for two-body interaction


Newton’s Second Law applied to body 1 is
2

→ d r 1
F 2,1 = m1
2
dt

and applied to body 2 is


2

→ d r 2
F 1,2 = m2
2
dt

Divide each side of Equation (13.1.2) by m ,


1


2

F 2,1 d r 1
=
2
m1 dt

and divide each side of Equation (13.1.3) by m , 2


2

F 1,2 d r 2
=
2
m2 dt

13.12.1 https://phys.libretexts.org/@go/page/26940
Subtract Equation (13.1.5) from Equation (13.1.4) yielding
→ →
→ → 2

2 2
F 2,1 F 1,2 d r 1 d r 2 d r 2,1
− = − =
m1 m2 dt2 dt2 dt2

→ → → → →
where r 2,1 = r 1 − r 2 . Use Newton’s Third Law F 2,1 = − F 1,2 on the left hand side of Equation (13.1.6) to obtain
→ → →
2 2 2
→ 1 1 d r 1 d r 2 d r 2,1
F 2,1 ( + ) = − =
m1 m2 dt2 dt2 dt2


The quantity d 2
r 1,2 /dt
2
is the relative acceleration of body 1 with respect to body 2.
Define
1 1 1
≡ +
μ m1 m2

The quantity μ is known as the reduced mass of the system. Equation (13.1.7) now takes the form
2

→ d r 2,1
F 2,1 = μ
dt2

The work done in the system in displacing the two masses from an initial state A to a final state B is given by
B B
→ → → →
W =∫ F 2,1 ⋅ d r 1 + ∫ F 1,2 ⋅ d r 2
A A

Recall by the work energy theorem that the LHS is the work done on the system,
B B
→ → → →
W =∫ F 2,1 ⋅ d r 1 + ∫ F 1,2 ⋅ d r 2 = ΔK
A A

From Newton’s Third Law, the sum in Equation (13.1.10) becomes


B B B B
→ → → → → → → → →
W =∫ F 2,1 ⋅ d r 1 − ∫ F 2,1 ⋅ d r 2 = ∫ F 2,1 ⋅ (d r 1 − d r 2 ) = ∫ F 2,1 ⋅ d r 2,1
A A A A


where d r 2,1is the relative displacement of the two bodies. We can now substitute Newton’s Second Law, Equation (13.1.9), for
the relative acceleration into Equation (13.1.12),
2
→ 2
→ →
B B B
→ → d r 2,1 → d r 2,1 d r 2,1
W =∫ F 2,1 ⋅ d r 2,1 = ∫ μ ⋅ d r 2,1 = μ ∫ ( ⋅ ) dt
2 2
A A dt A dt dt


d r 2,1
where we have used the relation between the differential elements dr2,1 =
dt
dt . The product rule for derivatives of the scalar
product of a vector with itself is given for this case by
→ → 2
→ →
1 d d r 2,1 d r 2,1 d r 2,1 d r 2,1
( ⋅ ) = ⋅
2
2 dt dt dt dt dt

Substitute Equation (13.1.14) into Equation (13.1.13), which then becomes


B
→ →
1 d d r 2,1 d r 2,1
W = μ∫ ( ⋅ ) dt
A
2 dt dt dt

Equation (13.1.15) is now the integral of an exact derivative, yielding


B
→ →
∣ B
1 d r 2,1 d r 2,1 1 → → ∣ 1
∣ 2 2
W = μ( ⋅ ) = μ ( v 2,1 ⋅ v 2,1 )∣ = μ (v −v )
∣ ∣ B A
2 dt dt 2 A 2
∣A

13.12.2 https://phys.libretexts.org/@go/page/26940

where V is the relative velocity between the two bodies. It’s important to note that in the above derivation had we exchanged the
2,1

roles of body 1 and 2 i.e. 1 → 2 and 2 → 1 we would have obtained the identical result because
→ →
F 1,2 = − F 2,1

→ → → →
r 1,2 = r 2 − r 1 = − r 2,1

→ → → →
d r 1,2 = d ( r 2 − r 1 ) = −d r

→ →
v 1,2 = − v 2,1

Equation (13.1.16) implies that the work done is the change in the kinetic energy of the system, which we can write in terms of the
reduced mass and the change in the square of relative speed of the two objects
1
2 2
ΔK = μ (v −v )
B A
2

This page titled 13.12: Appendix 13A Work Done on a System of Two Particles is shared under a not declared license and was authored, remixed,
and/or curated by Peter Dourmashkin (MIT OpenCourseWare) via source content that was edited to the style and standards of the LibreTexts
platform; a detailed edit history is available upon request.

13.12.3 https://phys.libretexts.org/@go/page/26940
CHAPTER OVERVIEW
14: Potential Energy and Conservation of Energy
14.1: Conservation of Energy
14.2: Conservative and Non-Conservative Forces
14.3: Changes in Potential Energies of a System
14.4: Change in Potential Energy and Zero Point for Potential Energy
14.5: Mechanical Energy and Conservation of Mechanical Energy
14.6: Spring Force Energy Diagram
14.7: Change of Mechanical Energy for Closed System with Internal Nonconservative Forces
14.8: Dissipative Forces- Friction
14.9: Worked Examples

This page titled 14: Potential Energy and Conservation of Energy is shared under a CC BY-NC-SA 4.0 license and was authored, remixed, and/or
curated by Peter Dourmashkin (MIT OpenCourseWare) via source content that was edited to the style and standards of the LibreTexts platform; a
detailed edit history is available upon request.

1
14.1: Conservation of Energy
There is a fact, or if you wish, a law, governing all natural phenomena that are known to date. There is no exception to this law —
it is exact as far as we know. The law is called the conservation of energy. It states that there is a certain quantity, which we call
energy that does not change in the manifold changes which nature undergoes. That is a most abstract idea, because it is a
mathematical principle; it says that there is a numerical quantity, which does not change when something happens. It is not a
description of a mechanism, or anything concrete; it is just a strange fact that we can calculate some number and when we finish
watching nature go through her tricks and calculate the number again, it is the same.
~Richard Feynman
So far we have analyzed the motion of point-like objects under the action of forces using Newton’s Laws of Motion. We shall now
introduce the Principle of Conservation of Energy to study the change in energy of a system between its initial and final states. In
particular we shall introduce the concept of potential energy to describe the effect of conservative internal forces acting on the
constituent components of a system.
Recall from Chapter 13.1, the principle of conservation of energy. When a system and its surroundings undergo a transition from an
initial state to a final state, the change in energy is zero,

ΔE = ΔEsystem  + ΔEsurroundings  = 0

Figure 14.1 Diagram of a system and its surroundings


We shall study types of energy transformations due to interactions both inside and across the boundary of a system.

This page titled 14.1: Conservation of Energy is shared under a CC BY-NC-SA 4.0 license and was authored, remixed, and/or curated by Peter
Dourmashkin (MIT OpenCourseWare) via source content that was edited to the style and standards of the LibreTexts platform; a detailed edit
history is available upon request.

14.1.1 https://phys.libretexts.org/@go/page/24508
14.2: Conservative and Non-Conservative Forces
Our first type of “energy accounting” involves mechanical energy. There are two types of mechanical energy, kinetic energy and
potential energy. Our first task is to define what we mean by the change of the potential energy of a system.

→ →
We defined the work done by a force F , on an object, which moves along a path from an initial position r to a final position r ,i f

as the integral of the component of the force tangent to the path with respect to the displacement of the point of contact of the force
and the object,
→ →
W =∫ F ⋅d r
path 

Does the work done on the object by the force depend on the path taken by the object?

Figure 14.2 (a) and (b) Two different paths connecting the same initial and final points
First consider the motion of an object under the influence of a gravitational force near the surface of the earth. Let’s consider two
paths 1 and 2 shown in Figure 14.2. Both paths begin at the initial point (x , y ) = (0, y ) and end at the final point
i i i


(xf , yf ) = (xf , 0) . The gravitational force always points downward, so with our choice of coordinates, ^
F = −mg j . The

infinitesimal displacement along path 1 (Figure 14.2a) is given by ^ ^
d r 1 = dx1 i + dy1 j . The scalar product is then


^ ^ ^
F ⋅ d r 1 = −mg j ⋅ (dx1 i + dy1 j ) = −mgdy1

The work done by gravity along path 1 is the integral


( xf ,0)
→ →
W1 = ∫ F ⋅d r =∫ −mgdy1 = −mg (0 − yi ) = mgyi
path 1 (0, yi )

Path 2 consists of two legs (Figure 14.2b), leg A goes from the initial point (0, y ) to the origin (0,0) , and leg B goes from the
i

origin (0,0) to the final point (x , 0). We shall calculate the work done along the two legs and then sum them up. The infinitesimal
f


displacement along leg A is given by d r A
^
= dyA j . The scalar product is then
→ →
^ ^
F ⋅ d r A = −mg j ⋅ dyA j = −mgdyA

The work done by gravity along leg A is the integral


(0,0)
→ →
WA = ∫ F ⋅d r A =∫ −mgdyA = −mg (0 − yi ) = mgyi
leg A (0, yi )


The infinitesimal displacement along leg B is given by d r B
^
= dxB i . The scalar product is then
→ →
^ ^
F ⋅ d r B = −mg j ⋅ dxB i = 0

14.2.1 https://phys.libretexts.org/@go/page/24509
Therefore the work done by gravity along leg B is zero, W = 0 , which is no surprise because leg B is perpendicular to the
B

direction of the gravitation force. Therefore the work done along path 2 is equal to the work along path 1,

W2 = WA + WB = mgyi = W1

Now consider the motion of an object on a surface with a kinetic frictional force between the object and the surface and denote the
coefficient of kinetic friction by μ . Let’s compare two paths from an initial point x to a final point x . The first path is a straight-
k i f

line path. Along this path the work done is just


→ →
f
W =∫ F ⋅d r =∫ Fx dx = −μk N s1 = −μk N Δx < 0
path 1 path1

where the length of the path is equal to the displacement, s = Δx . Note that the fact that the kinetic frictional force is directed
1

opposite to the displacement, which is reflected in the minus sign in Equation (14.2.8). The second path goes past x some distance f

and them comes back to x (Figure 14.3). Because the force of friction always opposes the motion, the work done by friction is
f

negative,
→ →
f
W =∫ F ⋅d r =∫ Fx dx = −μk N s2 < 0
path 2 path 2

The work depends on the total distance traveled s , and is greater than the displacement s
2 2 > Δx . The magnitude of the work
done along the second path is greater than the magnitude of the work done along the first path.

Figure 14.3 Two different paths from x to x


i f

These two examples typify two fundamentally different types of forces and their contribution to work. The work done by the
gravitational force near the surface of the earth is independent of the path taken between the initial and final points. In the case of
sliding friction, the work done depends on the path taken.
Whenever the work done by a force in moving an object from an initial point to a final point is independent of the path, the force is
called a conservative force.

The work done by a conservative force F in going around a closed path is zero. Consider the two paths shown in Figure 14.4 that
c

form a closed path starting and ending at the point A with Cartesian coordinates (1, 0).

14.2.2 https://phys.libretexts.org/@go/page/24509
Figure 14.4 Two paths in the presence of a conservative force.
The work done along path 1 (the upper path in the figure, blue if viewed in color) from point A to point B with coordinates (0,1) is
given by
B
→ →
W1 = ∫ F c (1) ⋅ d r 1
A

The work done along path 2 (the lower path, green in color) from B to A is given by
A
→ →
W2 = ∫ F c (2) ⋅ d r 2
B

The work done around the closed path is just the sum of the work along paths 1 and 2,
B A
→ → → →
W = W1 + W2 = ∫ F c (1) ⋅ d r 1 + ∫ F c (2) ⋅ d r 2
A B

If we reverse the endpoints of path 2, then the integral changes sign,


A B
→ → → →
W2 = ∫ F c (2) ⋅ d r 2 = − ∫ F c (2) ⋅ d r 2
B A

We can then substitute Equation (14.2.13) into Equation (14.2.12) to find that the work done around the closed path is
B B
→ → → →
W =∫ F c (1) ⋅ d r 1 − ∫ F c (2) ⋅ d r 2
A A

Since the force is conservative, the work done between the points A to B is independent of the path, so
B B
→ → → →
∫ F c (1) ⋅ d r 1 = ∫ F c (2) ⋅ d r 2
A A

We now use path independence of work for a conservative force (Equation (14.2.15) in Equation (14.2.14)) to conclude that the
work done by a conservative force around a closed path is zero,
→ →
W =∮ Fc ⋅d r = 0
closed 

14.2.3 https://phys.libretexts.org/@go/page/24509
This page titled 14.2: Conservative and Non-Conservative Forces is shared under a CC BY-NC-SA 4.0 license and was authored, remixed, and/or
curated by Peter Dourmashkin (MIT OpenCourseWare) via source content that was edited to the style and standards of the LibreTexts platform; a
detailed edit history is available upon request.

14.2.4 https://phys.libretexts.org/@go/page/24509
14.3: Changes in Potential Energies of a System
Consider an object near the surface of the earth as a system that is initially given a velocity directed upwards. Once the object is
released, the gravitation force, acting as an external force, does a negative amount of work on the object, and the kinetic energy
decreases until the object reaches its highest point, at which its kinetic energy is zero. The gravitational force then does positive
work until the object returns to its initial starting point with a velocity directed downward. If we ignore any effects of air resistance,
the descending object will then have the identical kinetic energy as when it was thrown. All the kinetic energy was completely
recovered.
Now consider both the earth and the object as a system and assume that there are no other external forces acting on the system.
Then the gravitational force is an internal conservative force, and does work on both the object and the earth during the motion. As
the object moves upward, the kinetic energy of the system decreases, primarily because the object slows down, but there is also an
imperceptible increase in the kinetic energy of the earth. The change in kinetic energy of the earth must also be included because
the earth is part of the system. When the object returns to its original height (vertical distance from the surface of the earth), all the
kinetic energy in the system is recovered, even though a very small amount has been transferred to the Earth.
If we included the air as part of the system, and the air resistance as a nonconservative internal force, then the kinetic energy lost
due to the work done by the air resistance is not recoverable. This lost kinetic energy, which we have called thermal energy, is
distributed as random kinetic energy in both the air molecules and the molecules that compose the object (and, to a smaller extent,
the earth).
We shall define a new quantity, the change in the internal potential energy of the system, which measures the amount of lost kinetic
energy that can be recovered during an interaction.
When only internal conservative forces act in a closed system, the sum of the changes of the kinetic and potential energies of the
system is zero.
Consider a closed system, ΔE = 0 , that consists of two objects with masses m and m respectively. Assume that there is only
sys 1 2

one conservative force (internal force) that is the source of the interaction between two objects. We denote the force on object 1 due
→ →
to the interaction with object 2 by F 2,1 and the force on object 2 due to the interaction with object 1 by F 1,2 . From Newton’s
Third Law,
→ →
F 2,1 = − F 1,2

The forces acting on the objects are shown in Figure 14.5.

Figure 14.5 Internal forces acting on two objects



Choose a coordinate system (Figure 14.6) in which the position vector of object 1 is given by r 1 and the position vector of object
→ → → →
2 is given by r 2 . The relative position of object 1 with respect to object 2 is given by r 2,1 = r 1 − r 2 . During the course of the
→ →
interaction, object 1 is displaced by d r 1 and object 2 is displaced by d r , so the relative displacement of the two objects during
2

→ → →
the interaction is given by d r 2,1 = d r 1 −d r 2 .

14.3.1 https://phys.libretexts.org/@go/page/24510
→ → →
Figure 14.6 Coordinate system for two objects with relative position vector r 2,1 = r 1 − r 2

Recall that the change in the kinetic energy of an object is equal to the work done by the forces in displacing the object. For two
objects displaced from an initial state A to a final state B ,
B B
→ → → →
ΔKsys = ΔK1 + ΔK2 = Wc = ∫ F 2,1 ⋅ d r 1 + ∫ F 1,2 ⋅ d r 2
A A

(In Equation (14.3.2), the labels “ A ” and “ B ” refer to initial and final states, not paths.) From Newton’s Third Law, Equation
(14.3.1), the sum in Equation (14.3.2) becomes
B B B B
→ → → → → → → → →
ΔKsys = Wc = ∫ F 2,1 ⋅ d r 1 − ∫ F 2,1 ⋅ d r 2 = ∫ F 2,1 ⋅ (d r 1 − d r 2 ) = ∫ F 2,1 ⋅ d r 2,1
A A A A

→ → →
where d r 2,1 = d r 1 − d r 2 is the relative displacement of the two objects. Note that since
B B
→ → → → → → → →
F 2,1 = − F 1,2  and d r 2,1 = −d r 1,2 , ∫ F 2,1 ⋅ d r 2,1 = ∫ F 1,2 ⋅ d r 1,2
A A


Consider a system consisting of two objects interacting through a conservative force. Let F 2,1 denote the force on object 1 due to
→ → →
the interaction with object 2 and let be the relative displacement of the two objects. The change in internal
d r 2,1 = d r 1 − d r 2

potential energy of the system is defined to be the negative of the work done by the conservative force when the objects undergo a
relative displacement from the initial state A to the final state B along any displacement that changes the initial state A to the final
state B ,
B B
→ → → →
ΔUsys = −Wc = − ∫ F 2,1 ⋅ d r 2,1 = − ∫ F 1,2 ⋅ d r 1,2
A A

Our definition of potential energy only holds for conservative forces, because the work done by a conservative force does not
depend on the path but only on the initial and final positions. Because the work done by the conservative force is equal to the
change in kinetic energy, we have that
\[\Delta U_{\mathrm{sys}}=-\Delta K_{\mathrm{sys}}\end{equation} (closed system with no non-conservative forces)
Recall that the work done by a conservative force in going around a closed path is zero (Equation (14.2.16)); therefore the change
in kinetic energy when a system returns to its initial state is zero. This means that the kinetic energy is completely recoverable.
In the Appendix 13A: Work Done on a System of Two Particles, we showed that the work done by an internal force in changing a
system of two particles of masses m and m respectively from an initial state A to a final state B is equal to
1 2

1
2 2
W = μ (v −v ) = ΔKsys
B A
2

14.3.2 https://phys.libretexts.org/@go/page/24510
where v
2
B
is the square of the relative velocity in state B, v is the square of the relative velocity in state A, and
2

μ = m1 m2 / (m + m ) is a quantity known as the reduced mass of the system.


1 2

Change in Potential Energy for Several Conservative Forces


When there are several internal conservative forces acting on the system we define a separate change in potential energy for the
work done by each conservative force,
B


ΔUsys,i = −Wc,i = − ∫ F c,i ⋅ d r i
A

→ → →
where F is a conservative internal force and d r a change in the relative positions of the objects on which F when the
c,i i c,i

system is changed from state A to state B. The work done is the sum of the work done by the individual conservative forces,

Wc = Wc,1 + Wc,2 + ⋯

Hence, the sum of the changes in potential energies for the system is the sum

ΔUsys = ΔUsys,1 + ΔUsys,2 + ⋯

Therefore the change in potential energy of the system is equal to the negative of the work done
B
→ →
ΔUsys = −Wc = − ∑ ∫ F c,i ⋅ d r i

i A

If the system is closed (external forces do no work), and there are no non-conservative internal forces then Equation (14.3.5) holds.

This page titled 14.3: Changes in Potential Energies of a System is shared under a CC BY-NC-SA 4.0 license and was authored, remixed, and/or
curated by Peter Dourmashkin (MIT OpenCourseWare) via source content that was edited to the style and standards of the LibreTexts platform; a
detailed edit history is available upon request.

14.3.3 https://phys.libretexts.org/@go/page/24510
14.4: Change in Potential Energy and Zero Point for Potential Energy
We already calculated the work done by different conservative forces: constant gravity near the surface of the earth, the spring
force, and the universal gravitation force. We chose the system in each case so that the conservative force was an external force. In
each case, there was no change of potential energy and the work done was equal to the change of kinetic energy,

Wext = ΔKsys

We now treat each of these conservative forces as internal forces and calculate the change in potential energy of the system
according to our definition
B
→ →
ΔUsys = −Wc = − ∫ Fc ⋅d r
A

We shall also choose a zero reference potential for the potential energy of the system, so that we can consider all changes in
potential energy relative to this reference potential.

Change in Gravitational Potential Energy Near Surface of the Earth


Let’s consider the example of an object falling near the surface of the earth. Choose our system to consist of the earth and the
object. The gravitational force is now an internal conservative force acting inside the system. The distance separating the object and
the center of mass of the earth, and the velocities of the earth and the object specifies the initial and final states.
Let’s choose a coordinate system with the origin on the surface of the earth and the + y - direction pointing away from the center of
the earth. Because the displacement of the earth is negligible, we need only consider the displacement of the object in order to
calculate the change in potential energy of the system.
Suppose the object starts at an initial height y above the surface of the earth and ends at final height y . The gravitational force on
i f

→g →
the object is given by F
^
= −mg j , the displacement is given by ^
d r = dy j , and the scalar product is given by
→g →
F
^ ^
⋅ d r = −mg j ⋅ dy j = −mgdy . The work done by the gravitational force on the object is then
yf g yf
→ →
g
W =∫ F ⋅d r =∫ −mgdy = −mg (yf − yi )
y y
i i

The change in potential energy is then given by


g g
ΔU = −W = mgΔy = mgyf − mgyi

We introduce a potential energy function U so that


g g g
ΔU ≡U −U
f i

Only differences in the function U have a physical meaning. We can choose a zero reference point for the potential energy
g

anywhere we like. We have some flexibility to adapt our choice of zero for the potential energy to best fit a particular problem.
Because the change in potential energy only depended on the displacement, Δy. In the above expression for the change of potential
energy (Equation (14.4.4)), let y = y be an arbitrary point and y = 0 denote the surface of the earth. Choose the zero reference
f i

potential for the potential energy to be at the surface of the earth corresponding to our origin y = 0 , with U (0) = 0 . Then g

g g g g
ΔU =U (y) − U (0) = U (y)

Substitute y = 0, y = y and Equation (14.4.6) into Equation (14.4.4) yielding a potential energy as a function of the height y
i f

above the surface of the earth,


g g
U (y) = mgy,  with U (y = 0) = 0

Hooke’s Law Spring-Object System


Consider a spring-object system lying on a frictionless horizontal surface with one end of the spring fixed to a wall and the other
end attached to an object of mass m (Figure 14.7). The spring force is an internal conservative force. The wall exerts an external

14.4.1 https://phys.libretexts.org/@go/page/24511
force on the spring-object system but since the point of contact of the wall with the spring undergoes no displacement, this external
force does no work.

Figure 14.7 A spring-object system.


Choose the origin at the position of the center of the object when the spring is relaxed (the equilibrium position). Let x be the
displacement of the object from the origin. We choose the +^i unit vector to point in the direction the object moves when the spring
→s
is being stretched (to the right of x = 0 in the figure). The spring force on a mass is then given by F
s^ ^
= Fx i = −kx i . The
→ → →
displacement is ^
d r = dx i . The scalar product is ^ ^
F ⋅ d r = −kx i ⋅ dx i = −kxdx . The work done by the spring force on the
mass is
x=xf x=xf
→ → 1 1 1
s 2 2
W =∫ F ⋅d r =− ∫ − (−kx)dx = − k (x −x )
f i
x=xi
2 x=x
2 2
i

We then define the change in potential energy in the spring-object system in moving the object from an initial position xi from
equilibrium to a final position x from equilibrium by
f

s s s s
1
2 2
ΔU ≡U (xf ) − U (xi ) = −W = k (x −x )
f i
2

Therefore an arbitrary stretch or compression of a spring-object system from equilibrium x i =0 to a final position x
f =x changes
the potential energy by

s s s
1 2
ΔU =U (xf ) − U (0) = kx
2

For the spring-object system, there is an obvious choice of position where the potential energy is zero, the equilibrium position of
the spring- object,
s
U (0) ≡ 0

Then with this choice of zero reference potential, the potential energy as a function of the displacement x from the equilibrium
position is given by
1
s 2 s
U (x) = kx ,  with U (0) ≡ 0
2

Inverse Square Gravitation Force


Consider a system consisting of two objects of masses m and m that are separated by a center-to-center distance r . A
1 2 2,1

coordinate system is shown in the Figure 14.8. The internal gravitational force on object 1 due to the interaction between the two
objects is given by
→G Gm1 m2
F 2,1 = − ^
r2,1
2
r
2,1


The displacement vector is given by d r 2,1 = dr2,1 ^
r2,1 . So the scalar product is

14.4.2 https://phys.libretexts.org/@go/page/24511
G
→ → Gm1 m2 Gm1 m2
F 2,1 ⋅ d r 2,1 = − ^
r2,1 ⋅ dr2,1 ^
r2,1 = − dr2,1
2 2
r r
2,1 2,1

Figure 14.8 Gravitational interaction


Using our definition of potential energy (Equation (14.3.4)), we have that the change in the gravitational potential energy of the
system in moving the two objects from an initial position in which the center of mass of the two objects are a distance r apart to a i

final position in which the center of mass of the two objects are a distance r apart is given by f

B rf
→G f
Gm1 m2 Gm1 m2 ∣ Gm1 m2 Gm1 m2
G →
ΔU = −∫ F 2,1 ⋅ d r 2,1 = − ∫ − dr2,1 = − ∣ =− +
2
A ri r r2,1 ∣ rf ri
2,1 ri

We now choose our reference point for the zero of the potential energy to be at infinity, r = ∞ with the choice that U (∞) ≡ 0 .
i
G

By making this choice, the term 1/r in the i expression for the change in potential energy vanishes when r = ∞ . The gravitational i

potential energy as a function of the relative distance r between the two objects is given by

G
Gm1 m2 G
U (r) = − ,  with U (∞) ≡ 0
r

This page titled 14.4: Change in Potential Energy and Zero Point for Potential Energy is shared under a CC BY-NC-SA 4.0 license and was
authored, remixed, and/or curated by Peter Dourmashkin (MIT OpenCourseWare) via source content that was edited to the style and standards of
the LibreTexts platform; a detailed edit history is available upon request.

14.4.3 https://phys.libretexts.org/@go/page/24511
14.5: Mechanical Energy and Conservation of Mechanical Energy
The total change in the mechanical energy of the system is defined to be the sum of the changes of the kinetic and the potential
energies,

ΔEm = ΔKsys + ΔUsys

For a closed system with only conservative internal forces, the total change in the mechanical energy is zero,

ΔEm = ΔKsys + ΔUsys = 0

Equation (14.4.18) is the symbolic statement of what is called conservation of mechanical energy. Recall that the work done by a
conservative force in going around a closed path is zero (Equation (14.2.16)), therefore both the changes in kinetic energy and
potential energy are zero when a closed system with only conservative internal forces returns to its initial state. Throughout the
process, the kinetic energy may change into internal potential energy but if the system returns to its initial state, the kinetic energy
is completely recoverable. We shall refer to a closed system in which processes take place in which only conservative forces act as
completely reversible processes.

Change in Gravitational potential Energy Near Surface of the Earth


Let’s consider the example of an object of mass m falling near the surface of the earth (mass m ). Choose our system to consist of
o e

the earth and the object. The gravitational force is now an internal conservative force acting inside the system. The initial and final
states are specified by the distance separating the object and the center of mass of the earth, and the velocities of the earth and the
object. The change in kinetic energy between the initial and final states for the system is

ΔKsys  = ΔKe + ΔKo

1 2
1 2
1 2
1 2
ΔKsys = ( me (ve,f ) − me (ve,i ) ) + ( mo (vo,f ) − mo (vo,i) )
2 2 2 2

The change of kinetic energy of the earth due to the gravitational interaction between the earth and the object is negligible. The
change in kinetic energy of the system is approximately equal to the change in kinetic energy of the object,
1 2 1 2
ΔKsys ≅ΔKo = mo (vo,f ) − mo (vo,i)
2 2

We now define the mechanical energy function for the system


1 2
g g
Em = K + U = mo (vb ) + mo gy,  with U (0) = 0
2

where K is the kinetic energy and U is the potential energy. The change in mechanical energy is then
g

g g
ΔEm ≡ Em,f − Em,i = (Kf + U ) − (Ki + U )
f i

When the work done by the external forces is zero and there are no internal nonconservative forces, the total mechanical energy of
the system is constant,

Em,f = Em,i

or equivalently

(Kf + Uf ) = (Ki + Ui )

This page titled 14.5: Mechanical Energy and Conservation of Mechanical Energy is shared under a CC BY-NC-SA 4.0 license and was authored,
remixed, and/or curated by Peter Dourmashkin (MIT OpenCourseWare) via source content that was edited to the style and standards of the
LibreTexts platform; a detailed edit history is available upon request.

14.5.1 https://phys.libretexts.org/@go/page/24512
14.6: Spring Force Energy Diagram
s

The spring force on an object is a restoring force F = F ^i = −kx ^i where we choose a coordinate system with the equilibrium
s
x

position at x = 0 and x is the amount the spring has been stretched (x > 0) or compressed (x < 0) from its equilibrium position.
i

We calculate the potential energy difference Equation (14.4.9) and found that
x
s s
1 2
s 2
U (x) − U (xi ) = − ∫ Fx dx = k (x −x )
i
xi
2

The first fundamental theorem of calculus states that

x =x
′ dU ′
U (x) − U (xi ) = ∫ x dx


x =xi
dx

Comparing Equation (14.5.1) with Equation (14.5.2) shows that the force is the negative derivative (with respect to position) of the
potential energy,
s
dU (x)
s
Fx = −
dx

Choose the zero reference point for the potential energy to be at the equilibrium position, U
s
(0) ≡ 0 Then the potential energy
function becomes
s 1 2
U (x) = kx
2

s
dU (x) d 1
s 2
Fx = − =− ( kx ) = −kx
dx dx 2

In Figure 14.9 we plot the potential energy function U


s
(x) for the spring force as function of x with U
s
(0) ≡ 0 (the units are
arbitrary).

Figure 14.9 Graph of potential energy function as function of x for the spring.
The minimum of the potential energy function occurs at the point where the first derivative vanishes
s
dU (x)
=0
dx

From Equation (14.5.4), the minimum occurs at x = 0,


s
dU (x)
0 = = kx
dx

Because the force is the negative derivative of the potential energy, and this derivative vanishes at the minimum, we have that the
spring force is zero at the minimum x = 0 agreeing with our force law, F | = −kx| =0 .
s
x x=0 x=0

The potential energy function has positive curvature in the neighborhood of a minimum equilibrium point. If the object is extended
a small distance x > 0 away from equilibrium, the slope of the potential energy function is positive, dU (x)/dx > 0 , hence the
component of the force is negative because F = −dU (x)/dx < 0 . Thus the object experiences a restoring force towards the
x

minimum point of the potential. If the object is compresses with x < 0 then dU (x)/dx < 0 , hence the component of the force is

14.6.1 https://phys.libretexts.org/@go/page/24513
positive, F = −dU (x)/dx > 0 and the object again experiences a restoring force back towards the minimum of the potential
x

energy as in Figure 14.10.

Figure 14.10 Stability diagram for the spring force


The mechanical energy at any time is the sum of the kinetic energy K x( ) and the potential energy U s
(x)

s
Em = K(x) + U (x)

Suppose our spring-object system has no loss of mechanical energy due to dissipative forces such as friction or air resistance. Both
the kinetic energy and the potential energy are functions of the position of the object with respect to equilibrium. The energy is a
constant of the motion and with our choice of U (0) ≡ 0 , the energy can be either a positive value or zero. When the energy is
s

zero, the object is at rest at the equilibrium position.


In Figure 14.10, we draw a straight horizontal line corresponding to a non-zero positive value for the energy E on the graph of m

potential energy as a function of x . The energy intersects the potential energy function at two points
{−x max ,x }  with x
max > 0 . These points correspond to the maximum compression and maximum extension of the spring,
max

which are called the turning points. The kinetic energy is the difference between the energy and the potential energy,
s
K(x) = Em − U (x)

At the turning points, where E = U (x) , the kinetic energy is zero. Regions where the kinetic energy is negative,
m
s

x < −x  or x > x


max are called the classically forbidden regions, which the object can never reach if subject to the laws of
max

classical mechanics. In quantum mechanics, with similar energy diagrams for quantum systems, there is a very small probability
that the quantum object can be found in a classically forbidden region.
Example 14.1 Energy Diagram
The potential energy function for a particle of mass m , moving in the x -direction is given by
3 2
x x
U (x) = −U1 ( ( ) −( ) )
x1 x1

where U and x are positive constants and U (0) = 0 . (a) Sketch U (x)/U as a function of x/x . (b) Find the points where the
1 1 1 1

force on the particle is zero. Classify them as stable or unstable. Calculate the value of U (x)/U at these equilibrium points. (c)
1

For energies E that lies in 0 < E < (4/27)U find an equation whose solution yields the turning points along the x-axis about
1

which the particle will undergo periodic motion. (d) Suppose E = (4/27)U and that the particle starts at x = 0 with speed v .
1 0

Find v .
0

Solution: a) Figure 14.11 shows a graph of U(x) vs. x , with the choice of values x 1 = 1.5m U1 = 27/4J,  and E = 0.2J .

14.6.2 https://phys.libretexts.org/@go/page/24513
Figure 14.11 Energy diagram for Example 14.1
b) The force on the particle is zero at the minimum of the potential which occurs at

dU 3 2
2
Fx (x) = − (x) = U1 (( )x −( ) x) = 0
3 2
dx x x
1 1

which becomes
2
x = (2 x1 /3) x

We can solve Equation (14.5.12) for the extrema. This has two solutions
x = (2 x1 /3) and \(\

x =0 (14.6.1)

)
The second derivative is given by
2
d U 6 2
(x) = −U1 (( ) x −( ))
dx2 3
x x
2
1 1

Evaluating the second derivative at x = (2x 1 /3) yields a negative quantity


2
d U 6 2x1 2 2U1
(x = (2 x1 /3)) = −U1 (( ) −( )) = − <0
2 3 2 2
dx x 3 x x
1 1 1

indicating the solution x = (2x /3) represents a local maximum and hence is an unstable point. At x = (2x /3), , the potential
1 1

energy is given by the value U ((2x /3)) = (4/27)U . Evaluating the second derivative at x = 0 yields a positive quantity
1 1

2
d U 6 2 2U1
(x = 0) = −U1 (( ) 0 −( )) = >0
2 3 2 2
dx x x x
1 1 1

indicating the solution x=0 represents a local minimum and is a stable point. At the local minimum x = 0, the potential energy U(0)
= 0.
c) Consider a fixed value of the energy of the particle within the range
4U1
U (0) = 0 < E < U (2 x1 /3) =
27

If the particle at any time is found in the region x a < x < xb < 2 x1 /3 , where x and x are the turning points and are solutions to
a b

the equation
3 2
x x
E = U (x) = −U1 ( ( ) −( ) )
x1 x1

14.6.3 https://phys.libretexts.org/@go/page/24513
then the particle will undergo periodic motion between the values x < x < x . Within this region x < x < x , the kinetic
a b a b

energy is always positive because K(x) = E − U (x) There is another solution x to Equation (14.5.18) somewhere in the region
c

x > 2 x /3 . If the particle at any time is in the region x > x then it at any later time it is restricted to the region x < x < +∞ .
c 1 c c

For E > U (2x /3) = (4/27)U , , Equation (14.5.18) has only one solution x . . For all values of x > x , the kinetic energy is
1 1 d d

positive, which means that the particle can “escape” to infinity but can never enter the region x < x . d

For E < U (0) = 0 the kinetic energy is negative for the range −∞ < x < x where x satisfies Equation (14.5.18) and therefore
e e

this region of space is forbidden.


(d) If the particle has speed v0 at x = 0 where the potential energy is zero, U (0) = 0 , the energy of the particle is constant and
equal to kinetic energy
1
2
E = K(0) = mv
0
2

Therefore
1
2
(4/27)U1 = mv
0
2

which we can solve for the speed


−−−−−−−−
v0 = √8 U1 /27m

This page titled 14.6: Spring Force Energy Diagram is shared under a CC BY-NC-SA 4.0 license and was authored, remixed, and/or curated by
Peter Dourmashkin (MIT OpenCourseWare) via source content that was edited to the style and standards of the LibreTexts platform; a detailed
edit history is available upon request.

14.6.4 https://phys.libretexts.org/@go/page/24513
14.7: Change of Mechanical Energy for Closed System with Internal
Nonconservative Forces
Consider a closed system (energy of the system is constant) that undergoes a transformation from an initial state to a final state by a
prescribed set of changes.
Whenever the work done by a force in moving an object from an initial point to a final point depends on the path, the force is called
a non-conservative force.
Suppose the internal forces are both conservative and non-conservative. The work W done by the forces is a sum of the
conservative work W which is path-independent, and the non-conservative work W which is path-dependent,
c nc

W = Wc + Wnc

The work done by the conservative forces is equal to the negative of the change in the potential energy

ΔU = −Wc

Substituting Equation (14.6.2) into Equation (14.6.1) yields

W = −ΔU + Wnc

The work done is equal to the change in the kinetic energy,

W = ΔK

Substituting Equation (14.6.4) into Equation (14.6.3) yields

ΔK = −ΔU + Wnc

which we can rearrange as

Wnc = ΔK + ΔU

We can now substitute Equation (14.6.4) into our expression for the change in the mechanical energy, Equation (14.4.17), with the
result

Wnc = ΔEm

The mechanical energy is no longer constant. The total change in energy of the system is zero,

ΔEsystem  = ΔEm − Wnc = 0

Energy is conserved but some mechanical energy has been transferred into nonrecoverable energy W . We shall refer to processes
nc

in which there is non-zero nonrecoverable energy as irreversible processes.

Change of Mechanical Energy for a Non-closed System


When the system is no longer closed but in contact with its surroundings, the change in energy of the system is equal to the
negative of the change in energy of the surroundings (Equation (14.1.1)),

ΔEsystem  = −ΔEsurroundings 

If the system is not isolated, the change in energy of the system can be the result of external work done by the surroundings on the
system (which can be positive or negative)
B
→ →
Wext = ∫ F ext ⋅ d r
A

This work will result in the system undergoing coherent motion. Note that W > 0 if work is done on the system
ext

(ΔE < 0) and W


surroundings  < 0 if the system does work on the surroundings (ΔE
ext  > 0) . If the system is in thermal
surroundings 

contact with the surroundings, then energy can flow into or out of the system. This energy flow due to thermal contact is often

14.7.1 https://phys.libretexts.org/@go/page/27793
denoted by Q with the convention that Q >0 if the energy flows into the system (ΔE surroundings  < 0) and Q <0 if the energy
flows out of the system (ΔE surroundings  > 0) . Then Equation (14.6.9) can be rewritten as
ext
W + Q = ΔEsys

Equation (14.6.11) is also called the first law of thermodynamics.


This will result in either an increase or decrease in random thermal motion of the molecules inside the system, There may also be
other forms of energy that enter the system, for example radiative energy.
Several questions naturally arise from this set of definitions and physical concepts. Is it possible to identify all the conservative
forces and calculate the associated changes in potential energies? How do we account for non-conservative forces such as friction
that act at the boundary of the system?

This page titled 14.7: Change of Mechanical Energy for Closed System with Internal Nonconservative Forces is shared under a not declared
license and was authored, remixed, and/or curated by Peter Dourmashkin (MIT OpenCourseWare) via source content that was edited to the style
and standards of the LibreTexts platform; a detailed edit history is available upon request.

14.7.2 https://phys.libretexts.org/@go/page/27793
14.8: Dissipative Forces- Friction
Suppose we consider an object moving on a rough surface. As the object slides it slows down and stops. While the sliding occurs
both the object and the surface increase in temperature. The increase in temperature is due to the molecules inside the materials
increasing their kinetic energy. This random kinetic energy is called thermal energy. Kinetic energy associated with the coherent
motion of the molecules of the object has been dissipated into kinetic energy associated with random motion of the molecules
composing the object and surface.
If we define the system to be just the object, then the friction force acts as an external force on the system and results in the
dissipation of energy into both the block and the surface. Without knowing further properties of the material we cannot determine
the exact changes in the energy of the system.
Friction introduces a problem in that the point of contact is not well defined because the surface of contact is constantly deforming
as the object moves along the surface. If we considered the object and the surface as the system, then the friction force is an internal
force, and the decrease in the kinetic energy of the moving object ends up as an increase in the internal random kinetic energy of
the constituent parts of the system. When there is dissipation at the boundary of the system, we need an additional model (thermal
equation of state) for how the dissipated energy distributes itself among the constituent parts of the system.

Source Energy
Consider a person walking. The frictional force between the person and the ground does no work because the point of contact
between the person’s foot and the ground undergoes no displacement as the person applies a force against the ground, (there may
be some slippage but that would be opposite the direction of motion of the person). However the kinetic energy of the object
increases. Have we disproved the work-energy theorem? The answer is no! The chemical energy stored in the body tissue is
converted to kinetic energy and thermal energy. Because the person-air-ground can be treated as a closed system, we have that

0 = ΔEsys  = ΔEchemical  + ΔEthermal  + ΔEmechanical 

If we assume that there is no change in the potential energy of the system, then ΔE = ΔK . Therefore some of the
mechanical 

internal chemical energy has been transformed into thermal energy and the rest has changed into the kinetic energy of the system,

−ΔEchemical  = ΔEthermal  + ΔK

This page titled 14.8: Dissipative Forces- Friction is shared under a not declared license and was authored, remixed, and/or curated by Peter
Dourmashkin (MIT OpenCourseWare) via source content that was edited to the style and standards of the LibreTexts platform; a detailed edit
history is available upon request.

14.8.1 https://phys.libretexts.org/@go/page/27794
14.9: Worked Examples
Example 14.2 Escape Velocity of Toro
The asteroid Toro, discovered in 1964, has a radius of about R = 5.0km and a mass of about m = 2.0 × 10 kg . Let’s assume t
15

that Toro is a perfectly uniform sphere. What is the escape velocity for an object of mass m on the surface of Toro? Could a person
reach this speed (on earth) by running?
Solution: The only potential energy in this problem is the gravitational potential energy. We choose the zero point for the potential
energy to be when the object and Toro are an infinite distance apart, U (∞) ≡ 0 . With this choice, the potential energy when the
G

object and Toro are a finite distance r apart is given by

G
Gmt m
U (r) = −
r

with U (∞) ≡ 0 The expression escape velocity refers to the minimum speed necessary for an object to escape the gravitational
G

interaction of the asteroid and move off to an infinite distance away. If the object has a speed less than the escape velocity, it will be
unable to escape the gravitational force and must return to Toro. If the object has a speed greater than the escape velocity, it will
have a non-zero kinetic energy at infinity. The condition for the escape velocity is that the object will have exactly zero kinetic
energy at infinity.
We choose our initial state, at time t , when the object is at the surface of the asteroid with speed equal to the escape velocity. We
i

choose our final state, at time t , to occur when the separation distance between the asteroid and the object is infinite.
f

The initial kinetic energy is Ki = (1/2)m vesc


2
. The initial potential energy is Ui = −Gmt m/R and so the initial mechanical
energy is
1 Gmt m
2
Ei = Ki + Ui = m vesc −
2 R

The final kinetic energy is K f =0 because this is the c


ondition that defines the escape velocity. The final potential energy is zero, Uf = 0 because we chose the zero point for potential
energy at infinity. The final mechanical energy is then

Ef = Kf + Uf = 0

There is no non-conservative work, so the change in mechanical energy is zero

0 = Wnc = ΔEm = Ef − Ei

Therefore
1 Gmt m
2
0 = −( m vesc − )
2 R

This can be solved for the escape velocity,


−−−−−−
2Gmt
vesc = √
R
−−−−−−−−−−−−−−−−−−−−−−−−−−−−−−−−−− −
 −11 −2 15
2
 2 (6.67 × 10 N ⋅ m ⋅ kg ) (2.0 × 10 kg)
−1
= = 7.3m ⋅ s
3
⎷ (5.0 × 10 m)

Considering that Olympic sprinters typically reach velocities of 12m ⋅ s , this is an easy speed to attain by running on earth. It
−1

may be harder on Toro to generate the acceleration necessary to reach this speed by pushing off the ground, since any slight upward
force will raise the runner’s center of mass and it will take substantially more time than on earth to come back down for another
push off the ground.
Example 14.3 Spring-Block-Loop-the-Loop
A small block of mass m is pushed against a spring with spring constant k and held in place with a catch. The spring is compressed
an unknown distance x (Figure 14.12). When the catch is removed, the block leaves the spring and slides along a frictionless

14.9.1 https://phys.libretexts.org/@go/page/27795
circular loop of radius r . When the block reaches the top of the loop, the force of the loop on the block (the normal force) is equal
to twice the gravitational force on the mass. (a) Using conservation of energy, find the kinetic energy of the block at the top of the
loop. (b) Using Newton’s Second Law, derive the equation of motion for the block when it is at the top of the loop. Specifically,
find the speed vtop  in terms of the gravitation constant g and the loop radius r . (c) What distance was the spring compressed?

Figure 14.12 Initial state for spring-block-loop-the-loop system


Solution: a) Choose for the initial state the instant before the catch is released. The initial kinetic energy is Ki = 0 . The initial
potential energy is non-zero, U = (1/2)kx . The initial mechanical energy is then
i
2

1 2
Ei = Ki + Ui = kx
2

Choose for the final state the instant the block is at the top of the loop. The final kinetic energy is K = (1/2)mv ; the block is
f
2
top 

in motion with speed v . The final potential energy is non-zero, U = (mg)(2R) . The final mechanical energy is then
top  f

1
2
Ef = Kf + Uf = 2mgR + m vtop
2

Because we are assuming the track is frictionless and neglecting air resistance, there is no non- conservative work. The change in
mechanical energy is therefore zero,

0 = Wnc = ΔEm = Ef − Ei

Mechanical energy is conserved, E f = Ei , therefore


1 1 2
2
2mgR + m vtop = kx
2 2

From Equation (14.8.10), the kinetic energy at the top of the loop is
1 1
2 2
mv = kx − 2mgR
top
2 2

b) At the top of the loop, the forces on the block are the gravitational force of magnitude mg and the normal force of magnitude N ,
both directed down. Newton’s Second Law in the radial direction, which is the downward direction, is
2
mv
top
−mg − N = −
R

In this problem, we are given that when the block reaches the top of the loop, the force of the loop on the block (the normal force,
downward in this case) is equal to twice the weight of the block, N = 2mg . The Second Law, Equation (14.8.12), then becomes
2
mvtop
3mg =
R

We can rewrite Equation (14.8.13) in terms of the kinetic energy as


3 1
2
mgR = mv
top
2 2

The speed at the top is therefore


−−−−−
vtop = √3mgR

14.9.2 https://phys.libretexts.org/@go/page/27795
c) Combing Equations (14.8.11) and (14.8.14) yields
7 1 2
mgR = kx
2 2

Thus the initial displacement of the spring from equilibrium is


−−−−−−
7mgR
x =√
k

Example 14.4 Mass-Spring on a Rough Surface


A block of mass m slides along a horizontal table with speed v . At x = 0 it hits a spring with spring constant k and begins to
0

experience a friction force. The coefficient of friction is variable and is given by μ = bx , where b is a positive constant. Find the
loss in mechanical energy when the block first momentarily comes to rest.

Figure 14.13 Spring-block system


Solution: From the model given for the frictional force, we could find the nonconservative work done, which is the same as the loss
of mechanical energy, if we knew the position x where the block first comes to rest. The most direct (and easiest) way to find x
f f

is to use the work-energy theorem. The initial mechanical energy is E = mv /2 and the final mechanical energy is E = kx /2
i
2
i f
2
f

(note that there is no potential energy term in E and no kinetic energy term E The difference between these two mechanical
i f

energies is the non-conservative work done by the frictional force,


x=xf x=xf x=xf

Wnc = ∫ Fnc dx = ∫ −Ff rictiondx = ∫ −μN dx


x=0 x=0 x=0
xf
1
2
= −∫ bxmgdx = − bmgx
f
0 2

We then have that

Wnc = ΔEm

Wnc = Ef − Ei

1 1 1
2 2 2
− bmgx = kx − mv
f f i
2 2 2

Solving the last of these equations for x yields


2
f

2
mv
2 0
x =
f
k + bmg

Substitute Equation (14.8.20) into Equation (14.8.18) gives the result that
2 2 −1
bmg mv mv k
0 0
Wnc = − =− (1 + )
2 k + bmg 2 bmg

It is worth checking that the above result is dimensionally correct. From the model, the parameter b must have dimensions of
inverse length (the coefficient of friction μ must be dimensionless), and so the product bmg has dimensions of force per length, as
does the spring constant k; the result is dimensionally consistent.
Example 14.5 Cart-Spring on an Inclined Plane

14.9.3 https://phys.libretexts.org/@go/page/27795
An object of mass m slides down a plane that is inclined at an angle θ from the horizontal (Figure 14.14). The object starts out at
rest. The center of mass of the cart is a distance d from an unstretched spring that lies at the bottom of the plane. Assume the spring
is massless, and has a spring constant k . Assume the inclined plane to be frictionless. (a) How far will the spring compress when
the mass first comes to rest? (b) Now assume that the inclined plane has a coefficient of kinetic friction μ How far will the spring
k

compress when the mass first comes to rest? The friction is primarily between the wheels and the bearings, not between the cart
and the plane, but the friction force may be modeled by a coefficient of friction μ . . (c) In case (b), how much energy has been lost
k

to friction?

Figure 14.14 Cart on inclined plane


Solution: Let x denote the displacement of the spring from the equilibrium position. Choose the zero point for the gravitational
potential energy U (0) = 0 not at the very bottom of the inclined plane, but at the location of the end of the unstretched spring.
g

Choose the zero point for the spring potential energy where the spring is at its equilibrium position, U (0) = 0
s

a) Choose for the initial state the instant the object is released (Figure 14.15). The initial kinetic energy is Ki = 0 . The initial
potential energy is non-zero, U = mgd sin θ . The initial mechanical energy is then
i

Ei = Ki + Ui = mgd sin θ

Choose for the final state the instant when the object first comes to rest and the spring is compressed a distance x at the bottom of
the inclined plane (Figure 14.16). The final kinetic energy is K = 0 since the mass is not in motion. The final potential energy is
f

non-zero, U = kx /2 − xmg sin θ Notice that the gravitational potential energy is negative because the object has dropped below
f
2

the height of the zero point of gravitational potential energy.

Figure 14.15 Initial state

Figure 14.16 Final state


The final mechanical energy is then
1 2
Ef = Kf + Uf = kx − xmg sin θ
2

Because we are assuming the track is frictionless and neglecting air resistance, there is no non- conservative work. The change in
mechanical energy is therefore zero,

0 = Wnc = ΔEm = Ef − Ei

14.9.4 https://phys.libretexts.org/@go/page/27795
Therefore
1 2
dmg sin θ = kx − xmg sin θ
2

This is a quadratic equation in x ,


2mg sin θ 2dmg sin θ
2
x − x− =0
k k

In the quadratic formula, we want the positive choice of square root for the solution to ensure a positive displacement of the spring
from equilibrium,
1/2
2 2 2
mg sin θ m g sin θ 2dmg sin θ
x = +( + )
2
k k k

mg −−−−−−−−−−−−−−−
= (sin θ + √ 1 + 2(kd/mg) sin θ )
k

(What would the solution with the negative root represent?)


b) The effect of kinetic friction is that there is now a non-zero non-conservative work done on the object, which has moved a
distance, d + x , given by

Wnc = −fk (d + x) = −μk N (d + x) = −μk mg cos θ(d + x)

Note the normal force is found by using Newton’s Second Law in the perpendicular direction to the inclined plane,

N − mg cos θ = 0

The change in mechanical energy is therefore

Wnc = ΔEm = Ef − Ei

which becomes
1
2
−μk mg cos θ(d + x) = ( kx − xmg sin θ) − dmg sin θ
2

Equation (14.8.31) simplifies to


1
2
0 =( kx − xmg (sin θ − μk cos θ)) − dmg (sin θ − μk cos θ)
2

This is the same as Equation (14.8.25) above, but with sin θ → sin θ − μ k cos θ . The maximum displacement of the spring is when
there is friction is then
mg −−−−−−−−−−−−−−−−−−−−−−−−
x = ((sin θ − μk cos θ) + √ 1 + 2(kd/mg) (sin θ − μk cos θ) )
k

c) The energy lost to friction is given by W nc = −μk mg cos θ(d + x) where x is given in part b).
Example 14.6 Object Sliding on a Sphere
A small point like object of mass m rests on top of a sphere of radius R . The object is released from the top of the sphere with a
negligible speed and it slowly starts to slide (Figure 14.17). Let g denote the gravitation constant. (a) Determine the angle θ with 1

respect to the vertical at which the object will lose contact with the surface of the sphere. (b) What is the speed v of the object at 1

the instant it loses contact with the surface of the sphere.

14.9.5 https://phys.libretexts.org/@go/page/27795
Figure 14.17 Object sliding on surface of sphere
Solution: We begin by identifying the forces acting on the object. There are two forces acting on the object, the gravitation and
radial normal force that the sphere exerts on the particle that we denote by N . We draw a free-body force diagram for the object
while it is sliding on the sphere. We choose polar coordinates as shown in Figure 14.18.

Figure 14.18 Free-body force diagram on object


The key constraint is that when the particle just leaves the surface the normal force is zero,

N (θ1 ) = 0

where θ denotes the angle with respect to the vertical at which the object will just lose contact with the surface of the sphere.
1

Because the normal force is perpendicular to the displacement of the object, it does no work on the object and hence conservation
of energy does not take into account the constraint on the motion imposed by the normal force. In order to analyze the effect of the
normal force we must use the radial component of Newton’s Second Law,
2
v
N − mg cos θ = −m
R

Then when the object just loses contact with the surface, Equations (14.8.34) and (14.8.35) require that
2
v
1
mg cos θ1 = m
R

where v denotes the speed of the object at the instant it loses contact with the surface of the sphere. Note that the constrain
1

condition Equation (14.8.36) can be rewritten as


2
mgR cos θ1 = m v
1

We can now apply conservation of energy. Choose the zero reference point U = 0 for potential energy to be the midpoint of the
sphere.
Identify the initial state as the instant the object is released (Figure 14.19). We can neglect the very small initial kinetic energy
needed to move the object away from the top of the sphere and so K = 0 . The initial potential energy is non-zero, U = mgR .
i i

The initial mechanical energy is then

Ei = Ki + Ui = mgR

14.9.6 https://phys.libretexts.org/@go/page/27795
Figure 14.19 Initial state

Figure 14.20 Final state


Choose for the final state the instant the object leaves the sphere (Figure 14.20). The final kinetic energy is K = mv /2 ; the
f
2
1

object is in motion with speed v . The final potential energy is non-zero, U = mgR cos θ . The final mechanical energy is then
1 f 1

1
2
Ef = Kf + Uf = mv + mgR cos θ1
1
2

Because we are assuming the contact surface is frictionless and neglecting air resistance, there is no non-conservative work. The
change in mechanical energy is therefore zero,

0 = Wnc = ΔEm = Ef − Ei

Therefore
1
2
mv + mgR cos θ1 = mgR
1
2

We now solve the constraint condition Equation (14.8.37) into Equation (14.8.41) yielding
1
mgR cos θ1 + mgR cos θ1 = mgR
2

We can now solve for the angle at which the object just leaves the surface
−1
θ1 = cos (2/3)

We now substitute this result into Equation (14.8.37) and solve for the speed
−−−−−
v1 = √2gR/3

This page titled 14.9: Worked Examples is shared under a not declared license and was authored, remixed, and/or curated by Peter Dourmashkin
(MIT OpenCourseWare) via source content that was edited to the style and standards of the LibreTexts platform; a detailed edit history is
available upon request.

14.9.7 https://phys.libretexts.org/@go/page/27795

You might also like